You are on page 1of 114

B G.R. No.

135385 December 6, 2000 IPRA is consistent with the Constitution and pray that the petition for prohibition and mandamus be
ISAGANI CRUZ and CESAR EUROPA, petitioners, dismissed.
vs.
SECRETARY OF ENVIRONMENT AND NATURAL RESOURCES, SECRETARY OF BUDGET AND The motions for intervention of the aforesaid groups and organizations were granted.
MANAGEMENT and CHAIRMAN and COMMISSIONERS OF THE NATIONAL COMMISSION ON
INDIGENOUS PEOPLES, respondents. Oral arguments were heard on April 13, 1999. Thereafter, the parties and intervenors filed their
RESOLUTION respective memoranda in which they reiterate the arguments adduced in their earlier pleadings and
during the hearing.
PER CURIAM:
Petitioners Isagani Cruz and Cesar Europa brought this suit for prohibition and mandamus as citizens Petitioners assail the constitutionality of the following provisions of the IPRA and its Implementing
and taxpayers, assailing the constitutionality of certain provisions of Republic Act No. 8371 (R.A. Rules on the ground that they amount to an unlawful deprivation of the State’s ownership over lands
8371), otherwise known as the Indigenous Peoples Rights Act of 1997 (IPRA), and its Implementing of the public domain as well as minerals and other natural resources therein, in violation of the
Rules and Regulations (Implementing Rules). regalian doctrine embodied in Section 2, Article XII of the Constitution:

In its resolution of September 29, 1998, the Court required respondents to comment.1 In compliance, "(1) Section 3(a) which defines the extent and coverage of ancestral domains, and Section 3(b) which,
respondents Chairperson and Commissioners of the National Commission on Indigenous Peoples in turn, defines ancestral lands;
(NCIP), the government agency created under the IPRA to implement its provisions, filed on October
13, 1998 their Comment to the Petition, in which they defend the constitutionality of the IPRA and "(2) Section 5, in relation to section 3(a), which provides that ancestral domains including inalienable
pray that the petition be dismissed for lack of merit. public lands, bodies of water, mineral and other resources found within ancestral domains are private
but community property of the indigenous peoples;
On October 19, 1998, respondents Secretary of the Department of Environment and Natural
Resources (DENR) and Secretary of the Department of Budget and Management (DBM) filed through "(3) Section 6 in relation to section 3(a) and 3(b) which defines the composition of ancestral domains
the Solicitor General a consolidated Comment. The Solicitor General is of the view that the IPRA is and ancestral lands;
partly unconstitutional on the ground that it grants ownership over natural resources to indigenous
peoples and prays that the petition be granted in part. "(4) Section 7 which recognizes and enumerates the rights of the indigenous peoples over the
ancestral domains;
On November 10, 1998, a group of intervenors, composed of Sen. Juan Flavier, one of the authors of
the IPRA, Mr. Ponciano Bennagen, a member of the 1986 Constitutional Commission, and the leaders (5) Section 8 which recognizes and enumerates the rights of the indigenous peoples over the ancestral
and members of 112 groups of indigenous peoples (Flavier, et. al), filed their Motion for Leave to lands;
Intervene. They join the NCIP in defending the constitutionality of IPRA and praying for the dismissal
of the petition. "(6) Section 57 which provides for priority rights of the indigenous peoples in the harvesting,
extraction, development or exploration of minerals and other natural resources within the areas
On March 22, 1999, the Commission on Human Rights (CHR) likewise filed a Motion to Intervene claimed to be their ancestral domains, and the right to enter into agreements with nonindigenous
and/or to Appear as Amicus Curiae. The CHR asserts that IPRA is an expression of the principle of peoples for the development and utilization of natural resources therein for a period not exceeding
parens patriae and that the State has the responsibility to protect and guarantee the rights of those 25 years, renewable for not more than 25 years; and
who are at a serious disadvantage like indigenous peoples. For this reason it prays that the petition
be dismissed. "(7) Section 58 which gives the indigenous peoples the responsibility to maintain, develop, protect
and conserve the ancestral domains and portions thereof which are found to be necessary for critical
On March 23, 1999, another group, composed of the Ikalahan Indigenous People and the Haribon watersheds, mangroves, wildlife sanctuaries, wilderness, protected areas, forest cover or
Foundation for the Conservation of Natural Resources, Inc. (Haribon, et al.), filed a motion to reforestation."2
Intervene with attached Comment-in-Intervention. They agree with the NCIP and Flavier, et al. that

1
Petitioners also content that, by providing for an all-encompassing definition of "ancestral domains" "(1) A declaration that Sections 3, 5, 6, 7, 8, 52[I], 57, 58, 59, 63, 65 and 66 and other related provisions
and "ancestral lands" which might even include private lands found within said areas, Sections 3(a) of R.A. 8371 are unconstitutional and invalid;
and 3(b) violate the rights of private landowners.3
"(2) The issuance of a writ of prohibition directing the Chairperson and Commissioners of the NCIP to
In addition, petitioners question the provisions of the IPRA defining the powers and jurisdiction of the cease and desist from implementing the assailed provisions of R.A. 8371 and its Implementing Rules;
NCIP and making customary law applicable to the settlement of disputes involving ancestral domains
and ancestral lands on the ground that these provisions violate the due process clause of the "(3) The issuance of a writ of prohibition directing the Secretary of the Department of Environment
Constitution.4 and Natural Resources to cease and desist from implementing Department of Environment and
Natural Resources Circular No. 2, series of 1998;
These provisions are:
"(4) The issuance of a writ of prohibition directing the Secretary of Budget and Management to cease
"(1) sections 51 to 53 and 59 which detail the process of delineation and recognition of ancestral and desist from disbursing public funds for the implementation of the assailed provisions of R.A. 8371;
domains and which vest on the NCIP the sole authority to delineate ancestral domains and ancestral and
lands;
"(5) The issuance of a writ of mandamus commanding the Secretary of Environment and Natural
"(2) Section 52[i] which provides that upon certification by the NCIP that a particular area is an Resources to comply with his duty of carrying out the State’s constitutional mandate to control and
ancestral domain and upon notification to the following officials, namely, the Secretary of supervise the exploration, development, utilization and conservation of Philippine natural
Environment and Natural Resources, Secretary of Interior and Local Governments, Secretary of Justice resources."7
and Commissioner of the National Development Corporation, the jurisdiction of said officials over
said area terminates; After due deliberation on the petition, the members of the Court voted as follows:

"(3) Section 63 which provides the customary law, traditions and practices of indigenous peoples shall Seven (7) voted to dismiss the petition. Justice Kapunan filed an opinion, which the Chief Justice and
be applied first with respect to property rights, claims of ownership, hereditary succession and Justices Bellosillo, Quisumbing, and Santiago join, sustaining the validity of the challenged provisions
settlement of land disputes, and that any doubt or ambiguity in the interpretation thereof shall be of R.A. 8371. Justice Puno also filed a separate opinion sustaining all challenged provisions of the law
resolved in favor of the indigenous peoples; with the exception of Section 1, Part II, Rule III of NCIP Administrative Order No. 1, series of 1998, the
Rules and Regulations Implementing the IPRA, and Section 57 of the IPRA which he contends should
"(4) Section 65 which states that customary laws and practices shall be used to resolve disputes be interpreted as dealing with the large-scale exploitation of natural resources and should be read in
involving indigenous peoples; and conjunction with Section 2, Article XII of the 1987 Constitution. On the other hand, Justice Mendoza
voted to dismiss the petition solely on the ground that it does not raise a justiciable controversy and
"(5) Section 66 which vests on the NCIP the jurisdiction over all claims and disputes involving rights of petitioners do not have standing to question the constitutionality of R.A. 8371.
the indigenous peoples."5
Seven (7) other members of the Court voted to grant the petition. Justice Panganiban filed a separate
Finally, petitioners assail the validity of Rule VII, Part II, Section 1 of the NCIP Administrative Order opinion expressing the view that Sections 3 (a)(b), 5, 6, 7 (a)(b), 8, and related provisions of R.A. 8371
No. 1, series of 1998, which provides that "the administrative relationship of the NCIP to the Office of are unconstitutional. He reserves judgment on the constitutionality of Sections 58, 59, 65, and 66 of
the President is characterized as a lateral but autonomous relationship for purposes of policy and the law, which he believes must await the filing of specific cases by those whose rights may have been
program coordination." They contend that said Rule infringes upon the President’s power of control violated by the IPRA. Justice Vitug also filed a separate opinion expressing the view that Sections 3(a),
over executive departments under Section 17, Article VII of the Constitution.6 7, and 57 of R.A. 8371 are unconstitutional. Justices Melo, Pardo, Buena, Gonzaga-Reyes, and De Leon
join in the separate opinions of Justices Panganiban and Vitug.
Petitioners pray for the following:

2
As the votes were equally divided (7 to 7) and the necessary majority was not obtained, the case was A. The Laws of the Indies
redeliberated upon. However, after redeliberation, the voting remained the same. Accordingly,
pursuant to Rule 56, Section 7 of the Rules of Civil Procedure, the petition is DISMISSED. B. Valenton v. Murciano

Attached hereto and made integral parts thereof are the separate opinions of Justices Puno, Vitug, C. The Public Land Acts and the Torrens System
Kapunan, Mendoza, and Panganiban.
D. The Philippine Constitutions
SO ORDERED.
II. The Indigenous Peoples Rights Act (IPRA).
Davide, Jr., C.J., Bellosillo, Melo, Quisumbing, Pardo, Buena, Gonzaga-Reyes, Ynares-Santiago, and De
Leon, Jr., JJ., concur. A. Indigenous Peoples
Puno, Vitug, Kapunan, Mendoza and Panganiban JJ., see separate opinion
1. Indigenous Peoples: Their History
SEPARATE OPINION
PUNO, J.: 2. Their Concept of Land
PRECIS
A classic essay on the utility of history was written in 1874 by Friedrich Nietzsche entitled "On the III. The IPRA is a Novel Piece of Legislation.
Uses and Disadvantages of History for Life." Expounding on Nietzsche's essay, Judge Richard Posner1
wrote:2 A. Legislative History

"Law is the most historically oriented, or if you like the most backward-looking, the most 'past- IV. The Provisions of the IPRA Do Not Contravene the Constitution.
dependent,' of the professions. It venerates tradition, precedent, pedigree, ritual, custom, ancient
practices, ancient texts, archaic terminology, maturity, wisdom, seniority, gerontocracy, and A. Ancestral domains and ancestral lands are the private property of indigenous peoples and do not
interpretation conceived of as a method of recovering history. It is suspicious of innovation, constitute part of the land of the public domain.
discontinuities, 'paradigm shifts,' and the energy and brashness of youth. These ingrained attitudes
are obstacles to anyone who wants to re-orient law in a more pragmatic direction. But, by the same 1. The right to ancestral domains and ancestral lands: how acquired
token, pragmatic jurisprudence must come to terms with history."
2. The concept of native title
When Congress enacted the Indigenous Peoples Rights Act (IPRA), it introduced radical concepts into
the Philippine legal system which appear to collide with settled constitutional and jural precepts on (a) Cariño v. Insular Government
state ownership of land and other natural resources. The sense and subtleties of this law cannot be
appreciated without considering its distinct sociology and the labyrinths of its history. This Opinion (b) Indian Title to land
attempts to interpret IPRA by discovering its soul shrouded by the mist of our history. After all, the
IPRA was enacted by Congress not only to fulfill the constitutional mandate of protecting the (c) Why the Cariño doctrine is unique
indigenous cultural communities' right to their ancestral land but more importantly, to correct a grave
historical injustice to our indigenous people. 3. The option of securing a torrens title to the ancestral land

This Opinion discusses the following: B. The right of ownership and possession by the ICCs/IPs to their ancestral domains is a limited form
of ownership and does not include the right to alienate the same.
I. The Development of the Regalian Doctrine in the Philippine Legal System.
1. The indigenous concept of ownership and customary law

3
may be necessary for tillage and pasturage, confirming them in what they now have and giving them
C. Sections 7 (a), 7 (b) and 57 of the IPRA do not violate the Regalian Doctrine enshrined in Section 2, more if necessary, all the rest of said lands may remain free and unencumbered for us to dispose of
Article XII of the 1987 Constitution. as we may wish.

1. The rights of ICCs/IPs over their ancestral domains and lands We therefore order and command that all viceroys and presidents of pretorial courts designate at
such time as shall to them seem most expedient, a suitable period within which all possessors of
2. The right of ICCs/IPs to develop lands and natural resources within the ancestral domains does not tracts, farms, plantations, and estates shall exhibit to them and to the court officers appointed by
deprive the State of ownership over the natural resources, control and supervision in their them for this purpose, their title deeds thereto. And those who are in possession by virtue of proper
development and exploitation. deeds and receipts, or by virtue of just prescriptive right shall be protected, and all the rest shall be
restored to us to be disposed of at our will."4
(a) Section 1, Part II, Rule III of the Implementing Rules goes beyond the parameters of Section 7(a)
of the law on ownership of ancestral domains and is ultra vires. The Philippines passed to Spain by virtue of "discovery" and conquest. Consequently, all lands became
the exclusive patrimony and dominion of the Spanish Crown. The Spanish Government took charge
(b) The small-scale utilization of natural resources in Section 7 (b) of the IPRA is allowed under of distributing the lands by issuing royal grants and concessions to Spaniards, both military and
Paragraph 3, Section 2, Article XII of the 1987 Consitution. civilian.5 Private land titles could only be acquired from the government either by purchase or by the
various modes of land grant from the Crown.6
(c) The large-scale utilization of natural resources in Section 57 of the IPRA may be harmonized with
Paragraphs 1 and 4, Section 2, Article XII of the 1987 Constitution. The Laws of the Indies were followed by the Ley Hipotecaria, or the Mortgage Law of 1893.7 The
Spanish Mortgage Law provided for the systematic registration of titles and deeds as well as
V. The IPRA is a Recognition of Our Active Participation in the International Indigenous Movement. possessory claims. The law sought to register and tax lands pursuant to the Royal Decree of 1880. The
Royal Decree of 1894, or the "Maura Law," was partly an amendment of the Mortgage Law as well as
DISCUSSION the Laws of the Indies, as already amended by previous orders and decrees.8 This was the last Spanish
land law promulgated in the Philippines. It required the "adjustment" or registration of all agricultural
I. THE DEVELOPMENT OF THE REGALIAN DOCTRINE IN THE PHILIPPINE LEGAL SYSTEM. lands, otherwise the lands shall revert to the state.

A. The Laws of the Indies Four years later, by the Treaty of Paris of December 10, 1898, Spain ceded to the government of the
United States all rights, interests and claims over the national territory of the Philippine Islands. In
The capacity of the State to own or acquire property is the state's power of dominium.3 This was the 1903, the United States colonial government, through the Philippine Commission, passed Act No. 926,
foundation for the early Spanish decrees embracing the feudal theory of jura regalia. The "Regalian the first Public Land Act.
Doctrine" or jura regalia is a Western legal concept that was first introduced by the Spaniards into the
country through the Laws of the Indies and the Royal Cedulas. The Laws of the Indies, i.e., more B. Valenton v. Murciano
specifically, Law 14, Title 12, Book 4 of the Novisima Recopilacion de Leyes de las Indias, set the policy
of the Spanish Crown with respect to the Philippine Islands in the following manner: In 1904, under the American regime, this Court decided the case of Valenton v. Murciano.9

"We, having acquired full sovereignty over the Indies, and all lands, territories, and possessions not Valenton resolved the question of which is the better basis for ownership of land: long-time
heretofore ceded away by our royal predecessors, or by us, or in our name, still pertaining to the royal occupation or paper title. Plaintiffs had entered into peaceful occupation of the subject land in 1860.
crown and patrimony, it is our will that all lands which are held without proper and true deeds of Defendant's predecessor-in-interest, on the other hand, purchased the land from the provincial
grant be restored to us as they belong to us, in order that after reserving before all what to us or to treasurer of Tarlac in 1892. The lower court ruled against the plaintiffs on the ground that they had
our viceroys, audiencias, and governors may seem necessary for public squares, ways, pastures, and lost all rights to the land by not objecting to the administrative sale. Plaintiffs appealed the judgment,
commons in those places which are peopled, taking into consideration not only their present asserting that their 30-year adverse possession, as an extraordinary period of prescription in the
condition, but also their future and their probable increase, and after distributing to the natives what

4
Partidas and the Civil Code, had given them title to the land as against everyone, including the State; virtue of which said lands are occupied. x x x. Said subdelegates will at the same time warn the parties
and that the State, not owning the land, could not validly transmit it. interested that in case of their failure to present their title deeds within the term designated, without
a just and valid reason therefor, they will be deprived of and evicted from their lands, and they will
The Court, speaking through Justice Willard, decided the case on the basis of "those special laws which be granted to others."15
from earliest time have regulated the disposition of the public lands in the colonies."10 The question
posed by the Court was: "Did these special laws recognize any right of prescription as against the On June 25, 1880, the Crown adopted regulations for the adjustment of lands "wrongfully occupied"
State as to these lands; and if so, to what extent was it recognized?" by private individuals in the Philippine Islands. Valenton construed these regulations together with
contemporaneous legislative and executive interpretations of the law, and concluded that plaintiffs'
Prior to 1880, the Court said, there were no laws specifically providing for the disposition of land in case fared no better under the 1880 decree and other laws which followed it, than it did under the
the Philippines. However, it was understood that in the absence of any special law to govern a specific earlier ones. Thus as a general doctrine, the Court stated:
colony, the Laws of the Indies would be followed. Indeed, in the Royal Order of July 5, 1862, it was
decreed that until regulations on the subject could be prepared, the authorities of the Philippine "While the State has always recognized the right of the occupant to a deed if he proves a possession
Islands should follow strictly the Laws of the Indies, the Ordenanza of the Intendentes of 1786, and for a sufficient length of time, yet it has always insisted that he must make that proof before the
the Royal Cedula of 1754.11 proper administrative officers, and obtain from them his deed, and until he did that the State
remained the absolute owner."16
Quoting the preamble of Law 14, Title 12, Book 4 of the Recopilacion de Leyes de las Indias, the court
interpreted it as follows: In conclusion, the Court ruled: "We hold that from 1860 to 1892 there was no law in force in these
Islands by which the plaintiffs could obtain the ownership of these lands by prescription, without any
"In the preamble of this law there is, as is seen, a distinct statement that all those lands belong to the action by the State."17 Valenton had no rights other than those which accrued to mere possession.
Crown which have not been granted by Philip, or in his name, or by the kings who preceded him. This Murciano, on the other hand, was deemed to be the owner of the land by virtue of the grant by the
statement excludes the idea that there might be lands not so granted, that did not belong to the king. provincial secretary. In effect, Valenton upheld the Spanish concept of state ownership of public land.
It excludes the idea that the king was not still the owner of all ungranted lands, because some private
person had been in the adverse occupation of them. By the mandatory part of the law all the As a fitting observation, the Court added that "[t]he policy pursued by the Spanish Government from
occupants of the public lands are required to produce before the authorities named, and within a earliest times, requiring settlers on the public lands to obtain title deeds therefor from the State, has
time to be fixed by them, their title papers. And those who had good title or showed prescription been continued by the American Government in Act No. 926."18
were to be protected in their holdings. It is apparent that it was not the intention of the law that mere
possession for a length of time should make the possessors the owners of the land possessed by them C. The Public Land Acts and the Torrens System
without any action on the part of the authorities."12
Act No. 926, the first Public Land Act, was passed in pursuance of the provisions of the the Philippine
The preamble stated that all those lands which had not been granted by Philip, or in his name, or by Bill of 1902. The law governed the disposition of lands of the public domain. It prescribed rules and
the kings who preceded him, belonged to the Crown.13 For those lands granted by the king, the regulations for the homesteading, selling, and leasing of portions of the public domain of the
decree provided for a system of assignment of such lands. It also ordered that all possessors of Philippine Islands, and prescribed the terms and conditions to enable persons to perfect their titles
agricultural land should exhibit their title deed, otherwise, the land would be restored to the to public lands in the Islands. It also provided for the "issuance of patents to certain native settlers
Crown.14 upon public lands," for the establishment of town sites and sale of lots therein, for the completion of
imperfect titles, and for the cancellation or confirmation of Spanish concessions and grants in the
The Royal Cedula of October 15, 1754 reinforced the Recopilacion when it ordered the Crown's Islands." In short, the Public Land Act operated on the assumption that title to public lands in the
principal subdelegate to issue a general order directing the publication of the Crown's instructions: Philippine Islands remained in the government;19 and that the government's title to public land
sprung from the Treaty of Paris and other subsequent treaties between Spain and the United
"x x x to the end that any and all persons who, since the year 1700, and up to the date of the States.20 The term "public land" referred to all lands of the public domain whose title still remained
promulgation and publication of said order, shall have occupied royal lands, whether or not x x x in the government and are thrown open to private appropriation and settlement,21 and excluded the
cultivated or tenanted, may x x x appear and exhibit to said subdelegates the titles and patents by patrimonial property of the government and the friar lands.22

5
be limited to citizens of the Philippines, or to corporations or associations at least sixty per centum of
Act No. 926 was superseded in 1919 by Act 2874, the second Public Land Act. This new law was passed the capital of which is owned by such citizens, subject to any existing right, grant, lease, or concession
under the Jones Law. It was more comprehensive in scope but limited the exploitation of agricultural at the time of the inauguration of the Government established under this Constitution. Natural
lands to Filipinos and Americans and citizens of other countries which gave Filipinos the same resources, with the exception of public agricultural land, shall not be alienated, and no license,
privileges.23 After the passage of the 1935 Constitution, Act 2874 was amended in 1936 by concession, or lease for the exploitation, development, or utilization of any of the natural resources
Commonwealth Act No. 141. Commonwealth Act No. 141 remains the present Public Land Law and it shall be granted for a period exceeding twenty-five years, except as to water rights for irrigation,
is essentially the same as Act 2874. The main difference between the two relates to the transitory water supply, fisheries, or industrial uses other than the development of water power, in which cases
provisions on the rights of American citizens and corporations during the Commonwealth period at beneficial use may be the measure and the limit of the grant."
par with Filipino citizens and corporations.24
The 1973 Constitution reiterated the Regalian doctrine in Section 8, Article XIV on the "National
Grants of public land were brought under the operation of the Torrens system under Act 496, or the Economy and the Patrimony of the Nation," to wit:
Land Registration Law of 1903. Enacted by the Philippine Commission, Act 496 placed all public and
private lands in the Philippines under the Torrens system. The law is said to be almost a verbatim copy "Sec. 8. All lands of the public domain, waters, minerals, coal, petroleum and other mineral oils, all
of the Massachussetts Land Registration Act of 1898,25 which, in turn, followed the principles and forces of potential energy, fisheries, wildlife, and other natural resources of the Philippines belong to
procedure of the Torrens system of registration formulated by Sir Robert Torrens who patterned it the State. With the exception of agricultural, industrial or commercial, residential, and resettlement
after the Merchant Shipping Acts in South Australia. The Torrens system requires that the government lands of the public domain, natural resources shall not be alienated, and no license, concession, or
issue an official certificate of title attesting to the fact that the person named is the owner of the lease for the exploration, development, exploitation, or utilization of any of the natural resources
property described therein, subject to such liens and encumbrances as thereon noted or the law shall be granted for a period exceeding twenty-five years, renewable for not more than twenty-five
warrants or reserves.26 The certificate of title is indefeasible and imprescriptible and all claims to the years, except as to water rights for irrigation, water supply, fisheries, or industrial uses other than the
parcel of land are quieted upon issuance of said certificate. This system highly facilitates land development of water power, in which cases beneficial use may be the measure and the limit of the
conveyance and negotiation.27 grant."

D. The Philippine Constitutions The 1987 Constitution reaffirmed the Regalian doctrine in Section 2 of Article XII on "National
Economy and Patrimony," to wit:
The Regalian doctrine was enshrined in the 1935 Constitution. One of the fixed and dominating
objectives of the 1935 Constitutional Convention was the nationalization and conservation of the "Sec. 2. All lands of the public domain, waters, minerals, coal, petroleum, and other mineral oils, all
natural resources of the country.28 There was an overwhelming sentiment in the Convention in favor forces of potential energy, fisheries, forests or timber, wildlife, flora and fauna, and other natural
of the principle of state ownership of natural resources and the adoption of the Regalian doctrine.29 resources are owned by the State. With the exception of agricultural lands, all other natural resources
State ownership of natural resources was seen as a necessary starting point to secure recognition of shall not be alienated. The exploration, development and utilization of natural resources shall be
the state's power to control their disposition, exploitation, development, or utilization.30 The under the full control and supervision of the State. The State may directly undertake such activities
delegates to the Constitutional Convention very well knew that the concept of State ownership of or it may enter into co-production, joint venture, or production-sharing agreements with Filipino
land and natural resources was introduced by the Spaniards, however, they were not certain whether citizens, or corporations or associations at least sixty per centum of whose capital is owned by such
it was continued and applied by the Americans. To remove all doubts, the Convention approved the citizens. Such agreements may be for a period not exceeding twenty-five years, renewable for not
provision in the Constitution affirming the Regalian doctrine.31 more than twenty-five years, and under such terms and conditions as may be provided by law. In
cases of water rights for irrigation, water supply, fisheries, or industrial uses other than the
Thus, the 1935 Constitution, in Section 1 of Article XIII on "Conservation and Utilization of Natural development of water power, beneficial use may be the measure and limit of the grant.
Resources," reads as follows:
x x x."
"Sec. 1. All agricultural, timber, and mineral lands of the public domain, waters, minerals, coal,
petroleum, and other mineral oils, all forces of potential energy, and other natural resources of the
Philippines belong to the State, and their disposition, exploitation, development, or utilization shall

6
Simply stated, all lands of the public domain as well as all natural resources enumerated therein, Within their ancestral domains and ancestral lands, the ICCs/IPs are given the right to self-governance
whether on public or private land, belong to the State. It is this concept of State ownership that and empowerment,34 social justice and human rights,35 the right to preserve and protect their
petitioners claim is being violated by the IPRA. culture, traditions, institutions and community intellectual rights, and the right to develop their own
sciences and technologies.36
II. THE INDIGENOUS PEOPLES RIGHTS ACT.
To carry out the policies of the Act, the law created the National Commission on Indigenous Peoples
Republic Act No. 8371 is entitled "An Act to Recognize, Protect and Promote the Rights of Indigenous (NCIP). The NCIP is an independent agency under the Office of the President and is composed of seven
Cultural Communities/ Indigenous Peoples, Creating a National Commission on Indigenous Peoples, (7) Commissioners belonging to ICCs/IPs from each of the ethnographic areas- Region I and the
Establishing Implementing Mechanisms, Appropriating Funds Therefor, and for Other Purposes." It is Cordilleras; Region II; the rest of Luzon; Island groups including Mindoro, Palawan, Romblon, Panay
simply known as "The Indigenous Peoples Rights Act of 1997" or the IPRA. and the rest of the Visayas; Northern and Western Mindanao; Southern and Eastern Mindanao; and
Central Mindanao.37 The NCIP took over the functions of the Office for Northern Cultural
The IPRA recognizes the existence of the indigenous cultural communities or indigenous peoples Communities and the Office for Southern Cultural Communities created by former President Corazon
(ICCs/IPs) as a distinct sector in Philippine society. It grants these people the ownership and Aquino which were merged under a revitalized structure.38
possession of their ancestral domains and ancestral lands, and defines the extent of these lands and
domains. The ownership given is the indigenous concept of ownership under customary law which Disputes involving ICCs/IPs are to be resolved under customary laws and practices. When still
traces its origin to native title. unresolved, the matter may be brought to the NCIP, which is granted quasi-judicial powers.39 The
NCIP's decisions may be appealed to the Court of Appeals by a petition for review.
Other rights are also granted the ICCs/IPs, and these are:
Any person who violates any of the provisions of the Act such as, but not limited to, unauthorized
- the right to develop lands and natural resources; and/or unlawful intrusion upon ancestral lands and domains shall be punished in accordance with
customary laws or imprisoned from 9 months to 12 years and/or fined from ₱100,000.00 to
- the right to stay in the territories; ₱500,000.00 and obliged to pay damages.40

- the right in case of displacement; A. Indigenous Peoples

- the right to safe and clean air and water; The IPRA is a law dealing with a specific group of people, i.e., the Indigenous Cultural Communities
(ICCs) or the Indigenous Peoples (IPs). The term "ICCs" is used in the 1987 Constitution while that of
- the right to claim parts of reservations; "IPs" is the contemporary international language in the International Labor Organization (ILO)
Convention 16941 and the United Nations (UN) Draft Declaration on the Rights of Indigenous
- the right to resolve conflict;32 Peoples.42

- the right to ancestral lands which include ICCs/IPs are defined by the IPRA as:

a. the right to transfer land/property to/among members of the same ICCs/IPs, subject to customary "Sec. 3 [h]. Indigenous Cultural Communities/ Indigenous Peoples- refer to a group of people or
laws and traditions of the community concerned; homogeneous societies identified by self-ascription and ascription by others, who have continuously
lived as organized community on communally bounded and defined territory, and who have, under
b. the right to redemption for a period not exceeding 15 years from date of transfer, if the transfer is claims of ownership since time immemorial, occupied, possessed and utilized such territories, sharing
to a non-member of the ICC/IP and is tainted by vitiated consent of the ICC/IP, or if the transfer is for common bonds of language, customs, traditions and other distinctive cultural traits, or who have,
an unconscionable consideration.33 through resistance to political, social and cultural inroads of colonization, non-indigenous religions
and cultures, became historically differentiated from the majority of Filipinos. ICCs/IPs shall likewise
include peoples who are regarded as indigenous on account of their descent from the populations

7
which inhabited the country, at the time of conquest or colonization, or at the time of inroads of non-
indigenous religions and cultures, or the establishment of present state boundaries, who retain some 7. In Region IX- the Badjao numbering about 192,000 in Tawi-Tawi, Zamboanga del Sur; the Kalibugan
or all of their own social, economic, cultural and political institutions, but who may have been of Basilan, the Samal, Subanon and Yakat.
displaced from their traditional domains or who may have resettled outside their ancestral domains."
8. Region X- Numbering 1.6 million in Region X alone, the IPs are: the Banwaon, Bukidnon, Matigsalog,
Indigenous Cultural Communities or Indigenous Peoples refer to a group of people or homogeneous Talaanding of Bukidnon; the Camiguin of Camiguin Island; the Higa-unon of Agusan del Norte, Agusan
societies who have continuously lived as an organized community on communally bounded and del Sur, Bukidnon and Misamis Occidental; the Tigwahanon of Agusan del Sur, Misamis Oriental and
defined territory. These groups of people have actually occupied, possessed and utilized their and Misamis Occidental, the Manobo of the Agusan provinces, and the Umayamnon of Agusan and
territories under claim of ownership since time immemorial. They share common bonds of language, Bukidnon.
customs, traditions and other distinctive cultural traits, or, they, by their resistance to political, social
and cultural inroads of colonization, non-indigenous religions and cultures, became historically 9. In Region XI- There are about 1,774,065 IPs in Region XI. They are tribes of the Dibabaon, Mansaka
differentiated from the Filipino majority. ICCs/IPs also include descendants of ICCs/IPs who inhabited of Davao del Norte; B'laan, Kalagan, Langilad, T'boli and Talaingod of Davao del Sur; Mamamanua of
the country at the time of conquest or colonization, who retain some or all of their own social, Surigao del Sur; Mandaya of the Surigao provinces and Davao Oriental; Manobo Blit of South
economic, cultural and political institutions but who may have been displaced from their traditional Cotabato; the Mangguangon of Davao and South Cotabato; Matigsalog of Davao del Norte and Del
territories or who may have resettled outside their ancestral domains. Sur; Tagakaolo, Tasaday and Ubo of South Cotabato; and Bagobo of Davao del sur and South
Cotabato.
1. Indigenous Peoples: Their History
10. In Region XII- Ilianen, Tiruray, Maguindanao, Maranao, Tausug, Yakan/Samal, and Iranon.43
Presently, Philippine indigenous peoples inhabit the interiors and mountains of Luzon, Mindanao,
Mindoro, Negros, Samar, Leyte, and the Palawan and Sulu group of islands. They are composed of How these indigenous peoples came to live in the Philippines goes back to as early as 25,000 to 30,000
110 tribes and are as follows: B.C.

1. In the Cordillera Autonomous Region- Kankaney, Ibaloi, Bontoc, Tinggian or Itneg, Ifugao, Kalinga, Before the time of Western contact, the Philippine archipelago was peopled largely by the Negritos,
Yapayao, Aeta or Agta or Pugot, and Bago of Ilocos Norte and Pangasinan; Ibanag of Isabela, Cagayan; Indonesians and Malays.44 The strains from these groups eventually gave rise to common cultural
Ilongot of Quirino and Nueva Vizcaya; Gaddang of Quirino, Nueva Vizcaya, Itawis of Cagayan; Ivatan features which became the dominant influence in ethnic reformulation in the archipelago. Influences
of Batanes, Aeta of Cagayan, Quirino and Isabela. from the Chinese and Indian civilizations in the third or fourth millenium B.C. augmented these ethnic
strains. Chinese economic and socio-cultural influences came by way of Chinese porcelain, silk and
2. In Region III- Aetas. traders. Indian influence found their way into the religious-cultural aspect of pre-colonial society.45

3. In Region IV- Dumagats of Aurora, Rizal; Remontado of Aurora, Rizal, Quezon; Alangan or Mangyan, The ancient Filipinos settled beside bodies of water. Hunting and food gathering became
Batangan, Buid or Buhid, Hanunuo and Iraya of Oriental and Occidental Mindoro; Tadyawan of supplementary activities as reliance on them was reduced by fishing and the cultivation of the soil.46
Occidental Mindoro; Cuyonon, Palawanon, Tagbanua and Tao't bato of Palawan. From the hinterland, coastal, and riverine communities, our ancestors evolved an essentially
homogeneous culture, a basically common way of life where nature was a primary factor. Community
4. In Region V- Aeta of Camarines Norte and Camarines Sur; Aeta-Abiyan, Isarog, and Kabihug of life throughout the archipelago was influenced by, and responded to, common ecology. The generally
Camarines Norte; Agta, and Mayon of Camarines Sur; Itom of Albay, Cimaron of Sorsogon; and the benign tropical climate and the largely uniform flora and fauna favored similarities, not differences.47
Pullon of Masbate and Camarines Sur. Life was essentially subsistence but not harsh.48

5. In Region VI- Ati of Negros Occidental, Iloilo and Antique, Capiz; the Magahat of Negros Occidental; The early Filipinos had a culture that was basically Malayan in structure and form. They had languages
the Corolano and Sulod. that traced their origin to the Austronesian parent-stock and used them not only as media of daily
communication but also as vehicles for the expression of their literary moods.49 They fashioned
6. In Region VII- Magahat of Negros Oriental and Eskaya of Bohol. concepts and beliefs about the world that they could not see, but which they sensed to be part of

8
their lives.50 They had their own religion and religious beliefs. They believed in the immortality of the Sometime in the 13th century, Islam was introduced to the archipelago in Maguindanao. The
soul and life after death. Their rituals were based on beliefs in a ranking deity whom they called Sultanate of Sulu was established and claimed jurisdiction over territorial areas represented today by
Bathalang Maykapal, and a host of other deities, in the environmental spirits and in soul spirits. The Tawi-tawi, Sulu, Palawan, Basilan and Zamboanga. Four ethnic groups were within this jurisdiction:
early Filipinos adored the sun, the moon, the animals and birds, for they seemed to consider the Sama, Tausug, Yakan and Subanon.62 The Sultanate of Maguindanao spread out from Cotabato
objects of Nature as something to be respected. They venerated almost any object that was close to toward Maranao territory, now Lanao del Norte and Lanao del Sur.63
their daily life, indicating the importance of the relationship between man and the object of nature.51
The Muslim societies evolved an Asiatic form of feudalism where land was still held in common but
The unit of government was the "barangay," a term that derived its meaning from the Malay word was private in use. This is clearly indicated in the Muslim Code of Luwaran. The Code contains a
"balangay," meaning, a boat, which transported them to these shores.52 The barangay was basically provision on the lease of cultivated lands. It, however, has no provision for the acquisition, transfer,
a family-based community and consisted of thirty to one hundred families. Each barangay was cession or sale of land.64
different and ruled by a chieftain called a "dato." It was the chieftain's duty to rule and govern his
subjects and promote their welfare and interests. A chieftain had wide powers for he exercised all the The societies encountered by Magellan and Legaspi therefore were primitive economies where most
functions of government. He was the executive, legislator and judge and was the supreme production was geared to the use of the producers and to the fulfillment of kinship obligations. They
commander in time of war.53 were not economies geared to exchange and profit.65 Moreover, the family basis of barangay
membership as well as of leadership and governance worked to splinter the population of the islands
Laws were either customary or written. Customary laws were handed down orally from generation into numerous small and separate communities.66
to generation and constituted the bulk of the laws of the barangay. They were preserved in songs and
chants and in the memory of the elder persons in the community.54 The written laws were those that When the Spaniards settled permanently in the Philippines in 1565, they found the Filipinos living in
the chieftain and his elders promulgated from time to time as the necessity arose.55 The oldest barangay settlements scattered along water routes and river banks. One of the first tasks imposed on
known written body of laws was the Maragtas Code by Datu Sumakwel at about 1250 A.D. Other old the missionaries and the encomenderos was to collect all scattered Filipinos together in a
codes are the Muslim Code of Luwaran and the Principal Code of Sulu.56 Whether customary or reduccion.67 As early as 1551, the Spanish government assumed an unvarying solicitous attitude
written, the laws dealt with various subjects, such as inheritance, divorce, usury, loans, partnership, towards the natives.68 The Spaniards regarded it a sacred "duty to conscience and humanity to civilize
crime and punishment, property rights, family relations and adoption. Whenever disputes arose, these less fortunate people living in the obscurity of ignorance" and to accord them the "moral and
these were decided peacefully through a court composed by the chieftain as "judge" and the material advantages" of community life and the "protection and vigilance afforded them by the same
barangay elders as "jury." Conflicts arising between subjects of different barangays were resolved by laws."69
arbitration in which a board composed of elders from neutral barangays acted as arbiters.57
The Spanish missionaries were ordered to establish pueblos where the church and convent would be
Baranganic society had a distinguishing feature: the absence of private property in land. The chiefs constructed. All the new Christian converts were required to construct their houses around the
merely administered the lands in the name of the barangay. The social order was an extension of the church and the unbaptized were invited to do the same.70 With the reduccion, the Spaniards
family with chiefs embodying the higher unity of the community. Each individual, therefore, attempted to "tame" the reluctant Filipinos through Christian indoctrination using the convento/casa
participated in the community ownership of the soil and the instruments of production as a member real/plaza complex as focal point. The reduccion, to the Spaniards, was a "civilizing" device to make
of the barangay.58 This ancient communalism was practiced in accordance with the concept of the Filipinos law-abiding citizens of the Spanish Crown, and in the long run, to make them ultimately
mutual sharing of resources so that no individual, regardless of status, was without sustenance. adopt Hispanic culture and civilization.71
Ownership of land was non-existent or unimportant and the right of usufruct was what regulated the
development of lands.59 Marine resources and fishing grounds were likewise free to all. Coastal All lands lost by the old barangays in the process of pueblo organization as well as all lands not
communities depended for their economic welfare on the kind of fishing sharing concept similar to assigned to them and the pueblos, were now declared to be crown lands or realengas, belonging to
those in land communities.60 Recognized leaders, such as the chieftains and elders, by virtue of their the Spanish king. It was from the realengas that land grants were made to non-Filipinos.72
positions of importance, enjoyed some economic privileges and benefits. But their rights, related to
either land and sea, were subject to their responsibility to protect the communities from danger and The abrogation of the Filipinos' ancestral rights in land and the introduction of the concept of public
to provide them with the leadership and means of survival.61 domain were the most immediate fundamental results of Spanish colonial theory and law.73 The

9
concept that the Spanish king was the owner of everything of value in the Indies or colonies was
imposed on the natives, and the natives were stripped of their ancestral rights to land.74 The Americans classified the Filipinos into two: the Christian Filipinos and the non-Christian Filipinos.
The term "non-Christian" referred not to religious belief, but to a geographical area, and more
Increasing their foothold in the Philippines, the Spanish colonialists, civil and religious, classified the directly, "to natives of the Philippine Islands of a low grade of civilization, usually living in tribal
Filipinos according to their religious practices and beliefs, and divided them into three types . First relationship apart from settled communities."82
were the Indios, the Christianized Filipinos, who generally came from the lowland populations.
Second, were the Moros or the Muslim communities, and third, were the infieles or the indigenous Like the Spaniards, the Americans pursued a policy of assimilation. In 1903, they passed Act No. 253
communities.75 creating the Bureau of Non-Christian Tribes (BNCT). Under the Department of the Interior, the BNCT's
primary task was to conduct ethnographic research among unhispanized Filipinos, including those in
The Indio was a product of the advent of Spanish culture. This class was favored by the Spaniards and Muslim Mindanao, with a "special view to determining the most practicable means for bringing about
was allowed certain status although below the Spaniards. The Moros and infieles were regarded as their advancement in civilization and prosperity." The BNCT was modeled after the bureau dealing
the lowest classes.76 with American Indians. The agency took a keen anthropological interest in Philippine cultural
minorities and produced a wealth of valuable materials about them.83
The Moros and infieles resisted Spanish rule and Christianity. The Moros were driven from Manila
and the Visayas to Mindanao; while the infieles, to the hinterlands. The Spaniards did not pursue The 1935 Constitution did not carry any policy on the non-Christian Filipinos. The raging issue then
them into the deep interior. The upland societies were naturally outside the immediate concern of was the conservation of the national patrimony for the Filipinos.
Spanish interest, and the cliffs and forests of the hinterlands were difficult and inaccessible, allowing
the infieles, in effect, relative security.77 Thus, the infieles, which were peripheral to colonial In 1957, the Philippine Congress passed R.A. No. 1888, an "Act to effectuate in a more rapid and
administration, were not only able to preserve their own culture but also thwarted the complete manner the economic, social, moral and political advancement of the non-Christian
Christianization process, separating themselves from the newly evolved Christian community.78 Their Filipinos or national cultural minorities and to render real, complete, and permanent the integration
own political, economic and social systems were kept constantly alive and vibrant. of all said national cultural minorities into the body politic, creating the Commission on National
Integration charged with said functions." The law called for a policy of integration of indigenous
The pro-Christian or pro-Indio attitude of colonialism brought about a generally mutual feeling of peoples into the Philippine mainstream and for this purpose created the Commission on National
suspicion, fear, and hostility between the Christians on the one hand and the non-Christians on the Integration (CNI).84 The CNI was given, more or less, the same task as the BNCT during the American
other. Colonialism tended to divide and rule an otherwise culturally and historically related populace regime. The post-independence policy of integration was like the colonial policy of assimilation
through a colonial system that exploited both the virtues and vices of the Filipinos.79 understood in the context of a guardian-ward relationship.85

President McKinley, in his instructions to the Philippine Commission of April 7, 1900, addressed the The policy of assimilation and integration did not yield the desired result. Like the Spaniards and
existence of the infieles: Americans, government attempts at integration met with fierce resistance. Since World War II, a tidal
wave of Christian settlers from the lowlands of Luzon and the Visayas swamped the highlands and
"In dealing with the uncivilized tribes of the Islands, the Commission should adopt the same course wide open spaces in Mindanao.86 Knowledge by the settlers of the Public Land Acts and the Torrens
followed by Congress in permitting the tribes of our North American Indians to maintain their tribal system resulted in the titling of several ancestral lands in the settlers' names. With government
organization and government, and under which many of those tribes are now living in peace and initiative and participation, this titling displaced several indigenous peoples from their lands. Worse,
contentment, surrounded by civilization to which they are unable or unwilling to conform. Such tribal these peoples were also displaced by projects undertaken by the national government in the name
government should, however, be subjected to wise and firm regulation; and, without undue or petty of national development.87
interference, constant and active effort should be exercised to prevent barbarous practices and
introduce civilized customs."80 It was in the 1973 Constitution that the State adopted the following provision:

Placed in an alternative of either letting the natives alone or guiding them in the path of civilization, "The State shall consider the customs, traditions, beliefs, and interests of national cultural
the American government chose "to adopt the latter measure as one more in accord with humanity communities in the formulation and implementation of State policies."88
and with the national conscience."81

10
For the first time in Philippine history, the "non-Christian tribes" or the "cultural minorities" were Indigenous peoples share distinctive traits that set them apart from the Filipino mainstream. They are
addressed by the highest law of the Republic, and they were referred to as "cultural communities." non-Christians. They live in less accessible, marginal, mostly upland areas. They have a system of self-
More importantly this time, their "uncivilized" culture was given some recognition and their government not dependent upon the laws of the central administration of the Republic of the
"customs, traditions, beliefs and interests" were to be considered by the State in the formulation and Philippines. They follow ways of life and customs that are perceived as different from those of the
implementation of State policies. President Marcos abolished the CNI and transferred its functions to rest of the population.97 The kind of response the indigenous peoples chose to deal with colonial
the Presidential Adviser on National Minorities (PANAMIN). The PANAMIN was tasked to integrate threat worked well to their advantage by making it difficult for Western concepts and religion to erode
the ethnic groups that sought full integration into the larger community, and at the same time their customs and traditions. The "infieles societies" which had become peripheral to colonial
"protect the rights of those who wish to preserve their original lifeways beside the larger administration, represented, from a cultural perspective, a much older base of archipelagic culture.
community."89 In short, while still adopting the integration policy, the decree recognized the right of The political systems were still structured on the patriarchal and kinship oriented arrangement of
tribal Filipinos to preserve their way of life.90 power and authority. The economic activities were governed by the concepts of an ancient
communalism and mutual help. The social structure which emphasized division of labor and
In 1974, President Marcos promulgated P.D. No. 410, otherwise known as the Ancestral Lands Decree. distinction of functions, not status, was maintained. The cultural styles and forms of life portraying
The decree provided for the issuance of land occupancy certificates to members of the national the varieties of social courtesies and ecological adjustments were kept constantly vibrant.98
cultural communities who were given up to 1984 to register their claims.91 In 1979, the Commission
on the Settlement of Land Problems was created under E.O. No. 561 which provided a mechanism for Land is the central element of the indigenous peoples' existence. There is no traditional concept of
the expeditious resolution of land problems involving small settlers, landowners, and tribal permanent, individual, land ownership. Among the Igorots, ownership of land more accurately applies
Filipinos.92 to the tribal right to use the land or to territorial control. The people are the secondary owners or
stewards of the land and that if a member of the tribe ceases to work, he loses his claim of ownership,
Despite the promulgation of these laws, from 1974 to the early 1980's, some 100,000 Kalingas and and the land reverts to the beings of the spirit world who are its true and primary owners. Under the
Bontoks of the Cordillera region were displaced by the Chico River dam project of the National Power concept of "trusteeship," the right to possess the land does not only belong to the present generation
Corporation (NPC). The Manobos of Bukidnon saw their land bulldozed by the Bukidnon Sugar but the future ones as well.99
Industries Company (BUSCO). In Agusan del Sur, the National Development Company was authorized
by law in 1979 to take approximately 40,550 hectares of land that later became the NDC-Guthrie Customary law on land rests on the traditional belief that no one owns the land except the gods and
plantation in Agusan del Sur. Most of the land was possessed by the Agusan natives.93 Timber spirits, and that those who work the land are its mere stewards.100 Customary law has a strong
concessions, water projects, plantations, mining, and cattle ranching and other projects of the preference for communal ownership, which could either be ownership by a group of individuals or
national government led not only to the eviction of the indigenous peoples from their land but also families who are related by blood or by marriage,101 or ownership by residents of the same locality
to the reduction and destruction of their natural environment.94 who may not be related by blood or marriage. The system of communal ownership under customary
laws draws its meaning from the subsistence and highly collectivized mode of economic production.
The Aquino government signified a total shift from the policy of integration to one of preservation. The Kalingas, for instance, who are engaged in team occupation like hunting, foraging for forest
Invoking her powers under the Freedom Constitution, President Aquino created the Office of Muslim products, and swidden farming found it natural that forest areas, swidden farms, orchards, pasture
Affairs, Office for Northern Cultural Communities and the Office for Southern Cultural Communities and burial grounds should be communally-owned.102 For the Kalingas, everybody has a common
all under the Office of the President.95 right to a common economic base. Thus, as a rule, rights and obligations to the land are shared in
common.
The 1987 Constitution carries at least six (6) provisions which insure the right of tribal Filipinos to
preserve their way of life.96 This Constitution goes further than the 1973 Constitution by expressly Although highly bent on communal ownership, customary law on land also sanctions individual
guaranteeing the rights of tribal Filipinos to their ancestral domains and ancestral lands. By ownership. The residential lots and terrace rice farms are governed by a limited system of individual
recognizing their right to their ancestral lands and domains, the State has effectively upheld their right ownership. It is limited because while the individual owner has the right to use and dispose of the
to live in a culture distinctly their own. property, he does not possess all the rights of an exclusive and full owner as defined under our Civil
Code.103 Under Kalinga customary law, the alienation of individually-owned land is strongly
2. Their Concept of Land discouraged except in marriage and succession and except to meet sudden financial needs due to
sickness, death in the family, or loss of crops.104 Moreover, and to be alienated should first be offered

11
to a clan-member before any village-member can purchase it, and in no case may land be sold to a "The IPs are the offsprings and heirs of the peoples who have first inhabited and cared for the land
non-member of the ili.105 long before any central government was established. Their ancestors had territories over which they
ruled themselves and related with other tribes. These territories- the land- include people, their
Land titles do not exist in the indigenous peoples' economic and social system. The concept of dwelling, the mountains, the water, the air, plants, forest and the animals. This is their environment
individual land ownership under the civil law is alien to them. Inherently colonial in origin, our national in its totality. Their existence as indigenous peoples is manifested in their own lives through political,
land laws and governmental policies frown upon indigenous claims to ancestral lands. Communal economic, socio-cultural and spiritual practices. The IPs culture is the living and irrefutable proof to
ownership is looked upon as inferior, if not inexistent.106 this.

III. THE IPRA IS A NOVEL PIECE OF LEGISLATION. Their survival depends on securing or acquiring land rights; asserting their rights to it; and depending
on it. Otherwise, IPs shall cease to exist as distinct peoples."110
A. The Legislative History of the IPRA
To recognize the rights of the indigenous peoples effectively, Senator Flavier proposed a bill based on
It was to address the centuries-old neglect of the Philippine indigenous peoples that the Tenth two postulates: (1) the concept of native title; and (2) the principle of parens patriae.
Congress of the Philippines, by their joint efforts, passed and approved R.A. No. 8371, the Indigenous
Peoples Rights Act (IPRA) of 1997. The law was a consolidation of two Bills- Senate Bill No. 1728 and According to Senator Flavier, "[w]hile our legal tradition subscribes to the Regalian Doctrine
House Bill No. 9125. reinstated in Section 2, Article XII of the 1987 Constitution," our "decisional laws" and jurisprudence
passed by the State have "made exception to the doctrine." This exception was first laid down in the
Principally sponsored by Senator Juan M. Flavier,107 Senate Bill No. 1728 was a consolidation of four case of Cariño v. Insular Government where:
proposed measures referred to the Committees on Cultural Communities, Environment and Natural
Resources, Ways and Means, as well as Finance. It adopted almost en toto the comprehensive version "x x x the court has recognized long occupancy of land by an indigenous member of the cultural
of Senate Bill Nos. 1476 and 1486 which was a result of six regional consultations and one national communities as one of private ownership, which, in legal concept, is termed "native title." This ruling
consultation with indigenous peoples nationwide.108 At the Second Regular Session of the Tenth has not been overturned. In fact, it was affirmed in subsequent cases."111
Congress, Senator Flavier, in his sponsorship speech, gave a background on the situation of indigenous
peoples in the Philippines, to wit: Following Cariño, the State passed Act No. 926, Act No. 2874, C.A. No. 141, P.D. 705, P.D. 410, P.D.
1529, R.A. 6734 (the Organic Act for the Autonomous Region of Muslim Mindanao). These laws,
"The Indigenous Cultural Communities, including the Bangsa Moro, have long suffered from the explicitly or implicitly, and liberally or restrictively, recognized "native title" or "private right" and the
dominance and neglect of government controlled by the majority. Massive migration of their existence of ancestral lands and domains. Despite the passage of these laws, however, Senator Flavier
Christian brothers to their homeland shrunk their territory and many of the tribal Filipinos were continued:
pushed to the hinterlands. Resisting the intrusion, dispossessed of their ancestral land and with the
massive exploitation of their natural resources by the elite among the migrant population, they "x x x the executive department of government since the American occupation has not implemented
became marginalized. And the government has been an indispensable party to this insidious the policy. In fact, it was more honored in its breach than in its observance, its wanton disregard
conspiracy against the Indigenous Cultural Communities (ICCs). It organized and supported the shown during the period unto the Commonwealth and the early years of the Philippine Republic when
resettlement of people to their ancestral land, which was massive during the Commonwealth and government organized and supported massive resettlement of the people to the land of the ICCs."
early years of the Philippine Republic. Pursuant to the Regalian Doctrine first introduced to our system
by Spain through the Royal Decree of 13 February 1894 or the Maura Law, the government passed Senate Bill No. 1728 seeks to genuinely recognize the IPs right to own and possess their ancestral
laws to legitimize the wholesale landgrabbing and provide for easy titling or grant of lands to migrant land. The bill was prepared also under the principle of parens patriae inherent in the supreme power
homesteaders within the traditional areas of the ICCs."109 of the State and deeply embedded in Philippine legal tradition. This principle mandates that persons
suffering from serious disadvantage or handicap, which places them in a position of actual inequality
Senator Flavier further declared: in their relation or transaction with others, are entitled to the protection of the State.

12
Senate Bill No. 1728 was passed on Third Reading by twenty-one (21) Senators voting in favor and interrupted by war, force majeure or displacement by force, deceit, stealth or as a consequence of
none against, with no abstention.112 government projects or any other voluntary dealings entered into by government and private
individuals/corporations, and which are necessary to ensure their economic, social and cultural
House Bill No. 9125 was sponsored by Rep. Zapata, Chairman of the Committee on Cultural welfare. It shall include ancestral lands, forests, pasture, residential, agricultural, and other lands
Communities. It was originally authored and subsequently presented and defended on the floor by individually owned whether alienable and disposable or otherwise, hunting grounds, burial grounds,
Rep. Gregorio Andolana of North Cotabato.113 worship areas, bodies of water, mineral and other natural resources, and lands which may no longer
be exclusively occupied by ICCs/IPs but from which they traditionally had access to for their
Rep. Andolana's sponsorhip speech reads as follows: subsistence and traditional activities, particularly the home ranges of ICCs/IPs who are still nomadic
and/or shifting cultivators;
"This Representation, as early as in the 8th Congress, filed a bill of similar implications that would
promote, recognize the rights of indigenous cultural communities within the framework of national b) Ancestral Lands.- Subject to Section 56 hereof, refers to land occupied, possessed and utilized by
unity and development. individuals, families and clans who are members of the ICCs/IPs since time immemorial, by
themselves or through their predecessors-in-interest, under claims of individual or traditional group
Apart from this, Mr. Speaker, is our obligation, the government's obligation to assure and ascertain ownership, continuously, to the present except when interrupted by war, force majeure or
that these rights shall be well-preserved and the cultural traditions as well as the indigenous laws that displacement by force, deceit, stealth, or as a consequence of government projects and other
remained long before this Republic was established shall be preserved and promoted. There is a need, voluntary dealings entered into by government and private individuals/corporations, including, but
Mr. Speaker, to look into these matters seriously and early approval of the substitute bill shall bring not limited to, residential lots, rice terraces or paddies, private forests, swidden farms and tree lots."
into reality the aspirations, the hope and the dreams of more than 12 million Filipinos that they be
considered in the mainstream of the Philippine society as we fashion for the year 2000." 114 Ancestral domains are all areas belonging to ICCs/IPs held under a claim of ownership, occupied or
possessed by ICCs/IPs by themselves or through their ancestors, communally or individually since
Rep. Andolana stressed that H.B. No. 9125 is based on the policy of preservation as mandated in the time immemorial, continuously until the present, except when interrupted by war, force majeure or
Constitution. He also emphasized that the rights of IPs to their land was enunciated in Cariño v. Insular displacement by force, deceit, stealth or as a consequence of government projects or any other
Government which recognized the fact that they had vested rights prior to the establishment of the voluntary dealings with government and/or private individuals or corporations. Ancestral domains
Spanish and American regimes.115 comprise lands, inland waters, coastal areas, and natural resources therein and includes ancestral
lands, forests, pasture, residential, agricultural, and other lands individually owned whether alienable
After exhaustive interpellation, House Bill No. 9125, and its corresponding amendments, was or not, hunting grounds, burial grounds, worship areas, bodies of water, mineral and other natural
approved on Second Reading with no objections. resources. They also include lands which may no longer be exclusively occupied by ICCs/IPs but from
which they traditionally had access to for their subsistence and traditional activities, particularly the
IV. THE PROVISIONS OF THE IPRA DO NOT CONTRAVENE THE CONSTITUTION. home ranges of ICCs/IPs who are still nomadic and/or shifting cultivators.116

A. Ancestral Domains and Ancestral Lands are the Private Property of Indigenous Peoples and Do Not Ancestral lands are lands held by the ICCs/IPs under the same conditions as ancestral domains except
Constitute Part of the Land of the Public Domain. that these are limited to lands and that these lands are not merely occupied and possessed but are
also utilized by the ICCs/IPs under claims of individual or traditional group ownership. These lands
The IPRA grants to ICCs/IPs a distinct kind of ownership over ancestral domains and ancestral lands. include but are not limited to residential lots, rice terraces or paddies, private forests, swidden farms
Ancestral lands are not the same as ancestral domains. These are defined in Section 3 [a] and [b] of and tree lots.117
the Indigenous Peoples Right Act, viz:
The procedures for claiming ancestral domains and lands are similar to the procedures embodied in
"Sec. 3 a) Ancestral Domains. - Subject to Section 56 hereof, refer to all areas generally belonging to Department Administrative Order (DAO) No. 2, series of 1993, signed by then Secretary of the
ICCs/IPs comprising lands, inland waters, coastal areas, and natural resources therein, held under a Department of Environment and Natural Resources (DENR) Angel Alcala.118 DAO No. 2 allowed the
claim of ownership, occupied or possessed by ICCs/IPs by themselves or through their ancestors, delineation of ancestral domains by special task forces and ensured the issuance of Certificates of
communally or individually since time immemorial, continuously to the present except when Ancestral Land Claims (CALC's) and Certificates of Ancestral Domain Claims (CADC's) to IPs.

13
shall be embodied in a Certificate of Ancestral Domain Title (CADT), which shall recognize the title of
The identification and delineation of these ancestral domains and lands is a power conferred by the the concerned ICCs/IPs over the territories identified and delineated.128
IPRA on the National Commission on Indigenous Peoples (NCIP).119 The guiding principle in
identification and delineation is self-delineation.120 This means that the ICCs/IPs have a decisive role Like a torrens title, a CADT is evidence of private ownership of land by native title. Native title,
in determining the boundaries of their domains and in all the activities pertinent thereto.121 however, is a right of private ownership peculiarly granted to ICCs/IPs over their ancestral lands and
domains. The IPRA categorically declares ancestral lands and domains held by native title as never to
The procedure for the delineation and recognition of ancestral domains is set forth in Sections 51 and have been public land. Domains and lands held under native title are, therefore, indisputably
52 of the IPRA. The identification, delineation and certification of ancestral lands is in Section 53 of presumed to have never been public lands and are private.
said law.
(a) Cariño v. Insular Government129
Upon due application and compliance with the procedure provided under the law and upon finding
by the NCIP that the application is meritorious, the NCIP shall issue a Certificate of Ancestral Domain The concept of native title in the IPRA was taken from the 1909 case of Cariño v. Insular
Title (CADT) in the name of the community concerned.122 The allocation of lands within the ancestral Government.130 Cariño firmly established a concept of private land title that existed irrespective of
domain to any individual or indigenous corporate (family or clan) claimants is left to the ICCs/IPs any royal grant from the State.
concerned to decide in accordance with customs and traditions.123 With respect to ancestral lands
outside the ancestral domain, the NCIP issues a Certificate of Ancestral Land Title (CALT).124 In 1903, Don Mateo Cariño, an Ibaloi, sought to register with the land registration court 146 hectares
of land in Baguio Municipality, Benguet Province. He claimed that this land had been possessed and
CADT's and CALT's issued under the IPRA shall be registered by the NCIP before the Register of Deeds occupied by his ancestors since time immemorial; that his grandfather built fences around the
in the place where the property is situated.125 property for the holding of cattle and that his father cultivated some parts of the land. Cariño
inherited the land in accordance with Igorot custom. He tried to have the land adjusted under the
(1) Right to Ancestral Domains and Ancestral Lands: How Acquired Spanish land laws, but no document issued from the Spanish Crown.131 In 1901, Cariño obtained a
possessory title to the land under the Spanish Mortgage Law.132 The North American colonial
The rights of the ICCs/IPs to their ancestral domains and ancestral lands may be acquired in two government, however, ignored his possessory title and built a public road on the land prompting him
modes: (1) by native title over both ancestral lands and domains; or (2) by torrens title under the to seek a Torrens title to his property in the land registration court. While his petition was pending, a
Public Land Act and the Land Registration Act with respect to ancestral lands only. U.S. military reservation133 was proclaimed over his land and, shortly thereafter, a military
detachment was detailed on the property with orders to keep cattle and trespassers, including Cariño,
(2) The Concept of Native Title off the land.134

Native title is defined as: In 1904, the land registration court granted Cariño's application for absolute ownership to the land.
Both the Government of the Philippine Islands and the U.S. Government appealed to the C.F.I. of
"Sec. 3 [l]. Native Title- refers to pre-conquest rights to lands and domains which, as far back as Benguet which reversed the land registration court and dismissed Cariño's application. The Philippine
memory reaches, have been held under a claim of private ownership by ICCs/IPs, have never been Supreme Court135 affirmed the C.F.I. by applying the Valenton ruling. Cariño took the case to the U.S.
public lands and are thus indisputably presumed to have been held that way since before the Spanish Supreme Court.136 On one hand, the Philippine government invoked the Regalian doctrine and
Conquest."126 contended that Cariño failed to comply with the provisions of the Royal Decree of June 25, 1880,
which required registration of land claims within a limited period of time. Cariño, on the other,
Native title refers to ICCs/IPs' preconquest rights to lands and domains held under a claim of private asserted that he was the absolute owner of the land jure gentium, and that the land never formed
ownership as far back as memory reaches. These lands are deemed never to have been public lands part of the public domain.
and are indisputably presumed to have been held that way since before the Spanish Conquest. The
rights of ICCs/IPs to their ancestral domains (which also include ancestral lands) by virtue of native In a unanimous decision written by Justice Oliver Wendell Holmes, the U.S. Supreme Court held:
title shall be recognized and respected.127 Formal recognition, when solicited by ICCs/IPs concerned,

14
"It is true that Spain, in its earlier decrees, embodied the universal feudal theory that all lands were "Every presumption is and ought to be against the government in a case like the present. It might,
held from the Crown, and perhaps the general attitude of conquering nations toward people not perhaps, be proper and sufficient to say that when, as far back as testimony or memory goes, the land
recognized as entitled to the treatment accorded to those in the same zone of civilization with has been held by individuals under a claim of private ownership, it will be presumed to have been
themselves. It is true, also, that in legal theory, sovereignty is absolute, and that, as against foreign held in the same way from before the Spanish conquest, and never to have been public land. Certainly
nations, the United States may assert, as Spain asserted, absolute power. But it does not follow that, in a case like this, if there is doubt or ambiguity in the Spanish law, we ought to give the applicant the
as against the inhabitants of the Philippines, the United States asserts that Spain had such power. benefit of the doubt."140
When theory is left on one side, sovereignty is a question of strength, and may vary in degree. How
far a new sovereign shall insist upon the theoretical relation of the subjects to the head in the past, The court thus laid down the presumption of a certain title held (1) as far back as testimony or
and how far it shall recognize actual facts, are matters for it to decide."137 memory went, and (2) under a claim of private ownership. Land held by this title is presumed to
"never have been public land."
The U.S. Supreme Court noted that it need not accept Spanish doctrines. The choice was with the new
colonizer. Ultimately, the matter had to be decided under U.S. law. Against this presumption, the U.S. Supreme Court analyzed the Spanish decrees upheld in the 1904
decision of Valenton v. Murciano. The U.S. Supreme Court found no proof that the Spanish decrees
The Cariño decision largely rested on the North American constitutionalist's concept of "due process" did not honor native title. On the contrary, the decrees discussed in Valenton appeared to recognize
as well as the pronounced policy "to do justice to the natives."138 It was based on the strong mandate that the natives owned some land, irrespective of any royal grant. The Regalian doctrine declared in
extended to the Islands via the Philippine Bill of 1902 that "No law shall be enacted in said islands the preamble of the Recopilacion was all "theory and discourse" and it was observed that titles were
which shall deprive any person of life, liberty, or property without due process of law, or deny to any admitted to exist beyond the powers of the Crown, viz:
person therein the equal protection of the laws." The court declared:
"If the applicant's case is to be tried by the law of Spain, we do not discover such clear proof that it
"The acquisition of the Philippines was not like the settlement of the white race in the United States. was bad by that law as to satisfy us that he does not own the land. To begin with, the older decrees
Whatever consideration may have been shown to the North American Indians, the dominant purpose and laws cited by the counsel for the plaintiff in error seem to indicate pretty clearly that the natives
of the whites in America was to occupy land. It is obvious that, however stated, the reason for our were recognized as owning some lands, irrespective of any royal grant. In other words, Spain did not
taking over the Philippines was different. No one, we suppose, would deny that, so far as consistent assume to convert all the native inhabitants of the Philippines into trespassers or even into tenants
with paramount necessities, our first object in the internal administration of the islands is to do justice at will. For instance, Book 4, title 12, Law 14 of the the Recopilacion de Leyes de las Indias, cited for a
to the natives, not to exploit their country for private gain. By the Organic Act of July 1, 1902, chapter contrary conclusion in Valenton v. Murciano, 3 Philippine 537, while it commands viceroys and others,
1369, section 12 (32 Statutes at Large, 691), all the property and rights acquired there by the United when it seems proper, to call for the exhibition of grants, directs them to confirm those who hold by
States are to be administered 'for the benefit of the inhabitants thereof.' It is reasonable to suppose good grants or justa prescripcion. It is true that it begins by the characteristic assertion of feudal
that the attitude thus assumed by the United States with regard to what was unquestionably its own overlordship and the origin of all titles in the King or his predecessors. That was theory and discourse.
is also its attitude in deciding what it will claim for its own. The same statute made a bill of rights, The fact was that titles were admitted to exist that owed nothing to the powers of Spain beyond this
embodying the safeguards of the Constitution, and, like the Constitution, extends those safeguards recognition in their books." (Emphasis supplied).141
to all. It provides that 'no law shall be enacted in said islands which shall deprive any person of life,
liberty, or property without due process of law, or deny to any person therein the equal protection The court further stated that the Spanish "adjustment" proceedings never held sway over
of the laws.' In the light of the declaration that we have quoted from section 12, it is hard to believe unconquered territories. The wording of the Spanish laws were not framed in a manner as to convey
that the United States was ready to declare in the next breath that "any person" did not embrace the to the natives that failure to register what to them has always been their own would mean loss of
inhabitants of Benguet, or that it meant by "property" only that which had become such by such land. The registration requirement was "not to confer title, but simply to establish it;" it was "not
ceremonies of which presumably a large part of the inhabitants never had heard, and that it proposed calculated to convey to the mind of an Igorot chief the notion that ancient family possessions were in
to treat as public land what they, by native custom and by long association,- of the profoundest danger, if he had read every word of it."
factors in human thought,- regarded as their own."139
By recognizing this kind of title, the court clearly repudiated the doctrine of Valenton. It was frank
The Court went further: enough, however, to admit the possibility that the applicant might have been deprived of his land
under Spanish law because of the inherent ambiguity of the decrees and concomitantly, the various

15
interpretations which may be given them. But precisely because of the ambiguity and of the strong In a footnote in the same article, Professor Lynch stated that the concept of "native title" as defined
"due process mandate" of the Constitution, the court validated this kind of title.142 This title was by Justice Holmes in Cariño "is conceptually similar to "aboriginal title" of the American Indians.148
sufficient, even without government administrative action, and entitled the holder to a Torrens This is not surprising, according to Prof. Lynch, considering that during the American regime,
certificate. Justice Holmes explained: government policy towards ICCs/IPs was consistently made in reference to native Americans.149 This
was clearly demonstrated in the case of Rubi v. Provincial Board of Mindoro.150
"It will be perceived that the rights of the applicant under the Spanish law present a problem not
without difficulties for courts of a legal tradition. We have deemed it proper on that account to notice In Rubi, the Provincial Board of Mindoro adopted a Resolution authorizing the provincial governor to
the possible effect of the change of sovereignty and the act of Congress establishing the fundamental remove the Mangyans from their domains and place them in a permanent reservation in Sitio Tigbao,
principles now to be observed. Upon a consideration of the whole case we are of the opinion that law Lake Naujan. Any Mangyan who refused to comply was to be imprisoned. Rubi and some Mangyans,
and justice require that the applicant should be granted what he seeks, and should not be deprived including one who was imprisoned for trying to escape from the reservation, filed for habeas corpus
of what, by the practice and belief of those among whom he lived, was his property, through a refined claiming deprivation of liberty under the Board Resolution. This Court denied the petition on the
interpretation of an almost forgotten law of Spain."143 ground of police power. It upheld government policy promoting the idea that a permanent settlement
was the only successful method for educating the Mangyans, introducing civilized customs, improving
Thus, the court ruled in favor of Cariño and ordered the registration of the 148 hectares in Baguio their health and morals, and protecting the public forests in which they roamed.151 Speaking through
Municipality in his name.144 Justice Malcolm, the court said:

Examining Cariño closer, the U.S. Supreme Court did not categorically refer to the title it upheld as "Reference was made in the President's instructions to the Commission to the policy adopted by the
"native title." It simply said: United States for the Indian Tribes. The methods followed by the Government of the Philippine Islands
in its dealings with the so-called non-Christian people is said, on argument, to be practically identical
"The Province of Benguet was inhabited by a tribe that the Solicitor-General, in his argument, with that followed by the United States Government in its dealings with the Indian tribes. Valuable
characterized as a savage tribe that never was brought under the civil or military government of the lessons, it is insisted, can be derived by an investigation of the American-Indian policy.
Spanish Crown. It seems probable, if not certain, that the Spanish officials would not have granted to
anyone in that province the registration to which formerly the plaintiff was entitled by the Spanish From the beginning of the United States, and even before, the Indians have been treated as "in a state
Laws, and which would have made his title beyond question good. Whatever may have been the of pupilage." The recognized relation between the Government of the United States and the Indians
technical position of Spain it does not follow that, in the view of the United States, he had lost all may be described as that of guardian and ward. It is for the Congress to determine when and how the
rights and was a mere trespasser when the present government seized his land. The argument to that guardianship shall be terminated. The Indians are always subject to the plenary authority of the
effect seems to amount to a denial of native titles through an important part of the Island of Luzon, United States.152
at least, for the want of ceremonies which the Spaniards would not have permitted and had not the
power to enforce."145 x x x.

This is the only instance when Justice Holmes used the term "native title" in the entire length of the As to the second point, the facts in the Standing Bear case and the Rubi case are not exactly identical.
Cariño decision. It is observed that the widespread use of the term "native title" may be traced to But even admitting similarity of facts, yet it is known to all that Indian reservations do exist in the
Professor Owen James Lynch, Jr., a Visiting Professor at the University of the Philippines College of United States, that Indians have been taken from different parts of the country and placed on these
Law from the Yale University Law School. In 1982, Prof. Lynch published an article in the Philippine reservations, without any previous consultation as to their own wishes, and that, when once so
Law Journal entitled Native Title, Private Right and Tribal Land Law.146 This article was made after located, they have been made to remain on the reservation for their own good and for the general
Professor Lynch visited over thirty tribal communities throughout the country and studied the origin good of the country. If any lesson can be drawn from the Indian policy of the United States, it is that
and development of Philippine land laws.147 He discussed Cariño extensively and used the term the determination of this policy is for the legislative and executive branches of the government and
"native title" to refer to Cariño's title as discussed and upheld by the U.S. Supreme Court in said case. that when once so decided upon, the courts should not interfere to upset a carefully planned
governmental system. Perhaps, just as many forceful reasons exist for the segregation of the
(b) Indian Title Manguianes in Mindoro as existed for the segregation of the different Indian tribes in the United
States."153

16
whomsoever they pleased, was denied by the fundamental principle that discovery gave exclusive
Rubi applied the concept of Indian land grants or reservations in the Philippines. An Indian reservation title to those who made it.
is a part of the public domain set apart by proper authority for the use and occupation of a tribe or
tribes of Indians.154 It may be set apart by an act of Congress, by treaty, or by executive order, but it While the different nations of Europe respected the right of the natives as occupants, they asserted
cannot be established by custom and prescription.155 the ultimate dominion to be in themselves; and claimed and exercised, as a consequence of this
ultimate dominion, a power to grant the soil, while yet in possession of the natives. These grants have
Indian title to land, however, is not limited to land grants or reservations. It also covers the "aboriginal been understood by all to convey a title to the grantees, subject only to the Indian right of
right of possession or occupancy."156 The aboriginal right of possession depends on the actual occupancy."161
occupancy of the lands in question by the tribe or nation as their ancestral home, in the sense that
such lands constitute definable territory occupied exclusively by the particular tribe or nation.157 It Thus, the discoverer of new territory was deemed to have obtained the exclusive right to acquire
is a right which exists apart from any treaty, statute, or other governmental action, although in Indian land and extinguish Indian titles. Only to the discoverer- whether to England, France, Spain or
numerous instances treaties have been negotiated with Indian tribes, recognizing their aboriginal Holland- did this right belong and not to any other nation or private person. The mere acquisition of
possession and delimiting their occupancy rights or settling and adjusting their boundaries.158 the right nonetheless did not extinguish Indian claims to land. Rather, until the discoverer, by
purchase or conquest, exercised its right, the concerned Indians were recognized as the "rightful
American jurisprudence recognizes the Indians' or native Americans' rights to land they have held and occupants of the soil, with a legal as well as just claim to retain possession of it." Grants made by the
occupied before the "discovery" of the Americas by the Europeans. The earliest definitive statement discoverer to her subjects of lands occupied by the Indians were held to convey a title to the grantees,
by the U.S. Supreme Court on the nature of aboriginal title was made in 1823 in Johnson & Graham's subject only to the Indian right of occupancy. Once the discoverer purchased the land from the Indians
Lessee v. M'Intosh.159 or conquered them, it was only then that the discoverer gained an absolute title unrestricted by Indian
rights.
In Johnson, the plaintiffs claimed the land in question under two (2) grants made by the chiefs of two
(2) Indian tribes. The U.S. Supreme Court refused to recognize this conveyance, the plaintiffs being The court concluded, in essence, that a grant of Indian lands by Indians could not convey a title
private persons. The only conveyance that was recognized was that made by the Indians to the paramount to the title of the United States itself to other parties, saying:
government of the European discoverer. Speaking for the court, Chief Justice Marshall pointed out
that the potentates of the old world believed that they had made ample compensation to the "It has never been contended that the Indian title amounted to nothing. Their right of possession has
inhabitants of the new world by bestowing civilization and Christianity upon them; but in addition, never been questioned. The claim of government extends to the complete ultimate title, charged with
said the court, they found it necessary, in order to avoid conflicting settlements and consequent war, this right of possession, and to the exclusive power of acquiring that right."162
to establish the principle that discovery gives title to the government by whose subjects, or by whose
authority, the discovery was made, against all other European governments, which title might be It has been said that the history of America, from its discovery to the present day, proves the universal
consummated by possession.160 The exclusion of all other Europeans gave to the nation making the recognition of this principle.163
discovery the sole right of acquiring the soil from the natives and establishing settlements upon it. As
regards the natives, the court further stated that: The Johnson doctrine was a compromise. It protected Indian rights and their native lands without
having to invalidate conveyances made by the government to many U.S. citizens.164
"Those relations which were to exist between the discoverer and the natives were to be regulated by
themselves. The rights thus acquired being exclusive, no other power could interpose between them. Johnson was reiterated in the case of Worcester v. Georgia.165 In this case, the State of Georgia
enacted a law requiring all white persons residing within the Cherokee nation to obtain a license or
In the establishment of these relations, the rights of the original inhabitants were, in no instance, permit from the Governor of Georgia; and any violation of the law was deemed a high misdemeanor.
entirely disregarded; but were necessarily, to a considerable extent, impaired. They were admitted The plaintiffs, who were white missionaries, did not obtain said license and were thus charged with a
to be the rightful occupants of the soil, with a legal as well as just claim to retain possession of it, and violation of the Act.
to use it according to their own discretion; but their rights to complete sovereignty, as independent
nations, were necessarily diminished, and their power to dispose of the soil at their own will, to The U.S. Supreme Court declared the Act as unconstitutional for interfering with the treaties
established between the United States and the Cherokee nation as well as the Acts of Congress

17
regulating intercourse with them. It characterized the relationship between the United States The discovery of the American continent gave title to the government of the discoverer as against all
government and the Indians as: other European governments. Designated as the naked fee,169 this title was to be consummated by
possession and was subject to the Indian title of occupancy. The discoverer acknowledged the Indians'
"The Indian nations were, from their situation, necessarily dependent on some foreign potentate for legal and just claim to retain possession of the land, the Indians being the original inhabitants of the
the supply of their essential wants, and for their protection from lawless and injurious intrusions into land. The discoverer nonetheless asserted the exclusive right to acquire the Indians' land- either by
their country. That power was naturally termed their protector. They had been arranged under the purchase, "defensive" conquest, or cession- and in so doing, extinguish the Indian title. Only the
protection of Great Britain; but the extinguishment of the British power in their neighborhood, and discoverer could extinguish Indian title because it alone asserted ultimate dominion in itself. Thus,
the establishment of that of the United States in its place, led naturally to the declaration, on the part while the different nations of Europe respected the rights of the natives as occupants, they all
of the Cherokees, that they were under the protection of the United States, and of no other power. asserted the ultimate dominion and title to be in themselves.170
They assumed the relation with the United States which had before subsisted with Great Britain.
As early as the 19th century, it became accepted doctrine that although fee title to the lands occupied
This relation was that of a nation claiming and receiving the protection of one more powerful, not by the Indians when the colonists arrived became vested in the sovereign- first the discovering
that of individuals abandoning their national character, and submitting as subjects to the laws of a European nation and later the original 13 States and the United States- a right of occupancy in the
master."166 Indian tribes was nevertheless recognized. The Federal Government continued the policy of
respecting the Indian right of occupancy, sometimes called Indian title, which it accorded the
It was the policy of the U.S. government to treat the Indians as nations with distinct territorial protection of complete ownership.171 But this aboriginal Indian interest simply constitutes
boundaries and recognize their right of occupancy over all the lands within their domains. Thus: "permission" from the whites to occupy the land, and means mere possession not specifically
recognized as ownership by Congress.172 It is clear that this right of occupancy based upon aboriginal
"From the commencement of our government Congress has passed acts to regulate trade and possession is not a property right.173 It is vulnerable to affirmative action by the federal government
intercourse with the Indians; which treat them as nations, respect their rights, and manifest a firm who, as sovereign, possessed exclusive power to extinguish the right of occupancy at will.174 Thus,
purpose to afford that protection which treaties stipulate. All these acts, and especially that of 1802, aboriginal title is not the same as legal title. Aboriginal title rests on actual, exclusive and continuous
which is still in force, manifestly consider the several Indian nations as distinct political communities, use and occupancy for a long time.175 It entails that land owned by Indian title must be used within
having territorial boundaries, within which their authority is exclusive, and having a right to all the the tribe, subject to its laws and customs, and cannot be sold to another sovereign government nor
lands within those boundaries, which is not only acknowledged, but guaranteed by the United States. to any citizen.176 Such title as Indians have to possess and occupy land is in the tribe, and not in the
individual Indian; the right of individual Indians to share in the tribal property usually depends upon
x x x. tribal membership, the property of the tribe generally being held in communal ownership.177

"The Indian nations had always been considered as distinct, independent political communities, As a rule, Indian lands are not included in the term "public lands," which is ordinarily used to designate
retaining their original natural rights, as the undisputed possessors of the soil from time immemorial, such lands as are subject to sale or other disposal under general laws.178 Indian land which has been
with the single exception of that imposed by irresistible power, which excluded them from abandoned is deemed to fall into the public domain.179 On the other hand, an Indian reservation is
intercourse with any other European potentate than the first discoverer of the coast of the particular a part of the public domain set apart for the use and occupation of a tribe of Indians.180 Once set
region claimed: and this was a restriction which those European potentates imposed on themselves, apart by proper authority, the reservation ceases to be public land, and until the Indian title is
as well as on the Indians. The very term "nation," so generally applied to them, means "a people extinguished, no one but Congress can initiate any preferential right on, or restrict the nation's power
distinct from others." x x x.167 to dispose of, them.181

The Cherokee nation, then, is a distinct community, occupying its own territory, with boundaries The American judiciary struggled for more than 200 years with the ancestral land claims of indigenous
accurately described, in which the laws of Georgia can have no force, and which the citizens of Georgia Americans.182 And two things are clear. First, aboriginal title is recognized. Second, indigenous
have no right to enter but with the assent of the Cherokees themselves or in conformity with treaties property systems are also recognized. From a legal point of view, certain benefits can be drawn from
and with the acts of Congress. The whole intercourse between the United States and this nation is, a comparison of Philippine IPs to native Americans.183 Despite the similarities between native title
by our Constitution and laws, vested in the government of the United States."168 and aboriginal title, however, there are at present some misgivings on whether jurisprudence on
American Indians may be cited authoritatively in the Philippines. The U.S. recognizes the possessory

18
rights of the Indians over their land; title to the land, however, is deemed to have passed to the U.S. (3) The Option of Securing a Torrens Title to the Ancestral Land Indicates that the Land is Private.
as successor of the discoverer. The aboriginal title of ownership is not specifically recognized as
ownership by action authorized by Congress.184 The protection of aboriginal title merely guards The private character of ancestral lands and domains as laid down in the IPRA is further strengthened
against encroachment by persons other than the Federal Government.185 Although there are by the option given to individual ICCs/IPs over their individually-owned ancestral lands. For purposes
criticisms against the refusal to recognize the native Americans' ownership of these lands,186 the of registration under the Public Land Act and the Land Registration Act, the IPRA expressly converts
power of the State to extinguish these titles has remained firmly entrenched.187 ancestral land into public agricultural land which may be disposed of by the State. The necessary
implication is that ancestral land is private. It, however, has to be first converted to public agricultural
Under the IPRA, the Philippine State is not barred form asserting sovereignty over the ancestral land simply for registration purposes. To wit:
domains and ancestral lands.188 The IPRA, however, is still in its infancy and any similarities between
its application in the Philippines vis-à-vis American Jurisprudence on aboriginal title will depend on "Sec. 12. Option to Secure Certificate of Title Under Commonwealth Act 141, as amended, or the Land
the peculiar facts of each case. Registration Act 496- Individual members of cultural communities, with respect to their individually-
owned ancestral lands who, by themselves or through their predecessors-in-interest, have been in
(c) Why the Cariño doctrine is unique continuous possession and occupation of the same in the concept of owner since time immemorial
or for a period of not less than thirty (30) years immediately preceding the approval of this Act and
In the Philippines, the concept of native title first upheld in Cariño and enshrined in the IPRA grants uncontested by the members of the same ICCs/IPs shall have the option to secure title to their
ownership, albeit in limited form, of the land to the ICCs/IPs. Native title presumes that the land is ancestral lands under the provisions of Commonwealth Act 141, as amended, or the Land Registration
private and was never public. Cariño is the only case that specifically and categorically recognizes Act 496.
native title. The long line of cases citing Cariño did not touch on native title and the private character
of ancestral domains and lands. Cariño was cited by the succeeding cases to support the concept of For this purpose, said individually-owned ancestral lands, which are agricultural in character and
acquisitive prescription under the Public Land Act which is a different matter altogether. Under the actually used for agricultural, residential, pasture, and tree farming purposes, including those with a
Public Land Act, land sought to be registered must be public agricultural land. When the conditions slope of eighteen percent (18%) or more, are hereby classified as alienable and disposable agricultural
specified in Section 48 [b] of the Public Land Act are complied with, the possessor of the land is lands.
deemed to have acquired, by operation of law, a right to a grant of the land.189 The land ceases to
be part of the public domain,190 ipso jure,191 and is converted to private property by the mere lapse The option granted under this section shall be exercised within twenty (20) years from the approval
or completion of the prescribed statutory period. of this Act."196

It was only in the case of Oh Cho v. Director of Lands192 that the court declared that the rule that all ICCs/IPs are given the option to secure a torrens certificate of title over their individually-owned
lands that were not acquired from the government, either by purchase or grant, belong to the public ancestral lands. This option is limited to ancestral lands only, not domains, and such lands must be
domain has an exception. This exception would be any land that should have been in the possession individually, not communally, owned.
of an occupant and of his predecessors-in-interest since time immemorial. It is this kind of possession
that would justify the presumption that the land had never been part of the public domain or that it Ancestral lands that are owned by individual members of ICCs/IPs who, by themselves or through
had been private property even before the Spanish conquest.193 Oh Cho, however, was decided their predecessors-in-interest, have been in continuous possession and occupation of the same in the
under the provisions of the Public Land Act and Cariño was cited to support the applicant's claim of concept of owner since time immemorial197 or for a period of not less than 30 years, which claims
acquisitive prescription under the said Act. are uncontested by the members of the same ICCs/IPs, may be registered under C.A. 141, otherwise
known as the Public Land Act, or Act 496, the Land Registration Act. For purposes of registration, the
All these years, Cariño had been quoted out of context simply to justify long, continuous, open and individually-owned ancestral lands are classified as alienable and disposable agricultural lands of the
adverse possession in the concept of owner of public agricultural land. It is this long, continuous, open public domain, provided, they are agricultural in character and are actually used for agricultural,
and adverse possession in the concept of owner of thirty years both for ordinary citizens194 and residential, pasture and tree farming purposes. These lands shall be classified as public agricultural
members of the national cultural minorities195 that converts the land from public into private and lands regardless of whether they have a slope of 18% or more.
entitles the registrant to a torrens certificate of title.

19
The classification of ancestral land as public agricultural land is in compliance with the requirements "Sec. 44. Any natural-born citizen of the Philippines who is not the owner of more than twenty-four
of the Public Land Act and the Land Registration Act. C.A. 141, the Public Land Act, deals specifically hectares and who since July fourth, 1926 or prior thereto, has continuously occupied and cultivated,
with lands of the public domain.198 Its provisions apply to those lands "declared open to disposition either by himself or through his predecessors-in-interest, a tract or tracts of agricultural public lands
or concession" x x x "which have not been reserved for public or quasi-public purposes, nor subject to disposition, or who shall have paid the real estate tax thereon while the same has not been
appropriated by the Government, nor in any manner become private property, nor those on which a occupied by any person shall be entitled, under the provisions of this chapter, to have a free patent
private right authorized and recognized by this Act or any other valid law x x x or which having been issued to him for such tract or tracts of such land not to exceed twenty-four hectares.
reserved or appropriated, have ceased to be so."199 Act 496, the Land Registration Act, allows
registration only of private lands and public agricultural lands. Since ancestral domains and lands are A member of the national cultural minorities who has continuously occupied and cultivated, either by
private, if the ICC/IP wants to avail of the benefits of C.A. 141 and Act 496, the IPRA itself converts his himself or through his predecessors-in-interest, a tract or tracts of land, whether disposable or not
ancestral land, regardless of whether the land has a slope of eighteen per cent (18%) or over,200 from since July 4, 1955, shall be entitled to the right granted in the preceding paragraph of this section:
private to public agricultural land for proper disposition. Provided, That at the time he files his free patent application he is not the owner of any real property
secured or disposable under the provision of the Public Land Law.203
The option to register land under the Public Land Act and the Land Registration Act has nonetheless
a limited period. This option must be exercised within twenty (20) years from October 29, 1997, the x x x.
date of approval of the IPRA.
"Sec. 48. The following described citizens of the Philippines, occupying lands of the public domain or
Thus, ancestral lands and ancestral domains are not part of the lands of the public domain. They are claiming to own any such lands or an interest therein, but whose titles have not been perfected or
private and belong to the ICCs/IPs. Section 3 of Article XII on National Economy and Patrimony of the completed, may apply to the Court of First Instance of the province where the land is located for
1987 Constitution classifies lands of the public domain into four categories: (a) agricultural, (b) forest confirmation of their claims and the issuance of a certificate of title therefor, under the Land
or timber, (c) mineral lands, and (d) national parks. Section 5 of the same Article XII mentions ancestral Registration Act, to wit:
lands and ancestral domains but it does not classify them under any of the said four categories. To
classify them as public lands under any one of the four classes will render the entire IPRA law a nullity. (a) [perfection of Spanish titles] xxx.
The spirit of the IPRA lies in the distinct concept of ancestral domains and ancestral lands. The IPRA
addresses the major problem of the ICCs/IPs which is loss of land. Land and space are of vital concern (b) Those who by themselves or through their predecessors-in-interest have been in open,
in terms of sheer survival of the ICCs/IPs.201 continuous, exclusive, and notorious possession and occupation of agricultural lands of the public
domain, under a bona fide claim of acquisition or ownership, for at least thirty years immediately
The 1987 Constitution mandates the State to "protect the rights of indigenous cultural communities preceding the filing of the application for confirmation of title except when prevented by war or force
to their ancestral lands" and that "Congress provide for the applicability of customary laws x x x in majeure. These shall be conclusively presumed to have performed all the conditions essential to a
determining the ownership and extent of ancestral domain."202 It is the recognition of the ICCs/IPs Government grant and shall be entitled to a certificate of title under the provisions of this Chapter.
distinct rights of ownership over their ancestral domains and lands that breathes life into this
constitutional mandate. (c) Members of the national cultural minorities who by themselves or through their predecessors-in-
interest have been in open, continuous, exclusive and notorious possession and occupation of lands
B. The right of ownership and possession by the ICCs/IPs of their ancestral domains is a limited form of the public domain suitable to agriculture, whether disposable or not, under a bona fide claim of
of ownership and does not include the right to alienate the same. ownership for at least 30 years shall be entitled to the rights granted in sub-section (b) hereof."204

Registration under the Public Land Act and Land Registration Act recognizes the concept of ownership Registration under the foregoing provisions presumes that the land was originally public agricultural
under the civil law. This ownership is based on adverse possession for a specified period, and harkens land but because of adverse possession since July 4, 1955 (free patent) or at least thirty years (judicial
to Section 44 of the Public Land Act on administrative legalization (free patent) of imperfect or confirmation), the land has become private. Open, adverse, public and continuous possession is
incomplete titles and Section 48 (b) and (c) of the same Act on the judicial confirmation of imperfect sufficient, provided, the possessor makes proper application therefor. The possession has to be
or incomplete titles. Thus: confirmed judicially or administratively after which a torrens title is issued.

20
A torrens title recognizes the owner whose name appears in the certificate as entitled to all the rights Communal rights to the land are held not only by the present possessors of the land but extends to
of ownership under the civil law. The Civil Code of the Philippines defines ownership in Articles 427, all generations of the ICCs/IPs, past, present and future, to the domain. This is the reason why the
428 and 429. This concept is based on Roman Law which the Spaniards introduced to the Philippines ancestral domain must be kept within the ICCs/IPs themselves. The domain cannot be transferred,
through the Civil Code of 1889. Ownership, under Roman Law, may be exercised over things or rights. sold or conveyed to other persons. It belongs to the ICCs/IPs as a community.
It primarily includes the right of the owner to enjoy and dispose of the thing owned. And the right to
enjoy and dispose of the thing includes the right to receive from the thing what it produces,205 the Ancestral lands are also held under the indigenous concept of ownership. The lands are communal.
right to consume the thing by its use,206 the right to alienate, encumber, transform or even destroy These lands, however, may be transferred subject to the following limitations: (a) only to the
the thing owned,207 and the right to exclude from the possession of the thing owned by any other members of the same ICCs/IPs; (b) in accord with customary laws and traditions; and (c) subject to
person to whom the owner has not transmitted such thing.208 the right of redemption of the ICCs/IPs for a period of 15 years if the land was transferred to a non-
member of the ICCs/IPs.
1. The Indigenous Concept of Ownership and Customary Law.
Following the constitutional mandate that "customary law govern property rights or relations in
Ownership of ancestral domains by native title does not entitle the ICC/IP to a torrens title but to a determining the ownership and extent of ancestral domains,"216 the IPRA, by legislative fiat,
Certificate of Ancestral Domain Title (CADT). The CADT formally recognizes the indigenous concept of introduces a new concept of ownership. This is a concept that has long existed under customary
ownership of the ICCs/IPs over their ancestral domain. Thus: law.217

"Sec. 5. Indigenous concept of ownership.- Indigenous concept of ownership sustains the view that Custom, from which customary law is derived, is also recognized under the Civil Code as a source of
ancestral domains and all resources found therein shall serve as the material bases of their cultural law.218 Some articles of the Civil Code expressly provide that custom should be applied in cases
integrity. The indigenous concept of ownership generally holds that ancestral domains are the where no codal provision is applicable.219 In other words, in the absence of any applicable provision
ICCs/IPs private but community property which belongs to all generations and therefore cannot be in the Civil Code, custom, when duly proven, can define rights and liabilities.220
sold, disposed or destroyed. It likewise covers sustainable traditional resource rights."
Customary law is a primary, not secondary, source of rights under the IPRA and uniquely applies to
The right of ownership and possession of the ICCs/IPs to their ancestral domains is held under the ICCs/IPs. Its recognition does not depend on the absence of a specific provision in the civil law. The
indigenous concept of ownership. This concept maintains the view that ancestral domains are the indigenous concept of ownership under customary law is specifically acknowledged and recognized,
ICCs/IPs private but community property. It is private simply because it is not part of the public and coexists with the civil law concept and the laws on land titling and land registration.221
domain. But its private character ends there. The ancestral domain is owned in common by the
ICCs/IPs and not by one particular person. The IPRA itself provides that areas within the ancestral To be sure, the indigenous concept of ownership exists even without a paper title. The CADT is merely
domains, whether delineated or not, are presumed to be communally held.209 These communal a "formal recognition" of native title. This is clear from Section 11 of the IPRA, to wit:
rights, however, are not exactly the same as co-ownership rights under the Civil Code.210 Co-
ownership gives any co-owner the right to demand partition of the property held in common. The "Sec. 11. Recognition of Ancestral Domain Rights.- The rights of ICCs/IPs to their ancestral domains
Civil Code expressly provides that "no co-owner shall be obliged to remain in the co-ownership." Each by virtue of Native Title shall be recognized and respected. Formal recognition, when solicited by
co-owner may demand at any time the partition of the thing in common, insofar as his share is ICCs/IPs concerned shall be embodied in a Certificate of Ancestral Domain Title, which shall recognize
concerned.211 To allow such a right over ancestral domains may be destructive not only of customary the title of the concerned ICCs/IPs over the territories identified and delineated."
law of the community but of the very community itself.212
The moral import of ancestral domain, native land or being native is "belongingness" to the land,
Communal rights over land are not the same as corporate rights over real property, much less being people of the land- by sheer force of having sprung from the land since time beyond recall, and
corporate condominium rights. A corporation can exist only for a maximum of fifty (50) years subject the faithful nurture of the land by the sweat of one's brow. This is fidelity of usufructuary relation to
to an extension of another fifty years in any single instance.213 Every stockholder has the right to the land- the possession of stewardship through perduring, intimate tillage, and the mutuality of
disassociate himself from the corporation.214 Moreover, the corporation itself may be dissolved blessings between man and land; from man, care for land; from the land, sustenance for man.222
voluntarily or involuntarily.215

21
C. Sections 7 (a), 7 (b) and 57 of the IPRA Do Not Violate the Regalian Doctrine Enshrined in Section g) Right to Claim Parts of Reservations.- The right to claim parts of the ancestral domains which have
2, Article XII of the 1987 Constitution. been reserved for various purposes, except those reserved and intended for common and public
welfare and service;
1. The Rights of ICCs/IPs Over Their Ancestral Domains and Lands
h) Right to Resolve Conflict.- Right to resolve land conflicts in accordance with customary laws of the
The IPRA grants the ICCs/IPs several rights over their ancestral domains and ancestral lands. Section area where the land is located, and only in default thereof shall the complaints be submitted to
7 provides for the rights over ancestral domains: amicable settlement and to the Courts of Justice whenever necessary."

"Sec. 7. Rights to Ancestral Domains.- The rights of ownership and possession of ICCs/IPs to their Section 8 provides for the rights over ancestral lands:
ancestral domains shall be recognized and protected. Such rights include:
"Sec. 8. Rights to Ancestral Lands.- The right of ownership and possession of the ICCs/IPs to their
a) Right of Ownership.- The right to claim ownership over lands, bodies of water traditionally and ancestral lands shall be recognized and protected.
actually occupied by ICCs/IPs, sacred places, traditional hunting and fishing grounds, and all
improvements made by them at any time within the domains; a) Right to transfer land/property.- Such right shall include the right to transfer land or property rights
to/among members of the same ICCs/IPs, subject to customary laws and traditions of the community
b) Right to Develop Lands and Natural Resources.- Subject to Section 56 hereof, the right to develop, concerned.
control and use lands and territories traditionally occupied, owned, or used; to manage and conserve
natural resources within the territories and uphold the responsibilities for future generations; to b) Right to Redemption.- In cases where it is shown that the transfer of land/property rights by virtue
benefit and share the profits from allocation and utilization of the natural resources found therein; of any agreement or devise, to a non-member of the concerned ICCs/IPs is tainted by the vitiated
the right to negotiate the terms and conditions for the exploration of natural resources in the areas consent of the ICCs/IPs, or is transferred for an unconscionable consideration or price, the transferor
for the purpose of ensuring ecological, environmental protection and the conservation measures, ICC/IP shall have the right to redeem the same within a period not exceeding fifteen (15) years from
pursuant to national and customary laws; the right to an informed and intelligent participation in the the date of transfer."
formulation and implementation of any project, government or private, that will affect or impact
upon the ancestral domains and to receive just and fair compensation for any damages which they Section 7 (a) defines the ICCs/IPs the right of ownership over their ancestral domains which covers
may sustain as a result of the project; and the right to effective measures by the government to (a) lands, (b) bodies of water traditionally and actually occupied by the ICCs/IPs, (c) sacred places, (d)
prevent any interference with, alienation and encroachment upon these rights;" traditional hunting and fishing grounds, and (e) all improvements made by them at any time within
the domains. The right of ownership includes the following rights: (1) the right to develop lands and
c) Right to Stay in the Territories.- The right to stay in the territory and not to be removed therefrom. natural resources; (b) the right to stay in the territories; (c) the right to resettlement in case of
No ICCs/IPs will be relocated without their free and prior informed consent, nor through any means displacement; (d) the right to regulate the entry of migrants; (e) the right to safe and clean air and
other than eminent domain. x x x; water; (f) the right to claim parts of the ancestral domains as reservations; and (g) the right to resolve
conflict in accordance with customary laws.
d) Right in Case of Displacement.- In case displacement occurs as a result of natural catastrophes, the
State shall endeavor to resettle the displaced ICCs/IPs in suitable areas where they can have Section 8 governs their rights to ancestral lands. Unlike ownership over the ancestral domains, Section
temporary life support systems: x x x; 8 gives the ICCs/IPs also the right to transfer the land or property rights to members of the same
ICCs/IPs or non-members thereof. This is in keeping with the option given to ICCs/IPs to secure a
e) Right to Regulate the Entry of Migrants.- Right to regulate the entry of migrant settlers and torrens title over the ancestral lands, but not to domains.
organizations into their domains;
2. The Right of ICCs/IPs to Develop Lands and Natural Resources Within the Ancestral Domains Does
f) Right to Safe and Clean Air and Water.-For this purpose, the ICCs/IPs shall have access to integrated Not Deprive the State of Ownership Over the Natural Resources and Control and Supervision in their
systems for the management of their inland waters and air space; Development and Exploitation.

22
The Regalian doctrine on the ownership, management and utilization of natural resources is declared 2. The State may enter into co-production, joint venture or production-sharing agreements with
in Section 2, Article XII of the 1987 Constitution, viz: Filipino citizens or qualified corporations;

"Sec. 2. All lands of the public domain, waters, minerals, coal, petroleum, and other mineral oils, all 3. Congress may, by law, allow small-scale utilization of natural resources by Filipino citizens;
forces of potential energy, fisheries, forests or timber, wildlife, flora and fauna, and other natural
resources are owned by the State. With the exception of agricultural lands, all other natural resources 4. For the large-scale exploration, development and utilization of minerals, petroleum and other
shall not be alienated. The exploration, development, and utilization of natural resources shall be mineral oils, the President may enter into agreements with foreign-owned corporations involving
under the full control and supervision of the State. The State may directly undertake such activities, technical or financial assistance.
or, it may enter into co-production, joint venture, or production-sharing agreements with Filipino
citizens, or corporations or associations at least sixty per centum of whose capital is owned by such As owner of the natural resources, the State is accorded primary power and responsibility in the
citizens. Such agreements may be for a period not exceeding twenty-five years, renewable for not exploration, development and utilization of these natural resources. The State may directly undertake
more than twenty-five years, and under such terms and conditions as may be provided by law. In the exploitation and development by itself, or, it may allow participation by the private sector through
cases of water rights for irrigation, water supply, fisheries, water supply, fisheries, or industrial uses co-production,224 joint venture,225 or production-sharing agreements.226 These agreements may
other than the development of water power, beneficial use may be the measure and limit of the be for a period of 25 years, renewable for another 25 years. The State, through Congress, may allow
grant. the small-scale utilization of natural resources by Filipino citizens. For the large-scale exploration of
these resources, specifically minerals, petroleum and other mineral oils, the State, through the
The State shall protect the nation's marine wealth in its archipelagic waters, territorial sea, and President, may enter into technical and financial assistance agreements with foreign-owned
exclusive economic zone, and reserve its use and enjoyment exclusively to Filipino citizens. corporations.

The Congress may, by law, allow small-scale utilization of natural resources by Filipino citizens, as well Under the Philippine Mining Act of 1995, (R.A. 7942) and the People's Small-Scale Mining Act of 1991
as cooperative fish farming, with priority to subsistence fishermen and fishworkers in rivers, lakes, (R.A. 7076) the three types of agreements, i.e., co-production, joint venture or production-sharing,
bays, and lagoons. may apply to both large-scale227 and small-scale mining.228 "Small-scale mining" refers to "mining
activities which rely heavily on manual labor using simple implements and methods and do not use
The President may enter into agreements with foreign-owned corporations involving either technical explosives or heavy mining equipment."229
or financial assistance for large-scale exploration, development, and utilization of minerals,
petroleum, and other mineral oils according to the general terms and conditions provided by law, Examining the IPRA, there is nothing in the law that grants to the ICCs/IPs ownership over the natural
based on real contributions to the economic growth and general welfare of the country. In such resources within their ancestral domains. The right of ICCs/IPs in their ancestral domains includes
agreements, the state shall promote the development and use of local scientific and technical ownership, but this "ownership" is expressly defined and limited in Section 7 (a) as:
resources.
"Sec. 7. a) Right of ownership- The right to claim ownership over lands, bodies of water traditionally
The President shall notify the Congress of every contract entered into in accordance with this and actually occupied by ICCs/IPs, sacred places, traditional hunting and fishing grounds, and all
provision, within thirty days from its execution."223 improvements made by them at any time within the domains;"

All lands of the public domain and all natural resources- waters, minerals, coal, petroleum, and other The ICCs/IPs are given the right to claim ownership over "lands, bodies of water traditionally and
mineral oils, all forces of potential energy, fisheries, forests or timber, wildlife, flora and fauna, and actually occupied by ICCs/IPs, sacred places, traditional hunting and fishing grounds, and all
other natural resources- are owned by the State. The Constitution provides that in the exploration, improvements made by them at any time within the domains." It will be noted that this enumeration
development and utilization of these natural resources, the State exercises full control and does not mention bodies of water not occupied by the ICCs/IPs, minerals, coal, wildlife, flora and
supervision, and may undertake the same in four (4) modes: fauna in the traditional hunting grounds, fish in the traditional fishing grounds, forests or timber in
the sacred places, etc. and all other natural resources found within the ancestral domains. Indeed,
1. The State may directly undertake such activities; or the right of ownership under Section 7 (a) does not cover "waters, minerals, coal, petroleum and
other mineral oils, all forces of potential energy, fisheries, forests or timber, wildlife, flora and fauna

23
and all other natural resources" enumerated in Section 2, Article XII of the 1987 Constitution as "Sec. 7 (b) Right to Develop Lands and Natural Resources.- Subject to Section 56 hereof, right to
belonging to the State. develop, control and use lands and territories traditionally occupied, owned, or used; to manage and
conserve natural resources within the territories and uphold the responsibilities for future
The non-inclusion of ownership by the ICCs/IPs over the natural resources in Section 7(a) complies generations; to benefit and share the profits from allocation and utilization of the natural resources
with the Regalian doctrine. found therein; the right to negotiate the terms and conditions for the exploration of natural resources
in the areas for the purpose of ensuring ecological, environmental protection and the conservation
(a) Section 1, Part II, Rule III of the Implementing Rules Goes Beyond the Parameters of Sec. 7 (a) of measures, pursuant to national and customary laws; the right to an informed and intelligent
the IPRA And is Unconstitutional. participation in the formulation and implementation of any project, government or private, that will
affect or impact upon the ancestral domains and to receive just and fair compensation for any
The Rules Implementing the IPRA230 in Section 1, Part II, Rule III reads: damages which they may sustain as a result of the project; and the right to effective measures by the
government to prevent any interference with, alienation and encroachment upon these rights;"
"Section 1. Rights of Ownership. ICCs/IPs have rights of ownership over lands, waters, and natural
resources and all improvements made by them at any time within the ancestral domains/ lands. These The right to develop lands and natural resources under Section 7 (b) of the IPRA enumerates the
rights shall include, but not limited to, the right over the fruits, the right to possess, the right to use, following rights:
right to consume, right to exclude and right to recover ownership, and the rights or interests over
land and natural resources. The right to recover shall be particularly applied to lands lost through a) the right to develop, control and use lands and territories traditionally occupied;
fraud or any form or vitiated consent or transferred for an unconscionable price."
b) the right to manage and conserve natural resources within the territories and uphold the
Section 1 of the Implementing Rules gives the ICCs/IPs rights of ownership over "lands, waters and responsibilities for future generations;
natural resources." The term "natural resources" is not one of those expressly mentioned in Section
7 (a) of the law. Our Constitution and jurisprudence clearly declare that the right to claim ownership c) the right to benefit and share the profits from the allocation and utilization of the natural resources
over land does not necessarily include the right to claim ownership over the natural resources found found therein;
on or under the land.231 The IPRA itself makes a distinction between land and natural resources.
Section 7 (a) speaks of the right of ownership only over the land within the ancestral domain. It is d) the right to negotiate the terms and conditions for the exploration of natural resources for the
Sections 7 (b) and 57 of the law that speak of natural resources, and these provisions, as shall be purpose of ensuring ecological, environmental protection and the conservation measures, pursuant
discussed later, do not give the ICCs/IPs the right of ownership over these resources. to national and customary laws;

The constitutionality of Section 1, Part II, Rule III of the Implementing Rules was not specifically and e) the right to an informed and intelligent participation in the formulation and implementation of any
categorically challenged by petitioners. Petitioners actually assail the constitutionality of the project, government or private, that will affect or impact upon the ancestral domains and to receive
Implementing Rules in general.232 Nevertheless, to avoid any confusion in the implementation of the just and fair compensation for any damages which they may sustain as a result of the project;
law, it is necessary to declare that the inclusion of "natural resources" in Section 1, Part II, Rule III of
the Implementing Rules goes beyond the parameters of Section 7 (b) of the law and is contrary to f) the right to effective measures by the government to prevent any interference with, alienation and
Section 2, Article XII of the 1987 Constitution. encroachment upon these rights.233

(b) The Small-Scale Utilization of Natural Resources In Sec. 7 (b) of the IPRA Is Allowed Under Ownership over the natural resources in the ancestral domains remains with the State and the
Paragraph 3, Section 2 of Article XII of the Constitution. ICCs/IPs are merely granted the right to "manage and conserve" them for future generations, "benefit
and share" the profits from their allocation and utilization, and "negotiate the terms and conditions
Ownership over natural resources remain with the State and the IPRA in Section 7 (b) merely grants for their exploration" for the purpose of "ensuring ecological and environmental protection and
the ICCs/IPs the right to manage them, viz: conservation measures." It must be noted that the right to negotiate the terms and conditions over
the natural resources covers only their exploration which must be for the purpose of ensuring

24
ecological and environmental protection of, and conservation measures in the ancestral domain. It than 25 years, renewable for another 25 years. This may be done on condition that a formal written
does not extend to the exploitation and development of natural resources. agreement be entered into by the non-member and members of the ICCs/IPs.

Simply stated, the ICCs/IPs' rights over the natural resources take the form of management or Section 57 of the IPRA does not give the ICCs/IPs the right to "manage and conserve" the natural
stewardship. For the ICCs/IPs may use these resources and share in the profits of their utilization or resources. Instead, the law only grants the ICCs/IPs "priority rights" in the development or
negotiate the terms for their exploration. At the same time, however, the ICCs/IPs must ensure that exploitation thereof. Priority means giving preference. Having priority rights over the natural
the natural resources within their ancestral domains are conserved for future generations and that resources does not necessarily mean ownership rights. The grant of priority rights implies that there
the "utilization" of these resources must not harm the ecology and environment pursuant to national is a superior entity that owns these resources and this entity has the power to grant preferential rights
and customary laws.234 over the resources to whosoever itself chooses.

The limited rights of "management and use" in Section 7 (b) must be taken to contemplate small- Section 57 is not a repudiation of the Regalian doctrine. Rather, it is an affirmation of the said doctrine
scale utilization of natural resources as distinguished from large-scale. Small-scale utilization of that all natural resources found within the ancestral domains belong to the State. It incorporates by
natural resources is expressly allowed in the third paragraph of Section 2, Article XII of the implication the Regalian doctrine, hence, requires that the provision be read in the light of Section 2,
Constitution "in recognition of the plight of forest dwellers, gold panners, marginal fishermen and Article XII of the 1987 Constitution. Interpreting Section 2, Article XII of the 1987 Constitution237 in
others similarly situated who exploit our natural resources for their daily sustenance and survival."235 relation to Section 57 of IPRA, the State, as owner of these natural resources, may directly undertake
Section 7 (b) also expressly mandates the ICCs/IPs to manage and conserve these resources and the development and exploitation of the natural resources by itself, or in the alternative, it may
ensure environmental and ecological protection within the domains, which duties, by their very recognize the priority rights of the ICCs/IPs as owners of the land on which the natural resources are
nature, necessarily reject utilization in a large-scale. found by entering into a co-production, joint venture, or production-sharing agreement with them.
The State may likewise enter into any of said agreements with a non-member of the ICCs/IPs, whether
(c) The Large-Scale Utilization of Natural Resources In Section 57 of the IPRA Is Allowed Under natural or juridical, or enter into agreements with foreign-owned corporations involving either
Paragraphs 1 and 4, Section 2, Article XII of the 1987 Constitution. technical or financial assistance for the large-scale exploration, development and utilization of
minerals, petroleum, and other mineral oils, or allow such non-member to participate in its
Section 57 of the IPRA provides: agreement with the ICCs/IPs. If the State decides to enter into an agreement with a non-ICC/IP
member, the National Commission on Indigenous Peoples (NCIP) shall ensure that the rights of the
"Sec. 57. Natural Resources within Ancestral Domains.- The ICCs/IPs shall have priority rights in the ICCs/IPs under the agreement shall be protected. The agreement shall be for a period of 25 years,
harvesting, extraction, development or exploitation of any natural resources within the ancestral renewable for another 25 years.
domains. A non-member of the ICCs/IPs concerned may be allowed to take part in the development
and utilization of the natural resources for a period of not exceeding twenty-five (25) years renewable To reiterate, in the large-scale utilization of natural resources within the ancestral domains, the State,
for not more than twenty-five (25) years: Provided, That a formal and written agreement is entered as owner of these resources, has four (4) options: (1) it may, of and by itself, directly undertake the
into with the ICCs/IPs concerned or that the community, pursuant to its own decision-making process, development and exploitation of the natural resources; or (2) it may recognize the priority rights of
has agreed to allow such operation: Provided finally, That the NCIP may exercise visitorial powers and the ICCs/IPs by entering into an agreement with them for such development and exploitation; or (3)
take appropriate action to safeguard the rights of the ICCs/IPs under the same contract." it may enter into an agreement with a non-member of the ICCs/IPs, whether natural or juridical, local
or foreign; or (4) it may allow such non-member to participate in the agreement with the ICCs/IPs.
Section 57 speaks of the "harvesting, extraction, development or exploitation of natural resources
within ancestral domains" and "gives the ICCs/IPs 'priority rights' therein." The terms "harvesting, The rights granted by the IPRA to the ICCs/IPs over the natural resources in their ancestral domains
extraction, development or exploitation" of any natural resources within the ancestral domains merely gives the ICCs/IPs, as owners and occupants of the land on which the resources are found, the
obviously refer to large-scale utilization. It is utilization not merely for subsistence but for commercial right to the small-scale utilization of these resources, and at the same time, a priority in their large-
or other extensive use that require technology other than manual labor.236 The law recognizes the scale development and exploitation. Section 57 does not mandate the State to automatically give
probability of requiring a non-member of the ICCs/IPs to participate in the development and priority to the ICCs/IPs. The State has several options and it is within its discretion to choose which
utilization of the natural resources and thereby allows such participation for a period of not more option to pursue. Moreover, there is nothing in the law that gives the ICCs/IPs the right to solely
undertake the large-scale development of the natural resources within their domains. The ICCs/IPs

25
must undertake such endeavour always under State supervision or control. This indicates that the
State does not lose control and ownership over the resources even in their exploitation. Sections 7 The indigenous movement can be seen as the heir to a history of anti-imperialism stretching back to
(b) and 57 of the law simply give due respect to the ICCs/IPs who, as actual occupants of the land prehistoric times. The movement received a massive impetus during the 1960's from two sources.
where the natural resources lie, have traditionally utilized these resources for their subsistence and First, the decolonization of Asia and Africa brought into the limelight the possibility of peoples
survival. controlling their own destinies. Second, the right of self-determination was enshrined in the UN
Declaration on Human Rights.238 The rise of the civil rights movement and anti-racism brought to
Neither is the State stripped of ownership and control of the natural resources by the following the attention of North American Indians, Aborigines in Australia, and Maori in New Zealand the
provision: possibility of fighting for fundamental rights and freedoms.

"Section 59. Certification Precondition.- All departments and other governmental agencies shall In 1974 and 1975, international indigenous organizations were founded,239 and during the 1980's,
henceforth be strictly enjoined from issuing, renewing or granting any concession, license or lease, or indigenous affairs were on the international agenda. The people of the Philippine Cordillera were the
entering into any production-sharing agreement. without prior certification from the NCIP that the first Asians to take part in the international indigenous movement. It was the Cordillera People's
area affected does not overlap with any ancestral domain. Such certification shall only be issued after Alliance that carried out successful campaigns against the building of the Chico River Dam in 1981-82
a field-based investigation is conducted by the Ancestral Domains Office of the area concerned: and they have since become one of the best-organized indigenous bodies in the world.240
Provided, That no certification shall be issued by the NCIP without the free and prior informed and
written consent of the ICCs/IPs concerned: Provided, further, That no department, government Presently, there is a growing concern for indigenous rights in the international scene. This came as a
agency or government-owned or -controlled corporation may issue new concession, license, lease, or result of the increased publicity focused on the continuing disrespect for indigenous human rights
production sharing agreement while there is a pending application for a CADT: Provided, finally, That and the destruction of the indigenous peoples' environment, together with the national governments'
the ICCs/IPs shall have the right to stop or suspend, in accordance with this Act, any project that has inability to deal with the situation.241 Indigenous rights came as a result of both human rights and
not satisfied the requirement of this consultation process." environmental protection, and have become a part of today's priorities for the international
agenda.242
Concessions, licenses, lease or production-sharing agreements for the exploitation of natural
resources shall not be issued, renewed or granted by all departments and government agencies International institutions and bodies have realized the necessity of applying policies, programs and
without prior certification from the NCIP that the area subject of the agreement does not overlap specific rules concerning IPs in some nations. The World Bank, for example, first adopted a policy on
with any ancestral domain. The NCIP certification shall be issued only after a field-based investigation IPs as a result of the dismal experience of projects in Latin America.243 The World Bank now seeks to
shall have been conducted and the free and prior informed written consent of the ICCs/IPs obtained. apply its current policy on IPs to some of its projects in Asia. This policy has provided an influential
Non-compliance with the consultation requirement gives the ICCs/IPs the right to stop or suspend model for the projects of the Asian Development Bank.244
any project granted by any department or government agency.
The 1987 Philippine Constitution formally recognizes the existence of ICCs/IPs and declares as a State
As its subtitle suggests, this provision requires as a precondition for the issuance of any concession, policy the promotion of their rights within the framework of national unity and development.245 The
license or agreement over natural resources, that a certification be issued by the NCIP that the area IPRA amalgamates the Philippine category of ICCs with the international category of IPs,246 and is
subject of the agreement does not lie within any ancestral domain. The provision does not vest the heavily influenced by both the International Labor Organization (ILO) Convention 169 and the United
NCIP with power over the other agencies of the State as to determine whether to grant or deny any Nations (UN) Draft Declaration on the Rights of Indigenous Peoples.247
concession or license or agreement. It merely gives the NCIP the authority to ensure that the ICCs/IPs
have been informed of the agreement and that their consent thereto has been obtained. Note that ILO Convention No. 169 is entitled the "Convention Concerning Indigenous and Tribal Peoples in
the certification applies to agreements over natural resources that do not necessarily lie within the Independent Countries"248 and was adopted on June 27, 1989. It is based on the Universal
ancestral domains. For those that are found within the said domains, Sections 7(b) and 57 of the IPRA Declaration of Human Rights, the International Covenant on Economic, Social and Cultural Rights, the
apply. International Covenant on Civil and Political Rights, and many other international instruments on the
prevention of discrimination.249 ILO Convention No. 169 revised the "Convention Concerning the
V. THE IPRA IS A RECOGNITION OF OUR ACTIVE PARTICIPATION IN THE INDIGENOUS INTERNATIONAL Protection and Integration of Indigenous and Other Tribal and Semi-Tribal Populations in Independent
MOVEMENT. Countries" (ILO No. 107) passed on June 26, 1957. Developments in international law made it

26
appropriate to adopt new international standards on indigenous peoples "with a view to removing that one is the proper and appropriate party to invoke judicial power.2 The rule requires a party to
the assimilationist orientation of the earlier standards," and recognizing the aspirations of these aptly show a personal stake in the outcome of the case or an injury to himself that can be redressed
peoples to exercise control over their own institutions, ways of life and economic development."250 by a favorable decision so as to warrant his invocation of the Court’s jurisdiction and to render legally
feasible the exercise of the Court’s remedial powers in his behalf. If it were otherwise, the exercise of
CONCLUSION that power can easily become too unwieldy by its sheer magnitude and scope to a point that may, in
no small measure, adversely affect its intended essentiality, stability and consequentiality.
The struggle of the Filipinos throughout colonial history had been plagued by ethnic and religious
differences. These differences were carried over and magnified by the Philippine government through Nevertheless, where a most compelling reason exits, such as when the matter is of transcendental
the imposition of a national legal order that is mostly foreign in origin or derivation.251 Largely importance and paramount interest to the nation,3 the Court must take the liberal approach that
unpopulist, the present legal system has resulted in the alienation of a large sector of society, recognizes the legal standing of nontraditional plaintiffs, such as citizens and taxpayers, to raise
specifically, the indigenous peoples. The histories and cultures of the indigenes are relevant to the constitutional issues that affect them.4 This Court thus did so in a case5 that involves the conservation
evolution of Philippine culture and are vital to the understanding of contemporary problems.252 It is of our forests for ecological needs. Until and exact balance is struck, the Court must accept an eclectic
through the IPRA that an attempt was made by our legislators to understand Filipino society not in notion that can free itself from the bondage of legal nicety and hold trenchant technicalities
terms of myths and biases but through common experiences in the course of history. The Philippines subordinate to what may be considered to be of overriding concern.
became a democracy a centennial ago and the decolonization process still continues. If the evolution
of the Filipino people into a democratic society is to truly proceed democratically, i.e., if the Filipinos The petition seeks a declaration by the Court of unconstitutionality of certain provisions of Republic
as a whole are to participate fully in the task of continuing democratization,253 it is this Court's duty Act No. 8371, a law that obviously is yet incapable of exact equation in its significance to the nation
to acknowledge the presence of indigenous and customary laws in the country and affirm their co- and its people now and in the generations yet to come. Republic Act No. 8371, otherwise also known
existence with the land laws in our national legal system. as the Indigenous Peoples Rights Act of 1997 ("IPRA"), enacted into law in 1997 and made effective
on 22 November 1997, is apparently intended to be a legislative response to the 1987 Constitution
With the foregoing disquisitions, I vote to uphold the constitutionality of the Indigenous Peoples which recognizes the rights of indigenous cultural communities "within the framework of national
Rights Act of 1997. unity and development"6 and commands the State, "subject to the provisions of this Constitution and
national development policies and programs," to protect the rights of indigenous cultural
communities to their ancestral lands in order to ensure their economic, social, and cultural well-
being.7

Among the assailed provisions in IPRA is its Section 3(a) which defines "ancestral domains" to embrace
"all areas generally belonging to ICCs/IPs comprising lands, inland waters, coastal areas, and natural
SEPARATE OPINION resources" including "ancestral lands, forest, pasture, residential, agricultural, and other lands
individually owned whether alienable and disposable or otherwise," over which indigenous cultural
VITUG, J.: communities/indigenous peoples ("ICCs/IPs") could exercise virtual ownership and control.

An issue of grave national interest indeed deserves a proper place in any forum and, when it shows IPRA effectively withdraws from the public domain the so-called ancestral domains covering literally
itself in a given judicial controversy, the rules of procedure, like locus standi, the propriety of the millions of hectares. The notion of community property would comprehend not only matters of
specific remedy invoked, or the principle of hierarchy of courts, that may ordinarily be raised by party- proprietary interest but also some forms of self-governance over the curved-out territory. This
litigants, should not be so perceived as good and inevitable justifications for advocating timidity, let concept is elaborated in Section 7 of the law which states that the "rights of ownership and possession
alone isolationism, by the Court. of ICCs/IPs to their ancestral domains shall be recognized and protected," subsumed under which
would encompass the right of ownership (paragraph a); the right to develop, control and use lands
A cardinal requirement, to which I agree, is that one who invokes the Court’s adjudication must have and natural resources, including "the right to negotiate the terms and conditions for the exploration
a personal and substantial interest in the dispute;1 indeed, the developing trend would require a of natural resources in the areas for the purpose of ensuring ecological, environmental protection
logical nexus between the status asserted and the claim sought to be adjudicated in order to ensure and the conservation measures, pursuant to national and customary laws;" (par. b); the right to stay

27
in the territories (par. c); the right to return to their abandoned lands in case of displacement (par. a court ruling or any piece of legislation to be conformed to by the fundamental law, but it is for the
d); the right to regulate entry of migrants (par. e); the right to claim parts of ancestral domains former to adapt to the latter, and it is the sovereign act that must, between them, stand inviolate.
previously reserved (par. g); and the right to resolve land conflicts in accordance primarily with
customary law (par. h). Concurrently, Section 57 states that ICCs/IPs shall be given "priority rights in The second paragraph of Section 5 of Article XII of the Constitution allows Congress to provide "for
the harvesting, extraction, development or exploitation of any natural resources within the ancestral the applicability of customary laws governing property rights or relations in determining the
domains." These provisions of IPRA, in their totality, are, in my view, beyond the context of the ownership and extent of ancestral domains." I do not see this statement as saying that Congress may
fundamental law and virtually amount to an undue delegation, if not an unacceptable abdication, of enact a law that would simply express that "customary laws shall govern" and end it there. Had it
State authority over a significant area of the country and its patrimony. been so, the Constitution could have itself easily provided without having to still commission Congress
to do it. Mr. Chief Justice Davide has explained this authority of Congress, during the deliberations of
Article XII of the 1987 Constitution expresses that all "lands of the public domain, waters, minerals, the 1986 Constitutional Convention, thus:
coal, petroleum, and other mineral oils, all forces of potential energy, fisheries, forest or timber,
wildlife, flora and fauna, and other natural resources are owned by the State," and, with the exception "Mr. Davide. x x x Insofar as the application of the customary laws governing property rights or
of agricultural lands, "shall not be alienated." It ordains that the "exploration, development, and relations in determining the ownership and extent of the ancestral domain is concerned, it is
utilization of natural resources shall be under the full control and supervision of the State."8 respectfully submitted that the particular matter must be submitted to Congress. I understand that
the idea of Comm. Bennagen is for the possibility of the codification of these customary laws. So
These provisions had roots in the 1935 Constitution which, along with some other specific mandates before these are codified, we cannot now mandate that the same must immediately be applicable.
in the 1935 Constitution, forming Article XII under the title "Conservation and Utilization of Natural We leave it to Congress to determine the extent of the ancestral domain and the ownership thereof
Resources", were derived largely from the report of the Committee on Nationalization and in relation to whatever may have been codified earlier. So, in short, let us not put the cart ahead of
Preservation of Lands and other Natural Resources.9 According to the Committee report, among the the horse."15
principles upon which these provisions were based, was "that the land, minerals, forest and other
natural resources constitute the exclusive heritage of the Filipino Nation," and should thereby "be The constitutional aim, it seems to me, is to get Congress to look closely into the customary laws and,
preserved for those under the sovereign authority of the Nation and for their posterity."10 The with specificity and by proper recitals, to hew them to, and make them part of, the stream of laws.
delegates to the 1934 Constitutional Convention were of the unanimous view that the "policy on The "due process clause," as I so understand it in Tanada vs. Tuvera16 would require an apt
natural resources, being fundamental to the nation’s survival should not be left to the changing mood publication of a legislative enactment before it is permitted to take force and effect. So, also,
of the lawmaking body."11 customary laws, when specifically enacted to become part of statutory law, must first undergo that
publication to render them correspondingly binding and effective as such.
The 1987 Constitution, like the precursor provisions in the 1935 and 1973 Constitutions, thus
expresses this regalian doctrine of the old, and the domainial doctrine of the new, that all lands and Undoubtedly, IPRA has several good points, and I would respectfully urge Congress to re-examine the
natural resources belong to the state other than those which it recognizes to be of private ownership. law. Indeed, the State is exhorted to protect the rights of indigenous cultural communities to their
Except for agricultural lands of the public domain which alone may be alienated, forest or timber, and ancestral lands, a task that would entail a balancing of interest between their specific needs and the
mineral lands, as well as all other natural resources, of the country must remain with the state, the imperatives of national interest.
exploration, development and utilization of which shall be subject to its full control and supervision
albeit allowing it to enter into co-production, joint venture or production-sharing agreements, or into WHEREFORE, I vote to grant the petition.
agreements with foreign-owned corporations involving technical or financial assistance for large-
scale exploration, development and utilization.12 SEPARATE OPINION

The decision of the United States Supreme Court in Cariño vs. Insular Government,13 holding that a KAPUNAN, J.:
parcel of land held since time immemorial by individuals under a claim of private ownership is
presumed never to have been public land and cited to downgrade the application of the regalian You ask if we own the land. . . How can you own that which will outlive you? Only the race own the
doctrine, cannot override the collective will of the people expressed in the Constitution. It is in them land because only the race lives forever. To claim a piece of land is a birthright of every man. The
that sovereignty resides and from them that all government authority emanates.14 It is not then for lowly animals claim their place; how much more man? Man is born to live. Apu Kabunian, lord of us

28
all, gave us life and placed us in the world to live human lives. And where shall we obtain life? From peoples" has been adopted by the United Nations (UN), although UN practice has been guided by a
the land. To work (the land) is an obligation, not merely a right. In tilling the land, you possess it. And working definition in the 1986 Report of UN Special Rapporteur Martinez Cobo:5
so land is a grace that must be nurtured. To enrich it and make it fructify is the eternal exhortation of
Apu Kabunian to all his children. Land is sacred. Land is beloved. From its womb springs …life. Indigenous communities, peoples and nations are those which, having a historical continuity with pre-
invasion and pre-colonial societies that developed on their territories, consider themselves distinct
- Macli-ing Dulag, Chieftain of the Kalinga Tribe (quoted in Ponciano L. Bennagen, "Tribal Filipinos" in from other sections of the societies now prevailing in those territories, or parts of them. They form at
Indigenous View of Land and the Environment, ed. Shelton H. Davis, the World Bank Discussion present non-dominant sections of society and are determined to preserve, develop and transmit to
Papers, No. 188, pp. 71-72.) future generations their ancestral territories, and their ethnic identity, as the basis of their continued
existence as peoples, in accordance with their own cultural patterns, social institutions and legal
It is established doctrine that a statute should be construed whenever possible in harmony with, systems.
rather than in violation of, the Constitution.1 The presumption is that the legislature intended to
enact a valid, sensible and just law and one which operates no further than may be necessary to This historical continuity may consist of the continuation, for an extended period reaching into the
effectuate the specific purpose of the law.2 present, of one or more of the following factors:

The challenged provisions of the Indigenous Peoples Rights Act (IPRA) must be construed in view of (a) Occupation of ancestral lands, or at least of part of them;
such presumption of constitutionality. Further, the interpretation of these provisions should take into
account the purpose of the law, which is to give life to the constitutional mandate that the rights of (b) Common ancestry with the original occupants of these lands;
the indigenous peoples be recognized and protected.
(c) Culture in general, or in specific manifestations (such as religion, living under a tribal system,
The struggle of our indigenous peoples to reclaim their ancestral lands and domains and therefore, membership of an indigenous community, dress, means of livelihood, life-style, etc.);
their heritage, is not unique. It is one that they share with the red-skinned "Indians" of the United
States, with the aborigines of Australia, the Maori of New Zealand and the Sazmi of Sweden, to name (d) Language (whether used as the only language, as mother-tongue, as the habitual means of
a few. Happily, the nations in which these indigenous peoples live all have enacted measures in an communication at home or in the family, or as the main, preferred, habitual, general or normal
attempt to heal an oppressive past by the promise of a progressive future. Thus has the international language);
community realized the injustices that have been perpetrated upon the indigenous peoples. This
sentiment among the family of nations is expressed in a number of documents, the most recent and (e) Residence in certain parts of the country; or in certain regions of the world;
most comprehensive of which is the Draft United Nations Declaration on the Rights of Indigenous
Peoples which was adopted by the UN Sub-Commission on Prevention of Discrimination and (f) Other relevant facts.6
Protection of Minorities by its resolution on August 26, 1994. Among the rights recognized by the UN
Draft is the restitution of lands, territories and even the resources which the indigenous peoples have In Philippine constitutional law, the term "indigenous peoples" pertains to those groups of Filipinos
traditionally owned or otherwise occupied or used, and which have been confiscated, occupied, used who have retained a high degree of continuity from pre-Conquest culture.7 Philippine legal history,
or damaged without the free and informed consent of the indigenous peoples. however, has not been kind to the indigenous peoples, characterized them as "uncivilized,"8
"backward people,"9 with "barbarous practices"10 and "a low order of intelligence."11
A Historical Backdrop on the Indigenous Peoples
Drawing inspiration from both our fundamental law and international law, IPRA now employs the
The term "indigenous" traces its origin to the Old Latin word indu, meaning "within." In the sense the politically-correct conjunctive term "indigenous peoples/indigenous cultural communities" as
term has come to be used, it is nearer in meaning to the Latin word indigenus, which means "native."3 follows:
"Indigenous" refers to that which originated or has been produced naturally in a particular land, and
has not been introduced from the outside.4 In international law, the definition of what constitutes Sec. 3. Definition of Terms.- For purposes of this Act, the following terms shall mean:
"indigenous peoples" attains some degree of controversy. No definition of the term "indigenous
xxx

29
(h) Indigenous peoples/Indigenous cultural communities. - refer to a group of people or homogenous The framers of the 1987 Constitution, looking back to the long destitution of our less fortunate
societies identified by self-ascription and ascription by others, who have continuously lived as brothers, fittingly saw the historic opportunity to actualize the ideals of people empowerment and
organized community on communally bounded and defined territory, and who have, under claims of social justice, and to reach out particularly to the marginalized sectors of society, including the
ownership since time immemorial, occupied, possessed and utilized such territories, sharing common indigenous peoples. They incorporated in the fundamental law several provisions recognizing and
bonds of language, customs, traditions, and other distinctive cultural traits, or who have, through protecting the rights and interests of the indigenous peoples, to wit:
resistance to political, social and cultural inroads of colonization, non-indigenous religions and
cultures, became historically differentiated from the majority of Filipinos. Indigenous peoples shall Sec. 22. The State recognizes and promotes the rights of indigenous peoples within the framework of
likewise include peoples who are regarded as indigenous on account of their descent from the national unity and development.17
populations which inhabited the country at the time of conquest or colonization, or at the time of
inroads of non-indigenous religions and cultures, or the establishment of present State boundaries, Sec. 5. The State, subject to the provisions of this Constitution and national development policies and
who retain some or all of their own social, economic, cultural and political institutions, but who may programs, shall protect the rights of indigenous cultural communities to their ancestral lands to
have been displaced from their traditional domains or who may have resettled outside their ancestral ensure their economic, social, and cultural well-being.
domains x x x.
The Congress may provide for the applicability of customary laws governing property rights and
Long before the Spaniards set foot in these islands, the indigenous peoples were already plowing our relations in determining the ownership and extent of ancestral domains.18
soil and hunting in our forests. The Filipinos of Aeta and Malay stock, who were the original
inhabitants of our archipelago, were, at that time, practicing a native culture. From the time the Sec. 1. The Congress shall give the highest priority to the enactment of measures that protect and
Spaniards arrived up to the early part of the American regime,12 these native inhabitants resisted enhance the right of all the people to human dignity, reduce social, economic and political
foreign invasion, relentlessly fighting for their lands. Today, from the remote uplands of Northern inequalities, and remove cultural inequities by equitably diffusing wealth and political power for the
Luzon, to Palawan, Mindoro and Mindanao, the indigenous peoples continue to live on and cultivate common good.
their ancestral lands, the lands of their forefathers.
To this end, the State shall regulate the acquisition, ownership, use and disposition of property and
Though Filipinos today are essentially of the same stock as the indigenous peoples, our national its increments.19
culture exhibits only the last vestiges of this native culture. Centuries of colonial rule and neocolonial
domination have created a discernible distinction between the cultural majority and the group of Sec. 6. The State shall apply the principles of agrarian reform or stewardship, whenever applicable in
cultural minorities.13 The extant Philippine national culture is the culture of the majority; its accordance with law, in the disposition and utilization of other natural resources, including lands of
indigenous roots were replaced by foreign cultural elements that are decidedly pronounced, if not the public domain under lease or concession, subject to prior rights, homestead rights of small
dominant.14 While the culture of the majority reoriented itself to Western influence, the culture of settlers, and the rights of indigenous communities to their ancestral lands.20
the minorities has retained its essentially native character.
Sec. 17. The State shall recognize, respect, and protect the rights of indigenous cultural communities
One of every six Filipinos is a member of an indigenous cultural community. Around twelve million to preserve and develop their cultures, traditions, and institutions. It shall consider these rights in the
Filipinos are members of the one hundred and ten or so indigenous cultural communities,15 formulation of national plans and policies.21
accounting for more than seventeen per centum of the estimated seventy million Filipinos16 in our
country. Sadly, the indigenous peoples are one of the poorest sectors of Philippine society. The Sec. 12. The Congress may create a consultative body to advise the President on policies affecting
incidence of poverty and malnutrition among them is significantly higher than the national average. indigenous cultural communities, the majority of the members of which shall come from such
The indigenous peoples are also among the most powerless. Perhaps because of their inability to communities.22
speak the language of law and power, they have been relegated to the fringes of society. They have
little, if any, voice in national politics and enjoy the least protection from economic exploitation. IPRA was enacted precisely to implement the foregoing constitutional provisions. It provides, among
others, that the State shall recognize and promote the rights of indigenous peoples within the
The Constitutional Policies on Indigenous Peoples framework of national unity and development, protect their rights over the ancestral lands and

30
ancestral domains and recognize the applicability of customary laws governing property rights or B. Petitioners, as citizens and taxpayers, have the requisite standing to raise the constitutional
relations in determining the ownership and extent of the ancestral domains.23 Moreover, IPRA questions herein.
enumerates the civil and political rights of the indigenous peoples;24 spells out their social and
cultural rights;25 acknowledges a general concept of indigenous property right and recognizes title In addition to the existence of an actual case or controversy, a person who assails the validity of a
thereto;26 and creates the NCIP as an independent agency under the Office of the President.27 statute must have a personal and substantial interest in the case, such that, he has sustained, or will
sustain, a direct injury as a result of its enforcement.35 Evidently, the rights asserted by petitioners
Preliminary Issues as citizens and taxpayers are held in common by all the citizens, the violation of which may result only
in a "generalized grievance".36 Yet, in a sense, all citizen’s and taxpayer’s suits are efforts to air
A. The petition presents an actual controversy. generalized grievances about the conduct of government and the allocation of power.37

The time-tested standards for the exercise of judicial review are: (1) the existence of an appropriate In several cases, the Court has adopted a liberal attitude with regard to standing.38 The proper party
case; (2) an interest personal and substantial by the party raising the constitutional question; (3) the requirement is considered as merely procedural,39 and the Court has ample discretion with regard
plea that the function be exercised at the earliest opportunity; and (4) the necessity that the thereto.40 As early as 1910, the Court in the case of Severino vs. Governor General 41 held:
constitutional question be passed upon in order to decide the case.28
x x x When the relief is sought merely for the protection of private rights, the relator must show some
Courts can only decide actual controversies, not hypothetical questions or cases.29 The threshold personal or special interest in the subject matter, since he is regarded as the real party in interest and
issue, therefore, is whether an "appropriate case" exists for the exercise of judicial review in the his right must clearly appear. Upon the other hand, when the question is one of public right and the
present case. object of the mandamus is to procure the enforcement of a public duty, the people are regarded as
the real party in interest, and the relator at whose instigation the proceedings are instituted need not
An "actual case or controversy" means an existing case or controversy which is both ripe for resolution show that he has any legal or special interest in the result, it being sufficient to show that he is a
and susceptible of judicial determination, and that which is not conjectural or anticipatory,30 or that citizen and as such interested in the execution of the laws.42
which seeks to resolve hypothetical or feigned constitutional problems.31 A petition raising a
constitutional question does not present an "actual controversy," unless it alleges a legal right or This Court has recognized that a "public right," or that which belongs to the people at large, may also
power. Moreover, it must show that a conflict of rights exists, for inherent in the term "controversy" be the subject of an actual case or controversy. In Severino, we ruled that a private citizen may
is the presence of opposing views or contentions.32 Otherwise, the Court will be forced to resolve enforce a "public right" in behalf of other citizens. We opined therein that:
issues which remain unfocused because they lack such concreteness provided when a question
emerges precisely framed from a clash of adversary arguments exploring every aspect of a multi- … The right which [petitioner] seeks to enforce is not greater or different from that of any other
faceted situation embracing conflicting and demanding interests.33 The controversy must also be qualified elector in the municipality of Silay. It is also true that the injury which he would suffer in
justiciable; that is, it must be susceptible of judicial determination.34 case he fails to obtain the relief sought would not be greater or different from that of the other
electors; but he is seeking to enforce a public right as distinguished from a private right. The real party
In the case at bar, there exists a live controversy involving a clash of legal rights. A law has been in interest is the public, or the qualified electors of the town of Silay. Each elector has the same right
enacted, and the Implementing Rules and Regulations approved. Money has been appropriated and and would suffer the same injury. Each elector stands on the same basis with reference to maintaining
the government agencies concerned have been directed to implement the statute. It cannot be a petition whether or not the relief sought by the relator should be granted.43
successfully maintained that we should await the adverse consequences of the law in order to
consider the controversy actual and ripe for judicial resolution. It is precisely the contention of the In Tañada v. Tuvera,44 the Court enforced the "public right" to due process and to be informed of
petitioners that the law, on its face, constitutes an unconstitutional abdication of State ownership matters of public concern.
over lands of the public domain and other natural resources. Moreover, when the State machinery is
set into motion to implement an alleged unconstitutional statute, this Court possesses sufficient In Garcia vs. Board of Investments,45 the Court upheld the "public right" to be heard or consulted on
authority to resolve and prevent imminent injury and violation of the constitutional process. matters of national concern.

31
In Oposa v. Factoran,46 the Court recognized the "public right" of citizens to "a balanced and healthful entity’s or person’s jurisdiction, or are accompanied with grave abuse of discretion, and there is no
ecology which, for the first time in our nation’s constitutional history, is solemnly incorporated in the appeal or any other plain, speedy and adequate remedy in the ordinary course of law.54 Mandamus,
fundamental law."47 Mr. Justice (now Chief Justice) Hilario G. Davide, Jr., delivering the opinion of on the other hand, is an extraordinary writ commanding a tribunal, corporation, board, officer or
the Court, stated that: person, immediately or at some other specified time, to do the act required to be done, when said
entity or person unlawfully neglects the performance of an act which the law specifically enjoins as a
Such a right belongs to a different category of rights altogether for it concerns nothing less than self- duty resulting from an office, trust or station, or when said entity or person unlawfully excludes
preservation and self-perpetuation-aptly and fittingly stressed by petitioners-the advancement of another from the use and enjoyment of a right or office to which such other is entitled, and there is
which may even be said to predate all governments and constitutions. As a matter of fact, these basic no other plain, speedy and adequate remedy in the ordinary course of law.55
rights need not even be written in the Constitution for they are assumed to exist from the inception
of humankind.48 In this case, the petitioners pray that respondents be restrained from implementing the challenged
provisions of the IPRA and its Implementing Rules and the assailed DENR Circular No. 2, series of 1998,
Petitioners, as citizens, possess the "public right" to ensure that the national patrimony is not and that the same officials be enjoined from disbursing public funds for the implementation of the
alienated and diminished in violation of the Constitution. Since the government, as the guardian of said law and rules. They further ask that the Secretary of the DENR be compelled to perform his duty
the national patrimony, holds it for the benefit of all Filipinos without distinction as to ethnicity, it to control and supervise the activities pertaining to natural resources.
follows that a citizen has sufficient interest to maintain a suit to ensure that any grant of concessions
covering the national economy and patrimony strictly complies with constitutional requirements. Prohibition will lie to restrain the public officials concerned from implementing the questioned
Thus, the preservation of the integrity and inviolability of the national patrimony is a proper subject provisions of the IPRA and from disbursing funds in connection therewith if the law is found to be
of a citizen’s suit. unconstitutional. Likewise, mandamus will lie to compel the Secretary of the DENR to perform his
duty to control and supervise the exploration, development, utilization and conservation of the
In addition, petitioners, as taxpayers, possess the right to restrain officials from wasting public funds country’s natural resources. Consequently, the petition for prohibition and mandamus is not an
through the enforcement of an unconstitutional statute. It is well-settled that a taxpayer has the right improper remedy for the relief sought.
to enjoin public officials from wasting public funds through the implementation of an unconstitutional
statute,49 and by necessity, he may assail the validity of a statute appropriating public funds.50 The D. Notwithstanding the failure of petitioners to observe the hierarchy of courts, the Court assumes
taxpayer has paid his taxes and contributed to the public coffers and, thus, may inquire into the jurisdiction over the petition in view of the importance of the issues raised therein.
manner by which the proceeds of his taxes are spent. The expenditure by an official of the State for
the purpose of administering an invalid law constitutes a misapplication of such funds.51 Between two courts of concurrent original jurisdiction, it is the lower court that should initially pass
upon the issues of a case. That way, as a particular case goes through the hierarchy of courts, it is
The IPRA appropriates funds as indicated in its title: "An Act to Recognize, Protect and Promote the shorn of all but the important legal issues or those of first impression, which are the proper subject
Rights of Indigenous Cultural Communities/Indigenous Peoples, Creating the National Commission on of attention of the appellate court. This is a procedural rule borne of experience and adopted to
Indigenous Peoples, Establishing Implementing Mechanisms, Appropriating Funds Therefor, and for improve the administration of justice.
Other Purposes." In the same manner, Section 79 authorizes for the expenditure of public funds by
providing that "the amount necessary to finance [its] initial implementation shall be charged against This Court has consistently enjoined litigants to respect the hierarchy of courts. Although this Court
the current year's appropriation for the Office for Northern Cultural Communities (the "ONCC") and has concurrent jurisdiction with the Regional Trial Courts and the Court of Appeals to issue writs of
the Office for Southern Cultural Communities (the "OSCC"),"52 which were merged as organic offices certiorari, prohibition, mandamus, quo warranto, habeas corpus and injunction,56 such concurrence
of the NCIP.53 Thus, the IPRA is a valid subject of a taxpayer’s suit. does not give a party unrestricted freedom of choice of court forum. The resort to this Court’s primary
jurisdiction to issue said writs shall be allowed only where the redress desired cannot be obtained in
C. The petition for prohibition and mandamus is not an improper remedy. the appropriate courts or where exceptional and compelling circumstances justify such invocation.57
We held in People v. Cuaresma58 that:
Prohibition is an extraordinary writ directed against any tribunal, corporation, board, officer or
person, whether exercising judicial, quasi-judicial or ministerial functions, ordering said entity or A becoming regard for judicial hierarchy most certainly indicates that petitions for the issuance of
person to desist from further proceedings when said proceedings are without or in excess of said extraordinary writs against first level ("inferior") courts should be filed with the Regional Trial Court,

32
and those against the latter, with the Court of Appeals. A direct invocation of the Supreme Court’s The President may enter into agreements with foreign-owned corporations involving either technical
original jurisdiction to issue these writs should be allowed only where there are special and important or financial assistance for large-scale exploration, development and utilization of minerals,
reasons therefor, clearly and specifically set out in the petition. This is established policy. It is a policy petroleum, and other mineral oils according to the general terms and conditions provided by law,
necessary to prevent inordinate demands upon the Court’s time and attention which are better based on real contributions to the economic growth and general welfare of the country. In such
devoted to those matters within its exclusive jurisdiction, and to prevent further over-crowding of agreements, the State shall promote the development and use of local scientific and technical
the Court’s docket x x x.59 (Emphasis supplied.) resources.

IPRA aims to rectify the historical injustice inflicted upon indigenous peoples. Its impact upon the lives The President shall notify the Congress of every contract entered into in accordance with this
not only of the indigenous peoples but also upon the lives of all Filipinos cannot be denied. The provision, within thirty days from its execution.
resolution of this case by the Court at the earliest opportunity is necessary if the aims of the law are
to be achieved. This reason is compelling enough to allow petitioners’ invocation of this Court’s Under IPRA, indigenous peoples may obtain the recognition of their right of ownership60 over
jurisdiction in the first instance. ancestral lands and ancestral domains by virtue of native title.61 The term "ancestral lands" under
the statute refers to lands occupied by individuals, families and clans who are members of indigenous
Substantive Issues cultural communities, including residential lots, rice terraces or paddies, private forests, swidden
farms and tree lots. These lands are required to have been "occupied, possessed and utilized" by them
Primary Issue or through their ancestors "since time immemorial, continuously to the present".62 On the other
hand, "ancestral domains" is defined as areas generally belonging to indigenous cultural communities,
The issue of prime concern raised by petitioners and the Solicitor General revolves around the including ancestral lands, forests, pasture, residential and agricultural lands, hunting grounds,
constitutionality of certain provisions of IPRA, specifically Sections 3(a), 3(b), 5, 6, 7, 8, 57, 58 and 59. worship areas, and lands no longer occupied exclusively by indigenous cultural communities but to
These provisions allegedly violate Section 2, Article XII of the Constitution, which states: which they had traditional access, particularly the home ranges of indigenous cultural communities
who are still nomadic or shifting cultivators. Ancestral domains also include inland waters, coastal
Sec. 2. All lands of the public domain, waters, minerals, coal, petroleum, and other mineral oils, all areas and natural resources therein.63 Again, the same are required to have been "held under a claim
forces of potential energy, fisheries, forests or timber, wildlife, flora and fauna, and other natural of ownership, occupied or possessed by ICCs/IPs, by themselves or through their ancestors,
resources are owned by the State. With the exception of agricultural lands, all other natural resources communally or individually since time immemorial, continuously to the present".64 Under Section
shall not be alienated. The exploration, development, and utilization of natural resources shall be 56, property rights within the ancestral domains already existing and/or vested upon effectivity of
under the full control and supervision of the State. The State may directly undertake such activities, said law "shall be recognized and respected."
or it may enter into co-production, joint venture, or production-sharing agreements with Filipino
citizens, or corporations or associations at least sixty per centum of whose capital is owned by such Ownership is the crux of the issue of whether the provisions of IPRA pertaining to ancestral lands,
citizens. Such agreements may be for a period not exceeding twenty-five years, renewable for not ancestral domains, and natural resources are unconstitutional. The fundamental question is, who,
more than twenty-five years, and under such terms and conditions as may be provided by law. In between the State and the indigenous peoples, are the rightful owners of these properties?
cases of water rights for irrigation, water supply, fisheries, or industrial uses other than the
development of water power, beneficial use may be the measure and limit of the grant. It bears stressing that a statute should be construed in harmony with, and not in violation, of the
fundamental law.65 The reason is that the legislature, in enacting a statute, is assumed to have acted
The State shall protect the nation’s marine wealth in its archipelagic waters, territorial sea, and within its authority and adhered to the constitutional limitations. Accordingly, courts should presume
exclusive economic zone, and reserve its use and enjoyment exclusively to Filipino citizens. that it was the intention of the legislature to enact a valid, sensible, and just law and one which
operates no further than may be necessary to effectuate the specific purpose of the law.66
The Congress, may, by law, allow small-scale utilization of natural resources by Filipino citizens, as
well as cooperative fish farming, with priority to subsistence fishermen and fishworkers in rivers, A. The provisions of IPRA recognizing the ownership of indigenous peoples over the ancestral lands
lakes, bays and lagoons. and ancestral domains are not unconstitutional.

33
In support of their theory that ancestral lands and ancestral domains are part of the public domain almost with reverence, as the immutable postulate of Philippine land law. It has been incorporated
and, thus, owned by the State, pursuant to Section 2, Article XII of the Constitution, petitioners and into our fundamental law and has been recognized by the Court.67
the Solicitor General advance the following arguments:
Generally, under the concept of jura regalia, private title to land must be traced to some grant,
First, according to petitioners, the King of Spain under international law acquired exclusive dominion express or implied, from the Spanish Crown or its successors, the American Colonial government, and
over the Philippines by virtue of discovery and conquest. They contend that the Spanish King under thereafter, the Philippine Republic. The belief that the Spanish Crown is the origin of all land titles in
the theory of jura regalia, which was introduced into Philippine law upon Spanish conquest in 1521, the Philippines has persisted because title to land must emanate from some source for it cannot issue
acquired title to all the lands in the archipelago. forth from nowhere.68

Second, petitioners and the Solicitor General submit that ancestral lands and ancestral domains are In its broad sense, the term "jura regalia" refers to royal rights,69 or those rights which the King has
owned by the State. They invoke the theory of jura regalia which imputes to the State the ownership by virtue of his prerogatives.70 In Spanish law, it refers to a right which the sovereign has over
of all lands and makes the State the original source of all private titles. They argue that the Philippine anything in which a subject has a right of property or propriedad.71 These were rights enjoyed during
State, as successor to Spain and the United States, is the source of any asserted right of ownership in feudal times by the king as the sovereign.
land.
The theory of the feudal system was that title to all lands was originally held by the King, and while
Third, petitioners and the Solicitor General concede that the Cariño doctrine exists. However, the use of lands was granted out to others who were permitted to hold them under certain conditions,
petitioners maintain that the doctrine merely states that title to lands of the public domain may be the King theoretically retained the title.72 By fiction of law, the King was regarded as the original
acquired by prescription. The Solicitor General, for his part, argues that the doctrine applies only to proprietor of all lands, and the true and only source of title, and from him all lands were held.73 The
alienable lands of the public domain and, thus, cannot be extended to other lands of the public theory of jura regalia was therefore nothing more than a natural fruit of conquest.74
domain such as forest or timber, mineral lands, and national parks.
The Regalian theory, however, does not negate native title to lands held in private ownership since
Fourth, the Solicitor General asserts that even assuming that native title over ancestral lands and time immemorial. In the landmark case of Cariño vs. Insular Government75 the United States
ancestral domains existed by virtue of the Cariño doctrine, such native title was extinguished upon Supreme Court, reversing the decision76of the pre-war Philippine Supreme Court, made the following
the ratification of the 1935 Constitution. pronouncement:

Fifth, petitioners admit that Congress is mandated under Section 5, Article XII of the Constitution to x x x Every presumption is and ought to be taken against the Government in a case like the present.
protect that rights of indigenous peoples to their ancestral lands and ancestral domains. However, It might, perhaps, be proper and sufficient to say that when, as far back as testimony or memory goes,
they contend that the mandate is subject to Section 2, Article XII and the theory of jura regalia the land has been held by individuals under a claim of private ownership, it will be presumed to have
embodied therein. According to petitioners, the recognition and protection under R.A. 8371 of the been held in the same way from before the Spanish conquest, and never to have been public land. x
right of ownership over ancestral lands and ancestral domains is far in excess of the legislative power x x.77 (Emphasis supplied.)
and constitutional mandate of Congress.
The above ruling institutionalized the recognition of the existence of native title to land, or ownership
Finally, on the premise that ancestral lands and ancestral domains are owned by the State, petitioners of land by Filipinos by virtue of possession under a claim of ownership since time immemorial and
posit that R.A. 8371 violates Section 2, Article XII of the Constitution which prohibits the alienation of independent of any grant from the Spanish Crown, as an exception to the theory of jura regalia.
non-agricultural lands of the public domain and other natural resources.
In Cariño, an Igorot by the name of Mateo Cariño applied for registration in his name of an ancestral
I am not persuaded by these contentions. land located in Benguet. The applicant established that he and his ancestors had lived on the land,
had cultivated it, and had used it as far they could remember. He also proved that they had all been
Undue reliance by petitioners and the Solicitor General on the theory of jura regalia is recognized as owners, the land having been passed on by inheritance according to native custom.
understandable. Not only is the theory well recognized in our legal system; it has been regarded, However, neither he nor his ancestors had any document of title from the Spanish Crown. The
government opposed the application for registration, invoking the theory of jura regalia. On appeal,

34
the United States Supreme Court held that the applicant was entitled to the registration of his native the Cariño doctrine because the Regalian doctrine which vests in the State ownership of lands of the
title to their ancestral land. public domain does not cover ancestral lands and ancestral domains.

Cariño was decided by the U.S. Supreme Court in 1909, at a time when decisions of the U.S. Court Legal history supports the Cariño doctrine.
were binding as precedent in our jurisdiction.78 We applied the Cariño doctrine in the 1946 case of
Oh Cho vs. Director of Lands,79 where we stated that "[a]ll lands that were not acquired from the When Spain acquired sovereignty over the Philippines by virtue of its discovery and occupation
Government either by purchase or by grant, belong to the public domain, but [a]n exception to the thereof in the 16th century and the Treaty of Tordesillas of 1494 which it entered into with
rule would be any land that should have been in the possession of an occupant and of his predecessors Portugal,83 the continents of Asia, the Americas and Africa were considered as terra nullius although
in interest since time immemorial, for such possession would justify the presumption that the land already populated by other peoples.84 The discovery and occupation by the European States, who
had never been part of the public domain or that it had been private property even before the Spanish were then considered as the only members of the international community of civilized nations, of
conquest."80 lands in the said continents were deemed sufficient to create title under international law.85

Petitioners however aver that the U.S. Supreme Court’s ruling in Cariño was premised on the fact that Although Spain was deemed to have acquired sovereignty over the Philippines, this did not mean that
the applicant had complied with the requisites of acquisitive prescription, having established that he it acquired title to all lands in the archipelago. By virtue of the colonial laws of Spain, the Spanish
and his predecessors-in-interest had been in possession of the property since time immemorial. In Crown was considered to have acquired dominion only over the unoccupied and unclaimed portions
effect, petitioners suggest that title to the ancestral land applied for by Cariño was transferred from of our islands.86
the State, as original owner, to Cariño by virtue of prescription. They conclude that the doctrine
cannot be the basis for decreeing "by mere legislative fiat…that ownership of vast tracts of land In sending the first expedition to the Philippines, Spain did not intend to deprive the natives of their
belongs to [indigenous peoples] without judicial confirmation."81 property. Miguel Lopez de Legazpi was under instruction of the Spanish King to do no harm to the
natives and to their property. In this regard, an authority on the early Spanish colonial period in the
The Solicitor General, for his part, claims that the Cariño doctrine applies only to alienable lands of Philippines wrote:
the public domain and, as such, cannot be extended to other lands of the public domain such as forest
or timber, mineral lands, and national parks. The government of [the King of Spain] Philip II regarded the Philippines as a challenging opportunity
to avoid a repetition of the sanguinary conquests of Mexico and Peru. In his written instructions for
There is no merit in these contentions. the Adelantado Legazpi, who commanded the expedition, Philip II envisaged a bloodless pacification
of the archipelago. This extraordinary document could have been lifted almost verbatim from the
A proper reading of Cariño would show that the doctrine enunciated therein applies only to lands lectures of the Dominican theologian, Francisco de Vitoria, delivered in the University of Salamanca.
which have always been considered as private, and not to lands of the public domain, whether The King instructed Legazpi to inform the natives that the Spaniards had come to do no harm to their
alienable or otherwise. A distinction must be made between ownership of land under native title and persons or to their property. The Spaniards intended to live among them in peace and in friendship
ownership by acquisitive prescription against the State. Ownership by virtue of native title and "to explain to them the law of Jesus Christ by which they will be saved." Although the Spanish
presupposes that the land has been held by its possessor and his predecessors-in-interest in the expedition could defend themselves if attacked, the royal instructions admonished the commander
concept of an owner since time immemorial. The land is not acquired from the State, that is, Spain or to commit no aggressive act which might arouse native hostility.87
its successors-in-interest, the United States and the Philippine Government. There has been no
transfer of title from the State as the land has been regarded as private in character as far back as Spanish colonial laws recognized and respected Filipino landholdings including native land
memory goes. In contrast, ownership of land by acquisitive prescription against the State involves a occupancy.88 Thus, the Recopilación de Leyes de las Indias expressly conferred ownership of lands
conversion of the character of the property from alienable public land to private land, which already held by the natives.89 The royal decrees of 1880 and 1894 did not extinguish native title to
presupposes a transfer of title from the State to a private person. Since native title assumes that the land in the Philippines. The earlier royal decree, dated June 25, 1880, provided that all those in
property covered by it is private land and is deemed never to have been part of the public domain, "unlawful possession of royal lands" must legalize their possession by means of adjustment
the Solicitor General’s thesis that native title under Cariño applies only to lands of the public domain proceedings,90 and within the period specified. The later royal decree, dated February 13, 1894,
is erroneous. Consequently, the classification of lands of the public domain into agricultural, forest or otherwise known as the Maura Law, declared that titles that were capable of adjustment under the
timber, mineral lands, and national parks under the Constitution82 is irrelevant to the application of royal decree of 1880, but for which adjustment was not sought, were forfeited. Despite the harsh

35
wording of the Maura Law, it was held in the case of Cariño that the royal decree of 1894 should not vested rights, he reasons out that even vested rights of ownership over ancestral lands and ancestral
be construed as confiscation of title, but merely as the withdrawal of the privilege of registering such domains are not absolute and may be impaired by the legitimate exercise of police power.
title.91
I cannot agree. The text of the provision of the 1935 Constitution invoked by the Solicitor General,
Neither was native title disturbed by the Spanish cession of the Philippines to the United States, while embodying the theory of jura regalia, is too clear for any misunderstanding. It simply declares
contrary to petitioners’ assertion that the US merely succeeded to the rights of Spain, including the that "all agricultural, timber, and mineral lands of the public domain, waters, minerals, coal,
latter’s rights over lands of the public domain.92 Under the Treaty of Paris of December 10, 1898, the petroleum, and other mineral oils, all forces of potential energy, and other natural resources of the
cession of the Philippines did not impair any right to property existing at the time.93 During the Philippines belong to the State."99 Nowhere does it state that certain lands which are "absolutely
American colonial regime, native title to land was respected, even protected. The Philippine Bill of necessary for social welfare and existence," including those which are not part of the public domain,
1902 provided that property and rights acquired by the US through cession from Spain were to be shall thereafter be owned by the State. If there is any room for constitutional construction, the
administered for the benefit of the Filipinos.94 In obvious adherence to libertarian principles, provision should be interpreted in favor of the preservation, rather than impairment or
McKinley’s Instructions, as well as the Philippine Bill of 1902, contained a bill of rights embodying the extinguishment, of vested rights. Stated otherwise, Section 1, Article XII of the 1935 Constitution
safeguards of the US Constitution. One of these rights, which served as an inviolable rule upon every cannot be construed to mean that vested right which had existed then were extinguished and that
division and branch of the American colonial government in the Philippines,95 was that "no person the landowners were divested of their lands, all in the guise of "wrest[ing] control of those portions
shall be deprived of life, liberty, or property without due process of law."96 These vested rights of the natural resources [which the State] deems absolutely necessary for social welfare and
safeguarded by the Philippine Bill of 1902 were in turn expressly protected by the due process clause existence." On the contrary, said Section restated the fundamental rule against the diminution of
of the 1935 Constitution. Resultantly, property rights of the indigenous peoples over their ancestral existing rights by expressly providing that the ownership of lands of the public domain and other
lands and ancestral domains were firmly established in law. natural resources by the State is "subject to any existing right, grant, lease, or concessions." The
"existing rights" that were intended to be protected must, perforce, include the right of ownership
Nonetheless, the Solicitor General takes the view that the vested rights of indigenous peoples to their by indigenous peoples over their ancestral lands and domains. The words of the law should be given
ancestral lands and domains were "abated by the direct act by the sovereign Filipino people of their ordinary or usual meaning,100 and the term "existing rights" cannot be assigned an unduly
ratifying the 1935 Constitution."97 He advances the following arguments: restrictive definition.

The Sovereign, which is the source of all rights including ownership, has the power to restructure the Petitioners concede that Congress is mandated under Section 5, Article XII of the 1987
consolidation of rights inherent in ownership in the State. Through the mandate of the Constitutions Constitution101to protect the rights of indigenous peoples to their ancestral lands and ancestral
that have been adopted, the State has wrested control of those portions of the natural resources it domains. Nonetheless, they contend that the recognition and protection under IPRA of the right of
deems absolutely necessary for social welfare and existence. It has been held that the State may ownership of indigenous peoples over ancestral lands and ancestral domains are far in excess of the
impair vested rights through a legitimate exercise of police power. legislative power and constitutional mandate of the Congress,102 since such recognition and
protection amount to the alienation of lands of the public domain, which is proscribed under Section
Vested rights do not prohibit the Sovereign from performing acts not only essential to but 2, Article XII of the Constitution.
determinative of social welfare and existence. To allow otherwise is to invite havoc in the established
social system. x x x Section 5, Article XII of the Constitution expresses the sovereign intent to "protect the rights of
indigenous peoples to their ancestral lands." In its general and ordinary sense, the term "right" refers
Time-immemorial possession does not create private ownership in cases of natural resources that to any legally enforceable claim.103 It is a power, privilege, faculty or demand inherent in one person
have been found from generation to generation to be critical to the survival of the Sovereign and its and incident upon another.104 When used in relation to property, "right" includes any interest in or
agent, the State.98 title to an object, or any just and legal claim to hold, use and enjoy it.105 Said provision in the
Constitution cannot, by any reasonable construction, be interpreted to exclude the protection of the
Stated simply, the Solicitor General’s argument is that the State, as the source of all titles to land, had right of ownership over such ancestral lands. For this reason, Congress cannot be said to have
the power to re-vest in itself, through the 1935 Constitution, title to all lands, including ancestral lands exceeded its constitutional mandate and power in enacting the provisions of IPRA, specifically
and ancestral domains. While the Solicitor General admits that such a theory would necessarily impair Sections 7(a) and 8, which recognize the right of ownership of the indigenous peoples over ancestral
lands.

36
There is a range of customary laws governing certain types of ownership. There would be ownership
The second paragraph of Section 5, Article XII also grants Congress the power to "provide for the based on individuals, on clan or lineage, or on community. And the thinking expressed in the
applicability of customary laws governing property rights or relations in determining the ownership consultation is that this should be codified and should be recognized in relation to existing national
and extent of ancestral domains." In light of this provision, does Congress have the power to decide laws. That is essentially the concept. 106 (Emphasis supplied.)
whether ancestral domains shall be private property or part of the public domain? Also, does
Congress have the power to determine whether the "extent" of ancestral domains shall include the The intention to treat ancestral domains as private property is also apparent from the following
natural resources found therein? exchange between Messrs. Suarez and Bennagen:

It is readily apparent from the constitutional records that the framers of the Constitution did not MR. SUAREZ. When we speak of customary laws governing property rights or relations in determining
intend Congress to decide whether ancestral domains shall be public or private property. Rather, they the ownership and extent of the ancestral domain, are we thinking in terms of the tribal ownership
acknowledged that ancestral domains shall be treated as private property, and that customary laws or community ownership or of private ownership within the ancestral lands or ancestral domain?
shall merely determine whether such private ownership is by the entire indigenous cultural
community, or by individuals, families, or clans within the community. The discussion below between MR. BENNAGEN. The concept of customary laws is that it is considered as ownership by private
Messrs. Regalado and Bennagen and Mr. Chief Justice Davide, then members of the 1986 individuals, clans and even communities.
Constitutional Commission, is instructive:
MR. SUAREZ. So, there will be two aspects to this situation. This means that the State will set aside
MR. REGALADO. Thank you, Madame President. May I seek some clarifications from either the ancestral domain and there is a separate law for that. Within the ancestral domain it could accept
Commissioner Bennagen or Commissioner Davide regarding this phrase "CONGRESS SHALL PROVIDE more specific ownership in terms of individuals within the ancestral lands.
FOR THE APPLICABILITY OF CUSTOMARY LAWS GOVERNING PROPERTY RIGHTS OR RELATIONS in
determining the ownership and extent of the ancestral domain," because ordinarily it is the law on MR. BENNAGEN. Individuals and groups within the ancestral domain. 107 (Emphasis supplied.)
ownership and the extent thereof which determine the property rights or relations arising therefrom.
On the other hand, in this proposed amendment the phraseology is that it is the property rights or It cannot be correctly argued that, because the framers of the Constitution never expressly mentioned
relations which shall be used as the basis in determining the ownership and extent of the ancestral Cariño in their deliberations, they did not intend to adopt the concept of native title to land, or that
domain. I assume there must be a certain difference in the customary laws and our regular civil laws they were unaware of native title as an exception to the theory of jura regalia.108 The framers of the
on property. Constitution, as well as the people adopting it, were presumed to be aware of the prevailing judicial
doctrines concerning the subject of constitutional provisions, and courts should take these doctrines
MR. DAVIDE. That is exactly the reason, Madam President, why we will leave it to Congress to make into consideration in construing the Constitution.109
the necessary exception to the general law on property relations.
Having thus recognized that ancestral domains under the Constitution are considered as private
MR. REGALADO. I was thinking if Commissioner Bennagen could give us an example of such a property of indigenous peoples, the IPRA, by affirming or acknowledging such ownership through its
customary law wherein it is the property rights and relations that determine the ownership and the various provisions, merely abides by the constitutional mandate and does not suffer any vice of
extent of that ownership, unlike the basic fundamental rule that it is the ownership and the extent of unconstitutionality.
ownership which determine the property rights and relations arising therefrom and consequent
thereto. Perhaps, these customary laws may have a different provision or thrust so that we could Petitioners interpret the phrase "subject to the provisions of this Constitution and national
make the corresponding suggestions also by way of an amendment. development policies and programs" in Section 5, Article XII of the Constitution to mean "as subject
to the provision of Section 2, Article XII of the Constitution," which vests in the State ownership of all
MR. DAVIDE. That is exactly my own perception. lands of the public domain, mineral lands and other natural resources. Following this interpretation,
petitioners maintain that ancestral lands and ancestral domains are the property of the State.
MR. BENNAGEN. Let me put it this way.
This proposition is untenable. Indeed, Section 2, Article XII reiterates the declarations made in the
1935 and 1973 Constitutions on the state policy of conservation and nationalization of lands of the

37
public domain and natural resources, and is of paramount importance to our national economy and Thus, the provisions of the Constitution on State ownership of public lands, mineral lands and other
patrimony. A close perusal of the records of the 1986 Constitutional Commission reveals that the natural resources should be read together with the other provisions thereof which firmly recognize
framers of the Constitution inserted the phrase "subject to the provisions of this Constitution" mainly the rights of the indigenous peoples. These, as set forth hereinbefore,112 include: Section 22, Article
to prevent the impairment of Torrens titles and other prior rights in the determination of what II, providing that the State recognizes and promotes the rights of indigenous peoples within the
constitutes ancestral lands and ancestral domains, to wit: framework of national unity and development; Section 5, Article XII, calling for the protection of the
rights of indigenous cultural communities to their ancestral lands to ensure their economic, social,
MR. NATIVIDAD. Just one question. I want to clear this section protecting ancestral lands. How does and cultural well-being, and for the applicability of customary laws governing property rights and
this affect the Torrens title and other prior rights? relations in determining the ownership and extent of ancestral domains; Section 1, Article XIII,
directing the removal or reduction of social, economic, political and cultural inequities and
MR. BENNAGEN. I think that was also discussed in the committee hearings and we did say that in inequalities by equitably diffusing wealth and political power for the common good; Section 6, Article
cases where due process is clearly established in terms of prior rights, these two have to be respected. XIII, directing the application of the principles of agrarian reform or stewardship in the disposition and
utilization of other natural resources, subject to prior rights, homestead rights of small settlers, and
MR. NATIVIDAD. The other point is: How vast is this ancestral land? Is it true that parts of Baguio City the rights of indigenous communities to their ancestral lands; Section 17, Article XIV, decreeing that
are considered as ancestral lands? the State shall recognize, respect, and protect the rights of indigenous cultural communities to
preserve and develop their cultures, traditions, and institutions; and Section 12, Article XVI,
MR. BENNAGEN. They could be regarded as such. If the Commissioner still recalls, in one of the authorizing the Congress to create a consultative body to advise the President on policies affecting
publications that I provided the Commissioners, the parts could be considered as ancestral domain in indigenous cultural communities.
relation to the whole population of Cordillera but not in relation to certain individuals or certain
groups. Again, as articulated in the Constitution, the first goal of the national economy is the more equitable
distribution of opportunities, income, and wealth.113 Equity is given prominence as the first objective
MR. NATIVIDAD. The Commissioner means that the whole Baguio City is considered as ancestral land? of national economic development.114 The framers of the Constitution did not, by the phrase
"subject to the provisions of this Constitution and national development policies and programs,"
MR. BENNAGEN. Yes, in the sense that it belongs to Cordillera or in the same manner that Filipinos intend to establish a hierarchy of constitutional norms. As explained by then Commissioner (now
can speak of the Philippine archipelago as ancestral land, but not in terms of the right of a particular Chief Justice) Hilario G. Davide, Jr., it was not their objective to make certain interests primary or
person or particular group to exploit, utilize, or sell it. paramount, or to create absolute limitations or outright prohibitions; rather, the idea is towards the
balancing of interests:
MR. NATIVIDAD. But is clear that the prior rights will be respected.
BISHOP BACANI. In Commissioner Davide’s formulation of the first sentence, he says: "The State,
MR. BENNAGEN. Definitely. 110 SUBJECT TO THE provisions of this Constitution AND NATIONAL DEVELOPMENT POLICIES AND
PROGRAMS shall guarantee the rights of cultural or tribal communities to their ancestral lands to
Thus, the phrase "subject to the provisions of this Constitution" was intended by the framers of the insure their economic, social and cultural well-being." There are at least two concepts here which
Constitution as a reiteration of the constitutional guarantee that no person shall be deprived of receive different weights very often. They are the concepts of national development policies and
property without due process of law. programs, and the rights of cultural or tribal communities to their ancestral lands, et cetera. I would
like to ask: When the Commissioner proposed this amendment, which was the controlling concept? I
There is another reason why Section 5 of Article XII mandating the protection of rights of the ask this because sometimes the rights of cultural minorities are precisely transgressed in the interest
indigenous peoples to their ancestral lands cannot be construed as subject to Section 2 of the same of national development policies and programs. Hence, I would like to know which is the controlling
Article ascribing ownership of all public lands to the State. The Constitution must be construed as a concept here. Is it the rights of indigenous peoples to their ancestral lands or is it national
whole. It is a rule that when construction is proper, the whole Constitution is examined in order to development policies and programs.
determine the meaning of any provision. That construction should be used which would give effect
to the entire instrument.111 MR. DAVIDE. It is not really a question of which is primary or which is more paramount. The concept
introduced here is really the balancing of interests. That is what we seek to attain. We have to balance

38
the interests taking into account the specific needs and the specific interests also of these cultural the possession the thing owned.123 In contrast, the indigenous peoples’ concept of ownership
communities in like manner that we did so in the autonomous regions.115 (Emphasis supplied.) emphasizes the importance of communal or group ownership. By virtue of the communal character
of ownership, the property held in common "cannot be sold, disposed or destroyed"124 because it
B. The provisions of R.A. 8371 do not infringe upon the State’s ownership over the natural resources was meant to benefit the whole indigenous community and not merely the individual member.125
within the ancestral domains.
That IPRA is not intended to bestow ownership over natural resources to the indigenous peoples is
Petitioners posit that IPRA deprives the State of its ownership over mineral lands of the public domain also clear from the deliberations of the bicameral conference committee on Section 7 which recites
and other natural resources,116 as well as the State’s full control and supervision over the the rights of indigenous peoples over their ancestral domains, to wit:
exploration, development and utilization of natural resources.117 Specifically, petitioners and the
Solicitor General assail Sections 3 (a),118 5,119 and 7120 of IPRA as violative of Section 2, Article XII CHAIRMAN FLAVIER. Accepted. Section 8126 rights to ancestral domain, this is where we transferred
of the Constitution which states, in part, that "[a]ll lands of the public domain, waters, minerals, coal, the other provision but here itself -
petroleum, and other mineral oils, all forces of potential energy, fisheries, forests or timber, wildlife,
flora and fauna, and other natural resources are owned by the State."121 They would have the Court HON. DOMINGUEZ. Mr. Chairman, if I maybe allowed to make a very short Statement. Earlier, Mr.
declare as unconstitutional Section 3(a) of IPRA because the inclusion of natural resources in the Chairman, we have decided to remove the provisions on natural resources because we all agree that
definition of ancestral domains purportedly results in the abdication of State ownership over these that belongs to the State. Now, the plight or the rights of those indigenous communities living in
resources. forest and areas where it could be exploited by mining, by dams, so can we not also provide a
provision to give little protection or either rights for them to be consulted before any mining areas
I am not convinced. should be done in their areas, any logging done in their areas or any dam construction because this
has been disturbing our people especially in the Cordilleras. So, if there could be, if our lawyers or the
Section 3(a) merely defines the coverage of ancestral domains, and describes the extent, limit and secretariat could just propose a provision for incorporation here so that maybe the right to
composition of ancestral domains by setting forth the standards and guidelines in determining consultation and the right to be compensated when there are damages within their ancestral lands.
whether a particular area is to be considered as part of and within the ancestral domains. In other
words, Section 3(a) serves only as a yardstick which points out what properties are within the CHAIRMAN FLAVIER. Yes, very well taken but to the best of my recollection both are already
ancestral domains. It does not confer or recognize any right of ownership over the natural resources considered in subsequent sections which we are now looking for.
to the indigenous peoples. Its purpose is definitional and not declarative of a right or title.
HON. DOMINGUEZ. Thank you.
The specification of what areas belong to the ancestral domains is, to our mind, important to ensure
that no unnecessary encroachment on private properties outside the ancestral domains will result CHAIRMAN FLAVIER. First of all there is a line that gives priority use for the indigenous people where
during the delineation process. The mere fact that Section 3(a) defines ancestral domains to include they are. Number two, in terms of the mines there is a need for prior consultation of source which is
the natural resources found therein does not ipso facto convert the character of such natural here already. So, anyway it is on the record that you want to make sure that the secretariat takes
resources as private property of the indigenous peoples. Similarly, Section 5 in relation to Section 3(a) note of those two issues and my assurance is that it is already there and I will make sure that they
cannot be construed as a source of ownership rights of indigenous people over the natural resources cross check.
simply because it recognizes ancestral domains as their "private but community property."
HON. ADAMAT. I second that, Mr. Chairman.
The phrase "private but community property" is merely descriptive of the indigenous peoples’
concept of ownership as distinguished from that provided in the Civil Code. In Civil Law, "ownership" CHAIRMAN FLAVIER. Okay, thank you. So we now move to Section 8, there is a Senate version you do
is the "independent and general power of a person over a thing for purposes recognized by law and not have and if you agree we will adopt that.127 (Emphasis supplied.)
within the limits established thereby."122 The civil law concept of ownership has the following
attributes: jus utendi or the right to receive from the thing that which it produces, jus abutendi or the Further, Section 7 makes no mention of any right of ownership of the indigenous peoples over the
right to consume the thing by its use, jus disponendi or the power to alienate, encumber, transform natural resources. In fact, Section 7(a) merely recognizes the "right to claim ownership over lands,
and even destroy that which is owned and jus vidicandi or the right to exclude other persons from bodies of water traditionally and actually occupied by indigenous peoples, sacred places, traditional

39
hunting and fishing grounds, and all improvements made by them at any time within the domains." to avoid situations whereby the Philippines would become a source of international conflicts, thereby
Neither does Section 7(b), which enumerates certain rights of the indigenous peoples over the natural posing danger to its internal security and independence.135
resources found within their ancestral domains, contain any recognition of ownership vis-a-vis the
natural resources. The declaration of State ownership and control over minerals and other natural resources in the 1935
Constitution was reiterated in both the 1973136 and 1987 Constitutions.137
What is evident is that the IPRA protects the indigenous peoples’ rights and welfare in relation to the
natural resources found within their ancestral domains,128 including the preservation of the Having ruled that the natural resources which may be found within the ancestral domains belong to
ecological balance therein and the need to ensure that the indigenous peoples will not be unduly the State, the Court deems it necessary to clarify that the jurisdiction of the NCIP with respect to
displaced when State-approved activities involving the natural resources located therein are ancestral domains under Section 52 [i] of IPRA extends only to the lands and not to the natural
undertaken. resources therein.

Finally, the concept of native title to natural resources, unlike native title to land, has not been Section 52[i] provides:
recognized in the Philippines. NCIP and Flavier, et al. invoke the case of Reavies v. Fianza129 in
support of their thesis that native title to natural resources has been upheld in this jurisdiction.130 Turnover of Areas Within Ancestral Domains Managed by Other Government Agencies. - The
They insist that "it is possible for rights over natural resources to vest on a private (as opposed to a Chairperson of the NCIP shall certify that the area covered is an ancestral domain. The secretaries of
public) holder if these were held prior to the 1935 Constitution."131 However, a judicious the Department of Agrarian Reform, Department of Environment and Natural Resources, Department
examination of Reavies reveals that, contrary to the position of NCIP and Flavier, et al., the Court did of Interior and Local Government, and Department of Justice, the Commissioner of the National
not recognize native title to natural resources. Rather, it merely upheld the right of the indigenous Development Corporation, and any other government agency claiming jurisdiction over the area shall
peoples to claim ownership of minerals under the Philippine Bill of 1902. be notified thereof. Such notification shall terminate any legal basis for the jurisdiction previously
claimed.
While as previously discussed, native title to land or private ownership by Filipinos of land by virtue
of time immemorial possession in the concept of an owner was acknowledged and recognized as far Undoubtedly, certain areas that are claimed as ancestral domains may still be under the
back during the Spanish colonization of the Philippines, there was no similar favorable treatment as administration of other agencies of the Government, such as the Department of Agrarian Reform,
regards natural resources. The unique value of natural resources has been acknowledged by the State with respect to agricultural lands, and the Department of Environment and Natural Resources with
and is the underlying reason for its consistent assertion of ownership and control over said natural respect to timber, forest and mineral lands. Upon the certification of these areas as ancestral domain
resources from the Spanish regime up to the present.132 Natural resources, especially minerals, were following the procedure outlined in Sections 51 to 53 of the IPRA, jurisdiction of the government
considered by Spain as an abundant source of revenue to finance its battles in wars against other agency or agencies concerned over lands forming part thereof ceases. Nevertheless, the jurisdiction
nations. Hence, Spain, by asserting its ownership over minerals wherever these may be found, of government agencies over the natural resources within the ancestral domains does not terminate
whether in public or private lands, recognized the separability of title over lands and that over by such certification because said agencies are mandated under existing laws to administer the
minerals which may be found therein. 133 natural resources for the State, which is the owner thereof. To construe Section 52[i] as divesting the
State, through the government agencies concerned, of jurisdiction over the natural resources within
On the other hand, the United States viewed natural resources as a source of wealth for its nationals. the ancestral domains would be inconsistent with the established doctrine that all natural resources
As the owner of natural resources over the Philippines after the latter’s cession from Spain, the United are owned by the State.
States saw it fit to allow both Filipino and American citizens to explore and exploit minerals in public
lands, and to grant patents to private mineral lands. A person who acquired ownership over a parcel C. The provisions of IPRA pertaining to the utilization of natural resources are not unconstitutional.
of private mineral land pursuant to the laws then prevailing could exclude other persons, even the
State, from exploiting minerals within his property.134 Although the United States made a distinction The IPRA provides that indigenous peoples shall have the right to manage and conserve the natural
between minerals found in public lands and those found in private lands, title in these minerals was resources found on the ancestral domains, to benefit from and share in the profits from the allocation
in all cases sourced from the State. The framers of the 1935 Constitution found it necessary to and utilization of these resources, and to negotiate the terms and conditions for the exploration of
maintain the State’s ownership over natural resources to insure their conservation for future such natural resources.138 The statute also grants them priority rights in the harvesting, extraction,
generations of Filipinos, to prevent foreign control of the country through economic domination; and development or exploitation of any natural resources within the ancestral domains.139 Before the

40
NCIP can issue a certification for the renewal, or grant of any concession, license or lease, or for the
perfection of any production-sharing agreement the prior informed written consent of the indigenous The rights given to the indigenous peoples regarding the exploitation of natural resources under
peoples concerned must be obtained.140 In return, the indigenous peoples are given the Sections 7(b) and 57 of IPRA amplify what has been granted to them under existing laws, such as the
responsibility to maintain, develop, protect and conserve the ancestral domains or portions thereof Small-Scale Mining Act of 1991 (R.A. 7076) and the Philippine Mining Act of 1995 (R.A. 7942). R.A.
which are found to be necessary for critical watersheds, mangroves, wildlife sanctuaries, wilderness, 7076 expressly provides that should an ancestral land be declared as a people’s small-scale mining
protected areas, forest cover, or reforestation.141 area, the members of the indigenous peoples living within said area shall be given priority in the
awarding of small-scale mining contracts.152 R.A. 7942 declares that no ancestral land shall be
The Solicitor General argues that these provisions deny the State an active and dominant role in the opened for mining operations without the prior consent of the indigenous cultural community
utilization of our country’s natural resources. Petitioners, on the other hand, allege that under the concerned153 and in the event that the members of such indigenous cultural community give their
Constitution the exploration, development and utilization of natural resources may only be consent to mining operations within their ancestral land, royalties shall be paid to them by the parties
undertaken by the State, either directly or indirectly through co-production, joint venture, or to the mining to the contract.154
production-sharing agreements.142 To petitioners, no other method is allowed by the Constitution.
They likewise submit that by vesting ownership of ancestral lands and ancestral domains in the In any case, a careful reading of Section 7(b) would reveal that the rights given to the indigenous
indigenous peoples, IPRA necessarily gives them control over the use and enjoyment of such natural peoples are duly circumscribed. These rights are limited only to the following: "to manage and
resources, to the prejudice of the State.143 conserve natural resources within territories and uphold it for future generations; to benefit and
share the profits from allocation and utilization of the natural resources found therein; to negotiate
Section 2, Article XII of the Constitution provides in paragraph 1 thereof that the exploration, the terms and conditions for the exploration of natural resources in the areas for the purpose of
development and utilization of natural resources must be under the full control and supervision of ensuring ecological, environmental protection and the conservation measures, pursuant to national
the State, which may directly undertake such activities or enter into co-production, joint venture, or and customary laws; to an informed and intelligent participation in the formulation and
production-sharing agreements. This provision, however, should not be read in isolation to avoid a implementation of any project, government or private, that will affect or impact upon the ancestral
mistaken interpretation that any and all forms of utilization of natural resources other than the domains and to receive just and fair compensation for any damages which they may sustain as a result
foregoing are prohibited. The Constitution must be regarded as consistent with itself throughout.144 of the project, and the right to effective measures by the government to prevent any interference
No constitutional provision is to be separated from all the others, or to be considered alone, all with, alienation and encroachment of these rights."
provisions bearing upon a particular subject are to be brought into view and to be so interpreted as
to effectuate the great purposes of the fundamental law.145 It must be noted that the right to negotiate terms and conditions granted under Section 7(b) pertains
only to the exploration of natural resources. The term "exploration" refers only to the search or
In addition to the means of exploration, development and utilization of the country’s natural prospecting of mineral resources, or any other means for the purpose of determining the existence
resources stated in paragraph 1, Section 2 of Article XII, the Constitution itself states in the third and the feasibility of mining them for profit.155 The exploration, which is merely a preliminary
paragraph of the same section that Congress may, by law, allow small-scale utilization of natural activity, cannot be equated with the entire process of "exploration, development and utilization" of
resources by its citizens.146 Further, Section 6, Article XIII, directs the State, in the disposition and natural resources which under the Constitution belong to the State.
utilization of natural resources, to apply the principles of agrarian reform or stewardship.147
Similarly, Section 7, Article XIII mandates the State to protect the rights of subsistence fishermen to Section 57, on the other hand, grants the indigenous peoples "priority rights" in the utilization of
the preferential use of marine and fishing resources.148 Clearly, Section 2, Article XII, when natural resources and not absolute ownership thereof. Priority rights does not mean exclusive rights.
interpreted in view of the pro-Filipino, pro-poor philosophy of our fundamental law, and in harmony What is granted is merely the right of preference or first consideration in the award of privileges
with the other provisions of the Constitution rather as a sequestered pronouncement,149 cannot be provided by existing laws and regulations, with due regard to the needs and welfare of indigenous
construed as a prohibition against any and all forms of utilization of natural resources without the peoples living in the area.
State’s direct participation.
There is nothing in the assailed law which implies an automatic or mechanical character in the grant
Through the imposition of certain requirements and conditions for the exploration, development and of concessions. Nor does the law negate the exercise of sound discretion by government entities.
utilization of the natural resources under existing laws,150 the State retains full control over such Several factors still have to be considered. For example, the extent and nature of utilization and the
activities, whether done on small-scale basis151 or otherwise. consequent impact on the environment and on the indigenous peoples’ way of life are important

41
considerations. Moreover, the indigenous peoples must show that they live in the area and that they and prior informed consent.157 As to non-members, the prior informed consent takes the form of a
are in the best position to undertake the required utilization. formal and written agreement between the indigenous peoples and non-members under the proviso
in Section 57 in case the State enters into a co-production, joint venture, or production-sharing
It must be emphasized that the grant of said priority rights to indigenous peoples is not a blanket agreement with Filipino citizens, or corporations. This requirement is not peculiar to IPRA. Existing
authority to disregard pertinent laws and regulations. The utilization of said natural resources is laws and regulations such as the Philippine Environmental Policy,158 the Environmental Impact
always subject to compliance by the indigenous peoples with existing laws, such as R.A. 7076 and R.A. System,159 the Local Government Code160 and the Philippine Mining Act of 1995161already require
7942 since it is not they but the State, which owns these resources. increased consultation and participation of stakeholders, such as indigenous peoples, in the planning
of activities with significant environment impact.
It also bears stressing that the grant of priority rights does not preclude the State from undertaking
activities, or entering into co-production, joint venture or production-sharing agreements with The requirement in Section 59 that prior written informed consent of the indigenous peoples must
private entities, to utilize the natural resources which may be located within the ancestral domains. be procured before the NCIP can issue a certification for the "issuance, renewal, or grant of any
There is no intention, as between the State and the indigenous peoples, to create a hierarchy of concession, license or lease, or to the perfection of any production-sharing agreement," must be
values; rather, the object is to balance the interests of the State for national development and those interpreted, not as a grant of the power to control the exploration, development and utilization of
of the indigenous peoples. natural resources, but merely the imposition of an additional requirement for such concession or
agreement. The clear intent of the law is to protect the rights and interests of the indigenous peoples
Neither does the grant of priority rights to the indigenous peoples exclude non-indigenous peoples which may be adversely affected by the operation of such entities or licensees.
from undertaking the same activities within the ancestral domains upon authority granted by the
proper governmental agency. To do so would unduly limit the ownership rights of the State over the Corollary Issues
natural resources.
A. IPRA does not violate the Due Process clause.
To be sure, the act of the State of giving preferential right to a particular sector in the utilization of
natural resources is nothing new. As previously mentioned, Section 7, Article XIII of the Constitution The first corollary issue raised by petitioners is whether IPRA violates Section 1, Article III of the
mandates the protection by the State of "the rights of subsistence fishermen, especially of local Constitution, which provides that "no person shall be deprived of life, liberty, or property without due
communities, to the preferential use of communal marine and fishing resources, both inland and process of law, nor shall any person be deprived the equal protection of the laws."
offshore."
Petitioners maintain that the broad definition of ancestral lands and ancestral domains under Section
Section 57 further recognizes the possibility that the exploration and exploitation of natural resources 3(a) and 3(b) of IPRA includes private lands. They argue that the inclusion of private lands in the
within the ancestral domains may disrupt the natural environment as well as the traditional activities ancestral lands and ancestral domains violates the due process clause.162 Petitioners’ contention is
of the indigenous peoples therein. Hence, the need for the prior informed consent of the indigenous erroneous.
peoples before any search for or utilization of the natural resources within their ancestral domains is
undertaken. Sections 3(a) and 3(b) expressly provide that the definition of ancestral lands and ancestral domains
are "subject to Section 56," which reads:
In a situation where the State intends to directly or indirectly undertake such activities, IPRA requires
that the prior informed consent of the indigenous peoples be obtained. The State must, as a matter Sec. 56. Existing Property Rights Regimes. – Property rights within the ancestral domains already
of policy and law, consult the indigenous peoples in accordance with the intent of the framers of the existing and/or vested upon effectivity of this Act, shall be recognized and protected.
Constitution that national development policies and programs should involve a systematic
consultation to balance local needs as well as national plans. As may be gathered from the discussion Petitioners, however, contend that Section 56 aims to protect only the vested rights of indigenous
of the framers of the Constitution on this point, the national plan presumably takes into account the peoples, but not those who are not members of such communities. Following their interpretation,
requirements of the region after thorough consultation.156 To this end, IPRA grants to the indigenous IPRA, under Section 56, recognizes the rights of indigenous peoples to their ancestral lands and
peoples the right to an informed and intelligent participation in the formulation and implementation ancestral domains, subject to the vested rights of the same communities to such ancestral lands and
of any project, government or private, and the right not to be removed therefrom without their free ancestral domains. Such interpretation is obviously incorrect.

42
customary laws. . . would clearly be a denial of due process. . . [because those who are not indigenous
The "property rights" referred to in Section 56 belong to those acquired by individuals, whether peoples] do not know what these customary laws are."175
indigenous or non-indigenous peoples. Said provision makes no distinction as to the ethnic origins of
the ownership of these "property rights." The IPRA thus recognizes and respects "vested rights" Petitioners’ concerns are unfounded. The fact that the NCIP is composed of members of the
regardless of whether they pertain to indigenous or non-indigenous peoples. Where the law does not indigenous peoples does not mean that it (the NCIP) is incapable, or will appear to be so incapable,
distinguish, the courts should not distinguish.163 What IPRA only requires is that these "property of delivering justice to the non-indigenous peoples. A person’s possession of the trait of impartiality
rights" already exist and/or vested upon its effectivity. desirable of a judge has nothing to do with his or her ethnic roots. In this wise, the indigenous peoples
are as capable of rendering justice as the non-indigenous peoples for, certainly, the latter have no
Further, by the enactment of IPRA, Congress did not purport to annul any and all Torrens titles within monopoly of the concept of justice.
areas claimed as ancestral lands or ancestral domains. The statute imposes strict procedural
requirements for the proper delineation of ancestral lands and ancestral domains as safeguards In any case, there are sufficient checks in the law against any abuse by the NCIP of its quasi-judicial
against the fraudulent deprivation of any landowner of his land, whether or not he is member of an powers. Section 67 states that the decision of the NCIP shall be appealable to the Court of Appeals by
indigenous cultural community. In all proceedings for delineation of ancestral lands and ancestral petition for review. The regular remedies under our rules of procedure are likewise available to any
domains, the Director of Lands shall appear to represent the interest of the Republic of the party aggrieved by the decision of the NCIP.
Philippines.164 With regard to ancestral domains, the following procedure is mandatory: first,
petition by an indigenous cultural community, or motu proprio by the NCIP; second, investigation and Anent the use of customary laws in determining the ownership and extent of ancestral domains,
census by the Ancestral domains Office ("ADO") of the NCIP; third, preliminary report by the ADO; suffice it to say that such is allowed under paragraph 2, Section 5 of Article XII of the Constitution.
fourth, posting and publication; and lastly, evaluation by the NCIP upon submission of the final report Said provision states, "The Congress may provide for the applicability of customary laws governing
of the ADO.165 With regard to ancestral lands, unless such lands are within an ancestral domain, the property rights and relations in determining the ownership and extent of the ancestral domains."
statute imposes the following procedural requirements: first, application; second, posting and Notably, the use of customary laws under IPRA is not absolute, for the law speaks merely of primacy
publication; third, investigation and inspection by the ADO; fourth, delineation; lastly, evaluation by of use.176 The IPRA prescribes the application of such customary laws where these present a
the NCIP upon submission of a report by the ADO.166 Hence, we cannot sustain the arguments of the workable solution acceptable to the parties, who are members of the same indigenous group. This
petitioners that the law affords no protection to those who are not indigenous peoples. interpretation is supported by Section 1, Rule IX of the Implementing Rules which states:

Neither do the questioned sections of IPRA on the composition and powers and jurisdiction of the RULE IX. JURISDICTION AND PROCEDURES FOR ENFORCEMENT OF RIGHTS
NCIP167 and the application of customary law,168 violate the due process clause of the Constitution.
Section 1. Primacy of Customary Law. All conflicts related to ancestral domains and lands, involving
Petitioners point out that IPRA provides that the NCIP shall be composed exclusively of members of ICCs/IPs, such as but not limited to conflicting claims and boundary disputes, shall be resolved by the
indigenous peoples,169 and that the NCIP shall have jurisdiction over all claims and disputes involving concerned parties through the application of customary laws in the area where the disputed ancestral
indigenous peoples,170 including even disputes between a member of such communities and one domain or land is located.
who is not a member, as well as over disputes in the delineation of ancestral domains.171 Petitioners
clarify that they do not claim that the members of the NCIP are incapable of being fair and impartial All conflicts related to the ancestral domains or lands where one of the parties is a non-ICC/IP or
judges. They merely contend that the NCIP will not appear to be impartial, because a party who is not where the dispute could not be resolved through customary law shall be heard and adjudicated in
a member of an indigenous cultural community "who must defend his case against [one who is] accordance with the Rules on Pleadings, Practice and Procedures Before the NCIP to be adopted
before judges who are all members of [indigenous peoples] cannot but harbor a suspicion that they hereafter. (Emphasis supplied.)
do not have the cold neutrality of an impartial judge."172
The application of customary law is limited to disputes concerning property rights or relations in
In addition, petitioners claim that IPRA prescribes that customary laws shall be applied first in disputes determining the ownership and extent of the ancestral domains,177 where all the parties involved
involving property, succession and land,173 and that such laws shall likewise be used in disputes are members of indigenous peoples,178 specifically, of the same indigenous group. It therefore
involving indigenous peoples.174 They assert that "[w]hen the dispute involves a member of an follows that when one of the parties to a dispute is a non-member of an indigenous group, or when
[indigenous cultural community and another who is not], a resolution of such a dispute based on

43
the indigenous peoples involved belong to different groups, the application of customary law is not
required. Petitioners asseverate that the aforecited rule infringes upon the power of control of the President
over the NCIP by characterizing the relationship of the NCIP to the Office of the President as "lateral
Like any other law, the objective of IPRA in prescribing the primacy of customary law in disputes but autonomous...for purposes of policy and program coordination."
concerning ancestral lands and domains where all parties involved are indigenous peoples is justice.
The utilization of customary laws is in line with the constitutional policy of recognizing the application Although both Section 40 of the IPRA and Section 1, Part II, Rule VII of the Implementing Rules
thereof through legislation passed by Congress. characterize the NCIP as an independent agency under the Office of the President, such
characterization does not remove said body from the President’s control and supervision.
Furthermore, the recognition and use of customary law is not a novel idea in this jurisdiction. Under
the Civil Code, use of customary law is sanctioned, as long as it is proved as a fact according to the The NCIP has been designated under IPRA as the primary government agency responsible for the
rules of evidence,179 and it is not contrary to law, public order or public policy.180 Moreover, the formulation and implementation of policies, plans and programs to promote and protect the rights
Local Government Code of 1991 calls for the recognition and application of customary laws to the and well being of the indigenous peoples and the recognition of their ancestral domain as well as
resolution of issues involving members of indigenous peoples. This law admits the operation of their rights thereto.182 It has been granted administrative,183 quasi-legislative184 and quasi-judicial
customary laws in the settling of disputes if such are ordinarily used in barangays where majority of powers185 to carry out its mandate. The diverse nature of the NCIP’s functions renders it impossible
the inhabitants are members of indigenous peoples.181 to place said agency entirely under the control of only one branch of government and this, apparently,
is the reason for its characterization by Congress as an independent agency. An "independent agency"
B. Section 1, Part II, Rule VII of the Implementing Rules of IPRA does not infringe upon the President’s is defined as an administrative body independent of the executive branch or one not subject to a
power of control over the Executive Department. superior head of department, as distinguished from a "subordinate agency" or an administrative body
whose action is subject to administrative review or revision.186
The second corollary issue is whether the Implementing Rules of IPRA violate Section 17, Article VII
of the Constitution, which provides that: That Congress did not intend to place the NCIP under the control of the President in all instances is
evident in the IPRA itself, which provides that the decisions of the NCIP in the exercise of its quasi-
The President shall have control of all the executive departments, bureaus, and offices. He shall judicial functions shall be appealable to the Court of Appeals,187 like those of the National Labor
ensure that the laws be faithfully executed. Relations Commission (NLRC) and the Securities and Exchange Commission (SEC). Nevertheless, the
NCIP, although independent to a certain degree, was placed by Congress "under the office of the
The assailed provision of the Implementing Rules provides: President" and, as such, is still subject to the President’s power of control and supervision granted
under Section 17, Article VII of the Constitution188 with respect to its performance of administrative
Rule VII. The National Commission on Indigenous Peoples (NCIP) functions, such as the following: (1) the NCIP must secure the President’s approval in obtaining loans
to finance its projects;189 (2) it must obtain the President’s approval for any negotiation for funds
xxx and for the acceptance of gifts and/or properties in whatever from and from whatever source;190 (3)
the NCIP shall submit annual reports of its operations and achievements to the President, and advise
Part II: NCIP as an Independent Agency Under the Office of the President the latter on all matters relating to the indigenous peoples;191 and (4) it shall exercise such other
powers as may be directed by the President.192 The President is also given the power to appoint the
Section 1. The NCIP is the primary agency of government for the formulation and implementation of Commissioners of the NCIP193 as well as to remove them from office for cause motu proprio or upon
policies, plans and programs to recognize, promote and protect the rights and well-being of the recommendation of any indigenous community.194
indigenous peoples. It shall be an independent agency under the Office of the President. As such, the
administrative relationship of the NCIP to the Office of the President is characterized as a lateral but To recapitulate:
autonomous relationship for purposes of policy and program coordination. This relationship shall be
carried out through a system of periodic reporting. Matters of day-to-day administration or all those (1) The provisions of the IPRA (specifically Sections 3, paragraphs (a) and (b), 5, 6, 7, and 8) affirming
pertaining to internal operations shall be left to the discretion of the Chairperson of the Commission, the ownership by the indigenous peoples of their ancestral lands and domains by virtue of native title
as the Chief Executive Officer. do not diminish the State’s ownership of lands of the public domain, because said ancestral lands and

44
domains are considered as private land, and never to have been part of the public domain, following This suit was instituted to determine the constitutionality of certain provisions of R.A. No. 8371,
the doctrine laid down in Cariño vs. Insular Government;195 otherwise known as the Indigenous Peoples Rights Act. Petitioners do not complain of any injury as a
result of the application of the statute to them. They assert a right to seek an adjudication of
(2) The constitutional provision vesting ownership over minerals, mineral lands and other natural constitutional questions as citizens and taxpayers, upon the plea that the questions raised are of
resources in the State is not violated by Sections 3, 5, 7, 56, 57, 58 and 59 of the IPRA which grant "transcendental importance."
certain rights to the indigenous peoples over the natural resources found within the ancestral
domains, e.g., to benefit from and share in the profits from the allocation and utilization of the same, The judicial power vested in this Court by Art. VIII, §1 extends only to cases and controversies for the
as well as priority rights in the harvesting, extraction, development or exploitation thereof. The State determination of such proceedings as are established by law for the protection or enforcement of
retains full control over the exploration, development and utilization of natural resources even with rights, or the prevention, redress or punishment of wrongs.1 In this case, the purpose of the suit is
the grant of said rights to the indigenous peoples, through the imposition of requirements and not to enforce a property right of petitioners against the government and other respondents or to
conditions for the utilization of natural resources under existing laws, such as the Small-Scale Mining demand compensation for injuries suffered by them as a result of the enforcement of the law, but
Act of 1991196and the Philippine Mining Act of 1995.197 Moreover, the rights granted to indigenous only to settle what they believe to be the doubtful character of the law in question. Any judgment
peoples for the utilization of natural resources within their ancestral domains merely amplify what that we render in this case will thus not conclude or bind real parties in the future, when actual
has been earlier granted to them under the aforesaid laws; litigation will bring to the Court the question of the constitutionality of such legislation. Such
judgment cannot be executed as it amounts to no more than an expression of opinion upon the
(3) While the IPRA recognizes the rights of indigenous peoples with regard to their ancestral lands validity of the provisions of the law in question.2
and domains, it also protects the vested rights of persons, whether indigenous or non-indigenous
peoples, who may have acquired rights of ownership lands or rights to explore and exploit natural I do not conceive it to be the function of this Court under Art. VIII, §1 of the Constitution to determine
resources within the ancestral lands and domains;198 in the abstract whether or not there has been a grave abuse of discretion amounting to lack or excess
of jurisdiction on the part of the legislative and executive departments in enacting the IPRA. Our
(4) The Due Process Clause of the Constitution is not violated by the provisions (Sections 40, 51-54, jurisdiction is confined to cases or controversies. No one reading Art. VIII, §5 can fail to note that, in
62, 63, 65 and 66) of the IPRA which, among others, establish the composition of the NCIP, and enumerating the matters placed in the keeping of this Court, it uniformly begins with the phrase "all
prescribe the application of customary law in certain disputes involving indigenous peoples. The fact cases. . . ."
the NCIP is composed wholly of indigenous peoples does not mean that it is incapable of being
impartial. Moreover, the use of customary laws is sanctioned by paragraph 2, Section 5 of Article XII The statement that the judicial power includes the duty to determine whether there has been a grave
of the Constitution; and abuse of discretion was inserted in Art. VIII, §1 not really to give the judiciary a roving commission to
right any wrong it perceives but to preclude courts from invoking the political question doctrine in
(5) The provision of the Implementing Rules characterizing the NCIP as an independent agency under order to evade the decision of certain cases even where violations of civil liberties are alleged.
the Office of the President does not infringe upon the President’s power of control under Section 17,
Article VII of the Constitution, since said provision as well as Section 40 of the IPRA expressly places The statement is based on the ruling of the Court in Lansang v. Garcia,3 in which this Court, adopting
the NCIP under the Office of the President, and therefore under the President’s control and the submission of the Solicitor General, formulated the following test of its jurisdiction in such cases:
supervision with respect to its administrative functions. However, insofar as the decisions of the NCIP
in the exercise of its quasi-judicial powers are concerned, the same are reviewable by the Court of [J]udicial inquiry into the basis of the questioned proclamation can go no further than to satisfy the
Appeals, like those of the NLRC and the SEC. Court not that the President’s decision is correct and that public safety was endangered by the
rebellion and justified the suspension of the writ, but that in suspending the writ, the President did
In view of the foregoing, I vote to DISMISS the petition. not act arbitrarily.

SEPARATE OPINION That is why Art. VII, §18 now confers on any citizen standing to question the proclamation of martial
law or the suspension of the privilege of the writ of habeas corpus. It is noteworthy that Chief Justice
MENDOZA, J.: Roberto Concepcion, who chaired the Committee on the Judiciary of the Constitutional Commission,
was the author of the opinions of the Court in Lopez v. Roxas and Lansang v. Garcia.

45
To decline, therefore, the exercise of jurisdiction where there is no genuine controversy is not to show
Indeed, the judicial power cannot be extended to matters which do not involve actual cases or timidity but respect for the judgment of a coequal department of government whose acts, unless
controversies without upsetting the balance of power among the three branches of the government shown to be clearly repugnant to the fundamental law, are presumed to be valid. The polestar of
and erecting, as it were, the judiciary, particularly the Supreme Court, as a third branch of Congress, constitutional adjudication was set forth by Justice Laurel in the Angara case when he said that "this
with power not only to invalidate statutes but even to rewrite them. Yet that is exactly what we would power of judicial review is limited to actual cases and controversies to be exercised after full
be permitting in this case were we to assume jurisdiction and decide wholesale the constitutional opportunity of argument by the parties, and limited further to the constitutional question raised or
validity of the IPRA contrary to the established rule that a party can question the validity of a statute the very lis mota, presented."8 For the exercise of this power is legitimate only in the last resort, and
only if, as applied to him, it is unconstitutional. Here the IPRA is sought to be declared void on its face. as a necessity in the determination of real, earnest, and vital controversy between individuals.9 Until,
therefore, an actual case is brought to test the constitutionality of the IPRA, the presumption of
The only instance where a facial challenge to a statute is allowed is when it operates in the area of constitutionality, which inheres in every statute, must be accorded to it.
freedom of expression. In such instance, the overbreadth doctrine permits a party to challenge the
validity of a statute even though as applied to him it is not unconstitutional but it might be if applied Justice Kapunan, on the other hand, cites the statement in Severino v. Governor General,10 reiterated
to others not before the Court whose activities are constitutionally protected. Invalidation of the in Tanada v. Tuvera,11 that "when the question is one of public right and the object of mandamus to
statute "on its face" rather than "as applied" is permitted in the interest of preventing a "chilling" procure the enforcement of a public duty, the people are regarded as the real party in interest, and
effect on freedom of expression. But in other cases, even if it is found that a provision of a statute is the relator at whose instigation the proceedings are instituted need not show that he has any legal
unconstitutional, courts will decree only partial invalidity unless the invalid portion is so far or special interest in the result, it being sufficient that he is a citizen and as such is interested in the
inseparable from the rest of the statute that a declaration of partial invalidity is not possible. execution of the laws." On the basis of this statement, he argues that petitioners have standing to
bring these proceedings.12
For the Court to exercise its power of review when there is no case or controversy is not only to act
without jurisdiction but also to run the risk that, in adjudicating abstract or hypothetical questions, In Severino v. Governor General,13 the question was whether mandamus lay to compel the Governor
its decision will be based on speculation rather than experience. Deprived of the opportunity to General to call a special election on the ground that it was his duty to do so. The ruling was that he
observe the impact of the law, the Court is likely to equate questions of constitutionality with did not have such a duty. On the other hand, although mandamus was issued in Tanada v. Tuvera, it
questions of wisdom and is thus likely to intrude into the domain of legislation. Constitutional was clear that petitioners had standing to bring the suit, because the public has a right to know and
adjudication, it cannot be too often repeated, cannot take place in a vacuum. the failure of respondents to publish all decrees and other presidential issuances in the Official
Gazette placed petitioners in danger of violating those decrees and issuances. But, in this case, what
Some of the brethren contend that not deciding the constitutional issues raised by petitioners will be public right is there for petitioners to enforce when the IPRA does not apply to them except in general
a "galling cop out"4 or an "advocacy of timidity, let alone isolationism."5 To decline the exercise of and in common with other citizens.
jurisdiction in this case is no more a "cop out" or a sign of "timidity" than it was for Chief Justice
Marshall in Marbury v. Madison6 to hold that petitioner had the right to the issuance of his For the foregoing reasons I vote to dismiss the petition in this case.
commission as justice of the peace of the District of Columbia only to declare in the end that after all
mandamus did not lie, because §13 of the Judiciary Act of 1789, which conferred original jurisdiction SEPARATE OPINION
on the United States Supreme Court to issue the writ of mandamus, was unconstitutional as the (Concurring and Dissenting)
court’s jurisdiction is mainly appellate.
PANGANIBAN, J.:
Today Marbury v. Madison is remembered for the institution of the power of judicial review, and so
that there can be no doubt of this power of our Court, we in this country have enshrined its principle I concur with the draft ponencia of Mr. Justice Santiago M. Kapunan in its well-crafted handling of the
in Art. VIII, §1. Now, the exercise of judicial review can result either in the invalidation of an act of procedural or preliminary issues. In particular, I agree that petitioners have shown an actual case or
Congress or in upholding it. Hence, the checking and legitimating functions of judicial review so well controversy involving at least two constitutional questions of transcendental importance,1 which
mentioned in the decisions7 of this Court. deserve judicious disposition on the merits directly by the highest court of the land.2 Further, I am
satisfied that the various aspects of this controversy have been fully presented and impressively
argued by the parties. Moreover, prohibition and mandamus are proper legal remedies3 to address

46
the problems raised by petitioners. In any event, this Court has given due course to the Petition, heard My basic premise is that the Constitution is the fundamental law of the land, to which all other laws
oral arguments and required the submission of memoranda. Indeed, it would then be a galling copout must conform.5 It is the people's quintessential act of sovereignty, embodying the principles upon
for us to dismiss it on mere technical or procedural grounds. which the State and the government are founded.6 Having the status of a supreme and all-
encompassing law, it speaks for all the people all the time, not just for the majority or for the minority
Protection of Indigenous Peoples’ Rights Must Be Within the Constitutional Framework at intermittent times. Every constitution is a compact made by and among the citizens of a State to
govern themselves in a certain manner.7 Truly, the Philippine Constitution is a solemn covenant made
With due respect, however, I dissent from the ponencia’s resolution of the two main substantive by all the Filipinos to govern themselves. No group, however blessed, and no sector, however
issues, which constitute the core of this case. Specifically, I submit that Republic Act (RA) No. 8371, distressed, is exempt from its compass.
otherwise known as the Indigenous Peoples’ Rights Act (IPRA) of 1997, violates and contravenes the
Constitution of the Philippines insofar as - RA 8371, which defines the rights of indigenous cultural communities and indigenous peoples,
admittedly professes a laudable intent. It was primarily enacted pursuant to the state policy enshrined
1. It recognizes or, worse, grants rights of ownership over "lands of the public domain, waters, x x x in our Constitution to "recognize and promote the rights of indigenous cultural communities within
and other natural resources" which, under Section 2, Article XII of the Constitution, "are owned by the framework of national unity and development."8 Though laudable and well-meaning, this statute,
the State" and "shall not be alienated." I respectfully reject the contention that "ancestral lands and however, has provisions that run directly afoul of our fundamental law from which it claims origin and
ancestral domains are not public lands and have never been owned by the State." Such sweeping authority. More specifically, Sections 3(a) and (b), 5, 6, 7(a) and (b), 8 and other related provisions
statement places substantial portions of Philippine territory outside the scope of the Philippine contravene the Regalian Doctrine - the basic foundation of the State's property regime.
Constitution and beyond the collective reach of the Filipino people. As will be discussed later, these
real properties constitute a third of the entire Philippine territory; and the resources, 80 percent of Public Domains and Natural Resources Are Owned by the State and Cannot Be Alienated or Ceded
the nation's natural wealth.
Jura regalia was introduced into our political system upon the "discovery" and the "conquest" of our
2. It defeats, dilutes or lessens the authority of the State to oversee the "exploration, development, country in the sixteenth century. Under this concept, the entire earthly territory known as the
and utilization of natural resources," which the Constitution expressly requires to "be under the full Philippine Islands was acquired and held by the Crown of Spain. The King, as then head of State, had
control and supervision of the State." the supreme power or exclusive dominion over all our lands, waters, minerals and other natural
resources. By royal decrees, though, private ownership of real property was recognized upon the
True, our fundamental law mandates the protection of the indigenous cultural communities’ right to showing of (1) a title deed; or (2) ancient possession in the concept of owner, according to which a
their ancestral lands, but such mandate is "subject to the provisions of this Constitution."4 I concede title could be obtained by prescription.9 Refusal to abide by the system and its implementing laws
that indigenous cultural communities and indigenous peoples (ICCs/IPs) may be accorded preferential meant the abandonment or waiver of ownership claims.
rights to the beneficial use of public domains, as well as priority in the exploration, development and
utilization of natural resources. Such privileges, however, must be subject to the fundamental law. By virtue of the 1898 Treaty of Paris, the Philippine archipelago was ceded to the United States. The
latter assumed administration of the Philippines and succeeded to the property rights of the Spanish
Consistent with the social justice principle of giving more in law to those who have less in life, Crown. But under the Philippine Bill of 1902, the US Government allowed and granted patents to
Congress in its wisdom may grant preferences and prerogatives to our marginalized brothers and Filipino and US citizens for the "free and open x x x exploration, occupation and purchase [of mines]
sisters, subject to the irreducible caveat that the Constitution must be respected. I personally believe and the land in which they are found."10 To a certain extent, private individuals were entitled to own,
in according every benefit to the poor, the oppressed and the disadvantaged, in order to empower exploit and dispose of mineral resources and other rights arising from mining patents.
them to equally enjoy the blessings of nationhood. I cannot, however, agree to legitimize perpetual
inequality of access to the nation's wealth or to stamp the Court's imprimatur on a law that offends This US policy was, however, rejected by the Philippine Commonwealth in 1935 when it crafted and
and degrades the repository of the very authority of this Court - the Constitution of the Philippines. ratified our first Constitution. Instead, the said Constitution embodied the Regalian Doctrine, which
more definitively declared as belonging to the State all lands of the public domain, waters, minerals
The Constitution Is a Compact and other natural resources.11 Although respecting mining patentees under the Philippine Bill of
1902, it restricted the further exploration, development and utilization of natural resources, both as
to who might be entitled to undertake such activities and for how long. The pertinent provision reads:

47
shall not be alienated. The exploration, development, and utilization of natural resources shall be
"SECTION 1 [Art. XIII]. All agricultural, timber, and mineral lands of the public domain, waters, under the full control and supervision of the State. The State may directly undertake such activities,
minerals, coal, petroleum, and other mineral oils, all forces of potential energy, and other natural or it may enter into co-production, joint venture, or production-sharing agreements with Filipino
resources of the Philippines belong to the State, and their disposition, exploitation, development, or citizen, or corporations or associations at least sixty per centum of whose capital is owned by such
utilization shall be limited to citizens of the Philippines, or to corporations or associations at least sixty citizens. Such agreements may be for a period not exceeding twenty-five years, renewable for not
per centum of the capital of which is owned by such citizens, subject to any existing right, grant, lease, more than twenty-five years, and under such terms and conditions as may be provided by law. In
or concession at the time of the inauguration of the Government established under this Constitution. cases of water rights for irrigation, water supply, fisheries, or industrial uses other than the
Natural resources, with the exception of public agricultural land, shall not be alienated, and license, development of water power, beneficial use may be the measure and limit of the grant.
concession, or lease for the exploitation, development, or utilization of any of the natural resources
shall be granted for a period exceeding twenty-five years, renewable for another twenty-five years, "The State shall protect the nation's marine wealth in its archipelagic waters, territorial sea, and
except as to water rights for irrigation, water supply, fisheries, or industrial uses other than the exclusive economic zone, and reserve its use and enjoyment exclusively to Filipino citizens.
development of water power, in which cases beneficial use may be the measure and the limit of the
grant." "The Congress may, by law, allow small-scale utilization of natural resources by Filipino citizens, as
well as cooperative fish farming, with priority to subsistence fishermen and fish workers in rivers,
The concept was carried over in the 1973 and the 1987 Constitutions. Hence, Sections 8 and 9, Article lakes, bays and lagoons.
XIV of the 1973 Constitution, state:
"The President may enter into agreements with foreign-owned corporations involving either technical
"SEC. 8. All lands of the public domain, waters, minerals, coal, petroleum and other mineral oils, all or financial assistance for large-scale exploration, development, and utilization of minerals,
forces of potential energy, fisheries, wildlife, and other natural resources of the Philippines belong to petroleum, and other mineral oils according to the general terms and conditions provided by law,
the State. With the exception of agricultural, industrial or commercial, residential, and resettlement based on real contributions to the economic growth and general welfare of the country. In such
lands of the public domain, natural resources shall not be alienated and no license, concession, or agreements, the State shall promote the development and use of local scientific and technical
lease for the exploration, development, exploitation, utilization of any of the natural resources shall resources.
be granted for a period exceeding twenty-five years, renewable for not more than twenty-five years,
except as to water rights for irrigation, water supply, fisheries, or industrial uses other than the "The President shall notify the Congress of every contract entered into in accordance with this
development of water power, in which cases beneficial use may be the measure and the limit of the provision, within thirty days from its execution."
grant.
The adoption of the Regalian Doctrine by the Philippine Commonwealth was initially impelled by the
SEC. 9. The disposition, exploration, development, exploitation, or utilization of any of the natural desire to preserve the nation's wealth in the hands of the Filipinos themselves. Nationalism was
resources of the Philippines shall be limited to citizens of the Philippines, or to corporations or fervent at the time, and our constitutional framers decided to embody the doctrine in our
associations at least sixty per centum of the capital of which is owned by such citizens. The National fundamental law. Charging the State with the conservation of the national patrimony was deemed
Assembly, in the national interest, may allow such citizens, corporations, or associations to enter into necessary for Filipino posterity. The arguments in support of the provision are encapsulated by
service contracts for financial, technical, management, or other forms of assistance with any foreign Aruego as follows: "[T]he natural resources, particularly the mineral resources which constituted a
person or entity for the exploration, development, exploitation, or utilization of any of the natural great source of wealth, belonged not only to the generation then but also to the succeeding
resources. Existing valid and binding service contracts for financial, technical, management, or other generation and consequently should be conserved for them."12
forms of assistance are hereby recognized as such."
Thus, after expressly declaring that all lands of the public domain, waters, minerals, all forces of
Similarly, Section 2, Article XII of the 1987 Constitution, provides: energy and other natural resources belonged to the Philippine State, the Commonwealth absolutely
prohibited the alienation of these natural resources. Their disposition, exploitation, development and
"SEC. 2. All lands of the public domain, waters, minerals, coal, petroleum, and other mineral oils, all utilization were further restricted only to Filipino citizens and entities that were 60 percent Filipino-
forces of potential energy, fisheries, forests or timber, wildlife, flora and fauna, and other natural owned. The present Constitution even goes further by declaring that such activities "shall be under
resources are owned by the State. With the exception of agricultural lands, all other natural resources the full control and supervision of the State." Additionally, it enumerates land classifications and

48
expressly states that only agricultural lands of the public domain shall be alienable. We quote below
the relevant provision:13 Respondent NCIP claims that IPRA does not violate the Constitution, because it does not grant
ownership of public domains and natural resources to ICCs/IPs. "Rather, it recognizes and mandates
"SEC. 3. Lands of the public domain are classified into agricultural, forest or timber, mineral lands, and respect for the rights of indigenous peoples over their ancestral lands and domains that had never
national parks. Agricultural lands of the public domain may be further classified by law according to been lands of the public domain."16 I say, however, that such claim finds no legal support. Nowhere
the uses to which they may be devoted. Alienable lands of the public domain shall be limited to in the Constitution is there a provision that exempts such lands and domains from its coverage. Quite
agricultural lands. Private corporations or associations may not hold such alienable lands of the public the contrary, it declares that all lands of the public domain and natural resources "are owned by the
domain except by lease, for a period not exceeding twenty-five years, renewable for not more than State"; and "with the exception of agricultural lands, all other natural resources shall not be
twenty-five years, and not to exceed one thousand hectares in area. x x x." alienated."

Mr. Justice Kapunan upholds private respondents and intervenors in their claim that all ancestral As early as Oh Cho v. Director of Lands,17 the Court declared as belonging to the public domain all
domains and lands are outside the coverage of public domain; and that these properties - including lands not acquired from the government, either by purchase or by grant under laws, orders or decrees
forests, bodies of water, minerals and parks found therein - are private and have never been part of promulgated by the Spanish government; or by possessory information under Act 496 (Mortgage
the public domain, because they have belonged to the indigenous people’s ancestors since time Law).
immemorial.
On the other hand, Intervenors Flavier et al.18 differentiate the concept of ownership of ICCs/IPs
I submit, however, that all Filipinos, whether indigenous or not, are subject to the Constitution. from that which is defined in Articles 427 and 428 of the Civil Code. They maintain that "[t]here are
Indeed, no one is exempt from its all-encompassing provisions. Unlike the 1935 Charter, which was variations among ethnolinguistic groups in the Cordillera, but a fair synthesis of these refers to ‘x x x
subject to "any existing right, grant, lease or concession," the 1973 and the 1987 Constitutions spoke the tribal right to use the land or to territorial control x x x, a collective right to freely use the particular
in absolute terms. Because of the State’s implementation of policies considered to be for the common territory x x x [in] the concept of trusteeship.'"
good, all those concerned have to give up, under certain conditions, even vested rights of ownership.
In other words, the "owner" is not an individual. Rather, it is a tribal community that preserves the
In Republic v. Court of Appeals,14 this Court said that once minerals are found even in private land, property for the common but nonetheless exclusive and perpetual benefit of its members, without
the State may intervene to enable it to extract the minerals in the exercise of its sovereign the attributes of alienation or disposition. This concept, however, still perpetually withdraws such
prerogative. The land is converted into mineral land and may not be used by any private person, property from the control of the State and from its enjoyment by other citizens of the Republic. The
including the registered owner, for any other purpose that would impede the mining operations. Such perpetual and exclusive character of private respondents’ claims simply makes them repugnant to
owner would be entitled to just compensation for the loss sustained. basic fairness and equality.

In Atok Big-Wedge Mining Company v. IAC,15 the Court clarified that while mining claim holders and Private respondents and intervenors trace their "ownership" of ancestral domains and lands to the
patentees have the exclusive right to the possession and enjoyment of the located claim, their rights pre-Spanish conquest. I should say that, at the time, their claims to such lands and domains was
are not absolute or strictly one of ownership. Thus, failure to comply with the requirements of limited to the surfaces thereof since their ancestors were agriculture-based. This must be the
pertinent mining laws was deemed an abandonment or a waiver of the claim. continuing scope of the indigenous groups’ ownership claims: limited to land, excluding the natural
resources found within.
Verily, as petitioners undauntedly point out, four hundred years of Philippine political history cannot
be set aside or ignored by IPRA, however well-intentioned it may be. The perceived lack of In any event, if all that the ICCs/IPs demand is preferential use - not ownership - of ancestral domains,
understanding of the cultural minorities cannot be remedied by conceding the nation’s resources to then I have no disagreement. Indeed, consistent with the Constitution is IPRA’s Section 5719- without
their exclusive advantage. They cannot be more privileged simply because they have chosen to ignore the too-broad definitions under Section 3 (a) and (b) - insofar as it grants them priority rights in
state laws. For having chosen not to be enfolded by statutes on perfecting land titles, ICCs/IPs cannot harvesting, extracting, developing or exploiting natural resources within ancestral domains.
now maintain their ownership of lands and domains by insisting on their concept of "native title"
thereto. It would be plain injustice to the majority of Filipinos who have abided by the law and, The concerted effort to malign the Regalian Doctrine as a vestige of the colonial past must fail. Our
consequently, deserve equal opportunity to enjoy the country’s resources. Constitution vests the ownership of natural resources, not in colonial masters, but in all the Filipino

49
people. As the protector of the Constitution, this Court has the sworn duty to uphold the tenets of force majeure or displacement x x x. It shall include ancestral lands, forests, pasture, residential,
that Constitution - not to dilute, circumvent or create exceptions to them. agricultural, and other lands individually owned whether alienable and disposable or otherwise,
hunting grounds x x x bodies of water, mineral and other natural resources x x x." (Emphasis ours.)
Cariño v. Insular Government Was Modified by the Constitution
Clearly, under the above-quoted provision of IPRA, ancestral domains of ICCs/IPs encompass the
In this connection, I submit that Cariño v. Insular Government20 has been modified or superseded by natural resources found therein. And Section 7 guarantees recognition and protection of their rights
our 1935, 1973 and 1987 Constitutions. Its ratio should be understood as referring only to a means of ownership and possession over such domains.
by which public agricultural land may be acquired by citizens. I must also stress that the claim of
Petitioner Cariño refers to land ownership only, not to the natural resources underneath or to the The indigenous concept of ownership, as defined under Section 5 of the law, "holds that ancestral
aerial and cosmic space above. domains are the ICC’s/IP’s private but community property which belongs to all generations and
therefore cannot be sold, disposed or destroyed." Simply put, the law declares that ancestral
Significantly, in Director of Land Management v. Court of Appeals,21 a Decision handed down after domains, including the natural resources found therein, are owned by ICCs/IPs and cannot be sold,
our three Constitutions had taken effect, the Court rejected a cultural minority member's registration disposed or destroyed. Not only does it vest ownership, as understood under the Civil Code; it adds
of land under CA 141, Section 48 (c).22 The reason was that the property fell within the Central perpetual exclusivity. This means that while ICCs/IPs could own vast ancestral domains, the majority
Cordillera Forest Reserve. This Court quoted with favor the solicitor general’s following statements: of Filipinos who are not indigenous can never own any part thereof.

"3. The construction given by respondent Court of Appeals to the particular provision of law involved, On the other hand, Section 3 (b)25 of IPRA defines ancestral lands as referring to "lands occupied,
as to include even forest reserves as susceptible to private appropriation, is to unconstitutionally possessed and utilized by individuals, families and clans of the ICCs/IPs since time immemorial x x x,
apply such provision. For, both the 1973 and present Constitutions do not include timber or forest under claims of individual or traditional group ownership, x x x including, but not limited to, residential
lands as alienable. Thus, Section 8, Article XIV of 1973 Constitution states that ‘with the exception of lots, rice terraces or paddies, private forests, swidden farms and tree lots." Section 8 recognizes and
agricultural, industrial or commercial, residential and resettlement lands of the public domain, natural protects "the right of ownership and possession of ICCs/IPs to their ancestral lands." Such ownership
resources shall not be alienated.’ The new Constitution, in its Article XII, Section 2, also expressly need not be by virtue of a certificate of title, but simply by possession since time immemorial.
states that ‘with the exception of agricultural lands, all other natural resources shall not be
alienated’." I believe these statutory provisions directly contravene Section 2, Article XII of the Constitution, more
specifically the declaration that the State owns all lands of the public domain, minerals and natural
Just recently, in Gordula v. Court of Appeals,23 the Court also stated that "forest land is incapable of resources – none of which, except agricultural lands, can be alienated. In several cases, this Court has
registration, and its inclusion in a title nullifies that title. To be sure, the defense of indefeasiblity of a consistently held that non-agricultural land must first be reclassified and converted into alienable or
certificate of title issued pursuant to a free patent does not lie against the state in an action for disposable land for agricultural purposes by a positive act of the government.26 Mere possession or
reversion of the land covered thereby when such land is a part of a public forest or of a forest utilization thereof, however long, does not automatically convert them into private properties.27 The
reservation, the patent covering forest land being void ab initio." presumption is that "all lands not appearing to be clearly within private ownership are presumed to
belong to the State. Hence, x x x all applicants in land registration proceedings have the burden of
RA 8371 Violates the Inalienability of Natural Resources and of Public Domains overcoming the presumption that the land thus sought to be registered forms part of the public
domain. Unless the applicant succeeds in showing by clear and convincing evidence that the property
The ponencia theorizes that RA 8371 does not grant to ICCs/IPs ownership of the natural resources involved was acquired by him or his ancestors either by composition title from the Spanish
found within ancestral domains. However, a simple reading of the very wordings of the law belies this Government or by possessory information title, or any other means for the proper acquisition of
statement. public lands, the property must be held to be part of the public domain. The applicant must present
competent and persuasive proof to substantiate his claim; he may not rely on general statements, or
Section 3 (a)24 defines and delineates ancestral domains as "all areas generally belonging to ICCs/IPs mere conclusions of law other than factual evidence of possession and title."28
comprising lands, inland waters, coastal areas, and natural resources therein, held under a claim of
ownership, occupied or possessed by ICCs/IPs, by themselves or through their ancestors, communally Respondents insist, and the ponencia agrees, that paragraphs (a) and (b) of Sections 3 are merely
or individually since time immemorial, continuously to the present except when interrupted by war, definitions and should not be construed independently of the other provisions of the law. But,

50
precisely, a definition is "a statement of the meaning of a word or word group."29 It determines or Already, as of June 1998, over 2.5 million hectares have been claimed by various ICCs/IPs as ancestral
settles the nature of the thing or person defined.30 Thus, after defining a term as encompassing domains; and over 10 thousand hectares, as ancestral lands.34 Based on ethnographic surveys, the
several items, one cannot thereafter say that the same term should be interpreted as excluding one solicitor general estimates that ancestral domains cover 80 percent of our mineral resources and
or more of the enumerated items in its definition. For that would be misleading the people who would between 8 and 10 million of the 30 million hectares of land in the country.35 This means that four
be bound by the law. In other words, since RA 8371 defines ancestral domains as including the natural fifths of its natural resources and one third of the country's land will be concentrated among 12
resources found therein and further states that ICCs/IPs own these ancestral domains, then it means million Filipinos constituting 110 ICCs,36 while over 60 million other Filipinos constituting the
that ICCs/IPs can own natural resources. overwhelming majority will have to share the remaining. These figures indicate a violation of the
constitutional principle of a "more equitable distribution of opportunities, income, and wealth"
In fact, Intervenors Flavier et al. submit that everything above and below these ancestral domains, among Filipinos.
with no specific limits, likewise belongs to ICCs/IPs. I say that this theory directly contravenes the
Constitution. Such outlandish contention further disregards international law which, by constitutional RA 8371 Abdicates the State Duty to Take Full Control and Supervision of Natural Resources
fiat, has been adopted as part of the law of the land.31
Section 2, Article XII of the Constitution, further provides that "[t]he exploration, development, and
No Land Area Limits Are Specified by RA 8371 utilization of natural resources shall be under the full control and supervision of the State." The State
may (1) directly undertake such activities; or (2) enter into co-production, joint venture or production-
Under Section 3, Article XII of the Constitution, Filipino citizens may acquire no more than 12 hectares sharing agreements with Filipino citizens or entities, 60 percent of whose capital is owned by
of alienable public land, whether by purchase, homestead or grant. More than that, but not exceeding Filipinos.37 Such agreements, however, shall not exceed 25 years, renewable for the same period and
500 hectares, they may hold by lease only. under terms and conditions as may be provided by law.

RA 8371, however, speaks of no area or term limits to ancestral lands and domains. In fact, by their But again, RA 8371 relinquishes this constitutional power of full control in favor of ICCs/IPs, insofar as
mere definitions, they could cover vast tracts of the nation's territory. The properties under the natural resources found within their territories are concerned. Pursuant to their rights of ownership
assailed law cover everything held, occupied or possessed "by themselves or through their ancestors, and possession, they may develop and manage the natural resources, benefit from and share in the
communally or individually since time immemorial." It also includes all "lands which may no longer profits from the allocation and the utilization thereof.38 And they may exercise such right without
be exclusively occupied by [them] but from which they traditionally had access to for their subsistence any time limit, unlike non-ICCs/IPs who may do so only for a period not exceeding 25 years, renewable
and traditional activities, particularly the home ranges of ICCs/IPs who are still nomadic and/or for a like period.39 Consistent with the Constitution, the rights of ICCs/IPs to exploit, develop and
shifting cultivators." utilize natural resources must also be limited to such period.

Nomadic groups have no fixed area within which they hunt or forage for food. As soon as they have In addition, ICCs/IPs are given the right to negotiate directly the terms and conditions for the
used up the resources of a certain area, they move to another place or go back to one they used to exploration of natural resources,40 a right vested by the Constitution only in the State. Congress,
occupy. From year to year, a growing tribe could occupy and use enormous areas, to which they could through IPRA, has in effect abdicated in favor of a minority group the State's power of ownership and
claim to have had "traditional access." If nomadic ICCs/IPs succeed in acquiring title to their enlarging full control over a substantial part of the national patrimony, in contravention of our most
ancestral domain or land, several thousands of hectares of land may yet be additionally delineated as fundamental law.
their private property.
I make clear, however, that to the extent that ICCs/IPs may undertake small-scale utilization of natural
Similarly, the Bangsa Moro people's claim to their ancestral land is not based on compounded or resources and cooperative fish farming, I absolutely have no objection. These undertakings are
consolidated title, but "on a collective stake to the right to claim what their forefathers secured for certainly allowed under the third paragraph of Section 2, Article XII of the Constitution.
them when they first set foot on our country."32 They trace their right to occupy what they deem to
be their ancestral land way back to their ancient sultans and datus, who had settled in many islands Having already disposed of the two major constitutional dilemmas wrought by RA 8371 – (1)
that have become part of Mindanao. This long history of occupation is the basis of their claim to their ownership of ancestral lands and domains and the natural resources therein; and (2) the ICCs/IPs'
ancestral lands.33 control of the exploration, development and utilization of such resources – I believe I should no longer
tackle the following collateral issues petitioners have brought up:

51
the nation's substantial wealth to them, to the exclusion of other Filipino citizens who have chosen
1. Whether the inclusion of private lands within the coverage of ancestral domains amounts to undue to live and abide by our previous and present Constitutions, would be not only unjust but also
deprivation of private property subversive of the rule of law.

2. Whether ICCs/IPs may regulate the entry/exit of migrants In giving ICCs/IPs rights in derogation of our fundamental law, Congress is effectively mandating
"reverse discrimination." In seeking to improve their lot, it would be doing so at the expense of the
3. Whether ancestral domains are exempt from real property taxes, special levies and other forms of majority of the Filipino people. Such short-sighted and misplaced generosity will spread the roots of
exaction discontent and, in the long term, fan the fires of turmoil to a conflagration of national proportions.

4. Whether customary laws and traditions of ICCs/IPs should first be applied in the settlements of Peace cannot be attained by brazenly and permanently depriving the many in order to coddle the
disputes over their rights and claims few, however disadvantaged they may have been. Neither can a just society be approximated by
maiming the healthy to place them at par with the injured. Nor can the nation survive by enclaving
5. Whether the composition and the jurisdiction of the National Commission of Indigenous Peoples its wealth for the exclusive benefit of favored minorities.
(NCIP) violate the due process and equal protection clauses
Rather, the law must help the powerless by enabling them to take advantage of opportunities and
6. Whether members of the ICCs/IPs may be recruited into the armed forces against their will privileges that are open to all and by preventing the powerful from exploiting and oppressing them.
This is the essence of social justice – empowering and enabling the poor to be able to compete with
I believe that the first three of the above collateral issues have been rendered academic or, at least, the rich and, thus, equally enjoy the blessings of prosperity, freedom and dignity.
no longer of "transcendental importance," in view of my contention that the two major IPRA
propositions are based on unconstitutional premises. On the other hand, I think that in the case of WHEREFORE, I vote to partially GRANT the Petition and to DECLARE as UNCONSTITUTIONAL Sections
the last three, it is best to await specific cases filed by those whose rights may have been injured by 3(a) and (b), 5, 6, 7(a) and (b), 8 and related provisions of RA 8371.
specific provisions of RA 8371.

Epilogue

Section 5, Article XII of the Constitution, provides:

"SEC. 5. The State, subject to the provisions of this Constitution and national development policies
and programs, shall protect the rights of indigenous cultural communities to their ancestral lands to
ensure their economic, social, and cultural well being.

"The Congress may provide for the applicability of customary laws governing property rights and
relations in determining the ownership and extent of ancestral domain."

Clearly, there are two parameters that must be observed in the protection of the rights of ICCs/IPs:
(1) the provisions of the 1987 Constitution and (2) national development policies and programs.

Indigenous peoples may have long been marginalized in Philippine politics and society. This does not,
however, give Congress any license to accord them rights that the Constitution withholds from the
rest of the Filipino people. I would concede giving them priority in the use, the enjoyment and the
preservation of their ancestral lands and domains.41 But to grant perpetual ownership and control of

52
G.R. No. 5246 September 16, 1910 therefore asked, under the provisions of section 38 of the Land Registration Act (No. 496), a revision
of the case, and that the said decree be modified so as to exclude the two parcels of land described
MANUELA GREY ALBA, ET AL., petitioners-appellants, in said motion. The Land Court upon this motion reopened the case, and after hearing the additional
vs. evidence presented by both parties, rendered, on the 23rd of November, 1908, its decision modifying
ANACLETO R. DE LA CRUZ, objector-appellee. the former decree by excluding from the same the two parcels of land claimed by Anacleto Ratilla de
la Cruz. From this decision and judgment the petitioners appealed and now insist, first, that the trial
Ramon Salinas, for appellants. court erred in reopening the case and modifying its decree dated the 12th of February, 1908, for the
Aniceto G. Reyes, for appellee. reason that said decree was not obtained by means of fraud; and, second, that the court erred in
holding that the two parcels of land described in the appellee's motion are not their property.
TRENT, J.:
It was agreed by counsel that the two small parcels now in dispute forma part of the land described
These petitioners, Manuela, Jose, Juan, and Francisco, surnamed Grey y Alba, are the only heirs of in the petition and were included in the decree of February 12, 1908, and that the petitioners are the
Doña Segunda Alba Clemente and Honorato Grey, deceased. Remedios Grey y Alba, a sister of the owners of the remainder of the land described in the said decree.
petitioners, was married on the 21st day of March, 1903, to Vicente Reyes and died on the 13th of
July, 1905, without leaving any heirs except her husband. The four petitioners, as coowners, sought The petitioners inherited this land from their parents, who acquired the same, including the two small
to have registered the following-described property: parcels in question, by purchase, as is evidenced by a public document dated the 26th of November,
1864, duly executed before Francisco Iriarte, alcalde mayor and judge of the Court of First Instance
A parcel of land situated in the barrio of Talampas, municipality of Baliuag, Province of Bulacan, upon of the Province of Bulacan.
which are situated three houses and one camarin of light material, having a superficial area of 52
hectares, 51 ares, and 22 centares; bounded on the north by the highway (calzada) of Talampas and Baldomero R. de la Cruz, father of the appellee, obtained in march, 1895, a state grant for several
the lands of Rita Ruiz Mateo; on the east by the lands of the said Rita Ruiz Mateo, Hermenegildo parcels of land, including the two parcels in question. This grant was duly inscribed in the old register
Prado, Policarpo de Jesus, and a stream called Sapang Buslut; on the south by the same stream and of property in Bulacan on the 6th of April of the same year.
the lands of the capellania; and on the west by the stream called Sapang Buslut, and the lands of
Vicente de la Cruz, Jose Camacho and Domingo Ruiz Mateo. It is admitted that at the time the appellants presented their petition in this case the appellee was
occupying the two parcels of land now in question. It is also admitted that the name of the appellee
This parcel of agricultural land is used for the raising of rice and sugar cane and is assessed at $1,000 does not appear in the said petition as an occupant of the said two parcels. The petitioners insist that
United States currency. The petition, which was filed on the 18th of December, 1906, was the appellee was occupying these parcels as their tenant and for this reason they did not include his
accompanied by a plan and technical description of the above-described parcel of land. name in their petition, as an occupant, while the appellee contends that he was occupying the said
parcels as the absolute owner under the estate grant by inheritance.
After hearing the proofs presented, the court entered, on the 12th of February, 1908, a decree in
accordance with the provisions of paragraph 6 of section 54 of Act No. 926, directing that the land The court below held that the failure on the part of the petitioners to include the name of the appellee
described in the petitioner be registered in the names of the four petitioners, as coowners, subject to in their petition, as an occupant of these two parcels of land, was a violation of section 21 of Act No.
the usufructuary right of Vicente Reyes, widower of Remedios Grey. 496, and that this constituted fraud within the meaning of section 38 of said Land Registration Act.
The trial court further held that the grant from the estate should prevail over the public document of
On the 16th of June, 1908, Anacleto Ratilla de la Cruz filed a motion in the Court of Land Registration purchase of 1864.
asking for a revision of the case, including the decision, upon the ground that he is the absolute owner
of the two parcels of land which are described in said motion, and which, according to his allegations, The mother of the petitioners died on November 15, 1881; their father died prior to that time.
are included in the lands decreed to the petitioners. He alleged that the decree of February 12, 1908, Manuela, the oldest of the petitioners, was about six years of age when their mother died. So these
was obtained maliciously and fraudulently by the petitioners, thereby depriving him of said two children were minors when the father of the appellee obtained the estate grant.
parcels of land. He further alleged that he was the absolute owner of the two parcels of land, having
inherited them from his father, Baldomero R. de la Cruz, who had a state grant for the same. He

53
On the 13th of June, 1882, Jose Grey, uncle and representative of the petitioners, who were then whom it may concern." In addition to the notice in the Official Gazette the Land Court shall, within
minors, rented the land owned by the petitioners' deceased parents to one Irineo Jose for a period of seven days after said publication, cause a copy of the notice, in Spanish, to be mailed by the clerk to
three years. On the 23d of March, 1895, the said Jose Grey, as the representative of the petitioners, every person named in the application whose address is known; to cause a duly attested copy of the
rented the same land for a period of six years to Baldomero R. de la Cruz, father of the appellee. This notice, in Spanish, to be posted in a conspicuous place on every parcel of land included in the
rental contract was duly executed in writing. This land was cultivated during these six years by application, and in a conspicuous place on the chief municipal building of the town in which the land
Baldomero R. de la Cruz and his children, one of whom is the appellee. On the 14th of December, is situated. The court may also cause other or further notice of the application to be given in such
1905, Jose Grey, for himself and the other petitioners, rented the same land to Estanislao R. de la manner and to such persons as it may deem proper. The certificate of the clerk that he has served
Cruz for a period of two years. Estanislao de la Cruz on entering into this rental contract with Jose the notice as directed by the court by publication or mailing shall be conclusive proof of such service.
Grey did so for himself and his brothers, one of whom is the appellee. While the appellee admits that Within the time allowed in the notices, if no person appears and answers, the court may at once,
his father and brother entered into these rental contracts and did, in fact, cultivate the petitioners' upon motion of the applicant, no reason to the contrary appearing, order a general default. By the
land, nevertheless he insists that the two small parcels in question were not included in these description in the published notice "to all whom it may concern," and by express provisions of law "all
contracts. In the rental contract between the uncle of the petitioners and he father of the appellee the word are made parties defendant and shall be concluded by the default an order." If the court,
the land is not described. In the rental contract between Jose Grey, one of the petitioners, and after hearing, finds that the applicant has title, as stated in his application, a decree or registration
Estanislao R. de la Cruz, brother of the appellee, the two small parcels of land in question are included, shall be entered.
according to the description given therein. This was found to be true by the court below, but the said
court held that as this contract was made by Estanislao R. de la Cruz it was not binding upon Anacleto Every decree of registration shall bind the land and quiet title thereto, subject only to the exceptions
R. de la Cruz, the appellee. stated in the following section. It shall be conclusive upon and against all persons, including the Insular
Government, and all the branches thereof, whether mentioned by name in the application, notice, or
The two small parcels of land in question were purchased by the parents of the petitioners in 1864, citation, or included in the general description "to all whom it may concern." Such decree shall not be
as is evidenced by the public document of purchase and sale of that year. The same two parcels of opened by reason of the absence, infancy, or other disability of any person affected thereby, nor by
land are included in the state grant issued in favor of Baldomero Ratilla de la Cruz in 1895. This grant any proceedings in any court for reversing judgments or decrees; subject, however, to the right of
was obtained after the death of the petitioners' parents and while they were minors. So it is clear that any person deprived of land or of any estate or interest therein by decree of registration obtained by
the petitioners honestly believed that the appellee was occupying the said parcels as their lessee at fraud to file in the Court of Land Registration a petition for review within one year. . . . (Sec. 38 of Act
the time they presented their application for registration. They did not act in bad faith, nor with any No. 496.)
fraudulent intent, when they omitted to include in their application the name of the appellee as one
of the occupants of the land. They believed that it was not necessary nor required that they include The appellee is not included in any of the exceptions named in section 38 referred to above.
in their application the names of their tenants. Under these circumstances, did the court below
commit an error in reopening this case in June, 1908, after its decree had been entered in February It will be seen that the applicant is required to mention not only the outstanding interest which he
of the same year? admits but also all claims of interest, though denied by him. By express provision of law the world are
made parties defendant by the description in the notice "to all whom it may concern."
The application for the registration is to be in writing, signed and sworn to by the applicant, or by
some person duly authorized in his behalf. It is to contain an accurate description of the land. It shall Although the appellee, occupying the two small parcels of land in question under the circumstances
contain the name in full and the address of the applicant, and also the names and addresses of all as we have set forth, was not served with notice, he was made a party defendant by publication; and
occupants of land and of all adjoining owners, if known; and, if not known, it shall state what search the entering of a decree on the 12th of February, 1908, must be held to be conclusive against all
has been made to find them. In the form of notice given by statute, which shall be sworn to, the persons, including the appellee, whether his (appellee's) name is mentioned in the application, notice,
applicant is required to state and set forth clearly all mortgages or encumbrances affecting said land, or citation.
if any, the rights and interests, legal or equitable, in the possession, remainder, reversion, or
expectancy of all persons, with their names in full, together with their place of residence and post The said decree of February 12, 1908, should not have been opened on account of the absence,
office addresses. Upon receipt of the application the clerk shall cause notice of the filling to be infancy, or other disability of any person affected thereby, and could have been opened only on the
published twice in the Official Gazette. This published notice shall be directed to all persons appearing ground that the said decree had been obtained by fraud. That decree was not obtained by fraud on
to have an interest in the land sought to be registered and to the adjoining owners, and also "to all the part of the applicants, inasmuch as they honestly believed that the appellee was occupying these

54
two small parcels of land as their tenant. One of the petitioner went upon the premises with the It might be urged that the appellee has been deprived of his property without due process of law, in
surveyor when the original plan was made. violation of section 5 of the Act of Congress of July 1, 1902, known as the Philippine Bill," which
provides "that no law shall be enacted in the said Islands which shall deprive any person of life, liberty,
Proof of constructive fraud is not sufficient to authorize the Court of Land Registration to reopen a or property without due process of law."
case and modify its decree. Specific, intentional acts to deceive and deprive anther of his right, or in
some manner injure him, must be alleged and proved; that is, there must be actual or positive fraud The Land Registration Act requires that all occupants be named in the petition and given notice by
as distinguished from constructive fraud. registered mail. This did not do the appellee any good, as he was not notified; but he was made a
party defendant, as we have said, by means of the publication "to all whom it may concern." If this
The question as to the meaning of the word "fraud" in the Australian statutes has been frequently section of the Act is to be upheld this must be declared to be due process of law.
raised. Two distinctions have been noted by the Australian courts; the first is the distinction between
the meaning of the word "fraud" in the sections relating to the conclusive effect of certificates of title, Before examining the validity of this part of the Act it might be well to note the history and purpose
and its meaning in the sections relating to the protection of bona fide purchasers from registered of what is known as the "Torrens Land Registration System." This system was introduced in South
proprietors. The second is the distinction between "legal," "equitable," or "constructive" fraud, and Australia by Sir Robert Torrens in 1857 and was there worked out in its practicable form.
"actual" or "moral" fraud. In none of the groups of the sections of the Australian statutes relating to
the conclusive effect of certificates of title, and in which fraud is referred to, is there any express The main principle of registration is to make registered titles indefeasible. As we have said, upon the
indication of the meaning of "fraud," with the sole exception of that of the South Australian group. presentation in the Court of Land Registration of an application for the registration of the title to
(Hogg on Australian Torrens System, p. 834.) lands, under this system, the theory of the law is that all occupants, adjoining owners, adverse
claimants, and other interested persons are notified of the proceedings, and have have a right to
With regard to decisions on the sections relating to the conclusive effect of certificates of title, it has appear in opposition to such application. In other words, the proceeding is against the whole word.
been held in some cases that the "fraud" there mentioned means actual or moral fraud, not merely This system was evidently considered by the Legislature to be a public project when it passed Act No.
constructive or legal fraud. In other cases "fraud" has been said to include constructive, legal, and 496. The interest of the community at large was considered to be preferred to that of private
every kind of fraud. In other cases, against, knowledge of other persons' right, and the deliberate individuals.
acquisition of registered title in the face of such knowledge, has been held to be "fraud" which
rendered voidable the certificates of title so obtained; and voluntary ignorance is, for this purpose, At the close of this nineteenth century, all civilized nations are coming to registration of title to land,
the same as knowledge. But in none of these three classes of cases was there absent the element of because immovable property is becoming more and more a matter of commercial dealing, and there
intention to deprive another of just rights, which constitutes the essential characteristics of actual — can be no trade without security. (Dumas's Lectures, p. 23.)
as distinguished from legal-fraud. (Id., p. 835, and cases cited in notes Nos. 85, 86, 87, 88, and 89 at
bottom of pages 835 and 836.) The registered proprietor will no longer have reasons to fear that he may evicted because his vendor
had, unknown to him, already sold the and to a third person. . . The registered proprietor may feel
By "fraud" is meant actual fraud-dishonesty of some sort. (Judgment of Privy Council in Assets Co. vs. himself protected against any defect in his vendor's title. (Id., p. 21.)
Mere Roihi, and Assets Co. vs. Panapa Waihopi, decided in March, 1905, cited by Hogg in his
Supplementary Addendum to his work on Australian Torrens System, supra.) The same meaning The following summary of benefits of the system of registration of titles, made by Sir Robert Torrens,
should be given to the word "fraud" used in section 38 of our statutes (Act No. 496). has been fully justified in its use:

The question as to whether any particular transaction shows fraud, within the meaning of the word First. It has substituted security for insecurity.
as used in our statutes, will in each case be a question of fact. We will not attempt to say what acts
would constitutes this kind of fraud in other cases. This must be determined from the fact an Second. It has reduced the costs of conveyances from pounds to shillings, and the time occupied from
circumstances in each particular case. The only question we are called upon to determine, and have months to days.
determined, is whether or not, under the facts and circumstances in this case, the petitioners did
obtain the decree of February 12, 1908, by means of fraud. Third. It has exchanged brevity and clearness for obscurity and verbiage.

55
Fourth. It has so simplified ordinary dealings that he who has mastered the "three R's" can transact
his own conveyancing. The Illinois and Massachusetts statutes were upheld by the supreme courts of those States.

Fifth. It affords protection against fraud. It is not enough to show a procedure to be unconstitutional to say that we never heard of it before.
(Tyler vs. Judges, supra; Hurtado vs. California, 110 U. S., 516.)
Sixth. It has restored to their just value many estates held under good holding titles, but depreciated
in consequence of some blur or technical defect, and has barred the reoccurrence of any similar faults. Looked at either from the point of view of history or of the necessary requirements of justice, a
(Sheldon on Land Registration, pp. 75, 76.) proceeding in rem dealing with a tangible res may be instituted and carried to judgment without
personal service upon claimants within the State or notice by name to those outside of it, and not
The boldest effort to grapple with the problem of simplification of title to land was made by Mr. encounter any provision of either constitution. Jurisdiction is secured by the power of the court over
(afterwards Sir Robert) Torrens, a layman, in South Australia in 1857. . . . In the Torrens system title the res. As we have said, such a proceeding would be impossible, were this not so, for it hardly would
by registration takes the place of "title by deeds" of the system under the "general" law. A sale of do to make a distinction between the constitutional rights of claimants who were known and those
land, for example, is effected by a registered transfer, upon which a certificate of title is issued. The who were not known to the plaintiff, when the proceeding is to bar all. (Tyler vs. Judges, supra.)
certificate is guaranteed by statute, and, with certain exceptions, constitutes indefeasible title to the
land mentioned therein. Under the old system the same sale would be effected by a conveyance, This same doctrine is annunciated in Pennoyer vs. Neff (95 U. S., 714); The Mary (9 Cranch, 126);
depending for its validity, apart from intrinsic flaws, on the correctness of a long series of prior deeds, Mankin vs. Chandler (2 Brock., 125); Brown vs. Levee Commission (50 Miss., 468); 2 Freeman,
wills, etc. . . . The object of the Torrens system, them, is to do away with the delay, uncertainty, and Judgments, 4th ed., secs. 606, 611.
expense of the old conveyancing system. (Duffy & Eagleson on The Transfer of Land Act, 1890, pp. 2,
3, 5, 7.) If the technical object of the suit is to establish a claim against some particular person, with a
judgment which generally, in theory at least, binds his body, or to bar some individual claim or
By "Torrens" system generally are meant those systems of registration of transactions with interest objection, so that only certain persons are entitled to be heard in defense, the action is in personam,
in land whose declared object . . . is, under governmental authority, to establish and certify to the although it may concern the right to or possession of a tangible thing. If, on the other hand, the object
ownership of an absolute and indefeasible title to realty, and to simplify its transfer. (Hogg on is to bar indifferently all who might be minded to make an objection of any sort against the right
Australian Torrens system, supra, pp. 1, 2.) sought to be established, and if anyone in the world has a right to be heard on the strenght of alleging
facts which, if true, show an inconsistent interest, the proceeding is in rem. (Tyler vs. Judges, supra.)
Compensation for errors from assurance funds is provided in all countries in which the Torrens system
has been enacted. Cases of error no doubt will always occur. The percentage of errors, as compared In the case of Hamilton vs. Brown (161 U. S., 256) a judgment of escheat was held conclusive upon
with the number of registered dealings in Australia, is very small. In New South Wales there were, in persons notified by advertisement to all persons interested. In this jurisdiction, by the provisions of
1889, 209, 894 registered dealings, the average risk of error being only 2 ½ cents for each dealing. In the Code of Civil Procedure, Act No. 190, a decree allowing or disallowing a will binds everybody,
Queensland the risk of error was only 1 ½ cents, the number of registered dealings being 233,309. In although the only notice of the proceedings given is by general notice to all persons interested.
Tasmania and in Western Australia not a cent was paid for compensation for errors during the whole
time of operation, (Dumas's Lectures, supra, p. 96.) This system has been adopted in various countries The supreme court Massachusetts, in the case of Tyler vs. Judges (supra), did not rest its judgment as
of the civilized world, including some of the States of the American Union, and practical experience to the conclusive effect of the decree upon the ground that the State has absolute power to
has demonstrated that it has been successful as a public project. determine the persons to whom a man's property shall go at his death, but upon the characteristics
of a proceeding in rem. So we conclude that the proceedings had in the case at bar, under all the facts
The validity of some of the provisions of the statutes adopting the Torrens system has been the and circumstances, especially the absolute lack on the part of the petitioners of any dishonest intent
subject of judicial decision in the courts of the United States. (People vs. Chase, 165 Ill., 527; State vs. to deprive the appellee of any right, or in any way injure him, constitute due process of law.
Guilbert, 56 Ohio St., 575; People vs. Simon, 176 Ill., 165; Tyler vs. Judges, 175 Mass., 71.)
As to whether or not the appellee can succesfully maintain an action under the provisions of sections
Act No. 496 of the Philippine Commission, known as the "Land Registration Act," was copied 101 and 102 of the Land Registration Act (secs. 2365, 2366, Compilation) we do not decide.
substantially from the Massachussetts law of 1898.

56
For these reasons we are of the opinion, and so hold, that the judgment appealed from should be,
and the same is hereby reversed and judgment entered in favor of the petitioners in conformity with
the decree of the lower court of February 12, 1908, without special ruling as to costs. It is so ordered.

Arellano, C.J., Torres, Johnson and Moreland, JJ., concur.

57
G.R. No. L-8936 October 2, 1915 Sixth. That the land occupied by t he wall is registered in the name of each of the owners of the
adjoining lots. The wall is not a joint wall.
CONSUELO LEGARDA, with her husband MAURO PRIETO, plaintiffs-appellants,
vs. Under these facts, who is the owner of the wall and the land occupied by it?
N.M. SALEEBY, defendant-appellee.
The decision of the lower court is based upon the theory that the action for the registration of the lot
Singson, Ledesma and Lim for appellants. of the defendant was a judicial proceeding and that the judgment or decree was binding upon all
D.R. Williams for appellee. parties who did not appear and oppose it. In other words, by reason of the fact that the plaintiffs had
not opposed the registration of that part of the lot on which the wall was situate they had lost it, even
though it had been theretofore registered in their name. Granting that theory to be correct one, and
JOHNSON, J.: granting even that the wall and the land occupied by it, in fact, belonged to the defendant and his
predecessors, then the same theory should be applied to the defendant himself. Applying that theory
From the record the following facts appear: to him, he had already lost whatever right he had therein, by permitting the plaintiffs to have the
same registered in their name, more than six years before. Having thus lost hid right, may he be
First. That the plaintiffs and the defendant occupy, as owners, adjoining lots in the district of Ermita permitted to regain it by simply including it in a petition for registration? The plaintiffs having secured
in the city of Manila. the registration of their lot, including the wall, were they obliged to constantly be on the alert and to
watch all the proceedings in the land court to see that some one else was not having all, or a portion
Second. That there exists and has existed a number of years a stone wall between the said lots. Said of the same, registered? If that question is to be answered in the affirmative, then the whole scheme
wall is located on the lot of the plaintiffs. and purpose of the torrens system of land registration must fail. The real purpose of that system is to
quiet title to land; to put a stop forever to any question of the legality of the title, except claims which
Third. That the plaintiffs, on the 2d day of March, 1906, presented a petition in the Court of Land were noted at the time of registration, in the certificate, or which may arise subsequent thereto. That
Registration for the registration of their lot. After a consideration of said petition the court, on the being the purpose of the law, it would seem that once a title is registered the owner may rest secure,
25th day of October, 1906, decreed that the title of the plaintiffs should be registered and issued to without the necessity of waiting in the portals of the court, or sitting in the "mirador de su casa," to
them the original certificate provided for under the torrens system. Said registration and certificate avoid the possibility of losing his land. Of course, it can not be denied that the proceeding for the
included the wall. registration of land under the torrens system is judicial (Escueta vs. .Director of Lands, 16 Phil. Rep.,
482). It is clothed with all the forms of an action and the result is final and binding upon all the world.
Fourth. Later the predecessor of the defendant presented a petition in the Court of Land Registration It is an action in rem. (Escueta vs. Director of Lands (supra); Grey Alba vs. De la Cruz, 17 Phil. rep., 49
for the registration of the lot now occupied by him. On the 25th day of March, 1912, the court decreed Roxas vs. Enriquez, 29 Phil. Rep., 31; Tyler vs. Judges, 175 Mass., 51 American Land Co. vs. Zeiss, 219
the registration of said title and issued the original certificate provided for under the torrens system. U.S., 47.)
The description of the lot given in the petition of the defendant also included said wall.
While the proceeding is judicial, it involves more in its consequences than does an ordinary action. All
Fifth. Several months later (the 13th day of December, 1912) the plaintiffs discovered that the wall the world are parties, including the government. After the registration is complete and final and there
which had been included in the certificate granted to them had also been included in the certificate exists no fraud, there are no innocent third parties who may claim an interest. The rights of all the
granted to the defendant .They immediately presented a petition in the Court of Land Registration world are foreclosed by the decree of registration. The government itself assumes the burden of
for an adjustment and correction of the error committed by including said wall in the registered title giving notice to all parties. To permit persons who are parties in the registration proceeding (and they
of each of said parties. The lower court however, without notice to the defendant, denied said are all the world) to again litigate the same questions, and to again cast doubt upon the validity of
petition upon the theory that, during the pendency of the petition for the registration of the the registered title, would destroy the very purpose and intent of the law. The registration, under the
defendant's land, they failed to make any objection to the registration of said lot, including the wall, torrens system, does not give the owner any better title than he had. If he does not already have a
in the name of the defendant. perfect title, he can not have it registered. Fee simple titles only may be registered. The certificate of
registration accumulates in open document a precise and correct statement of the exact status of the
fee held by its owner. The certificate, in the absence of fraud, is the evidence of title and shows exactly

58
the real interest of its owner. The title once registered, with very few exceptions, should not registered owner shall hold the title, and the effect of this undoubtedly is that where two certificates
thereafter be impugned, altered, changed, modified, enlarged, or diminished, except in some direct purport to include the same registered land, the holder of the earlier one continues to hold the title"
proceeding permitted by law. Otherwise all security in registered titles would be lost. A registered (p. 237).
title can not be altered, modified, enlarged, or diminished in a collateral proceeding and not even by
a direct proceeding, after the lapse of the period prescribed by law. Section 38 of Act No. 496, provides that; "It (the decree of registration) shall be conclusive upon and
against all persons, including the Insular Government and all the branches thereof, whether
For the difficulty involved in the present case the Act (No. 496) providing for the registration of titles mentioned by name in the application, notice, or citation, or included in the general description "To
under the torrens system affords us no remedy. There is no provision in said Act giving the parties all whom it may concern." Such decree shall not be opened by reason of the absence, infancy, or
relief under conditions like the present. There is nothing in the Act which indicates who should be the other disability of any person affected thereby, nor by any proceeding in any court for reversing
owner of land which has been registered in the name of two different persons. judgments or decrees; subject, however, to the right of any person deprived of land or of any estate
or interest therein by decree of registration obtained by fraud to file in the Court of Land Registration
The rule, we think, is well settled that the decree ordering the registration of a particular parcel of a petition for review within one year after entry of the decree (of registration), provided no innocent
land is a bar to future litigation over the same between the same parties .In view of the fact that all purchaser for value has acquired an interest.
the world are parties, it must follow that future litigation over the title is forever barred; there can be
no persons who are not parties to the action. This, we think, is the rule, except as to rights which are It will be noted, from said section, that the "decree of registration" shall not be opened, for any
noted in the certificate or which arise subsequently, and with certain other exceptions which need reason, in any court, except for fraud, and not even for fraud, after the lapse of one year. If then the
not be dismissed at present. A title once registered can not be defeated, even by an adverse, open, decree of registration can not be opened for any reason, except for fraud, in a direct proceeding for
and notorious possession. Registered title under the torrens system can not be defeated by that purpose, may such decree be opened or set aside in a collateral proceeding by including a portion
prescription (section 46, Act No. 496). The title, once registered, is notice to the world. All persons of the land in a subsequent certificate or decree of registration? We do not believe the law
must take notice. No one can plead ignorance of the registration. contemplated that a person could be deprived of his registered title in that way.

The question, who is the owner of land registered in the name of two different persons, has been We have in this jurisdiction a general statutory provision which governs the right of the ownership of
presented to the courts in other jurisdictions. In some jurisdictions, where the "torrens" system has land when the same is registered in the ordinary registry in the name of two persons. Article 1473 of
been adopted, the difficulty has been settled by express statutory provision. In others it has been the Civil Code provides, among other things, that when one piece of real property had been sold to
settled by the courts. Hogg, in his excellent discussion of the "Australian Torrens System," at page two different persons it shall belong to the person acquiring it, who first inscribes it in the registry.
823, says: "The general rule is that in the case of two certificates of title, purporting to include the This rule, of course, presupposes that each of the vendees or purchasers has acquired title to the
same land, the earlier in date prevails, whether the land comprised in the latter certificate be wholly, land. The real ownership in such a case depends upon priority of registration. While we do not now
or only in part, comprised in the earlier certificate. (Oelkers vs. Merry, 2 Q.S.C.R., 193; Miller vs. Davy, decide that the general provisions of the Civil Code are applicable to the Land Registration Act, even
7 N.Z.R., 155; Lloyd vs. Myfield, 7 A.L.T. (V.) 48; Stevens vs. Williams, 12 V.L. R., 152; Register of Titles, though we see no objection thereto, yet we think, in the absence of other express provisions, they
vs. Esperance Land Co., 1 W.A.R., 118.)" Hogg adds however that, "if it can be very clearly ascertained should have a persuasive influence in adopting a rule for governing the effect of a double registration
by the ordinary rules of construction relating to written documents, that the inclusion of the land in under said Act. Adopting the rule which we believe to be more in consonance with the purposes and
the certificate of title of prior date is a mistake, the mistake may be rectified by holding the latter of the real intent of the torrens system, we are of the opinion and so decree that in case land has been
the two certificates of title to be conclusive." (See Hogg on the "Australian torrens System," supra, registered under the Land Registration Act in the name of two different persons, the earlier in date
and cases cited. See also the excellent work of Niblack in his "Analysis of the Torrens System," page shall prevail.
99.) Niblack, in discussing the general question, said: "Where two certificates purport to include the
same land the earlier in date prevails. ... In successive registrations, where more than one certificate In reaching the above conclusion, we have not overlooked the forceful argument of the appellee. He
is issued in respect of a particular estate or interest in land, the person claiming under the prior says, among other things; "When Prieto et al. were served with notice of the application of Teus (the
certificates is entitled to the estate or interest; and that person is deemed to hold under the prior predecessor of the defendant) they became defendants in a proceeding wherein he, Teus, was
certificate who is the holder of, or whose claim is derived directly or indirectly from the person who seeking to foreclose their right, and that of orders, to the parcel of land described in his application.
was the holder of the earliest certificate issued in respect thereof. While the acts in this country do Through their failure to appear and contest his right thereto, and the subsequent entry of a default
not expressly cover the case of the issue of two certificates for the same land, they provide that a judgment against them, they became irrevocably bound by the decree adjudicating such land to Teus.

59
They had their day in court and can not set up their own omission as ground for impugning the validity "innocent purchaser," by virtue of the provisions of said sections. In the present case Teus had his
of a judgment duly entered by a court of competent jurisdiction. To decide otherwise would be to land, including the wall, registered in his name. He subsequently sold the same to the appellee. Is the
hold that lands with torrens titles are above the law and beyond the jurisdiction of the courts". appellee an "innocent purchaser," as that phrase is used in said sections? May those who have been
deprived of their land by reason of a mistake in the original certificate in favor of Teus be deprived of
As was said above, the primary and fundamental purpose of the torrens system is to quiet title. If the their right to the same, by virtue of the sale by him to the appellee? Suppose the appellants had sold
holder of a certificate cannot rest secure in this registered title then the purpose of the law is their lot, including the wall, to an "innocent purchaser," would such purchaser be included in the
defeated. If those dealing with registered land cannot rely upon the certificate, then nothing has been phrase "innocent purchaser," as the same is used in said sections? Under these examples there would
gained by the registration and the expense incurred thereby has been in vain. If the holder may lose be two innocent purchasers of the same land, is said sections are to be applied .Which of the two
a strip of his registered land by the method adopted in the present case, he may lose it all. Suppose innocent purchasers, if they are both to be regarded as innocent purchasers, should be protected
within the six years which elapsed after the plaintiff had secured their title, they had mortgaged or under the provisions of said sections? These questions indicate the difficulty with which we are met
sold their right, what would be the position or right of the mortgagee or vendee? That mistakes are in giving meaning and effect to the phrase "innocent purchaser," in said sections.
bound to occur cannot be denied, and sometimes the damage done thereby is irreparable. It is the
duty of the courts to adjust the rights of the parties under such circumstances so as to minimize such May the purchaser of land which has been included in a "second original certificate" ever be regarded
damages, taking into consideration al of the conditions and the diligence of the respective parties to as an "innocent purchaser," as against the rights or interest of the owner of the first original
avoid them. In the present case, the appellee was the first negligent (granting that he was the real certificate, his heirs, assigns, or vendee? The first original certificate is recorded in the public registry.
owner, and if he was not the real owner he can not complain) in not opposing the registration in the It is never issued until it is recorded. The record notice to all the world. All persons are charged with
name of the appellants. He was a party-defendant in an action for the registration of the lot in the knowledge of what it contains. All persons dealing with the land so recorded, or any portion of it,
question, in the name of the appellants, in 1906. "Through his failure to appear and to oppose such must be charged with notice of whatever it contains. The purchaser is charged with notice of every
registration, and the subsequent entry of a default judgment against him, he became irrevocably fact shown by the record and is presumed to know every fact which the record discloses .This rule is
bound by the decree adjudicating such land to the appellants. He had his day in court and should not so well established that it is scarcely necessary to cite authorities in its support (Northwestern
be permitted to set up his own omissions as the ground for impugning the validity of a judgment duly National Bank vs. Freeman, 171 U.S., 620, 629; Delvin on Real Estate, sections 710, 710 [a]).
entered by a court of competent jurisdiction." Granting that he was the owner of the land upon which
the wall is located, his failure to oppose the registration of the same in the name of the appellants, in When a conveyance has been properly recorded such record is constructive notice of its contents and
the absence of fraud, forever closes his mouth against impugning the validity of that judgment. There all interests, legal and equitable, included therein. (Grandin vs. Anderson, 15 Ohio State, 286, 289;
is no more reason why the doctrine invoked by the appellee should be applied to the appellants than Orvis vs. Newell, 17 Conn., 97; Buchanan vs. Intentional Bank, 78 Ill., 500; Youngs vs. Wilson, 27 N.Y.,
to him. 351; McCabe vs. Grey, 20 Cal., 509; Montefiore vs. Browne, 7 House of Lords Cases, 341.)

We have decided, in case of double registration under the Land Registration Act, that the owner of Under the rule of notice, it is presumed that the purchaser has examined every instrument of record
the earliest certificate is the owner of the land. That is the rule between original parties. May this rule affecting the title. Such presumption is irrebutable. He is charged with notice of every fact shown by
be applied to successive vendees of the owners of such certificates? Suppose that one or the other the record and is presumed to know every fact which an examination of the record would have
of the parties, before the error is discovered, transfers his original certificate to an "innocent disclosed. This presumption cannot be overcome by proof of innocence or good faith. Otherwise the
purchaser." The general rule is that the vendee of land has no greater right, title, or interest than his very purpose and object of the law requiring a record would be destroyed. Such presumption cannot
vendor; that he acquires the right which his vendor had, only. Under that rule the vendee of the earlier be defeated by proof of want of knowledge of what the record contains any more than one may be
certificate would be the owner as against the vendee of the owner of the later certificate. permitted to show that he was ignorant of the provisions of the law. The rule that all persons must
take notice of the facts which the public record contains is a rule of law. The rule must be absolute.
We find statutory provisions which, upon first reading, seem to cast some doubt upon the rule that Any variation would lead to endless confusion and useless litigation.
the vendee acquires the interest of the vendor only. Sections 38, 55, and 112 of Act No. 496 indicate
that the vendee may acquire rights and be protected against defenses which the vendor would not. While there is no statutory provision in force here requiring that original deeds of conveyance of real
Said sections speak of available rights in favor of third parties which are cut off by virtue of the sale property be recorded, yet there is a rule requiring mortgages to be recorded. (Arts. 1875 and 606 of
of the land to an "innocent purchaser." That is to say, persons who had had a right or interest in land the Civil Code.) The record of a mortgage is indispensable to its validity. (Art .1875.) In the face of that
wrongfully included in an original certificate would be unable to enforce such rights against an statute would the courts allow a mortgage to be valid which had not been recorded, upon the plea of

60
ignorance of the statutory provision, when third parties were interested? May a purchaser of land, one who had acquired rights in conflict therewith and who had full and complete knowledge of their
subsequent to the recorded mortgage, plead ignorance of its existence, and by reason of such rights. The purchaser of land included in the second original certificate, by reason of the facts
ignorance have the land released from such lien? Could a purchaser of land, after the recorded contained in the public record and the knowledge with which he is charged and by reason of his
mortgage, be relieved from the mortgage lien by the plea that he was a bona fide purchaser? May negligence, should suffer the loss, if any, resulting from such purchase, rather than he who has
there be a bona fide purchaser of said land, bona fide in the sense that he had no knowledge of the obtained the first certificate and who was innocent of any act of negligence.
existence of the mortgage? We believe the rule that all persons must take notice of what the public
record contains in just as obligatory upon all persons as the rule that all men must know the law; that The foregoing decision does not solve, nor pretend to solve, all the difficulties resulting from double
no one can plead ignorance of the law. The fact that all men know the law is contrary to the registration under the torrens system and the subsequent transfer of the land. Neither do we now
presumption. The conduct of men, at times, shows clearly that they do not know the law. The rule, attempt to decide the effect of the former registration in the ordinary registry upon the registration
however, is mandatory and obligatory, notwithstanding. It would be just as logical to allow the under the torrens system. We are inclined to the view, without deciding it, that the record under the
defense of ignorance of the existence and contents of a public record. torrens system, supersede all other registries. If that view is correct then it will be sufficient, in dealing
with land registered and recorded alone. Once land is registered and recorded under the torrens
In view, therefore, of the foregoing rules of law, may the purchaser of land from the owner of the system, that record alone can be examined for the purpose of ascertaining the real status of the title
second original certificate be an "innocent purchaser," when a part or all of such land had theretofore to the land.
been registered in the name of another, not the vendor? We are of the opinion that said sections 38,
55, and 112 should not be applied to such purchasers. We do not believe that the phrase "innocent It would be seen to a just and equitable rule, when two persons have acquired equal rights in the
purchaser should be applied to such a purchaser. He cannot be regarded as an "innocent purchaser" same thing, to hold that the one who acquired it first and who has complied with all the requirements
because of the facts contained in the record of the first original certificate. The rule should not be of the law should be protected.
applied to the purchaser of a parcel of land the vendor of which is not the owner of the original
certificate, or his successors. He, in nonsense, can be an "innocent purchaser" of the portion of the In view of our conclusions, above stated, the judgment of the lower court should be and is hereby
land included in another earlier original certificate. The rule of notice of what the record contains revoked. The record is hereby returned to the court now having and exercising the jurisdiction
precludes the idea of innocence. By reason of the prior registry there cannot be an innocent purchaser heretofore exercised by the land court, with direction to make such orders and decrees in the
of land included in a prior original certificate and in a name other than that of the vendor, or his premises as may correct the error heretofore made in including the land in the second original
successors. In order to minimize the difficulties we think this is the safe rule to establish. We believe certificate issued in favor of the predecessor of the appellee, as well as in all other duplicate
the phrase "innocent purchaser," used in said sections, should be limited only to cases where certificates issued.
unregistered land has been wrongfully included in a certificate under the torrens system. When land
is once brought under the torrens system, the record of the original certificate and all subsequent Without any findings as to costs, it is so ordered.
transfers thereof is notice to all the world. That being the rule, could Teus even regarded as the holder
in good fifth of that part of the land included in his certificate of the appellants? We think not. Arellano, C.J., Torrens, and Araullo, JJ., concur.
Suppose, for example, that Teus had never had his lot registered under the torrens system. Suppose
he had sold his lot to the appellee and had included in his deed of transfer the very strip of land now Separate Opinions
in question. Could his vendee be regarded as an "innocent purchaser" of said strip? Would his vendee
be an "innocent purchaser" of said strip? Certainly not. The record of the original certificate of the TRENT, J., dissenting:
appellants precludes the possibility. Has the appellee gained any right by reason of the registration of
the strip of land in the name of his vendor? Applying the rule of notice resulting from the record of I dissent.
the title of the appellants, the question must be answered in the negative. We are of the opinion that
these rules are more in harmony with the purpose of Act No. 496 than the rule contended for by the In cases of double or overlapping registration, I am inclined to agree with the reasoning and authority
appellee. We believe that the purchaser from the owner of the later certificate, and his successors, on which it is held in the majority opinion (first) that the original holder of the prior certificate is
should be required to resort to his vendor for damages, in case of a mistake like the present, rather entitled to the land as against the original holder of the later certificate, where there has been no
than to molest the holder of the first certificate who has been guilty of no negligence. The holder of transfer of title by either party to an innocent purchaser; both, as is shown in the majority opinion,
the first original certificate and his successors should be permitted to rest secure in their title, against being at fault in permitting the double registration to take place; (second) that an innocent purchaser

61
claiming under the prior certificate is entitled to the land as against the original holder of the later registration the earlier certificate should be protected, ought not to prevail so as to deprive an
certificate, and also as against innocent purchasers from the holder of the later certificate; the innocent purchaser under the later certificate of his title of the earlier certificate contributed to the
innocent purchaser being in no wise at fault in connection with the issuance of the later certificate. issuance of the later certificate. Hence the holder of the earlier certificate of title should not be heard
to invoke the "just and equitable rule" as laid down in the majority opinion, in order to have his own
But I am of opinion that neither the authorities cited, nor the reasoning of the majority opinion title protected and the title of an innocent purchaser of a later certificate cancelled or annulled, in
sustains the proposition that the original holder of the prior certificate is entitled to the land as against any case wherein it appears that the holder of the later certificate was wholly without fault, while the
an innocent purchaser from the holder of the later certificate. holder of the issuance of the later certificate, in that he might have prevented its issuance by merely
entering his appearance in court in response to lawful summons personally served upon him in the
As to the text-book authorities cited in the majority opinion, it is sufficient to say that the rules laid course of the proceedings for the issuance of the second certificate, and pleading his superior rights
down by both Hogg and Niblack are mere general rules, admittedly subject to exception, and of course under the earlier certificate, instead of keeping silent and by his silence permitting a default judgment
of no binding force or authority where the reasoning upon which these rules are based is applicable to be entered against him adjudicating title in favor of the second applicant.
to the facts developed in a particular case.
The majority opinion clearly recognizes the soundness of the principles I am contending for by
In its last analysis the general rule laid down in the majority opinion rests upon the proposition set reasoning (with which I am inclined to agree) whereby it undertakes to demonstrate that as between
forth in the last page of the opinion wherein it is said that "it would seem to be a just and equitable the original holders of the double or overlapping registration the general rule should prevail, because
rule, when two persons have acquired equal rights in the same thing, to hold that the one who both such original parties must held to have been fault and, their equities being equal, preference
acquired it first and who has complied with all the requirements of the law should be protected." The should be given to the earlier title.
rule, as applied to the matter in hand, may be stated as follows: It would seem to be a just and
equitable rule when two persons have acquired separate and independent registered titles to the The majority opinion further recognizes the soundness of my contention by the reasoning whereby it
same land, under the Land Registration Act, to hold that the one who first acquired registered title undertakes to sustain the application of the general rule in favor of the original holder of the earlier
and who has complied with all the requirements of the law in that regard should be protected, in the certificate against purchasers from the original holder of the later certificate, by an attempt to
absence of any express statutory provision to the contrary. demonstrate that such purchasers can in no event be held to be innocent purchasers; because, as it
is said, negligence may and should always be imputed to such a purchaser, so that in no event can he
Thus stated I have no quarrel with the doctrine as a statement of the general rule to be applied in claim to be without fault when it appears that the lands purchased by him from the holder of a duly
cases of double or overlapping registration under the Land Registration Act; for it is true as stated in registered certificate of title are included within the bounds of the lands described in a certificate of
the majority opinion that in the adjudication and registration of titles by the Courts of Land title of an earlier date.
Registration "mistakes are bound to occur, and sometimes the damage done thereby is irreparable;"
and that in the absence of statutory provisions covering such cases, "it is the duty of the courts to At considerable length the majority opinion (in reliance upon the general rule laid down under the
adjust the rights of the parties, under such circumstances, so as to minimize such damages, taking various systems of land registration, other than those based on the torrens system) insists that a
into consideration all of the conditions, and the diligence of the respective parties to avoid them." purchaser of land land duly registered in the Land Registration Court, is charged with notice of the
contents of each and every one of the thousands and tens of thousands of certificates of registry on
But like most such general rules, it has its exceptions and should not be applied in a case wherein the file in the land registry office, so that negligence may be imputed to him if he does not ascertain that
reasons on which it is based do not exist, or in cases wherein still more forceful reasons demand the all or any part of the land purchased by him is included within the boundary lines of any one of the
application of a contrary rule. thousands or tens of thousands of tracts of land whose original registry bears an earlier date than the
date of the original registry of the land purchased by him. It is contended that he cannot claim to be
The general rule relied upon in the majority opinion is a mere application of a well settled equity rule without fault should he buy such land because, as it is said, it was possible for him to discover that
that: "Where conflicting equities are otherwise equal in merit, that which first occurred will be given the land purchased by him had been made the subject of double or overlapping registration by a
the preference." But it is universally laid down by all the courts which have had occasion to apply this comparison of the description and boundary lines of the thousands of tracts and parcels of land to be
equity rule that "it should be the last test resorted to," and that "it never prevails when any other found in the land registry office.
equitable ground for preference exists." (See 19 Cent. Dig., tit. Equity, par. 181; and may cases cited
in 16 Cyc., 139, note 57.) It follows that the general rules, that in cases of double or overlapping

62
But such ruling goes far to defeat one of the principal objects sought to be attained by the On the other hand, I think that negligence and fault may fairly be imputed to a holder of a registered
introduction and adoption of the so-called torrens system for the registration of land. The avowed certificate of title who stood supinely by and let a default judgment be entered against him,
intent of that system of land registration is to relieve the purchase of registered lands from the adjudicating all or any part of his registered lands to another applicant, if it appears that he was served
necessity of looking farther than the certificate of title of the vendor in order that he may rest secure with notice or had actual notice of the pendency of the proceedings in the Court of Land Registration
as to the validity of the title to the lands conveyed to him. And yet it is said in the majority opinion wherein such default judgment was entered.
that he is charged with notice of the contents of every other certificate of title in the office of the
registrar so that his failure to acquaint himself with its contents may be imputed to him as negligence. The owner of land who enjoys the benefits secured to him by its registry in the Court of Land
Registration may reasonably be required to appear and defend his title when he has actual notice
If the rule announced in the majority opinion is to prevail, the new system of land registration, instead that proceedings are pending in that court wherein another applicant, claiming the land as his own,
of making transfers of real estate simple, expenditious and secure, and instead of avoiding the is seeking to secure its registry in his name. All that is necessary for him to do is to enter his
necessity for expensive and oftimes uncertain searches of the land record and registries, in order to appearance in those proceedings, invite the court's attention to the certificate of title registered in
ascertain the true condition of the title before purchase, will, in many instances, add to the labor, his name, and thus, at the cost of the applicant, avoid all the damage and inconvenience flowing from
expense and uncertainty of any attempt by a purchaser to satisfy himself as to the validity of the title the double or overlapping registration of the land in question. There is nothing in the new system of
to lands purchased by him. land registration which seems to render it either expedient or necessary to relieve a holder of a
registered title of the duty of appearing and defending that title, when he has actual notice that it is
As I have said before, one of the principal objects, if not the principal object, of the torrens system of being attacked in a court of competent jurisdiction, and if, as a result of his neglect or failure so to do,
land registration upon which our Land Registration Act is avowedly modelled is to facilitate the his lands become subject to double or overlapping registration, he should not be permitted to subject
transfer of real estate. To that end the Legislature undertakes to relieve prospective purchasers and an innocent purchaser, holding under the later certificate, to all the loss and damage resulting from
all others dealing in registered lands from the necessity of looking farther than the certificate of title the double or overlapping registration, while he goes scot free and holds the land under a manifest
to such lands furnished by the Court of Land Registration, and I cannot, therefore, give my consent to misapplication of the equitable rule that "where conflicting equities are otherwise equal in merit, that
a ruling which charges a purchaser or mortgage of registered lands with notice of the contents of which first accrued will be given the preference." It is only where both or neither of the parties are at
every other certificate of title in the land registry, so that negligence and fault may be imputed to him fault that the rule is properly applicable as between opposing claimants under an earlier and a later
should he be exposed to loss or damages as a result of the lack of such knowledge. certificate of registry to the same land.

Suppose a prospective purchaser of lands registered under the Land Registration Act desires to avoid Of course all that is said in the briefs of counsel and the majority opinion as to the right of the holder
the imputation of negligence in the event that, unknown to him, such lands have been made the of a certificate to rest secure in his registered title so that those dealing with registered lands can
subject of double or overlapping registration, what course should he pursue? What measures should confidently rely upon registry certificates thereto is equally forceful by way of argument in favor of
he adopt in order to search out the information with notice of which he is charged? There are no the holder of one or the other certificate in case of double or overlapping registration. The problem
indexes to guide him nor is there anything in the record or the certificate of title of the land he is to determine which of the certificate holders is entitled to the land. The decision of that question
proposes to buy which necessarily or even with reasonable probability will furnish him a clue as to in favor of either one must necessarily have the effect of destroying the value of the registered title
the fact of the existence of such double or overlapping registration. Indeed the only course open to of the other and to that extent shaking the public confidence in the value of the whole system for the
him, if he desires to assure himself against the possibility of double or overlapping registration, would registration of lands. But, in the language of the majority opinion, "that mistakes are bound to occur
even seem to be a careful, laborious and extensive comparison of the registered boundary lines cannot be denied and sometimes the damage done thereby is irreparable. It is the duty of the courts
contained in the certificate of title of the tract of land he proposes to buy with those contained in all to adjust the rights of the parties under such circumstances so as to minimize the damages, taking
the earlier certificates of title to be found in the land registry. Assuredly it was never the intention of into consideration all the conditions and the diligence of the respective parties to avoid
the author of the new Land Registration Act to impose such a burden on a purchaser of duly registered them."lawphil.net
real estate, under penalty that a lack of the knowledge which might thus be acquired may be imputed
to him by this court as negligence in ruling upon the respective equities of the holders of lands which It will be observed that I limit the exception to the general equitable rule, as laid down in the majority
have been the subject of double or overlapping registration. opinion, to case wherein the holder of the earlier certificate of title has actual notice of the pendency
of the proceedings in the course of which the latter certificate of title was issued, or to cases in which
he has received personal notice of the pendency of those proceedings. Unless he has actual notice of

63
the pendency of such proceedings I readily agree with the reasoning of the majority opinion so far as lands in favor of other applicants, despite actual notice of the pendency of judicial proceedings had
it holds that negligence, culpable negligence, should not be imputed to him for failure to appear and for that purpose, and this, without adding in any appreciable degree to the security of thir titles, and
defend his title so as to defeat his right to the benefit of the equitable rule. It is true that the order of merely to save them the very slight trouble or inconvenience incident to an entry of appearance in
publication in such cases having been duly complied with, all the world is charged with notice thereof, the court in which their own titles were secured, and inviting attention to the fact that their right,
but it does not necessarily follow that, in the absence of actual notice, culpable negligence in title and ownership in the lands in questions has already been conclusively adjudicated.
permitting a default judgment to be entered against him may be imputed to the holder of the earlier
certificate so as to defeat his right to the land under the equitable rule favoring the earlier certificate. The cases wherein there is a practical possibility of double or overlapping registration without actual
Such a holding would have the effect (to quote the language of the majority opinion) of requiring the notice to the holder of the earlier certificate must in the very nature of things to be so rare as to be
holder of a certificate of title to wait indefinitely "in the portals of the court" and to sit in the "mirador practically negligible. Double or overlapping registration almost invariably occurs in relation to lands
de su casa" in order to avoid the possibility of losing his lands; and I agree with the writer of the held by adjoining occupants or claimants. It is difficult to conceive of a case wherein double
majority opinion that to do so would place an unreasonable burden on the holders of such certificate, registration can take place, in the absence of fraud, without personal service of notice of the
which was not contemplated by the authors of the Land Registration Act. But no unreasonable burden pendency of the proceedings upon the holder of the earlier certificate, the statute requiring such
is placed upon the holder of a registered title by a rule which imputes culpable negligence to him notice to be served upon the owner or occupant of all lands adjoining those for which application for
when he sits supinely by and lets a judgment in default be entered against him adjudicating title to registration is made; and the cases wherein an adjoining land owner can, even by the use of fraud,
his lands in favor of another applicant, despite the fact that he has actual knowledge of the pendency conduct proceedings for the registration of his land to a successful conclusion without actual notice
of the proceedings in which such judgment is entered and despite the fact that he has been personally to the adjoining property owners must be rare indeed.
served with summons to appear and default his title.
In the case at bar the defendant purchased the land in question from the original holder of a
"Taking into consideration all of the conditions and the diligence of the respective parties," it seems certificate of title issued by the Court of Land Registration, relying upon the records of the Court of
to me that there is no "equality in merit" between the conflicting equities set up by an innocent Land Registration with reference thereto and with no knowledge that any part of the land thus
purchaser who acquires title to the land under a registered certificate, and the holder of an earlier purchased was included in an earlier certificate of title issued to the plaintiff. The plaintiff, the holder
certificate who permitted a default judgment to be entered against him, despite actual notice of the of the earlier certificate of title, negligently permitted a default judgment to be entered against him
pendency of the proceedings in the course of which the later certificate was issued. in the Court of Land Registration, adjudicating part of the lands included in his own certificate of title
in favor of another applicant, from whom the defendant in this action acquired title, and this despite
I am convinced, furthermore, that aside from the superior equities of the innocent purchaser in cases the fact that he was an adjoining land owner, had actual notice of the pendency of the proceedings
such as that now under discussion, there are strong reasons of convenience and public policy which and was personally served with summons to appear and defends his rights in the premises. It seems
militate in favor of the recognition of his title rather than that of the holder of the earlier title. to me that there can be no reason for doubt as to the respective merits of the equities of the parties,
and further that the judgment of the majority in favor of the plaintiff will inevitably tend to increase
One ruling exposes all persons purchasing or dealing in registered lands to unknown, unspecified and the number of cases wherein registered land owners in the future will fail to appear and defend their
uncertain dangers, to guard against which all such persons will be put to additional cost, annoyance titles when challenged in other proceedings in the Courts of Land Registration, thereby enormously
and labor on every occasion when any transaction is had with regard to such lands; while the other increasing the possibility and probability of loss and damage to innocent third parties and dealers in
ruling tends to eliminate consequences so directly adverse to the purpose and object for which the registered lands generally, arising out of erroneous, double or overlapping registration of lands by the
land registration law was enacted, and imposes no burden upon any holder of a certificate of Courts of Land Registration.
registered lands other than that of defending his title on those rare, definite and specific occasions
wherein he has actual notice that his title is being challenged in a Court of Land Registration, a Carson, J., concurs.
proceeding in which the cost and expense is reduced to the minimum by the conclusive character of
his certificate of title in support of his claim of ownership. Furthermore, judgment against the
innocent purchaser and in favor of the holder of the earlier certificate in a case such as that under
consideration must inevitably tend to increase the danger of double or overlapping registrations by
encouraging holders of registered titles, negligently or fraudulently and conclusively, to permit
default judgments to be entered against them adjudicating title to all or a part of their registered

64
.R. No. 2869 March 25, 1907 From the testimony given by Cariño as well as from that of several of the witnesses for the
Government it is deduced, that in or about the year 1884 Cariño erected and utilized as a domicile a
MATEO CARIÑO, petitioner-appellant, house on the property situated to the north of that property now in question, property which,
vs. according to the plan attached to expediente No. 561, appears to be property belonging to Donaldson
THE INSULAR GOVERNMENT, respondent-appellee. Sim; that during the year 1893 Cariño sold said house to one Cristobal Ramos, who in turn sold the
same to Donaldson Sim, moving to and living on the adjoining property, which appears on the plan
Coudert Brothers for appellant. aforesaid to be the property of H. Phelps Whitmarsh, a place where the father and the grandfather
Office of the Solicitor-General Araneta for appellee. of his wife, that is to say, Ortega and Minse, had lived . . ..

ARELLANO, C.J.: In or about the years 1898 Cariño abandoned the property of Whitmarsh and located on the property
described in the plan attached to expediente No. 561, having constructed a house thereon in which
Mateo Cariño, the appellant herein, on the 23d of February, 1904, filed his petition in the Court of he now lives, and which house is situated in the center of the property, as is indicated on the plan;
Land Registration praying that there be granted to him title to a parcel of land consisting of 40 and since which time he has undoubtedly occupied some portion of the property now claimed by him.
hectares, 1 are, and 13 centares, and situated in the town of Baguio, Province of Benguet, together (Bill of exceptions, pp. 11 and 12.)
with a house erected thereon and constructed of wood and roofed with rimo, and bounded as
follows: On the north, in lines running 1,048 metes and 20 decimeters with the lands of Sepa Cariño, 1. Therefore it is evident that this court can not decree the registration of all of the superficial
H. Phelps Whitmarsh, and Calsi; on the east, in lines running 991 meters and 50 decimeters with the extension of the land described in the petition and as appears on the plan filed herein, such extension
land of Kuidno, Esteban Gonzales, and of the Civil Government; on the south, in lines of 115 meters containing 40 hectares, 1 are, and 13 centares, inasmuch as the documentary evidence accompanying
and 60 decimeters, with the lands of Talaca; and on the west, in lines running 982 meters and 20 the petition is conclusive proof against the petitioners; this documentary proof consists of a
decimeters, with the lands of Sisco Cariño and Mayengmeng. possessory information under date of March 7, 1901, and registered on the 11th day of the same
month and year; and, according to such possessory information, the land therein described contains
By order of the court the hearing of this petition, No. 561, and that of Antonio Rebollo and Vicente an extension of only 28 hectares limited by "the country road to the barrio of Pias," a road appearing
Valpiedad filed under No. 834, were heard together for the reason that the latter petition claimed a on the plan now presented and cutting the land, as might be said, in half, or running through its center
small portion of land included in the parcel set out in the former petition. from north to south, a considerable extension of land remaining on the other side of the said road,
the west side, and which could not have been included in the possessory information mentioned.
The Insular Government opposed the granting of these petitions, alleging that the whole parcel of
land is public property of the Government and that the same was never acquired in any manner or 2. As has been shown during the trial of this case, this land, of which mention is made in said
through any title of egresion from the State. possessory information, and upon which is situated the house now actually occupied by the
petitioner, all of which is set forth as argument as to the possession in the judgment, is "used for
After trial, and the hearing of documentary and oral proof, the court of Land Registration rendered pasture and sowing," and belongs to the class called public lands.
its judgment in these terms:
3. Under the express provisions of law, a parcel of land, being of common origin,
Therefore the court finds that Cariño and his predecessors have not possessed exclusively and presumptively belonged to the State during its sovereignty, and, in order to perfect the legitimate
adversely any part of the said property prior to the date on which Cariño constructed the house now acquisition of such land by private persons, it was necessary that the possession of the same pass
there — that is to say, for the years 1897 and 1898, and Cariño held possession for some years from the State. And there is no evidence or proof of title of egresion of this land from the domain of
afterwards of but a part of the property to which he claims title. Both petitions are dismissed and the the Spanish Government, nor is there any possessory information equivalent to title by composicion
property in question is adjudged to be public land. (Bill of exceptions, p. 15.) or under agreement. 4, The possessory information filed herein is not the title to property authorized
in substitution for that of adjustment by the royal decree of February 13, 1894, this being the last law
The conclusions arrived at the set forth in definite terms in the decision of the court below are the or legal disposition of the former sovereignty applicable to the present subject-matter of common
following: lands: First, for the reason that the land referred to herein is not covered nor does it come within any
one of the three conditions required by article 19 of the said royal decree, to wit, that the land has

65
been in an uninterrupted state of cultivation during a period of six years last past; or that the same 1, 1902,1 and in conformity with other laws enacted under this act of Congress by the Philippine
has been possessed without interruption during a period of twelve years and has been in a state of Commission prescribing rules for the execution thereof, one of which is Act No. 648,2 herein
cultivation up to the date of the information and during the three years immediately preceding such mentioned by the petitioner, in connection with Act No. 627,3 which appears to be the law upon
information; or that such land had been possessed openly without interruption during a period of which the petition herein is founded.
thirty or more years, notwithstanding the land had not been cultivated; nor is it necessary to refer to
the testimony given by the two witnesses to the possessory information for the following reason: 8. Section 6 of Act No. 627 admits prescription, in accordance with the provisions contained
Second, because the possessory information authorized by said royal decree or last legal disposition in Act No. 190, as a basis for obtaining the right of ownership. "The petitioners claims title under the
of the Spanish Government, as title or for the purpose of acquiring actual proprietary right, equivalent period of prescription of ten years established by that act, as well as by reason of his occupancy and
to that of adjustment with the Spanish Government and required and necessary at all times until the use thereof from time immemorial." (Allegation 1.) But said act admits such prescription for the
publication of said royal decree was limited in time to one year, in accordance with article 21, which purpose of obtaining title and ownership to lands "not exceeding more that sixteen hectares in
is as follows: " A period of one year, not to be extended, is allowed to verify the possessory extent." (Sec. 6 of said act.) The land claimed by Cariño is 40 hectares in extent, if we take into
informations which are referred to in articles 19 and 20. After the expiration of this period of the right consideration his petition, or an extension of 28 hectares, according to the possessory information,
of the cultivators and persons in possession to obtain gratuitous title thereto lapses and the land the only thing that can be considered. Therefore, it follows that the judgment denying the petition
together with full possession reverts to the state, or, as the case may be, to the community, and the herein and now appealed from was strictly in accordance with the law invoked herein.
said possessors and cultivators or their assigns would simply have rights under universal or general
title of average in the event that the land is sold within a period of five years immediately following 9. And of the 28 hectares of land as set out in the possessory information, one part of same,
the cancellation. The possessors not included under this chapter can only acquire by time the according to the testimony of Cariño, belongs to Vicente Valpiedad, the extent of which is not
ownership and title to unappropriated or royal lands in accordance with common law." determined. From all of which it follows that the precise extent has not been determined in the trial
of this case on which judgment might be based in the event that the judgment and title be declared
5. In accordance with the preceding provisions, the right that remained to Cariño, if it be in favor of the petitioner, Mateo Cariño. And we should not lose sight of the fact that, considering the
certain that he was the true possessor of the land in question, was the right of average in case the intention of Congress in granting ownership and title to 16 hectares, that Mateo Cariño and his
Government or State could have sold the same within the period of five years immediately following children have already exceeded such amount in various acquirements of lands, all of which is shown
for example, if the denouncement of purchase had been carried out by Felipe Zafra or any other in different cases decided by the said Court of Land Registration, donations or gifts of land that could
person, as appears from the record of the trial of the case. Aside from this right, in such event, his only have been made efficacious as to the conveyance thereof with the assistance of these new laws.
possession as attested in the possessory information herein could not, in accordance with common
law, go to show any right of ownership until after the expiration of twenty years from the expiration By reason of the findings set forth it is clearly seen that the court below did not err:
of twenty years from the verification and registry of the same in conformity with the provisions of
article 393 of the Mortgage Law and other conditions prescribe by this law. 1. In finding that Mateo Cariño and those from whom he claims his right had not possessed
and claimed as owners the lands in question since time immemorial;
6. The right of possession in accordance with common law — that is to say, civil law — remains
at all times subordinate to the Spanish administrative law, inasmuch as it could only be of force when 2. In finding that the land in question did not belong to the petitioner, but that, on the
pertaining to royal transferable or alienable lands, which condition and the determination thereof is contrary, it was the property of the Government. (Allegation 21.)
reversed to the government, which classified and designated the royal alienable lands for the purpose
of distinguishing them from those lands strictly public, and from forestry lands which could at no time Wherefore, the judgment appealed from is affirmed with the costs of this instance against the
pass to private ownership nor be acquired through time even after the said royal decree of February appellant. After the expiration of twenty days from the notification of this decision let judgment be
13, 1894. entered in accordance herewith, and ten days thereafter let the case be remanded to the court from
whence it came for proper action. So ordered.
7. The advent of the new sovereignty necessarily brought a new method of dealing with lands
and particularly as to the classification and manner of transfer and acquisition of royal or common Torres, Mapa, Willard, and Tracey, JJ., concur.
lands then appropriated, which were thenceforth merely called public lands, the alienation of which Johnson, J., reserves his vote.
was reserved to the Government, in accordance with section 12 and 13 of the act of Congress of July

66
G.R. No. 133250 July 9, 2002
(iii) x x x CDCP shall give up all its development rights and hereby agrees to cede and transfer in favor
FRANCISCO I. CHAVEZ, petitioner, of PEA, all of the rights, title, interest and participation of CDCP in and to all the areas of land reclaimed
vs. by CDCP in the MCCRRP as of December 30, 1981 which have not yet been sold, transferred or
PUBLIC ESTATES AUTHORITY and AMARI COASTAL BAY DEVELOPMENT CORPORATION, respondents. otherwise disposed of by CDCP as of said date, which areas consist of approximately Ninety-Nine
Thousand Four Hundred Seventy Three (99,473) square meters in the Financial Center Area covered
CARPIO, J.: by land pledge No. 5 and approximately Three Million Three Hundred Eighty Two Thousand Eight
Hundred Eighty Eight (3,382,888) square meters of reclaimed areas at varying elevations above Mean
This is an original Petition for Mandamus with prayer for a writ of preliminary injunction and a Low Water Level located outside the Financial Center Area and the First Neighborhood Unit."3
temporary restraining order. The petition seeks to compel the Public Estates Authority ("PEA" for
brevity) to disclose all facts on PEA's then on-going renegotiations with Amari Coastal Bay and On January 19, 1988, then President Corazon C. Aquino issued Special Patent No. 3517, granting and
Development Corporation ("AMARI" for brevity) to reclaim portions of Manila Bay. The petition transferring to PEA "the parcels of land so reclaimed under the Manila-Cavite Coastal Road and
further seeks to enjoin PEA from signing a new agreement with AMARI involving such reclamation. Reclamation Project (MCCRRP) containing a total area of one million nine hundred fifteen thousand
eight hundred ninety four (1,915,894) square meters." Subsequently, on April 9, 1988, the Register of
The Facts Deeds of the Municipality of Parañaque issued Transfer Certificates of Title Nos. 7309, 7311, and
7312, in the name of PEA, covering the three reclaimed islands known as the "Freedom Islands"
On November 20, 1973, the government, through the Commissioner of Public Highways, signed a located at the southern portion of the Manila-Cavite Coastal Road, Parañaque City. The Freedom
contract with the Construction and Development Corporation of the Philippines ("CDCP" for brevity) Islands have a total land area of One Million Five Hundred Seventy Eight Thousand Four Hundred and
to reclaim certain foreshore and offshore areas of Manila Bay. The contract also included the Forty One (1,578,441) square meters or 157.841 hectares.
construction of Phases I and II of the Manila-Cavite Coastal Road. CDCP obligated itself to carry out
all the works in consideration of fifty percent of the total reclaimed land. On April 25, 1995, PEA entered into a Joint Venture Agreement ("JVA" for brevity) with AMARI, a
private corporation, to develop the Freedom Islands. The JVA also required the reclamation of an
On February 4, 1977, then President Ferdinand E. Marcos issued Presidential Decree No. 1084 additional 250 hectares of submerged areas surrounding these islands to complete the configuration
creating PEA. PD No. 1084 tasked PEA "to reclaim land, including foreshore and submerged areas," in the Master Development Plan of the Southern Reclamation Project-MCCRRP. PEA and AMARI
and "to develop, improve, acquire, x x x lease and sell any and all kinds of lands."1 On the same date, entered into the JVA through negotiation without public bidding.4 On April 28, 1995, the Board of
then President Marcos issued Presidential Decree No. 1085 transferring to PEA the "lands reclaimed Directors of PEA, in its Resolution No. 1245, confirmed the JVA.5 On June 8, 1995, then President Fidel
in the foreshore and offshore of the Manila Bay"2 under the Manila-Cavite Coastal Road and V. Ramos, through then Executive Secretary Ruben Torres, approved the JVA.6
Reclamation Project (MCCRRP).
On November 29, 1996, then Senate President Ernesto Maceda delivered a privilege speech in the
On December 29, 1981, then President Marcos issued a memorandum directing PEA to amend its Senate and denounced the JVA as the "grandmother of all scams." As a result, the Senate Committee
contract with CDCP, so that "[A]ll future works in MCCRRP x x x shall be funded and owned by PEA." on Government Corporations and Public Enterprises, and the Committee on Accountability of Public
Accordingly, PEA and CDCP executed a Memorandum of Agreement dated December 29, 1981, which Officers and Investigations, conducted a joint investigation. The Senate Committees reported the
stated: results of their investigation in Senate Committee Report No. 560 dated September 16, 1997.7 Among
the conclusions of their report are: (1) the reclaimed lands PEA seeks to transfer to AMARI under the
"(i) CDCP shall undertake all reclamation, construction, and such other works in the MCCRRP as may JVA are lands of the public domain which the government has not classified as alienable lands and
be agreed upon by the parties, to be paid according to progress of works on a unit price/lump sum therefore PEA cannot alienate these lands; (2) the certificates of title covering the Freedom Islands
basis for items of work to be agreed upon, subject to price escalation, retention and other terms and are thus void, and (3) the JVA itself is illegal.
conditions provided for in Presidential Decree No. 1594. All the financing required for such works shall
be provided by PEA. On December 5, 1997, then President Fidel V. Ramos issued Presidential Administrative Order No.
365 creating a Legal Task Force to conduct a study on the legality of the JVA in view of Senate
xxx Committee Report No. 560. The members of the Legal Task Force were the Secretary of Justice,8 the

67
Chief Presidential Legal Counsel,9 and the Government Corporate Counsel.10 The Legal Task Force The Issues
upheld the legality of the JVA, contrary to the conclusions reached by the Senate Committees.11
The issues raised by petitioner, PEA15 and AMARI16 are as follows:
On April 4 and 5, 1998, the Philippine Daily Inquirer and Today published reports that there were on-
going renegotiations between PEA and AMARI under an order issued by then President Fidel V. I. WHETHER THE PRINCIPAL RELIEFS PRAYED FOR IN THE PETITION ARE MOOT AND ACADEMIC
Ramos. According to these reports, PEA Director Nestor Kalaw, PEA Chairman Arsenio Yulo and retired BECAUSE OF SUBSEQUENT EVENTS;
Navy Officer Sergio Cruz composed the negotiating panel of PEA.
II. WHETHER THE PETITION MERITS DISMISSAL FOR FAILING TO OBSERVE THE PRINCIPLE GOVERNING
On April 13, 1998, Antonio M. Zulueta filed before the Court a Petition for Prohibition with Application THE HIERARCHY OF COURTS;
for the Issuance of a Temporary Restraining Order and Preliminary Injunction docketed as G.R. No.
132994 seeking to nullify the JVA. The Court dismissed the petition "for unwarranted disregard of III. WHETHER THE PETITION MERITS DISMISSAL FOR NON-EXHAUSTION OF ADMINISTRATIVE
judicial hierarchy, without prejudice to the refiling of the case before the proper court."12 REMEDIES;

On April 27, 1998, petitioner Frank I. Chavez ("Petitioner" for brevity) as a taxpayer, filed the instant IV. WHETHER PETITIONER HAS LOCUS STANDI TO BRING THIS SUIT;
Petition for Mandamus with Prayer for the Issuance of a Writ of Preliminary Injunction and Temporary
Restraining Order. Petitioner contends the government stands to lose billions of pesos in the sale by V. WHETHER THE CONSTITUTIONAL RIGHT TO INFORMATION INCLUDES OFFICIAL INFORMATION ON
PEA of the reclaimed lands to AMARI. Petitioner prays that PEA publicly disclose the terms of any ON-GOING NEGOTIATIONS BEFORE A FINAL AGREEMENT;
renegotiation of the JVA, invoking Section 28, Article II, and Section 7, Article III, of the 1987
Constitution on the right of the people to information on matters of public concern. Petitioner assails VI. WHETHER THE STIPULATIONS IN THE AMENDED JOINT VENTURE AGREEMENT FOR THE TRANSFER
the sale to AMARI of lands of the public domain as a blatant violation of Section 3, Article XII of the TO AMARI OF CERTAIN LANDS, RECLAIMED AND STILL TO BE RECLAIMED, VIOLATE THE 1987
1987 Constitution prohibiting the sale of alienable lands of the public domain to private corporations. CONSTITUTION; AND
Finally, petitioner asserts that he seeks to enjoin the loss of billions of pesos in properties of the State
that are of public dominion. VII. WHETHER THE COURT IS THE PROPER FORUM FOR RAISING THE ISSUE OF WHETHER THE
AMENDED JOINT VENTURE AGREEMENT IS GROSSLY DISADVANTAGEOUS TO THE GOVERNMENT.
After several motions for extension of time,13 PEA and AMARI filed their Comments on October 19,
1998 and June 25, 1998, respectively. Meanwhile, on December 28, 1998, petitioner filed an Omnibus The Court's Ruling
Motion: (a) to require PEA to submit the terms of the renegotiated PEA-AMARI contract; (b) for
issuance of a temporary restraining order; and (c) to set the case for hearing on oral argument. First issue: whether the principal reliefs prayed for in the petition are moot and academic because of
Petitioner filed a Reiterative Motion for Issuance of a TRO dated May 26, 1999, which the Court subsequent events.
denied in a Resolution dated June 22, 1999.
The petition prays that PEA publicly disclose the "terms and conditions of the on-going negotiations
In a Resolution dated March 23, 1999, the Court gave due course to the petition and required the for a new agreement." The petition also prays that the Court enjoin PEA from "privately entering into,
parties to file their respective memoranda. perfecting and/or executing any new agreement with AMARI."

On March 30, 1999, PEA and AMARI signed the Amended Joint Venture Agreement ("Amended JVA," PEA and AMARI claim the petition is now moot and academic because AMARI furnished petitioner on
for brevity). On May 28, 1999, the Office of the President under the administration of then President June 21, 1999 a copy of the signed Amended JVA containing the terms and conditions agreed upon in
Joseph E. Estrada approved the Amended JVA. the renegotiations. Thus, PEA has satisfied petitioner's prayer for a public disclosure of the
renegotiations. Likewise, petitioner's prayer to enjoin the signing of the Amended JVA is now moot
Due to the approval of the Amended JVA by the Office of the President, petitioner now prays that on because PEA and AMARI have already signed the Amended JVA on March 30, 1999. Moreover, the
"constitutional and statutory grounds the renegotiated contract be declared null and void."14 Office of the President has approved the Amended JVA on May 28, 1999.

68
Petitioner counters that PEA and AMARI cannot avoid the constitutional issue by simply fast-tracking reclaimed lands. Under the Amended JVA, PEA is obligated to transfer to AMARI the latter's seventy
the signing and approval of the Amended JVA before the Court could act on the issue. Presidential percent proportionate share in the reclaimed areas as the reclamation progresses. The Amended JVA
approval does not resolve the constitutional issue or remove it from the ambit of judicial review. even allows AMARI to mortgage at any time the entire reclaimed area to raise financing for the
reclamation project.21
We rule that the signing of the Amended JVA by PEA and AMARI and its approval by the President
cannot operate to moot the petition and divest the Court of its jurisdiction. PEA and AMARI have still Second issue: whether the petition merits dismissal for failing to observe the principle governing the
to implement the Amended JVA. The prayer to enjoin the signing of the Amended JVA on hierarchy of courts.
constitutional grounds necessarily includes preventing its implementation if in the meantime PEA and
AMARI have signed one in violation of the Constitution. Petitioner's principal basis in assailing the PEA and AMARI claim petitioner ignored the judicial hierarchy by seeking relief directly from the
renegotiation of the JVA is its violation of Section 3, Article XII of the Constitution, which prohibits the Court. The principle of hierarchy of courts applies generally to cases involving factual questions. As it
government from alienating lands of the public domain to private corporations. If the Amended JVA is not a trier of facts, the Court cannot entertain cases involving factual issues. The instant case,
indeed violates the Constitution, it is the duty of the Court to enjoin its implementation, and if already however, raises constitutional issues of transcendental importance to the public.22 The Court can
implemented, to annul the effects of such unconstitutional contract. resolve this case without determining any factual issue related to the case. Also, the instant case is a
petition for mandamus which falls under the original jurisdiction of the Court under Section 5, Article
The Amended JVA is not an ordinary commercial contract but one which seeks to transfer title and VIII of the Constitution. We resolve to exercise primary jurisdiction over the instant case.
ownership to 367.5 hectares of reclaimed lands and submerged areas of Manila Bay to a single private
corporation. It now becomes more compelling for the Court to resolve the issue to insure the Third issue: whether the petition merits dismissal for non-exhaustion of administrative remedies.
government itself does not violate a provision of the Constitution intended to safeguard the national
patrimony. Supervening events, whether intended or accidental, cannot prevent the Court from PEA faults petitioner for seeking judicial intervention in compelling PEA to disclose publicly certain
rendering a decision if there is a grave violation of the Constitution. In the instant case, if the Amended information without first asking PEA the needed information. PEA claims petitioner's direct resort to
JVA runs counter to the Constitution, the Court can still prevent the transfer of title and ownership of the Court violates the principle of exhaustion of administrative remedies. It also violates the rule that
alienable lands of the public domain in the name of AMARI. Even in cases where supervening events mandamus may issue only if there is no other plain, speedy and adequate remedy in the ordinary
had made the cases moot, the Court did not hesitate to resolve the legal or constitutional issues raised course of law.
to formulate controlling principles to guide the bench, bar, and the public.17
PEA distinguishes the instant case from Tañada v. Tuvera23 where the Court granted the petition for
Also, the instant petition is a case of first impression. All previous decisions of the Court involving mandamus even if the petitioners there did not initially demand from the Office of the President the
Section 3, Article XII of the 1987 Constitution, or its counterpart provision in the 1973 Constitution,18 publication of the presidential decrees. PEA points out that in Tañada, the Executive Department had
covered agricultural lands sold to private corporations which acquired the lands from private parties. an affirmative statutory duty under Article 2 of the Civil Code24 and Section 1 of Commonwealth Act
The transferors of the private corporations claimed or could claim the right to judicial confirmation No. 63825 to publish the presidential decrees. There was, therefore, no need for the petitioners in
of their imperfect titles19 under Title II of Commonwealth Act. 141 ("CA No. 141" for brevity). In the Tañada to make an initial demand from the Office of the President. In the instant case, PEA claims it
instant case, AMARI seeks to acquire from PEA, a public corporation, reclaimed lands and submerged has no affirmative statutory duty to disclose publicly information about its renegotiation of the JVA.
areas for non-agricultural purposes by purchase under PD No. 1084 (charter of PEA) and Title III of CA Thus, PEA asserts that the Court must apply the principle of exhaustion of administrative remedies to
No. 141. Certain undertakings by AMARI under the Amended JVA constitute the consideration for the the instant case in view of the failure of petitioner here to demand initially from PEA the needed
purchase. Neither AMARI nor PEA can claim judicial confirmation of their titles because the lands information.
covered by the Amended JVA are newly reclaimed or still to be reclaimed. Judicial confirmation of
imperfect title requires open, continuous, exclusive and notorious occupation of agricultural lands of The original JVA sought to dispose to AMARI public lands held by PEA, a government corporation.
the public domain for at least thirty years since June 12, 1945 or earlier. Besides, the deadline for Under Section 79 of the Government Auditing Code,26 the disposition of government lands to private
filing applications for judicial confirmation of imperfect title expired on December 31, 1987.20 parties requires public bidding. PEA was under a positive legal duty to disclose to the public the terms
and conditions for the sale of its lands. The law obligated PEA to make this public disclosure even
Lastly, there is a need to resolve immediately the constitutional issue raised in this petition because without demand from petitioner or from anyone. PEA failed to make this public disclosure because
of the possible transfer at any time by PEA to AMARI of title and ownership to portions of the the original JVA, like the Amended JVA, was the result of a negotiated contract, not of a public bidding.

69
Considering that PEA had an affirmative statutory duty to make the public disclosure, and was even
in breach of this legal duty, petitioner had the right to seek direct judicial intervention. xxx

Moreover, and this alone is determinative of this issue, the principle of exhaustion of administrative In Tañada v. Tuvera, the Court asserted that when the issue concerns a public right and the object of
remedies does not apply when the issue involved is a purely legal or constitutional question.27 The mandamus is to obtain the enforcement of a public duty, the people are regarded as the real parties
principal issue in the instant case is the capacity of AMARI to acquire lands held by PEA in view of the in interest; and because it is sufficient that petitioner is a citizen and as such is interested in the
constitutional ban prohibiting the alienation of lands of the public domain to private corporations. execution of the laws, he need not show that he has any legal or special interest in the result of the
We rule that the principle of exhaustion of administrative remedies does not apply in the instant case. action. In the aforesaid case, the petitioners sought to enforce their right to be informed on matters
of public concern, a right then recognized in Section 6, Article IV of the 1973 Constitution, in
Fourth issue: whether petitioner has locus standi to bring this suit connection with the rule that laws in order to be valid and enforceable must be published in the
Official Gazette or otherwise effectively promulgated. In ruling for the petitioners' legal standing, the
PEA argues that petitioner has no standing to institute mandamus proceedings to enforce his Court declared that the right they sought to be enforced 'is a public right recognized by no less than
constitutional right to information without a showing that PEA refused to perform an affirmative duty the fundamental law of the land.'
imposed on PEA by the Constitution. PEA also claims that petitioner has not shown that he will suffer
any concrete injury because of the signing or implementation of the Amended JVA. Thus, there is no Legaspi v. Civil Service Commission, while reiterating Tañada, further declared that 'when a
actual controversy requiring the exercise of the power of judicial review. mandamus proceeding involves the assertion of a public right, the requirement of personal interest
is satisfied by the mere fact that petitioner is a citizen and, therefore, part of the general 'public' which
The petitioner has standing to bring this taxpayer's suit because the petition seeks to compel PEA to possesses the right.'
comply with its constitutional duties. There are two constitutional issues involved here. First is the
right of citizens to information on matters of public concern. Second is the application of a Further, in Albano v. Reyes, we said that while expenditure of public funds may not have been
constitutional provision intended to insure the equitable distribution of alienable lands of the public involved under the questioned contract for the development, management and operation of the
domain among Filipino citizens. The thrust of the first issue is to compel PEA to disclose publicly Manila International Container Terminal, 'public interest [was] definitely involved considering the
information on the sale of government lands worth billions of pesos, information which the important role [of the subject contract] . . . in the economic development of the country and the
Constitution and statutory law mandate PEA to disclose. The thrust of the second issue is to prevent magnitude of the financial consideration involved.' We concluded that, as a consequence, the
PEA from alienating hundreds of hectares of alienable lands of the public domain in violation of the disclosure provision in the Constitution would constitute sufficient authority for upholding the
Constitution, compelling PEA to comply with a constitutional duty to the nation. petitioner's standing.

Moreover, the petition raises matters of transcendental importance to the public. In Chavez v. Similarly, the instant petition is anchored on the right of the people to information and access to
PCGG,28 the Court upheld the right of a citizen to bring a taxpayer's suit on matters of transcendental official records, documents and papers — a right guaranteed under Section 7, Article III of the 1987
importance to the public, thus - Constitution. Petitioner, a former solicitor general, is a Filipino citizen. Because of the satisfaction of
the two basic requisites laid down by decisional law to sustain petitioner's legal standing, i.e. (1) the
"Besides, petitioner emphasizes, the matter of recovering the ill-gotten wealth of the Marcoses is an enforcement of a public right (2) espoused by a Filipino citizen, we rule that the petition at bar should
issue of 'transcendental importance to the public.' He asserts that ordinary taxpayers have a right to be allowed."
initiate and prosecute actions questioning the validity of acts or orders of government agencies or
instrumentalities, if the issues raised are of 'paramount public interest,' and if they 'immediately We rule that since the instant petition, brought by a citizen, involves the enforcement of
affect the social, economic and moral well being of the people.' constitutional rights - to information and to the equitable diffusion of natural resources - matters of
transcendental public importance, the petitioner has the requisite locus standi.
Moreover, the mere fact that he is a citizen satisfies the requirement of personal interest, when the
proceeding involves the assertion of a public right, such as in this case. He invokes several decisions Fifth issue: whether the constitutional right to information includes official information on on-going
of this Court which have set aside the procedural matter of locus standi, when the subject of the case negotiations before a final agreement.
involved public interest.

70
Section 7, Article III of the Constitution explains the people's right to information on matters of public Also, AMARI contends that petitioner cannot invoke the right at the pre-decisional stage or before
concern in this manner: the closing of the transaction. To support its contention, AMARI cites the following discussion in the
1986 Constitutional Commission:
"Sec. 7. The right of the people to information on matters of public concern shall be recognized.
Access to official records, and to documents, and papers pertaining to official acts, transactions, or "Mr. Suarez. And when we say 'transactions' which should be distinguished from contracts,
decisions, as well as to government research data used as basis for policy development, shall be agreements, or treaties or whatever, does the Gentleman refer to the steps leading to the
afforded the citizen, subject to such limitations as may be provided by law." (Emphasis supplied) consummation of the contract, or does he refer to the contract itself?

The State policy of full transparency in all transactions involving public interest reinforces the people's Mr. Ople: The 'transactions' used here, I suppose is generic and therefore, it can cover both steps
right to information on matters of public concern. This State policy is expressed in Section 28, Article leading to a contract and already a consummated contract, Mr. Presiding Officer.
II of the Constitution, thus:
Mr. Suarez: This contemplates inclusion of negotiations leading to the consummation of the
"Sec. 28. Subject to reasonable conditions prescribed by law, the State adopts and implements a transaction.
policy of full public disclosure of all its transactions involving public interest." (Emphasis supplied)
Mr. Ople: Yes, subject only to reasonable safeguards on the national interest.
These twin provisions of the Constitution seek to promote transparency in policy-making and in the
operations of the government, as well as provide the people sufficient information to exercise Mr. Suarez: Thank you."32 (Emphasis supplied)
effectively other constitutional rights. These twin provisions are essential to the exercise of freedom
of expression. If the government does not disclose its official acts, transactions and decisions to AMARI argues there must first be a consummated contract before petitioner can invoke the right.
citizens, whatever citizens say, even if expressed without any restraint, will be speculative and Requiring government officials to reveal their deliberations at the pre-decisional stage will degrade
amount to nothing. These twin provisions are also essential to hold public officials "at all times x x x the quality of decision-making in government agencies. Government officials will hesitate to express
accountable to the people,"29 for unless citizens have the proper information, they cannot hold their real sentiments during deliberations if there is immediate public dissemination of their
public officials accountable for anything. Armed with the right information, citizens can participate in discussions, putting them under all kinds of pressure before they decide.
public discussions leading to the formulation of government policies and their effective
implementation. An informed citizenry is essential to the existence and proper functioning of any We must first distinguish between information the law on public bidding requires PEA to disclose
democracy. As explained by the Court in Valmonte v. Belmonte, Jr.30 – publicly, and information the constitutional right to information requires PEA to release to the public.
Before the consummation of the contract, PEA must, on its own and without demand from anyone,
"An essential element of these freedoms is to keep open a continuing dialogue or process of disclose to the public matters relating to the disposition of its property. These include the size,
communication between the government and the people. It is in the interest of the State that the location, technical description and nature of the property being disposed of, the terms and conditions
channels for free political discussion be maintained to the end that the government may perceive and of the disposition, the parties qualified to bid, the minimum price and similar information. PEA must
be responsive to the people's will. Yet, this open dialogue can be effective only to the extent that the prepare all these data and disclose them to the public at the start of the disposition process, long
citizenry is informed and thus able to formulate its will intelligently. Only when the participants in the before the consummation of the contract, because the Government Auditing Code requires public
discussion are aware of the issues and have access to information relating thereto can such bear bidding. If PEA fails to make this disclosure, any citizen can demand from PEA this information at any
fruit." time during the bidding process.

PEA asserts, citing Chavez v. PCGG,31 that in cases of on-going negotiations the right to information Information, however, on on-going evaluation or review of bids or proposals being undertaken by the
is limited to "definite propositions of the government." PEA maintains the right does not include bidding or review committee is not immediately accessible under the right to information. While the
access to "intra-agency or inter-agency recommendations or communications during the stage when evaluation or review is still on-going, there are no "official acts, transactions, or decisions" on the bids
common assertions are still in the process of being formulated or are in the 'exploratory stage'." or proposals. However, once the committee makes its official recommendation, there arises a
"definite proposition" on the part of the government. From this moment, the public's right to

71
information attaches, and any citizen can access all the non-proprietary information leading to such information does not compel PEA to prepare lists, abstracts, summaries and the like relating to the
definite proposition. In Chavez v. PCGG,33 the Court ruled as follows: renegotiation of the JVA.34 The right only affords access to records, documents and papers, which
means the opportunity to inspect and copy them. One who exercises the right must copy the records,
"Considering the intent of the framers of the Constitution, we believe that it is incumbent upon the documents and papers at his expense. The exercise of the right is also subject to reasonable
PCGG and its officers, as well as other government representatives, to disclose sufficient public regulations to protect the integrity of the public records and to minimize disruption to government
information on any proposed settlement they have decided to take up with the ostensible owners operations, like rules specifying when and how to conduct the inspection and copying.35
and holders of ill-gotten wealth. Such information, though, must pertain to definite propositions of
the government, not necessarily to intra-agency or inter-agency recommendations or The right to information, however, does not extend to matters recognized as privileged information
communications during the stage when common assertions are still in the process of being under the separation of powers.36 The right does not also apply to information on military and
formulated or are in the "exploratory" stage. There is need, of course, to observe the same restrictions diplomatic secrets, information affecting national security, and information on investigations of
on disclosure of information in general, as discussed earlier – such as on matters involving national crimes by law enforcement agencies before the prosecution of the accused, which courts have long
security, diplomatic or foreign relations, intelligence and other classified information." (Emphasis recognized as confidential.37 The right may also be subject to other limitations that Congress may
supplied) impose by law.

Contrary to AMARI's contention, the commissioners of the 1986 Constitutional Commission There is no claim by PEA that the information demanded by petitioner is privileged information rooted
understood that the right to information "contemplates inclusion of negotiations leading to the in the separation of powers. The information does not cover Presidential conversations,
consummation of the transaction." Certainly, a consummated contract is not a requirement for the correspondences, or discussions during closed-door Cabinet meetings which, like internal
exercise of the right to information. Otherwise, the people can never exercise the right if no contract deliberations of the Supreme Court and other collegiate courts, or executive sessions of either house
is consummated, and if one is consummated, it may be too late for the public to expose its of Congress,38 are recognized as confidential. This kind of information cannot be pried open by a co-
defects.1âwphi1.nêt equal branch of government. A frank exchange of exploratory ideas and assessments, free from the
glare of publicity and pressure by interested parties, is essential to protect the independence of
Requiring a consummated contract will keep the public in the dark until the contract, which may be decision-making of those tasked to exercise Presidential, Legislative and Judicial power.39 This is not
grossly disadvantageous to the government or even illegal, becomes a fait accompli. This negates the the situation in the instant case.
State policy of full transparency on matters of public concern, a situation which the framers of the
Constitution could not have intended. Such a requirement will prevent the citizenry from participating We rule, therefore, that the constitutional right to information includes official information on on-
in the public discussion of any proposed contract, effectively truncating a basic right enshrined in the going negotiations before a final contract. The information, however, must constitute definite
Bill of Rights. We can allow neither an emasculation of a constitutional right, nor a retreat by the State propositions by the government and should not cover recognized exceptions like privileged
of its avowed "policy of full disclosure of all its transactions involving public interest." information, military and diplomatic secrets and similar matters affecting national security and public
order.40 Congress has also prescribed other limitations on the right to information in several
The right covers three categories of information which are "matters of public concern," namely: (1) legislations.41
official records; (2) documents and papers pertaining to official acts, transactions and decisions; and
(3) government research data used in formulating policies. The first category refers to any document Sixth issue: whether stipulations in the Amended JVA for the transfer to AMARI of lands, reclaimed or
that is part of the public records in the custody of government agencies or officials. The second to be reclaimed, violate the Constitution.
category refers to documents and papers recording, evidencing, establishing, confirming, supporting,
justifying or explaining official acts, transactions or decisions of government agencies or officials. The The Regalian Doctrine
third category refers to research data, whether raw, collated or processed, owned by the government
and used in formulating government policies. The ownership of lands reclaimed from foreshore and submerged areas is rooted in the Regalian
doctrine which holds that the State owns all lands and waters of the public domain. Upon the Spanish
The information that petitioner may access on the renegotiation of the JVA includes evaluation conquest of the Philippines, ownership of all "lands, territories and possessions" in the Philippines
reports, recommendations, legal and expert opinions, minutes of meetings, terms of reference and passed to the Spanish Crown.42 The King, as the sovereign ruler and representative of the people,
other documents attached to such reports or minutes, all relating to the JVA. However, the right to

72
acquired and owned all lands and territories in the Philippines except those he disposed of by grant 1. That devoted to public use, such as roads, canals, rivers, torrents, ports and bridges constructed by
or sale to private individuals. the State, riverbanks, shores, roadsteads, and that of a similar character;

The 1935, 1973 and 1987 Constitutions adopted the Regalian doctrine substituting, however, the 2. That belonging exclusively to the State which, without being of general public use, is employed in
State, in lieu of the King, as the owner of all lands and waters of the public domain. The Regalian some public service, or in the development of the national wealth, such as walls, fortresses, and other
doctrine is the foundation of the time-honored principle of land ownership that "all lands that were works for the defense of the territory, and mines, until granted to private individuals."
not acquired from the Government, either by purchase or by grant, belong to the public domain."43
Article 339 of the Civil Code of 1889, which is now Article 420 of the Civil Code of 1950, incorporated Property devoted to public use referred to property open for use by the public. In contrast, property
the Regalian doctrine. devoted to public service referred to property used for some specific public service and open only to
those authorized to use the property.
Ownership and Disposition of Reclaimed Lands
Property of public dominion referred not only to property devoted to public use, but also to property
The Spanish Law of Waters of 1866 was the first statutory law governing the ownership and not so used but employed to develop the national wealth. This class of property constituted property
disposition of reclaimed lands in the Philippines. On May 18, 1907, the Philippine Commission enacted of public dominion although employed for some economic or commercial activity to increase the
Act No. 1654 which provided for the lease, but not the sale, of reclaimed lands of the government to national wealth.
corporations and individuals. Later, on November 29, 1919, the Philippine Legislature approved Act
No. 2874, the Public Land Act, which authorized the lease, but not the sale, of reclaimed lands of the Article 341 of the Civil Code of 1889 governed the re-classification of property of public dominion into
government to corporations and individuals. On November 7, 1936, the National Assembly passed private property, to wit:
Commonwealth Act No. 141, also known as the Public Land Act, which authorized the lease, but not
the sale, of reclaimed lands of the government to corporations and individuals. CA No. 141 continues "Art. 341. Property of public dominion, when no longer devoted to public use or to the defense of the
to this day as the general law governing the classification and disposition of lands of the public territory, shall become a part of the private property of the State."
domain.
This provision, however, was not self-executing. The legislature, or the executive department
The Spanish Law of Waters of 1866 and the Civil Code of 1889 pursuant to law, must declare the property no longer needed for public use or territorial defense
before the government could lease or alienate the property to private parties.45
Under the Spanish Law of Waters of 1866, the shores, bays, coves, inlets and all waters within the
maritime zone of the Spanish territory belonged to the public domain for public use.44 The Spanish Act No. 1654 of the Philippine Commission
Law of Waters of 1866 allowed the reclamation of the sea under Article 5, which provided as follows:
On May 8, 1907, the Philippine Commission enacted Act No. 1654 which regulated the lease of
"Article 5. Lands reclaimed from the sea in consequence of works constructed by the State, or by the reclaimed and foreshore lands. The salient provisions of this law were as follows:
provinces, pueblos or private persons, with proper permission, shall become the property of the party
constructing such works, unless otherwise provided by the terms of the grant of authority." "Section 1. The control and disposition of the foreshore as defined in existing law, and the title to all
Government or public lands made or reclaimed by the Government by dredging or filling or otherwise
Under the Spanish Law of Waters, land reclaimed from the sea belonged to the party undertaking the throughout the Philippine Islands, shall be retained by the Government without prejudice to vested
reclamation, provided the government issued the necessary permit and did not reserve ownership of rights and without prejudice to rights conceded to the City of Manila in the Luneta Extension.
the reclaimed land to the State.
Section 2. (a) The Secretary of the Interior shall cause all Government or public lands made or
Article 339 of the Civil Code of 1889 defined property of public dominion as follows: reclaimed by the Government by dredging or filling or otherwise to be divided into lots or blocks, with
the necessary streets and alleyways located thereon, and shall cause plats and plans of such surveys
"Art. 339. Property of public dominion is – to be prepared and filed with the Bureau of Lands.

73
(b) Upon completion of such plats and plans the Governor-General shall give notice to the public that Sec. 8. Only those lands shall be declared open to disposition or concession which have been officially
such parts of the lands so made or reclaimed as are not needed for public purposes will be leased for delimited or classified x x x.
commercial and business purposes, x x x.
xxx
xxx
Sec. 55. Any tract of land of the public domain which, being neither timber nor mineral land, shall be
(e) The leases above provided for shall be disposed of to the highest and best bidder therefore, subject classified as suitable for residential purposes or for commercial, industrial, or other productive
to such regulations and safeguards as the Governor-General may by executive order prescribe." purposes other than agricultural purposes, and shall be open to disposition or concession, shall be
(Emphasis supplied) disposed of under the provisions of this chapter, and not otherwise.

Act No. 1654 mandated that the government should retain title to all lands reclaimed by the Sec. 56. The lands disposable under this title shall be classified as follows:
government. The Act also vested in the government control and disposition of foreshore lands.
Private parties could lease lands reclaimed by the government only if these lands were no longer (a) Lands reclaimed by the Government by dredging, filling, or other means;
needed for public purpose. Act No. 1654 mandated public bidding in the lease of government
reclaimed lands. Act No. 1654 made government reclaimed lands sui generis in that unlike other (b) Foreshore;
public lands which the government could sell to private parties, these reclaimed lands were available
only for lease to private parties. (c) Marshy lands or lands covered with water bordering upon the shores or banks of navigable lakes
or rivers;
Act No. 1654, however, did not repeal Section 5 of the Spanish Law of Waters of 1866. Act No. 1654
did not prohibit private parties from reclaiming parts of the sea under Section 5 of the Spanish Law (d) Lands not included in any of the foregoing classes.
of Waters. Lands reclaimed from the sea by private parties with government permission remained
private lands. x x x.

Act No. 2874 of the Philippine Legislature Sec. 58. The lands comprised in classes (a), (b), and (c) of section fifty-six shall be disposed of to private
parties by lease only and not otherwise, as soon as the Governor-General, upon recommendation by
On November 29, 1919, the Philippine Legislature enacted Act No. 2874, the Public Land Act.46 The the Secretary of Agriculture and Natural Resources, shall declare that the same are not necessary for
salient provisions of Act No. 2874, on reclaimed lands, were as follows: the public service and are open to disposition under this chapter. The lands included in class (d) may
be disposed of by sale or lease under the provisions of this Act." (Emphasis supplied)
"Sec. 6. The Governor-General, upon the recommendation of the Secretary of Agriculture and Natural
Resources, shall from time to time classify the lands of the public domain into – Section 6 of Act No. 2874 authorized the Governor-General to "classify lands of the public domain
into x x x alienable or disposable"47 lands. Section 7 of the Act empowered the Governor-General to
(a) Alienable or disposable, "declare what lands are open to disposition or concession." Section 8 of the Act limited alienable or
disposable lands only to those lands which have been "officially delimited and classified."
(b) Timber, and
Section 56 of Act No. 2874 stated that lands "disposable under this title48 shall be classified" as
(c) Mineral lands, x x x. government reclaimed, foreshore and marshy lands, as well as other lands. All these lands, however,
must be suitable for residential, commercial, industrial or other productive non-agricultural purposes.
Sec. 7. For the purposes of the government and disposition of alienable or disposable public lands, These provisions vested upon the Governor-General the power to classify inalienable lands of the
the Governor-General, upon recommendation by the Secretary of Agriculture and Natural Resources, public domain into disposable lands of the public domain. These provisions also empowered the
shall from time to time declare what lands are open to disposition or concession under this Act." Governor-General to classify further such disposable lands of the public domain into government
reclaimed, foreshore or marshy lands of the public domain, as well as other non-agricultural lands.

74
development of water power, in which cases beneficial use may be the measure and limit of the
Section 58 of Act No. 2874 categorically mandated that disposable lands of the public domain grant." (Emphasis supplied)
classified as government reclaimed, foreshore and marshy lands "shall be disposed of to private
parties by lease only and not otherwise." The Governor-General, before allowing the lease of these The 1935 Constitution barred the alienation of all natural resources except public agricultural lands,
lands to private parties, must formally declare that the lands were "not necessary for the public which were the only natural resources the State could alienate. Thus, foreshore lands, considered
service." Act No. 2874 reiterated the State policy to lease and not to sell government reclaimed, part of the State's natural resources, became inalienable by constitutional fiat, available only for lease
foreshore and marshy lands of the public domain, a policy first enunciated in 1907 in Act No. 1654. for 25 years, renewable for another 25 years. The government could alienate foreshore lands only
Government reclaimed, foreshore and marshy lands remained sui generis, as the only alienable or after these lands were reclaimed and classified as alienable agricultural lands of the public domain.
disposable lands of the public domain that the government could not sell to private parties. Government reclaimed and marshy lands of the public domain, being neither timber nor mineral
lands, fell under the classification of public agricultural lands.50 However, government reclaimed and
The rationale behind this State policy is obvious. Government reclaimed, foreshore and marshy public marshy lands, although subject to classification as disposable public agricultural lands, could only be
lands for non-agricultural purposes retain their inherent potential as areas for public service. This is leased and not sold to private parties because of Act No. 2874.
the reason the government prohibited the sale, and only allowed the lease, of these lands to private
parties. The State always reserved these lands for some future public service. The prohibition on private parties from acquiring ownership of government reclaimed and marshy
lands of the public domain was only a statutory prohibition and the legislature could therefore
Act No. 2874 did not authorize the reclassification of government reclaimed, foreshore and marshy remove such prohibition. The 1935 Constitution did not prohibit individuals and corporations from
lands into other non-agricultural lands under Section 56 (d). Lands falling under Section 56 (d) were acquiring government reclaimed and marshy lands of the public domain that were classified as
the only lands for non-agricultural purposes the government could sell to private parties. Thus, under agricultural lands under existing public land laws. Section 2, Article XIII of the 1935 Constitution
Act No. 2874, the government could not sell government reclaimed, foreshore and marshy lands to provided as follows:
private parties, unless the legislature passed a law allowing their sale.49
"Section 2. No private corporation or association may acquire, lease, or hold public agricultural lands
Act No. 2874 did not prohibit private parties from reclaiming parts of the sea pursuant to Section 5 of in excess of one thousand and twenty four hectares, nor may any individual acquire such lands by
the Spanish Law of Waters of 1866. Lands reclaimed from the sea by private parties with government purchase in excess of one hundred and forty hectares, or by lease in excess of one thousand and
permission remained private lands. twenty-four hectares, or by homestead in excess of twenty-four hectares. Lands adapted to grazing,
not exceeding two thousand hectares, may be leased to an individual, private corporation, or
Dispositions under the 1935 Constitution association." (Emphasis supplied)

On May 14, 1935, the 1935 Constitution took effect upon its ratification by the Filipino people. The Still, after the effectivity of the 1935 Constitution, the legislature did not repeal Section 58 of Act No.
1935 Constitution, in adopting the Regalian doctrine, declared in Section 1, Article XIII, that – 2874 to open for sale to private parties government reclaimed and marshy lands of the public domain.
On the contrary, the legislature continued the long established State policy of retaining for the
"Section 1. All agricultural, timber, and mineral lands of the public domain, waters, minerals, coal, government title and ownership of government reclaimed and marshy lands of the public domain.
petroleum, and other mineral oils, all forces of potential energy and other natural resources of the
Philippines belong to the State, and their disposition, exploitation, development, or utilization shall Commonwealth Act No. 141 of the Philippine National Assembly
be limited to citizens of the Philippines or to corporations or associations at least sixty per centum of
the capital of which is owned by such citizens, subject to any existing right, grant, lease, or concession On November 7, 1936, the National Assembly approved Commonwealth Act No. 141, also known as
at the time of the inauguration of the Government established under this Constitution. Natural the Public Land Act, which compiled the then existing laws on lands of the public domain. CA No. 141,
resources, with the exception of public agricultural land, shall not be alienated, and no license, as amended, remains to this day the existing general law governing the classification and disposition
concession, or lease for the exploitation, development, or utilization of any of the natural resources of lands of the public domain other than timber and mineral lands.51
shall be granted for a period exceeding twenty-five years, renewable for another twenty-five years,
except as to water rights for irrigation, water supply, fisheries, or industrial uses other than the Section 6 of CA No. 141 empowers the President to classify lands of the public domain into "alienable
or disposable"52 lands of the public domain, which prior to such classification are inalienable and

75
outside the commerce of man. Section 7 of CA No. 141 authorizes the President to "declare what (a) Lands reclaimed by the Government by dredging, filling, or other means;
lands are open to disposition or concession." Section 8 of CA No. 141 states that the government can
declare open for disposition or concession only lands that are "officially delimited and classified." (b) Foreshore;
Sections 6, 7 and 8 of CA No. 141 read as follows:
(c) Marshy lands or lands covered with water bordering upon the shores or banks of navigable lakes
"Sec. 6. The President, upon the recommendation of the Secretary of Agriculture and Commerce, shall or rivers;
from time to time classify the lands of the public domain into –
(d) Lands not included in any of the foregoing classes.
(a) Alienable or disposable,
Sec. 60. Any tract of land comprised under this title may be leased or sold, as the case may be, to any
(b) Timber, and person, corporation, or association authorized to purchase or lease public lands for agricultural
purposes. x x x.
(c) Mineral lands,
Sec. 61. The lands comprised in classes (a), (b), and (c) of section fifty-nine shall be disposed of to
and may at any time and in like manner transfer such lands from one class to another,53 for the private parties by lease only and not otherwise, as soon as the President, upon recommendation by
purpose of their administration and disposition. the Secretary of Agriculture, shall declare that the same are not necessary for the public service and
are open to disposition under this chapter. The lands included in class (d) may be disposed of by sale
Sec. 7. For the purposes of the administration and disposition of alienable or disposable public lands, or lease under the provisions of this Act." (Emphasis supplied)
the President, upon recommendation by the Secretary of Agriculture and Commerce, shall from time
to time declare what lands are open to disposition or concession under this Act. Section 61 of CA No. 141 readopted, after the effectivity of the 1935 Constitution, Section 58 of Act
No. 2874 prohibiting the sale of government reclaimed, foreshore and marshy disposable lands of the
Sec. 8. Only those lands shall be declared open to disposition or concession which have been officially public domain. All these lands are intended for residential, commercial, industrial or other non-
delimited and classified and, when practicable, surveyed, and which have not been reserved for public agricultural purposes. As before, Section 61 allowed only the lease of such lands to private parties.
or quasi-public uses, nor appropriated by the Government, nor in any manner become private The government could sell to private parties only lands falling under Section 59 (d) of CA No. 141, or
property, nor those on which a private right authorized and recognized by this Act or any other valid those lands for non-agricultural purposes not classified as government reclaimed, foreshore and
law may be claimed, or which, having been reserved or appropriated, have ceased to be so. x x x." marshy disposable lands of the public domain. Foreshore lands, however, became inalienable under
the 1935 Constitution which only allowed the lease of these lands to qualified private parties.
Thus, before the government could alienate or dispose of lands of the public domain, the President
must first officially classify these lands as alienable or disposable, and then declare them open to Section 58 of CA No. 141 expressly states that disposable lands of the public domain intended for
disposition or concession. There must be no law reserving these lands for public or quasi-public uses. residential, commercial, industrial or other productive purposes other than agricultural "shall be
disposed of under the provisions of this chapter and not otherwise." Under Section 10 of CA No. 141,
The salient provisions of CA No. 141, on government reclaimed, foreshore and marshy lands of the the term "disposition" includes lease of the land. Any disposition of government reclaimed, foreshore
public domain, are as follows: and marshy disposable lands for non-agricultural purposes must comply with Chapter IX, Title III of
CA No. 141,54 unless a subsequent law amended or repealed these provisions.
"Sec. 58. Any tract of land of the public domain which, being neither timber nor mineral land, is
intended to be used for residential purposes or for commercial, industrial, or other productive In his concurring opinion in the landmark case of Republic Real Estate Corporation v. Court of
purposes other than agricultural, and is open to disposition or concession, shall be disposed of under Appeals,55 Justice Reynato S. Puno summarized succinctly the law on this matter, as follows:
the provisions of this chapter and not otherwise.
"Foreshore lands are lands of public dominion intended for public use. So too are lands reclaimed by
Sec. 59. The lands disposable under this title shall be classified as follows: the government by dredging, filling, or other means. Act 1654 mandated that the control and
disposition of the foreshore and lands under water remained in the national government. Said law

76
allowed only the 'leasing' of reclaimed land. The Public Land Acts of 1919 and 1936 also declared that the public interest; but the land so granted, donated, or transferred to a province, municipality or
the foreshore and lands reclaimed by the government were to be "disposed of to private parties by branch or subdivision of the Government shall not be alienated, encumbered, or otherwise disposed
lease only and not otherwise." Before leasing, however, the Governor-General, upon of in a manner affecting its title, except when authorized by Congress: x x x." (Emphasis supplied)
recommendation of the Secretary of Agriculture and Natural Resources, had first to determine that
the land reclaimed was not necessary for the public service. This requisite must have been met before The congressional authority required in Section 60 of CA No. 141 mirrors the legislative authority
the land could be disposed of. But even then, the foreshore and lands under water were not to be required in Section 56 of Act No. 2874.
alienated and sold to private parties. The disposition of the reclaimed land was only by lease. The land
remained property of the State." (Emphasis supplied) One reason for the congressional authority is that Section 60 of CA No. 141 exempted government
units and entities from the maximum area of public lands that could be acquired from the State. These
As observed by Justice Puno in his concurring opinion, "Commonwealth Act No. 141 has remained in government units and entities should not just turn around and sell these lands to private parties in
effect at present." violation of constitutional or statutory limitations. Otherwise, the transfer of lands for non-
agricultural purposes to government units and entities could be used to circumvent constitutional
The State policy prohibiting the sale to private parties of government reclaimed, foreshore and limitations on ownership of alienable or disposable lands of the public domain. In the same manner,
marshy alienable lands of the public domain, first implemented in 1907 was thus reaffirmed in CA No. such transfers could also be used to evade the statutory prohibition in CA No. 141 on the sale of
141 after the 1935 Constitution took effect. The prohibition on the sale of foreshore lands, however, government reclaimed and marshy lands of the public domain to private parties. Section 60 of CA No.
became a constitutional edict under the 1935 Constitution. Foreshore lands became inalienable as 141 constitutes by operation of law a lien on these lands.57
natural resources of the State, unless reclaimed by the government and classified as agricultural lands
of the public domain, in which case they would fall under the classification of government reclaimed In case of sale or lease of disposable lands of the public domain falling under Section 59 of CA No.
lands. 141, Sections 63 and 67 require a public bidding. Sections 63 and 67 of CA No. 141 provide as follows:

After the effectivity of the 1935 Constitution, government reclaimed and marshy disposable lands of "Sec. 63. Whenever it is decided that lands covered by this chapter are not needed for public
the public domain continued to be only leased and not sold to private parties.56 These lands purposes, the Director of Lands shall ask the Secretary of Agriculture and Commerce (now the
remained sui generis, as the only alienable or disposable lands of the public domain the government Secretary of Natural Resources) for authority to dispose of the same. Upon receipt of such authority,
could not sell to private parties. the Director of Lands shall give notice by public advertisement in the same manner as in the case of
leases or sales of agricultural public land, x x x.
Since then and until now, the only way the government can sell to private parties government
reclaimed and marshy disposable lands of the public domain is for the legislature to pass a law Sec. 67. The lease or sale shall be made by oral bidding; and adjudication shall be made to the highest
authorizing such sale. CA No. 141 does not authorize the President to reclassify government reclaimed bidder. x x x." (Emphasis supplied)
and marshy lands into other non-agricultural lands under Section 59 (d). Lands classified under
Section 59 (d) are the only alienable or disposable lands for non-agricultural purposes that the Thus, CA No. 141 mandates the Government to put to public auction all leases or sales of alienable or
government could sell to private parties. disposable lands of the public domain.58

Moreover, Section 60 of CA No. 141 expressly requires congressional authority before lands under Like Act No. 1654 and Act No. 2874 before it, CA No. 141 did not repeal Section 5 of the Spanish Law
Section 59 that the government previously transferred to government units or entities could be sold of Waters of 1866. Private parties could still reclaim portions of the sea with government permission.
to private parties. Section 60 of CA No. 141 declares that – However, the reclaimed land could become private land only if classified as alienable agricultural land
of the public domain open to disposition under CA No. 141. The 1935 Constitution prohibited the
"Sec. 60. x x x The area so leased or sold shall be such as shall, in the judgment of the Secretary of alienation of all natural resources except public agricultural lands.
Agriculture and Natural Resources, be reasonably necessary for the purposes for which such sale or
lease is requested, and shall not exceed one hundred and forty-four hectares: Provided, however, The Civil Code of 1950
That this limitation shall not apply to grants, donations, or transfers made to a province, municipality
or branch or subdivision of the Government for the purposes deemed by said entities conducive to

77
The Civil Code of 1950 readopted substantially the definition of property of public dominion found in development of water power, in which cases, beneficial use may be the measure and the limit of the
the Civil Code of 1889. Articles 420 and 422 of the Civil Code of 1950 state that – grant." (Emphasis supplied)

"Art. 420. The following things are property of public dominion: The 1973 Constitution prohibited the alienation of all natural resources with the exception of
"agricultural, industrial or commercial, residential, and resettlement lands of the public domain." In
(1) Those intended for public use, such as roads, canals, rivers, torrents, ports and bridges constructed contrast, the 1935 Constitution barred the alienation of all natural resources except "public
by the State, banks, shores, roadsteads, and others of similar character; agricultural lands." However, the term "public agricultural lands" in the 1935 Constitution
encompassed industrial, commercial, residential and resettlement lands of the public domain.60 If
(2) Those which belong to the State, without being for public use, and are intended for some public the land of public domain were neither timber nor mineral land, it would fall under the classification
service or for the development of the national wealth. of agricultural land of the public domain. Both the 1935 and 1973 Constitutions, therefore, prohibited
the alienation of all natural resources except agricultural lands of the public domain.
x x x.
The 1973 Constitution, however, limited the alienation of lands of the public domain to individuals
Art. 422. Property of public dominion, when no longer intended for public use or for public service, who were citizens of the Philippines. Private corporations, even if wholly owned by Philippine citizens,
shall form part of the patrimonial property of the State." were no longer allowed to acquire alienable lands of the public domain unlike in the 1935
Constitution. Section 11, Article XIV of the 1973 Constitution declared that –
Again, the government must formally declare that the property of public dominion is no longer
needed for public use or public service, before the same could be classified as patrimonial property "Sec. 11. The Batasang Pambansa, taking into account conservation, ecological, and development
of the State.59 In the case of government reclaimed and marshy lands of the public domain, the requirements of the natural resources, shall determine by law the size of land of the public domain
declaration of their being disposable, as well as the manner of their disposition, is governed by the which may be developed, held or acquired by, or leased to, any qualified individual, corporation, or
applicable provisions of CA No. 141. association, and the conditions therefor. No private corporation or association may hold alienable
lands of the public domain except by lease not to exceed one thousand hectares in area nor may any
Like the Civil Code of 1889, the Civil Code of 1950 included as property of public dominion those citizen hold such lands by lease in excess of five hundred hectares or acquire by purchase, homestead
properties of the State which, without being for public use, are intended for public service or the or grant, in excess of twenty-four hectares. No private corporation or association may hold by lease,
"development of the national wealth." Thus, government reclaimed and marshy lands of the State, concession, license or permit, timber or forest lands and other timber or forest resources in excess of
even if not employed for public use or public service, if developed to enhance the national wealth, one hundred thousand hectares. However, such area may be increased by the Batasang Pambansa
are classified as property of public dominion. upon recommendation of the National Economic and Development Authority." (Emphasis supplied)

Dispositions under the 1973 Constitution Thus, under the 1973 Constitution, private corporations could hold alienable lands of the public
domain only through lease. Only individuals could now acquire alienable lands of the public domain,
The 1973 Constitution, which took effect on January 17, 1973, likewise adopted the Regalian doctrine. and private corporations became absolutely barred from acquiring any kind of alienable land of the
Section 8, Article XIV of the 1973 Constitution stated that – public domain. The constitutional ban extended to all kinds of alienable lands of the public domain,
while the statutory ban under CA No. 141 applied only to government reclaimed, foreshore and
"Sec. 8. All lands of the public domain, waters, minerals, coal, petroleum and other mineral oils, all marshy alienable lands of the public domain.
forces of potential energy, fisheries, wildlife, and other natural resources of the Philippines belong to
the State. With the exception of agricultural, industrial or commercial, residential, and resettlement PD No. 1084 Creating the Public Estates Authority
lands of the public domain, natural resources shall not be alienated, and no license, concession, or
lease for the exploration, development, exploitation, or utilization of any of the natural resources On February 4, 1977, then President Ferdinand Marcos issued Presidential Decree No. 1084 creating
shall be granted for a period exceeding twenty-five years, renewable for not more than twenty-five PEA, a wholly government owned and controlled corporation with a special charter. Sections 4 and 8
years, except as to water rights for irrigation, water supply, fisheries, or industrial uses other than the of PD No. 1084, vests PEA with the following purposes and powers:

78
"Sec. 4. Purpose. The Authority is hereby created for the following purposes: excess of the area permitted to private corporations by statute." Thus, PEA can hold title to private
lands, as well as title to lands of the public domain.
(a) To reclaim land, including foreshore and submerged areas, by dredging, filling or other means, or
to acquire reclaimed land; In order for PEA to sell its reclaimed foreshore and submerged alienable lands of the public domain,
there must be legislative authority empowering PEA to sell these lands. This legislative authority is
(b) To develop, improve, acquire, administer, deal in, subdivide, dispose, lease and sell any and all necessary in view of Section 60 of CA No.141, which states –
kinds of lands, buildings, estates and other forms of real property, owned, managed, controlled
and/or operated by the government; "Sec. 60. x x x; but the land so granted, donated or transferred to a province, municipality, or branch
or subdivision of the Government shall not be alienated, encumbered or otherwise disposed of in a
(c) To provide for, operate or administer such service as may be necessary for the efficient, manner affecting its title, except when authorized by Congress; x x x." (Emphasis supplied)
economical and beneficial utilization of the above properties.
Without such legislative authority, PEA could not sell but only lease its reclaimed foreshore and
Sec. 5. Powers and functions of the Authority. The Authority shall, in carrying out the purposes for submerged alienable lands of the public domain. Nevertheless, any legislative authority granted to
which it is created, have the following powers and functions: PEA to sell its reclaimed alienable lands of the public domain would be subject to the constitutional
ban on private corporations from acquiring alienable lands of the public domain. Hence, such
(a)To prescribe its by-laws. legislative authority could only benefit private individuals.

xxx Dispositions under the 1987 Constitution

(i) To hold lands of the public domain in excess of the area permitted to private corporations by The 1987 Constitution, like the 1935 and 1973 Constitutions before it, has adopted the Regalian
statute. doctrine. The 1987 Constitution declares that all natural resources are "owned by the State," and
except for alienable agricultural lands of the public domain, natural resources cannot be alienated.
(j) To reclaim lands and to construct work across, or otherwise, any stream, watercourse, canal, ditch, Sections 2 and 3, Article XII of the 1987 Constitution state that –
flume x x x.
"Section 2. All lands of the public domain, waters, minerals, coal, petroleum and other mineral oils,
xxx all forces of potential energy, fisheries, forests or timber, wildlife, flora and fauna, and other natural
resources are owned by the State. With the exception of agricultural lands, all other natural resources
(o) To perform such acts and exercise such functions as may be necessary for the attainment of the shall not be alienated. The exploration, development, and utilization of natural resources shall be
purposes and objectives herein specified." (Emphasis supplied) under the full control and supervision of the State. x x x.

PD No. 1084 authorizes PEA to reclaim both foreshore and submerged areas of the public domain. Section 3. Lands of the public domain are classified into agricultural, forest or timber, mineral lands,
Foreshore areas are those covered and uncovered by the ebb and flow of the tide.61 Submerged and national parks. Agricultural lands of the public domain may be further classified by law according
areas are those permanently under water regardless of the ebb and flow of the tide.62 Foreshore and to the uses which they may be devoted. Alienable lands of the public domain shall be limited to
submerged areas indisputably belong to the public domain63 and are inalienable unless reclaimed, agricultural lands. Private corporations or associations may not hold such alienable lands of the public
classified as alienable lands open to disposition, and further declared no longer needed for public domain except by lease, for a period not exceeding twenty-five years, renewable for not more than
service. twenty-five years, and not to exceed one thousand hectares in area. Citizens of the Philippines may
lease not more than five hundred hectares, or acquire not more than twelve hectares thereof by
The ban in the 1973 Constitution on private corporations from acquiring alienable lands of the public purchase, homestead, or grant.
domain did not apply to PEA since it was then, and until today, a fully owned government corporation.
The constitutional ban applied then, as it still applies now, only to "private corporations and Taking into account the requirements of conservation, ecology, and development, and subject to the
associations." PD No. 1084 expressly empowers PEA "to hold lands of the public domain" even "in requirements of agrarian reform, the Congress shall determine, by law, the size of lands of the public

79
domain which may be acquired, developed, held, or leased and the conditions therefor." (Emphasis However, if the constitutional intent is to prevent huge landholdings, the Constitution could have
supplied) simply limited the size of alienable lands of the public domain that corporations could acquire. The
Constitution could have followed the limitations on individuals, who could acquire not more than 24
The 1987 Constitution continues the State policy in the 1973 Constitution banning private hectares of alienable lands of the public domain under the 1973 Constitution, and not more than 12
corporations from acquiring any kind of alienable land of the public domain. Like the 1973 hectares under the 1987 Constitution.
Constitution, the 1987 Constitution allows private corporations to hold alienable lands of the public
domain only through lease. As in the 1935 and 1973 Constitutions, the general law governing the If the constitutional intent is to encourage economic family-size farms, placing the land in the name
lease to private corporations of reclaimed, foreshore and marshy alienable lands of the public domain of a corporation would be more effective in preventing the break-up of farmlands. If the farmland is
is still CA No. 141. registered in the name of a corporation, upon the death of the owner, his heirs would inherit shares
in the corporation instead of subdivided parcels of the farmland. This would prevent the continuing
The Rationale behind the Constitutional Ban break-up of farmlands into smaller and smaller plots from one generation to the next.

The rationale behind the constitutional ban on corporations from acquiring, except through lease, In actual practice, the constitutional ban strengthens the constitutional limitation on individuals from
alienable lands of the public domain is not well understood. During the deliberations of the 1986 acquiring more than the allowed area of alienable lands of the public domain. Without the
Constitutional Commission, the commissioners probed the rationale behind this ban, thus: constitutional ban, individuals who already acquired the maximum area of alienable lands of the
public domain could easily set up corporations to acquire more alienable public lands. An individual
"FR. BERNAS: Mr. Vice-President, my questions have reference to page 3, line 5 which says: could own as many corporations as his means would allow him. An individual could even hide his
ownership of a corporation by putting his nominees as stockholders of the corporation. The
`No private corporation or association may hold alienable lands of the public domain except by lease, corporation is a convenient vehicle to circumvent the constitutional limitation on acquisition by
not to exceed one thousand hectares in area.' individuals of alienable lands of the public domain.

If we recall, this provision did not exist under the 1935 Constitution, but this was introduced in the The constitutional intent, under the 1973 and 1987 Constitutions, is to transfer ownership of only a
1973 Constitution. In effect, it prohibits private corporations from acquiring alienable public lands. limited area of alienable land of the public domain to a qualified individual. This constitutional intent
But it has not been very clear in jurisprudence what the reason for this is. In some of the cases decided is safeguarded by the provision prohibiting corporations from acquiring alienable lands of the public
in 1982 and 1983, it was indicated that the purpose of this is to prevent large landholdings. Is that the domain, since the vehicle to circumvent the constitutional intent is removed. The available alienable
intent of this provision? public lands are gradually decreasing in the face of an ever-growing population. The most effective
way to insure faithful adherence to this constitutional intent is to grant or sell alienable lands of the
MR. VILLEGAS: I think that is the spirit of the provision. public domain only to individuals. This, it would seem, is the practical benefit arising from the
constitutional ban.
FR. BERNAS: In existing decisions involving the Iglesia ni Cristo, there were instances where the Iglesia
ni Cristo was not allowed to acquire a mere 313-square meter land where a chapel stood because the The Amended Joint Venture Agreement
Supreme Court said it would be in violation of this." (Emphasis supplied)
The subject matter of the Amended JVA, as stated in its second Whereas clause, consists of three
In Ayog v. Cusi,64 the Court explained the rationale behind this constitutional ban in this way: properties, namely:

"Indeed, one purpose of the constitutional prohibition against purchases of public agricultural lands 1. "[T]hree partially reclaimed and substantially eroded islands along Emilio Aguinaldo Boulevard in
by private corporations is to equitably diffuse land ownership or to encourage 'owner-cultivatorship Paranaque and Las Pinas, Metro Manila, with a combined titled area of 1,578,441 square meters;"
and the economic family-size farm' and to prevent a recurrence of cases like the instant case. Huge
landholdings by corporations or private persons had spawned social unrest." 2. "[A]nother area of 2,421,559 square meters contiguous to the three islands;" and

80
3. "[A]t AMARI's option as approved by PEA, an additional 350 hectares more or less to regularize the The Amended JVA is the product of a renegotiation of the original JVA dated April 25, 1995 and its
configuration of the reclaimed area."65 supplemental agreement dated August 9, 1995.

PEA confirms that the Amended JVA involves "the development of the Freedom Islands and further The Threshold Issue
reclamation of about 250 hectares x x x," plus an option "granted to AMARI to subsequently reclaim
another 350 hectares x x x."66 The threshold issue is whether AMARI, a private corporation, can acquire and own under the
Amended JVA 367.5 hectares of reclaimed foreshore and submerged areas in Manila Bay in view of
In short, the Amended JVA covers a reclamation area of 750 hectares. Only 157.84 hectares of the Sections 2 and 3, Article XII of the 1987 Constitution which state that:
750-hectare reclamation project have been reclaimed, and the rest of the 592.15 hectares are still
submerged areas forming part of Manila Bay. "Section 2. All lands of the public domain, waters, minerals, coal, petroleum, and other mineral oils,
all forces of potential energy, fisheries, forests or timber, wildlife, flora and fauna, and other natural
Under the Amended JVA, AMARI will reimburse PEA the sum of P1,894,129,200.00 for PEA's "actual resources are owned by the State. With the exception of agricultural lands, all other natural resources
cost" in partially reclaiming the Freedom Islands. AMARI will also complete, at its own expense, the shall not be alienated. x x x.
reclamation of the Freedom Islands. AMARI will further shoulder all the reclamation costs of all the
other areas, totaling 592.15 hectares, still to be reclaimed. AMARI and PEA will share, in the xxx
proportion of 70 percent and 30 percent, respectively, the total net usable area which is defined in
the Amended JVA as the total reclaimed area less 30 percent earmarked for common areas. Title to Section 3. x x x Alienable lands of the public domain shall be limited to agricultural lands. Private
AMARI's share in the net usable area, totaling 367.5 hectares, will be issued in the name of AMARI. corporations or associations may not hold such alienable lands of the public domain except by lease,
Section 5.2 (c) of the Amended JVA provides that – x x x."(Emphasis supplied)

"x x x, PEA shall have the duty to execute without delay the necessary deed of transfer or conveyance Classification of Reclaimed Foreshore and Submerged Areas
of the title pertaining to AMARI's Land share based on the Land Allocation Plan. PEA, when requested
in writing by AMARI, shall then cause the issuance and delivery of the proper certificates of title PEA readily concedes that lands reclaimed from foreshore or submerged areas of Manila Bay are
covering AMARI's Land Share in the name of AMARI, x x x; provided, that if more than seventy percent alienable or disposable lands of the public domain. In its Memorandum,67 PEA admits that –
(70%) of the titled area at any given time pertains to AMARI, PEA shall deliver to AMARI only seventy
percent (70%) of the titles pertaining to AMARI, until such time when a corresponding proportionate "Under the Public Land Act (CA 141, as amended), reclaimed lands are classified as alienable and
area of additional land pertaining to PEA has been titled." (Emphasis supplied) disposable lands of the public domain:

Indisputably, under the Amended JVA AMARI will acquire and own a maximum of 367.5 hectares of 'Sec. 59. The lands disposable under this title shall be classified as follows:
reclaimed land which will be titled in its name.
(a) Lands reclaimed by the government by dredging, filling, or other means;
To implement the Amended JVA, PEA delegated to the unincorporated PEA-AMARI joint venture
PEA's statutory authority, rights and privileges to reclaim foreshore and submerged areas in Manila x x x.'" (Emphasis supplied)
Bay. Section 3.2.a of the Amended JVA states that –
Likewise, the Legal Task Force68 constituted under Presidential Administrative Order No. 365
"PEA hereby contributes to the joint venture its rights and privileges to perform Rawland Reclamation admitted in its Report and Recommendation to then President Fidel V. Ramos, "[R]eclaimed lands are
and Horizontal Development as well as own the Reclamation Area, thereby granting the Joint Venture classified as alienable and disposable lands of the public domain."69 The Legal Task Force concluded
the full and exclusive right, authority and privilege to undertake the Project in accordance with the that –
Master Development Plan."
"D. Conclusion

81
Reclaimed lands are lands of the public domain. However, by statutory authority, the rights of of PEA pursuant to Section 103 of PD No. 1529 authorizing the issuance of certificates of title
ownership and disposition over reclaimed lands have been transferred to PEA, by virtue of which PEA, corresponding to land patents. To this day, these certificates of title are still in the name of PEA.
as owner, may validly convey the same to any qualified person without violating the Constitution or
any statute. PD No. 1085, coupled with President Aquino's actual issuance of a special patent covering the
Freedom Islands, is equivalent to an official proclamation classifying the Freedom Islands as alienable
The constitutional provision prohibiting private corporations from holding public land, except by lease or disposable lands of the public domain. PD No. 1085 and President Aquino's issuance of a land
(Sec. 3, Art. XVII,70 1987 Constitution), does not apply to reclaimed lands whose ownership has patent also constitute a declaration that the Freedom Islands are no longer needed for public service.
passed on to PEA by statutory grant." The Freedom Islands are thus alienable or disposable lands of the public domain, open to disposition
or concession to qualified parties.
Under Section 2, Article XII of the 1987 Constitution, the foreshore and submerged areas of Manila
Bay are part of the "lands of the public domain, waters x x x and other natural resources" and At the time then President Aquino issued Special Patent No. 3517, PEA had already reclaimed the
consequently "owned by the State." As such, foreshore and submerged areas "shall not be alienated," Freedom Islands although subsequently there were partial erosions on some areas. The government
unless they are classified as "agricultural lands" of the public domain. The mere reclamation of these had also completed the necessary surveys on these islands. Thus, the Freedom Islands were no longer
areas by PEA does not convert these inalienable natural resources of the State into alienable or part of Manila Bay but part of the land mass. Section 3, Article XII of the 1987 Constitution classifies
disposable lands of the public domain. There must be a law or presidential proclamation officially lands of the public domain into "agricultural, forest or timber, mineral lands, and national parks."
classifying these reclaimed lands as alienable or disposable and open to disposition or concession. Being neither timber, mineral, nor national park lands, the reclaimed Freedom Islands necessarily fall
Moreover, these reclaimed lands cannot be classified as alienable or disposable if the law has under the classification of agricultural lands of the public domain. Under the 1987 Constitution,
reserved them for some public or quasi-public use.71 agricultural lands of the public domain are the only natural resources that the State may alienate to
qualified private parties. All other natural resources, such as the seas or bays, are "waters x x x owned
Section 8 of CA No. 141 provides that "only those lands shall be declared open to disposition or by the State" forming part of the public domain, and are inalienable pursuant to Section 2, Article XII
concession which have been officially delimited and classified."72 The President has the authority to of the 1987 Constitution.
classify inalienable lands of the public domain into alienable or disposable lands of the public domain,
pursuant to Section 6 of CA No. 141. In Laurel vs. Garcia,73 the Executive Department attempted to AMARI claims that the Freedom Islands are private lands because CDCP, then a private corporation,
sell the Roppongi property in Tokyo, Japan, which was acquired by the Philippine Government for use reclaimed the islands under a contract dated November 20, 1973 with the Commissioner of Public
as the Chancery of the Philippine Embassy. Although the Chancery had transferred to another location Highways. AMARI, citing Article 5 of the Spanish Law of Waters of 1866, argues that "if the ownership
thirteen years earlier, the Court still ruled that, under Article 42274 of the Civil Code, a property of of reclaimed lands may be given to the party constructing the works, then it cannot be said that
public dominion retains such character until formally declared otherwise. The Court ruled that – reclaimed lands are lands of the public domain which the State may not alienate."75 Article 5 of the
Spanish Law of Waters reads as follows:
"The fact that the Roppongi site has not been used for a long time for actual Embassy service does
not automatically convert it to patrimonial property. Any such conversion happens only if the "Article 5. Lands reclaimed from the sea in consequence of works constructed by the State, or by the
property is withdrawn from public use (Cebu Oxygen and Acetylene Co. v. Bercilles, 66 SCRA 481 provinces, pueblos or private persons, with proper permission, shall become the property of the party
[1975]. A property continues to be part of the public domain, not available for private appropriation constructing such works, unless otherwise provided by the terms of the grant of authority." (Emphasis
or ownership 'until there is a formal declaration on the part of the government to withdraw it from supplied)
being such' (Ignacio v. Director of Lands, 108 Phil. 335 [1960]." (Emphasis supplied)
Under Article 5 of the Spanish Law of Waters of 1866, private parties could reclaim from the sea only
PD No. 1085, issued on February 4, 1977, authorized the issuance of special land patents for lands with "proper permission" from the State. Private parties could own the reclaimed land only if not
reclaimed by PEA from the foreshore or submerged areas of Manila Bay. On January 19, 1988 then "otherwise provided by the terms of the grant of authority." This clearly meant that no one could
President Corazon C. Aquino issued Special Patent No. 3517 in the name of PEA for the 157.84 reclaim from the sea without permission from the State because the sea is property of public
hectares comprising the partially reclaimed Freedom Islands. Subsequently, on April 9, 1999 the dominion. It also meant that the State could grant or withhold ownership of the reclaimed land
Register of Deeds of the Municipality of Paranaque issued TCT Nos. 7309, 7311 and 7312 in the name because any reclaimed land, like the sea from which it emerged, belonged to the State. Thus, a private
person reclaiming from the sea without permission from the State could not acquire ownership of

82
the reclaimed land which would remain property of public dominion like the sea it replaced.76 Article classifying these submerged areas as alienable or disposable lands of the public domain open to
5 of the Spanish Law of Waters of 1866 adopted the time-honored principle of land ownership that disposition. These submerged areas are not covered by any patent or certificate of title. There can be
"all lands that were not acquired from the government, either by purchase or by grant, belong to the no dispute that these submerged areas form part of the public domain, and in their present state are
public domain."77 inalienable and outside the commerce of man. Until reclaimed from the sea, these submerged areas
are, under the Constitution, "waters x x x owned by the State," forming part of the public domain and
Article 5 of the Spanish Law of Waters must be read together with laws subsequently enacted on the consequently inalienable. Only when actually reclaimed from the sea can these submerged areas be
disposition of public lands. In particular, CA No. 141 requires that lands of the public domain must classified as public agricultural lands, which under the Constitution are the only natural resources that
first be classified as alienable or disposable before the government can alienate them. These lands the State may alienate. Once reclaimed and transformed into public agricultural lands, the
must not be reserved for public or quasi-public purposes.78 Moreover, the contract between CDCP government may then officially classify these lands as alienable or disposable lands open to
and the government was executed after the effectivity of the 1973 Constitution which barred private disposition. Thereafter, the government may declare these lands no longer needed for public service.
corporations from acquiring any kind of alienable land of the public domain. This contract could not Only then can these reclaimed lands be considered alienable or disposable lands of the public domain
have converted the Freedom Islands into private lands of a private corporation. and within the commerce of man.

Presidential Decree No. 3-A, issued on January 11, 1973, revoked all laws authorizing the reclamation The classification of PEA's reclaimed foreshore and submerged lands into alienable or disposable
of areas under water and revested solely in the National Government the power to reclaim lands. lands open to disposition is necessary because PEA is tasked under its charter to undertake public
Section 1 of PD No. 3-A declared that – services that require the use of lands of the public domain. Under Section 5 of PD No. 1084, the
functions of PEA include the following: "[T]o own or operate railroads, tramways and other kinds of
"The provisions of any law to the contrary notwithstanding, the reclamation of areas under water, land transportation, x x x; [T]o construct, maintain and operate such systems of sanitary sewers as
whether foreshore or inland, shall be limited to the National Government or any person authorized may be necessary; [T]o construct, maintain and operate such storm drains as may be necessary." PEA
by it under a proper contract. (Emphasis supplied) is empowered to issue "rules and regulations as may be necessary for the proper use by private
parties of any or all of the highways, roads, utilities, buildings and/or any of its properties and to
x x x." impose or collect fees or tolls for their use." Thus, part of the reclaimed foreshore and submerged
lands held by the PEA would actually be needed for public use or service since many of the functions
PD No. 3-A repealed Section 5 of the Spanish Law of Waters of 1866 because reclamation of areas imposed on PEA by its charter constitute essential public services.
under water could now be undertaken only by the National Government or by a person contracted
by the National Government. Private parties may reclaim from the sea only under a contract with the Moreover, Section 1 of Executive Order No. 525 provides that PEA "shall be primarily responsible for
National Government, and no longer by grant or permission as provided in Section 5 of the Spanish integrating, directing, and coordinating all reclamation projects for and on behalf of the National
Law of Waters of 1866. Government." The same section also states that "[A]ll reclamation projects shall be approved by the
President upon recommendation of the PEA, and shall be undertaken by the PEA or through a proper
Executive Order No. 525, issued on February 14, 1979, designated PEA as the National Government's contract executed by it with any person or entity; x x x." Thus, under EO No. 525, in relation to PD No.
implementing arm to undertake "all reclamation projects of the government," which "shall be 3-A and PD No.1084, PEA became the primary implementing agency of the National Government to
undertaken by the PEA or through a proper contract executed by it with any person or entity." Under reclaim foreshore and submerged lands of the public domain. EO No. 525 recognized PEA as the
such contract, a private party receives compensation for reclamation services rendered to PEA. government entity "to undertake the reclamation of lands and ensure their maximum utilization in
Payment to the contractor may be in cash, or in kind consisting of portions of the reclaimed land, promoting public welfare and interests."79 Since large portions of these reclaimed lands would
subject to the constitutional ban on private corporations from acquiring alienable lands of the public obviously be needed for public service, there must be a formal declaration segregating reclaimed
domain. The reclaimed land can be used as payment in kind only if the reclaimed land is first classified lands no longer needed for public service from those still needed for public service.1âwphi1.nêt
as alienable or disposable land open to disposition, and then declared no longer needed for public
service. Section 3 of EO No. 525, by declaring that all lands reclaimed by PEA "shall belong to or be owned by
the PEA," could not automatically operate to classify inalienable lands into alienable or disposable
The Amended JVA covers not only the Freedom Islands, but also an additional 592.15 hectares which lands of the public domain. Otherwise, reclaimed foreshore and submerged lands of the public
are still submerged and forming part of Manila Bay. There is no legislative or Presidential act

83
domain would automatically become alienable once reclaimed by PEA, whether or not classified as and 782 of CA No. 141. Once DENR decides that the reclaimed lands should be so classified, it then
alienable or disposable. recommends to the President the issuance of a proclamation classifying the lands as alienable or
disposable lands of the public domain open to disposition. We note that then DENR Secretary
The Revised Administrative Code of 1987, a later law than either PD No. 1084 or EO No. 525, vests in Fulgencio S. Factoran, Jr. countersigned Special Patent No. 3517 in compliance with the Revised
the Department of Environment and Natural Resources ("DENR" for brevity) the following powers Administrative Code and Sections 6 and 7 of CA No. 141.
and functions:
In short, DENR is vested with the power to authorize the reclamation of areas under water, while PEA
"Sec. 4. Powers and Functions. The Department shall: is vested with the power to undertake the physical reclamation of areas under water, whether directly
or through private contractors. DENR is also empowered to classify lands of the public domain into
(1) x x x alienable or disposable lands subject to the approval of the President. On the other hand, PEA is
tasked to develop, sell or lease the reclaimed alienable lands of the public domain.
xxx
Clearly, the mere physical act of reclamation by PEA of foreshore or submerged areas does not make
(4) Exercise supervision and control over forest lands, alienable and disposable public lands, mineral the reclaimed lands alienable or disposable lands of the public domain, much less patrimonial lands
resources and, in the process of exercising such control, impose appropriate taxes, fees, charges, of PEA. Likewise, the mere transfer by the National Government of lands of the public domain to PEA
rentals and any such form of levy and collect such revenues for the exploration, development, does not make the lands alienable or disposable lands of the public domain, much less patrimonial
utilization or gathering of such resources; lands of PEA.

xxx Absent two official acts – a classification that these lands are alienable or disposable and open to
disposition and a declaration that these lands are not needed for public service, lands reclaimed by
(14) Promulgate rules, regulations and guidelines on the issuance of licenses, permits, concessions, PEA remain inalienable lands of the public domain. Only such an official classification and formal
lease agreements and such other privileges concerning the development, exploration and utilization declaration can convert reclaimed lands into alienable or disposable lands of the public domain, open
of the country's marine, freshwater, and brackish water and over all aquatic resources of the country to disposition under the Constitution, Title I and Title III83 of CA No. 141 and other applicable laws.84
and shall continue to oversee, supervise and police our natural resources; cancel or cause to cancel
such privileges upon failure, non-compliance or violations of any regulation, order, and for all other PEA's Authority to Sell Reclaimed Lands
causes which are in furtherance of the conservation of natural resources and supportive of the
national interest; PEA, like the Legal Task Force, argues that as alienable or disposable lands of the public domain, the
reclaimed lands shall be disposed of in accordance with CA No. 141, the Public Land Act. PEA, citing
(15) Exercise exclusive jurisdiction on the management and disposition of all lands of the public Section 60 of CA No. 141, admits that reclaimed lands transferred to a branch or subdivision of the
domain and serve as the sole agency responsible for classification, sub-classification, surveying and government "shall not be alienated, encumbered, or otherwise disposed of in a manner affecting its
titling of lands in consultation with appropriate agencies."80 (Emphasis supplied) title, except when authorized by Congress: x x x."85 (Emphasis by PEA)

As manager, conservator and overseer of the natural resources of the State, DENR exercises In Laurel vs. Garcia,86 the Court cited Section 48 of the Revised Administrative Code of 1987, which
"supervision and control over alienable and disposable public lands." DENR also exercises "exclusive states that –
jurisdiction on the management and disposition of all lands of the public domain." Thus, DENR decides
whether areas under water, like foreshore or submerged areas of Manila Bay, should be reclaimed or "Sec. 48. Official Authorized to Convey Real Property. Whenever real property of the Government is
not. This means that PEA needs authorization from DENR before PEA can undertake reclamation authorized by law to be conveyed, the deed of conveyance shall be executed in behalf of the
projects in Manila Bay, or in any part of the country. government by the following: x x x."

DENR also exercises exclusive jurisdiction over the disposition of all lands of the public domain. Hence, Thus, the Court concluded that a law is needed to convey any real property belonging to the
DENR decides whether reclaimed lands of PEA should be classified as alienable under Sections 681 Government. The Court declared that -

84
Presidential Decree No. 1084. Any and all income that the PEA may derive from the sale, lease or use
"It is not for the President to convey real property of the government on his or her own sole will. Any of reclaimed lands shall be used in accordance with the provisions of Presidential Decree No. 1084."
such conveyance must be authorized and approved by a law enacted by the Congress. It requires
executive and legislative concurrence." (Emphasis supplied) There is no express authority under either PD No. 1085 or EO No. 525 for PEA to sell its reclaimed
lands. PD No. 1085 merely transferred "ownership and administration" of lands reclaimed from
PEA contends that PD No. 1085 and EO No. 525 constitute the legislative authority allowing PEA to Manila Bay to PEA, while EO No. 525 declared that lands reclaimed by PEA "shall belong to or be
sell its reclaimed lands. PD No. 1085, issued on February 4, 1977, provides that – owned by PEA." EO No. 525 expressly states that PEA should dispose of its reclaimed lands "in
accordance with the provisions of Presidential Decree No. 1084," the charter of PEA.
"The land reclaimed in the foreshore and offshore area of Manila Bay pursuant to the contract for the
reclamation and construction of the Manila-Cavite Coastal Road Project between the Republic of the PEA's charter, however, expressly tasks PEA "to develop, improve, acquire, administer, deal in,
Philippines and the Construction and Development Corporation of the Philippines dated November subdivide, dispose, lease and sell any and all kinds of lands x x x owned, managed, controlled and/or
20, 1973 and/or any other contract or reclamation covering the same area is hereby transferred, operated by the government."87 (Emphasis supplied) There is, therefore, legislative authority granted
conveyed and assigned to the ownership and administration of the Public Estates Authority to PEA to sell its lands, whether patrimonial or alienable lands of the public domain. PEA may sell to
established pursuant to PD No. 1084; Provided, however, That the rights and interests of the private parties its patrimonial properties in accordance with the PEA charter free from constitutional
Construction and Development Corporation of the Philippines pursuant to the aforesaid contract shall limitations. The constitutional ban on private corporations from acquiring alienable lands of the public
be recognized and respected. domain does not apply to the sale of PEA's patrimonial lands.

Henceforth, the Public Estates Authority shall exercise the rights and assume the obligations of the PEA may also sell its alienable or disposable lands of the public domain to private individuals since,
Republic of the Philippines (Department of Public Highways) arising from, or incident to, the aforesaid with the legislative authority, there is no longer any statutory prohibition against such sales and the
contract between the Republic of the Philippines and the Construction and Development Corporation constitutional ban does not apply to individuals. PEA, however, cannot sell any of its alienable or
of the Philippines. disposable lands of the public domain to private corporations since Section 3, Article XII of the 1987
Constitution expressly prohibits such sales. The legislative authority benefits only individuals. Private
In consideration of the foregoing transfer and assignment, the Public Estates Authority shall issue in corporations remain barred from acquiring any kind of alienable land of the public domain, including
favor of the Republic of the Philippines the corresponding shares of stock in said entity with an issued government reclaimed lands.
value of said shares of stock (which) shall be deemed fully paid and non-assessable.
The provision in PD No. 1085 stating that portions of the reclaimed lands could be transferred by PEA
The Secretary of Public Highways and the General Manager of the Public Estates Authority shall to the "contractor or his assignees" (Emphasis supplied) would not apply to private corporations but
execute such contracts or agreements, including appropriate agreements with the Construction and only to individuals because of the constitutional ban. Otherwise, the provisions of PD No. 1085 would
Development Corporation of the Philippines, as may be necessary to implement the above. violate both the 1973 and 1987 Constitutions.

Special land patent/patents shall be issued by the Secretary of Natural Resources in favor of the Public The requirement of public auction in the sale of reclaimed lands
Estates Authority without prejudice to the subsequent transfer to the contractor or his assignees of
such portion or portions of the land reclaimed or to be reclaimed as provided for in the above- Assuming the reclaimed lands of PEA are classified as alienable or disposable lands open to
mentioned contract. On the basis of such patents, the Land Registration Commission shall issue the disposition, and further declared no longer needed for public service, PEA would have to conduct a
corresponding certificate of title." (Emphasis supplied) public bidding in selling or leasing these lands. PEA must observe the provisions of Sections 63 and 67
of CA No. 141 requiring public auction, in the absence of a law exempting PEA from holding a public
On the other hand, Section 3 of EO No. 525, issued on February 14, 1979, provides that - auction.88 Special Patent No. 3517 expressly states that the patent is issued by authority of the
Constitution and PD No. 1084, "supplemented by Commonwealth Act No. 141, as amended." This is
"Sec. 3. All lands reclaimed by PEA shall belong to or be owned by the PEA which shall be responsible an acknowledgment that the provisions of CA No. 141 apply to the disposition of reclaimed alienable
for its administration, development, utilization or disposition in accordance with the provisions of lands of the public domain unless otherwise provided by law. Executive Order No. 654,89 which
authorizes PEA "to determine the kind and manner of payment for the transfer" of its assets and

85
properties, does not exempt PEA from the requirement of public auction. EO No. 654 merely 350 hectares. The original JVA, a negotiated contract, enlarged the reclamation area to 750
authorizes PEA to decide the mode of payment, whether in kind and in installment, but does not hectares.94 The failure of public bidding on December 10, 1991, involving only 407.84 hectares,95 is
authorize PEA to dispense with public auction. not a valid justification for a negotiated sale of 750 hectares, almost double the area publicly
auctioned. Besides, the failure of public bidding happened on December 10, 1991, more than three
Moreover, under Section 79 of PD No. 1445, otherwise known as the Government Auditing Code, the years before the signing of the original JVA on April 25, 1995. The economic situation in the country
government is required to sell valuable government property through public bidding. Section 79 of had greatly improved during the intervening period.
PD No. 1445 mandates that –
Reclamation under the BOT Law and the Local Government Code
"Section 79. When government property has become unserviceable for any cause, or is no longer
needed, it shall, upon application of the officer accountable therefor, be inspected by the head of the The constitutional prohibition in Section 3, Article XII of the 1987 Constitution is absolute and clear:
agency or his duly authorized representative in the presence of the auditor concerned and, if found "Private corporations or associations may not hold such alienable lands of the public domain except
to be valueless or unsaleable, it may be destroyed in their presence. If found to be valuable, it may by lease, x x x." Even Republic Act No. 6957 ("BOT Law," for brevity), cited by PEA and AMARI as
be sold at public auction to the highest bidder under the supervision of the proper committee on legislative authority to sell reclaimed lands to private parties, recognizes the constitutional ban.
award or similar body in the presence of the auditor concerned or other authorized representative of Section 6 of RA No. 6957 states –
the Commission, after advertising by printed notice in the Official Gazette, or for not less than three
consecutive days in any newspaper of general circulation, or where the value of the property does "Sec. 6. Repayment Scheme. - For the financing, construction, operation and maintenance of any
not warrant the expense of publication, by notices posted for a like period in at least three public infrastructure projects undertaken through the build-operate-and-transfer arrangement or any of its
places in the locality where the property is to be sold. In the event that the public auction fails, the variations pursuant to the provisions of this Act, the project proponent x x x may likewise be repaid
property may be sold at a private sale at such price as may be fixed by the same committee or body in the form of a share in the revenue of the project or other non-monetary payments, such as, but
concerned and approved by the Commission." not limited to, the grant of a portion or percentage of the reclaimed land, subject to the constitutional
requirements with respect to the ownership of the land: x x x." (Emphasis supplied)
It is only when the public auction fails that a negotiated sale is allowed, in which case the Commission
on Audit must approve the selling price.90 The Commission on Audit implements Section 79 of the A private corporation, even one that undertakes the physical reclamation of a government BOT
Government Auditing Code through Circular No. 89-29691 dated January 27, 1989. This circular project, cannot acquire reclaimed alienable lands of the public domain in view of the constitutional
emphasizes that government assets must be disposed of only through public auction, and a ban.
negotiated sale can be resorted to only in case of "failure of public auction."
Section 302 of the Local Government Code, also mentioned by PEA and AMARI, authorizes local
At the public auction sale, only Philippine citizens are qualified to bid for PEA's reclaimed foreshore governments in land reclamation projects to pay the contractor or developer in kind consisting of a
and submerged alienable lands of the public domain. Private corporations are barred from bidding at percentage of the reclaimed land, to wit:
the auction sale of any kind of alienable land of the public domain.
"Section 302. Financing, Construction, Maintenance, Operation, and Management of Infrastructure
PEA originally scheduled a public bidding for the Freedom Islands on December 10, 1991. PEA Projects by the Private Sector. x x x
imposed a condition that the winning bidder should reclaim another 250 hectares of submerged areas
to regularize the shape of the Freedom Islands, under a 60-40 sharing of the additional reclaimed xxx
areas in favor of the winning bidder.92 No one, however, submitted a bid. On December 23, 1994,
the Government Corporate Counsel advised PEA it could sell the Freedom Islands through In case of land reclamation or construction of industrial estates, the repayment plan may consist of
negotiation, without need of another public bidding, because of the failure of the public bidding on the grant of a portion or percentage of the reclaimed land or the industrial estate constructed."
December 10, 1991.93
Although Section 302 of the Local Government Code does not contain a proviso similar to that of the
However, the original JVA dated April 25, 1995 covered not only the Freedom Islands and the BOT Law, the constitutional restrictions on land ownership automatically apply even though not
additional 250 hectares still to be reclaimed, it also granted an option to AMARI to reclaim another expressly mentioned in the Local Government Code.

86
"When the lots in dispute were certified as disposable on May 19, 1971, and free patents were issued
Thus, under either the BOT Law or the Local Government Code, the contractor or developer, if a covering the same in favor of the private respondents, the said lots ceased to be part of the public
corporate entity, can only be paid with leaseholds on portions of the reclaimed land. If the contractor domain and, therefore, the Director of Lands lost jurisdiction over the same."
or developer is an individual, portions of the reclaimed land, not exceeding 12 hectares96 of non-
agricultural lands, may be conveyed to him in ownership in view of the legislative authority allowing 5.Republic v. Court of Appeals,101 where the Court stated –
such conveyance. This is the only way these provisions of the BOT Law and the Local Government
Code can avoid a direct collision with Section 3, Article XII of the 1987 Constitution. "Proclamation No. 350, dated October 9, 1956, of President Magsaysay legally effected a land grant
to the Mindanao Medical Center, Bureau of Medical Services, Department of Health, of the whole lot,
Registration of lands of the public domain validly sufficient for initial registration under the Land Registration Act. Such land grant is constitutive
of a 'fee simple' title or absolute title in favor of petitioner Mindanao Medical Center. Thus, Section
Finally, PEA theorizes that the "act of conveying the ownership of the reclaimed lands to public 122 of the Act, which governs the registration of grants or patents involving public lands, provides
respondent PEA transformed such lands of the public domain to private lands." This theory is echoed that 'Whenever public lands in the Philippine Islands belonging to the Government of the United
by AMARI which maintains that the "issuance of the special patent leading to the eventual issuance States or to the Government of the Philippines are alienated, granted or conveyed to persons or to
of title takes the subject land away from the land of public domain and converts the property into public or private corporations, the same shall be brought forthwith under the operation of this Act
patrimonial or private property." In short, PEA and AMARI contend that with the issuance of Special (Land Registration Act, Act 496) and shall become registered lands.'"
Patent No. 3517 and the corresponding certificates of titles, the 157.84 hectares comprising the
Freedom Islands have become private lands of PEA. In support of their theory, PEA and AMARI cite The first four cases cited involve petitions to cancel the land patents and the corresponding
the following rulings of the Court: certificates of titles issued to private parties. These four cases uniformly hold that the Director of
Lands has no jurisdiction over private lands or that upon issuance of the certificate of title the land
1. Sumail v. Judge of CFI of Cotabato,97 where the Court held – automatically comes under the Torrens System. The fifth case cited involves the registration under
the Torrens System of a 12.8-hectare public land granted by the National Government to Mindanao
"Once the patent was granted and the corresponding certificate of title was issued, the land ceased Medical Center, a government unit under the Department of Health. The National Government
to be part of the public domain and became private property over which the Director of Lands has transferred the 12.8-hectare public land to serve as the site for the hospital buildings and other
neither control nor jurisdiction." facilities of Mindanao Medical Center, which performed a public service. The Court affirmed the
registration of the 12.8-hectare public land in the name of Mindanao Medical Center under Section
2. Lee Hong Hok v. David,98 where the Court declared - 122 of Act No. 496. This fifth case is an example of a public land being registered under Act No. 496
without the land losing its character as a property of public dominion.
"After the registration and issuance of the certificate and duplicate certificate of title based on a
public land patent, the land covered thereby automatically comes under the operation of Republic In the instant case, the only patent and certificates of title issued are those in the name of PEA, a
Act 496 subject to all the safeguards provided therein."3. Heirs of Gregorio Tengco v. Heirs of Jose wholly government owned corporation performing public as well as proprietary functions. No patent
Aliwalas,99 where the Court ruled - or certificate of title has been issued to any private party. No one is asking the Director of Lands to
cancel PEA's patent or certificates of title. In fact, the thrust of the instant petition is that PEA's
"While the Director of Lands has the power to review homestead patents, he may do so only so long certificates of title should remain with PEA, and the land covered by these certificates, being alienable
as the land remains part of the public domain and continues to be under his exclusive control; but lands of the public domain, should not be sold to a private corporation.
once the patent is registered and a certificate of title is issued, the land ceases to be part of the public
domain and becomes private property over which the Director of Lands has neither control nor Registration of land under Act No. 496 or PD No. 1529 does not vest in the registrant private or public
jurisdiction." ownership of the land. Registration is not a mode of acquiring ownership but is merely evidence of
ownership previously conferred by any of the recognized modes of acquiring ownership. Registration
4. Manalo v. Intermediate Appellate Court,100 where the Court held – does not give the registrant a better right than what the registrant had prior to the registration.102
The registration of lands of the public domain under the Torrens system, by itself, cannot convert
public lands into private lands.103

87
Whereas, there are several reclamation projects which are ongoing or being proposed to be
Jurisprudence holding that upon the grant of the patent or issuance of the certificate of title the undertaken in various parts of the country which need to be evaluated for consistency with national
alienable land of the public domain automatically becomes private land cannot apply to government programs;
units and entities like PEA. The transfer of the Freedom Islands to PEA was made subject to the
provisions of CA No. 141 as expressly stated in Special Patent No. 3517 issued by then President Whereas, there is a need to give further institutional support to the Government's declared policy to
Aquino, to wit: provide for a coordinated, economical and efficient reclamation of lands;

"NOW, THEREFORE, KNOW YE, that by authority of the Constitution of the Philippines and in Whereas, Presidential Decree No. 3-A requires that all reclamation of areas shall be limited to the
conformity with the provisions of Presidential Decree No. 1084, supplemented by Commonwealth National Government or any person authorized by it under proper contract;
Act No. 141, as amended, there are hereby granted and conveyed unto the Public Estates Authority
the aforesaid tracts of land containing a total area of one million nine hundred fifteen thousand eight Whereas, a central authority is needed to act on behalf of the National Government which shall
hundred ninety four (1,915,894) square meters; the technical description of which are hereto ensure a coordinated and integrated approach in the reclamation of lands;
attached and made an integral part hereof." (Emphasis supplied)
Whereas, Presidential Decree No. 1084 creates the Public Estates Authority as a government
Thus, the provisions of CA No. 141 apply to the Freedom Islands on matters not covered by PD No. corporation to undertake reclamation of lands and ensure their maximum utilization in promoting
1084. Section 60 of CA No. 141 prohibits, "except when authorized by Congress," the sale of alienable public welfare and interests; and
lands of the public domain that are transferred to government units or entities. Section 60 of CA No.
141 constitutes, under Section 44 of PD No. 1529, a "statutory lien affecting title" of the registered Whereas, Presidential Decree No. 1416 provides the President with continuing authority to
land even if not annotated on the certificate of title.104 Alienable lands of the public domain held by reorganize the national government including the transfer, abolition, or merger of functions and
government entities under Section 60 of CA No. 141 remain public lands because they cannot be offices.
alienated or encumbered unless Congress passes a law authorizing their disposition. Congress,
however, cannot authorize the sale to private corporations of reclaimed alienable lands of the public NOW, THEREFORE, I, FERDINAND E. MARCOS, President of the Philippines, by virtue of the powers
domain because of the constitutional ban. Only individuals can benefit from such law. vested in me by the Constitution and pursuant to Presidential Decree No. 1416, do hereby order and
direct the following:
The grant of legislative authority to sell public lands in accordance with Section 60 of CA No. 141 does
not automatically convert alienable lands of the public domain into private or patrimonial lands. The Section 1. The Public Estates Authority (PEA) shall be primarily responsible for integrating, directing,
alienable lands of the public domain must be transferred to qualified private parties, or to and coordinating all reclamation projects for and on behalf of the National Government. All
government entities not tasked to dispose of public lands, before these lands can become private or reclamation projects shall be approved by the President upon recommendation of the PEA, and shall
patrimonial lands. Otherwise, the constitutional ban will become illusory if Congress can declare lands be undertaken by the PEA or through a proper contract executed by it with any person or entity;
of the public domain as private or patrimonial lands in the hands of a government agency tasked to Provided, that, reclamation projects of any national government agency or entity authorized under
dispose of public lands. This will allow private corporations to acquire directly from government its charter shall be undertaken in consultation with the PEA upon approval of the President.
agencies limitless areas of lands which, prior to such law, are concededly public lands.
x x x ."
Under EO No. 525, PEA became the central implementing agency of the National Government to
reclaim foreshore and submerged areas of the public domain. Thus, EO No. 525 declares that – As the central implementing agency tasked to undertake reclamation projects nationwide, with
authority to sell reclaimed lands, PEA took the place of DENR as the government agency charged with
"EXECUTIVE ORDER NO. 525 leasing or selling reclaimed lands of the public domain. The reclaimed lands being leased or sold by
PEA are not private lands, in the same manner that DENR, when it disposes of other alienable lands,
Designating the Public Estates Authority as the Agency Primarily Responsible for all Reclamation does not dispose of private lands but alienable lands of the public domain. Only when qualified private
Projects parties acquire these lands will the lands become private lands. In the hands of the government

88
agency tasked and authorized to dispose of alienable of disposable lands of the public domain, these
lands are still public, not private lands. PD No. 1529

Furthermore, PEA's charter expressly states that PEA "shall hold lands of the public domain" as well "Sec. 103. Certificate of Title to Patents. Whenever public land is by the Government alienated,
as "any and all kinds of lands." PEA can hold both lands of the public domain and private lands. Thus, granted or conveyed to any person, the same shall be brought forthwith under the operation of this
the mere fact that alienable lands of the public domain like the Freedom Islands are transferred to Decree." (Emphasis supplied)
PEA and issued land patents or certificates of title in PEA's name does not automatically make such
lands private. Based on its legislative history, the phrase "conveyed to any person" in Section 103 of PD No. 1529
includes conveyances of public lands to public corporations.
To allow vast areas of reclaimed lands of the public domain to be transferred to PEA as private lands
will sanction a gross violation of the constitutional ban on private corporations from acquiring any Alienable lands of the public domain "granted, donated, or transferred to a province, municipality, or
kind of alienable land of the public domain. PEA will simply turn around, as PEA has now done under branch or subdivision of the Government," as provided in Section 60 of CA No. 141, may be registered
the Amended JVA, and transfer several hundreds of hectares of these reclaimed and still to be under the Torrens System pursuant to Section 103 of PD No. 1529. Such registration, however, is
reclaimed lands to a single private corporation in only one transaction. This scheme will effectively expressly subject to the condition in Section 60 of CA No. 141 that the land "shall not be alienated,
nullify the constitutional ban in Section 3, Article XII of the 1987 Constitution which was intended to encumbered or otherwise disposed of in a manner affecting its title, except when authorized by
diffuse equitably the ownership of alienable lands of the public domain among Filipinos, now Congress." This provision refers to government reclaimed, foreshore and marshy lands of the public
numbering over 80 million strong. domain that have been titled but still cannot be alienated or encumbered unless expressly authorized
by Congress. The need for legislative authority prevents the registered land of the public domain from
This scheme, if allowed, can even be applied to alienable agricultural lands of the public domain since becoming private land that can be disposed of to qualified private parties.
PEA can "acquire x x x any and all kinds of lands." This will open the floodgates to corporations and
even individuals acquiring hundreds of hectares of alienable lands of the public domain under the The Revised Administrative Code of 1987 also recognizes that lands of the public domain may be
guise that in the hands of PEA these lands are private lands. This will result in corporations amassing registered under the Torrens System. Section 48, Chapter 12, Book I of the Code states –
huge landholdings never before seen in this country - creating the very evil that the constitutional
ban was designed to prevent. This will completely reverse the clear direction of constitutional "Sec. 48. Official Authorized to Convey Real Property. Whenever real property of the Government is
development in this country. The 1935 Constitution allowed private corporations to acquire not more authorized by law to be conveyed, the deed of conveyance shall be executed in behalf of the
than 1,024 hectares of public lands.105 The 1973 Constitution prohibited private corporations from government by the following:
acquiring any kind of public land, and the 1987 Constitution has unequivocally reiterated this
prohibition. (1) x x x

The contention of PEA and AMARI that public lands, once registered under Act No. 496 or PD No. (2) For property belonging to the Republic of the Philippines, but titled in the name of any political
1529, automatically become private lands is contrary to existing laws. Several laws authorize lands of subdivision or of any corporate agency or instrumentality, by the executive head of the agency or
the public domain to be registered under the Torrens System or Act No. 496, now PD No. 1529, instrumentality." (Emphasis supplied)
without losing their character as public lands. Section 122 of Act No. 496, and Section 103 of PD No.
1529, respectively, provide as follows: Thus, private property purchased by the National Government for expansion of a public wharf may
be titled in the name of a government corporation regulating port operations in the country. Private
Act No. 496 property purchased by the National Government for expansion of an airport may also be titled in the
name of the government agency tasked to administer the airport. Private property donated to a
"Sec. 122. Whenever public lands in the Philippine Islands belonging to the x x x Government of the municipality for use as a town plaza or public school site may likewise be titled in the name of the
Philippine Islands are alienated, granted, or conveyed to persons or the public or private corporations, municipality.106 All these properties become properties of the public domain, and if already
the same shall be brought forthwith under the operation of this Act and shall become registered registered under Act No. 496 or PD No. 1529, remain registered land. There is no requirement or
lands." provision in any existing law for the de-registration of land from the Torrens System.

89
lands retain their inherent potential as areas for public use or public service. Alienable lands of the
Private lands taken by the Government for public use under its power of eminent domain become public domain, increasingly becoming scarce natural resources, are to be distributed equitably among
unquestionably part of the public domain. Nevertheless, Section 85 of PD No. 1529 authorizes the our ever-growing population. To insure such equitable distribution, the 1973 and 1987 Constitutions
Register of Deeds to issue in the name of the National Government new certificates of title covering have barred private corporations from acquiring any kind of alienable land of the public domain.
such expropriated lands. Section 85 of PD No. 1529 states – Those who attempt to dispose of inalienable natural resources of the State, or seek to circumvent the
constitutional ban on alienation of lands of the public domain to private corporations, do so at their
"Sec. 85. Land taken by eminent domain. Whenever any registered land, or interest therein, is own risk.
expropriated or taken by eminent domain, the National Government, province, city or municipality,
or any other agency or instrumentality exercising such right shall file for registration in the proper We can now summarize our conclusions as follows:
Registry a certified copy of the judgment which shall state definitely by an adequate description, the
particular property or interest expropriated, the number of the certificate of title, and the nature of 1. The 157.84 hectares of reclaimed lands comprising the Freedom Islands, now covered by
the public use. A memorandum of the right or interest taken shall be made on each certificate of title certificates of title in the name of PEA, are alienable lands of the public domain. PEA may lease these
by the Register of Deeds, and where the fee simple is taken, a new certificate shall be issued in favor lands to private corporations but may not sell or transfer ownership of these lands to private
of the National Government, province, city, municipality, or any other agency or instrumentality corporations. PEA may only sell these lands to Philippine citizens, subject to the ownership limitations
exercising such right for the land so taken. The legal expenses incident to the memorandum of in the 1987 Constitution and existing laws.
registration or issuance of a new certificate of title shall be for the account of the authority taking the
land or interest therein." (Emphasis supplied) 2. The 592.15 hectares of submerged areas of Manila Bay remain inalienable natural resources of the
public domain until classified as alienable or disposable lands open to disposition and declared no
Consequently, lands registered under Act No. 496 or PD No. 1529 are not exclusively private or longer needed for public service. The government can make such classification and declaration only
patrimonial lands. Lands of the public domain may also be registered pursuant to existing laws. after PEA has reclaimed these submerged areas. Only then can these lands qualify as agricultural lands
of the public domain, which are the only natural resources the government can alienate. In their
AMARI makes a parting shot that the Amended JVA is not a sale to AMARI of the Freedom Islands or present state, the 592.15 hectares of submerged areas are inalienable and outside the commerce of
of the lands to be reclaimed from submerged areas of Manila Bay. In the words of AMARI, the man.
Amended JVA "is not a sale but a joint venture with a stipulation for reimbursement of the original
cost incurred by PEA for the earlier reclamation and construction works performed by the CDCP under 3. Since the Amended JVA seeks to transfer to AMARI, a private corporation, ownership of 77.34
its 1973 contract with the Republic." Whether the Amended JVA is a sale or a joint venture, the fact hectares110 of the Freedom Islands, such transfer is void for being contrary to Section 3, Article XII
remains that the Amended JVA requires PEA to "cause the issuance and delivery of the certificates of of the 1987 Constitution which prohibits private corporations from acquiring any kind of alienable
title conveying AMARI's Land Share in the name of AMARI."107 land of the public domain.

This stipulation still contravenes Section 3, Article XII of the 1987 Constitution which provides that 4. Since the Amended JVA also seeks to transfer to AMARI ownership of 290.156 hectares111 of still
private corporations "shall not hold such alienable lands of the public domain except by lease." The submerged areas of Manila Bay, such transfer is void for being contrary to Section 2, Article XII of the
transfer of title and ownership to AMARI clearly means that AMARI will "hold" the reclaimed lands 1987 Constitution which prohibits the alienation of natural resources other than agricultural lands of
other than by lease. The transfer of title and ownership is a "disposition" of the reclaimed lands, a the public domain. PEA may reclaim these submerged areas. Thereafter, the government can classify
transaction considered a sale or alienation under CA No. 141,108 the Government Auditing Code,109 the reclaimed lands as alienable or disposable, and further declare them no longer needed for public
and Section 3, Article XII of the 1987 Constitution. service. Still, the transfer of such reclaimed alienable lands of the public domain to AMARI will be void
in view of Section 3, Article XII of the 1987 Constitution which prohibits private corporations from
The Regalian doctrine is deeply implanted in our legal system. Foreshore and submerged areas form acquiring any kind of alienable land of the public domain.
part of the public domain and are inalienable. Lands reclaimed from foreshore and submerged areas
also form part of the public domain and are also inalienable, unless converted pursuant to law into Clearly, the Amended JVA violates glaringly Sections 2 and 3, Article XII of the 1987 Constitution.
alienable or disposable lands of the public domain. Historically, lands reclaimed by the government Under Article 1409112 of the Civil Code, contracts whose "object or purpose is contrary to law," or
are sui generis, not available for sale to private parties unlike other alienable public lands. Reclaimed whose "object is outside the commerce of men," are "inexistent and void from the beginning." The

90
Court must perform its duty to defend and uphold the Constitution, and therefore declares the
Amended JVA null and void ab initio.

Seventh issue: whether the Court is the proper forum to raise the issue of whether the Amended JVA
is grossly disadvantageous to the government.

Considering that the Amended JVA is null and void ab initio, there is no necessity to rule on this last
issue. Besides, the Court is not a trier of facts, and this last issue involves a determination of factual
matters.

WHEREFORE, the petition is GRANTED. The Public Estates Authority and Amari Coastal Bay
Development Corporation are PERMANENTLY ENJOINED from implementing the Amended Joint
Venture Agreement which is hereby declared NULL and VOID ab initio.

SO ORDERED.

Davide, Jr., C.J., Bellosillo, Puno, Vitug, Kapunan, Mendoza, Panganiban, Quisumbing, Ynares-
Santiago, Sandoval-Gutierrez, Austria-Martinez, and Corona, JJ., concur.

91
G.R. No. 166865 March 2, 2007 007604-000176-D, Parañaque Cadastre, located in San Dionisio, Parañaque City, with an area of
3,520.92 square meters, more or less. Petitioners alleged that "they and their predecessors-in-
ANGELITA F. BUENAVENTURA and PRECIOSA F. BUENAVENTURA, Petitioners, interest acquired title to the said parcel of land thru inheritance, transfer, and possession as owners
vs. of the same since time immemorial and/or within the period provided for by law."8
REPUBLIC OF THE PHILIPPINES, Respondent.
As the trial court found the application to be sufficient in form and substance, it thereby set the case
DECISION for hearing, and directed the service and publication of the notice thereof pursuant to Section 239 of
the Property Registration Decree (Presidential Decree No. 1529).
CHICO-NAZARIO, J.:
On 27 September 2001, when the case was called for hearing, no interested party appeared before
The case before this Court is a Petition for Review on Certiorari under Rule 45 of the 1997 Revised the trial court other than the petitioners. Consequently, petitioners proceeded to present several
Rules of Civil Procedure seeking to annul and set aside the Decision1 and Resolution2 of the Court of documents in order to establish compliance with the jurisdictional requirements. The same were
Appeals in CA-G.R. CV No. 72925 entitled, Angelita F. Buenaventura and Preciosa F. Buenaventura vs. marked and offered in evidence before the court a quo.
Republic of the Philippines, dated 23 August 2004 and 25 January 2005, respectively, which granted
the appeal filed by the Republic of the Philippines (Republic) and declared the parcel of land subject No formal opposition had been filed and no oppositor appeared in any of the previously set hearings
matter of this Petition as public land, thus, reversing the Order3 of the Regional Trial Court (RTC) of of the case; hence, petitioners’ counsel moved for the declaration of general default except for the
Parañaque City dated 29 October 2001, which recognized and confirmed the rights of herein Republic. The same was granted by the court a quo. The case was then referred to a commissioner,
petitioners Angelita F. Buenaventura (Angelita) and Preciosa F. Buenaventura (Preciosa), over the who directly received petitioners’ evidence in chief.
subject property, and issued a decree of registration of the same in their favor.
Petitioners presented five witnesses, namely: Aniceta C. Capiral, Engr. Teofilo R. La Guardia, Atty.
The antecedent facts of the case are as follows: Reginald L. Hernandez, Ricardo H. Lopez, and herein petitioner Angelita, in order to establish the fact
that petitioners and their predecessors have acquired vested right over the subject property by their
Petitioners Angelita and Preciosa are the applicants for registration of title over the subject property. open, continuous, and exclusive possession under a bona fide claim of ownership for over 50 years
They are the heirs of spouses Amado Buenaventura and Irene Flores (spouses Buenaventura) from completely unmolested by any adverse claim, meaning, their possession of the subject property was
whom they acquired the subject property. in the manner and for the period required by law; likewise, to prove the alienable and disposable
character of the subject property.
The facts reveal that the subject property was acquired by the spouses Buenaventura from the Heirs
of Lazaro de Leon, namely: Aurelio de Leon and his sister Rodencia Sta. Agueda even before World Other than the respective testimonies of the above-named witnesses, they also presented and
War II. However, it was only on 30 January 1948 that the corresponding Deed of Sale4 was executed identified several documents10 offered in evidence, which tend to establish further the following: (1)
in favor of the spouses Buenaventura. After the execution of the said Deed of Sale, the spouses petitioners’ fee simple title over the subject property; (2) the nature of the possession and occupation
Buenaventura transferred the tax declaration in their name. Consequently, Tax Declaration (T.D.) No. of the property; (3) its classification as part of the alienable and disposable zone of the government;
5492 covering the subject property in the names of Aurelio and Rodencia was cancelled and T.D. No. and (4) the improvements introduced thereon and the taxes paid on the subject property. Said
61035 was issued in the name of spouses Buenaventura. documents were duly admitted by the trial court.

In 1978, the spouses Buenaventura transferred, by way of Deed of Sale,6 the subject property, On 29 October 2001, based on the pieces of evidence presented by petitioners, the court a quo issued
together with the adjacent property, which they previously acquired from Mariano Pascual, to their an Order granting the application for registration of title of the subject property, the decretal portion
children, among whom are herein petitioners. As a result thereof, a new tax declaration (T.D. No. A- of which reads as follows:
004-05698)7 was issued in the name of the spouses Buenaventura’s children.
WHEREFORE, finding the application of registration of title to the subject parcel of land, known as Lot
Petitioners then filed an Application for Registration of Title on 5 June 2000 before the RTC of 5001-B Cad 299, Parañaque Cadastre, and more particularly described in approved Survey Plan Csd
Parañaque City of the subject property, more particularly described as Cadastral Lot No. 5001-B, Csd- 007604-000176 is hereby confirmed and ordered registered in the names of [petitioners] Preciosa,

92
Angelita, [and in the names of their other siblings] Crisostomo, and Alfredo, all surnamed
Buenaventura, free from all liens and encumbrances. In the Memorandum13 of the petitioners, they allege that the appellate court committed grave error
when it nullified the trial court’s Order dated 29 October 2001, which confirmed their title to the
ONCE THIS DECISION has become final, let another one issue directing the Land Registration Authority subject property. Petitioners claim that contrary to the findings of the Court of Appeals that the
to issue the corresponding decree. above-mentioned Order was not supported by evidence, the records of the case clearly speak of the
existence, not absence, of sufficient evidence to sustain the findings of the court a quo that
Let copies of this [D]ecision be furnished to the adjoining owners, Land Registration Authority, Land petitioners have established possession of the subject property in the manner and for the period
Management Bureau, Office of the Solicitor General, Sec. of Public Works and Highways, Department required by law, that is by open, continuous, exclusive, and notorious possession in the concept of an
of Agrarian Reform, the Director, Forest Management Bureau, Chairman Metropolitan Manila owner since 12 June 1945 or earlier, to warrant the registration of their title to the subject property.
Development Authority, DENR [Department of Environment and Natural Resources], South CENRO,
Land Management Sector, City Mayor of Parañaque and Registry of Deeds, Parañaque City.11 Petitioners likewise argue that the appellate court gravely erred when it declared as public land the
subject property despite the fact that they were able to prove by clear and convincing evidence that
Feeling aggrieved with the aforementioned Order of the trial court, the Republic appealed to the their possession of the subject property was indeed in the manner and within the period required by
Court of Appeals. According to the Republic, petitioners failed to prove continuous, open, exclusive law. Having been in possession of the subject property for more than 30 years, they have already
and notorious possession by their predecessors-in-interest and by themselves; hence, the trial court acquired vested right or title over the subject property by operation of law based on the period
erred in granting petitioners’ application for registration of the subject property. The Republic prayed provided for under the prevailing land registration and property laws; hence, the Decision of the Court
for the reversal of the Order of the trial court and for the dismissal of the application for registration of Appeals is inconsistent with the facts and the law.
filed by petitioners.
The Petition is meritorious.
On 23 August 2004, the Court of Appeals rendered a Decision in favor of the Republic, thus,
overturning the Order of the court a quo. The dispositive portion of the Decision reads as: In resolving the issues involved in the present case, there is a need for this Court to re-examine the
facts of the case for the proper determination of the issues raised herein.
WHEREFORE, the appeal is GRANTED and the Decision of the Regional Trial Court, Branch 274,
Parañaque City dated October 29, 2001 is REVERSED and SET ASIDE and the parcel of land subject As a rule, in the exercise of the Supreme Court’s power of review, the Court is not a trier of facts and
matter of the application is declared public land.12 does not normally undertake the re-examination of the evidence presented by the contending parties
during the trial of the case considering that the findings of fact of the Court of Appeals are conclusive
Petitioners filed a Motion for Reconsideration of the aforesaid Decision on 20 September 2004. In a and binding on the Court.14 However, the rule is not without exceptions. There are several recognized
Resolution dated 25 January 2005 rendered by the appellate court, said Motion for Reconsideration exceptions15 in which factual issues may be resolved by this Court and two of these exceptions find
was forthwith denied for lack of merit. application in this present case, to wit: (1) when the findings of the appellate court are contrary to
those of the trial court; and (2) when the findings of fact of the appellate court are premised on the
Hence, this Petition. supposed absence of evidence but contradicted by the evidence on record.

Petitioners raise the following issues for the resolution of this Court: The issues presented by petitioners will be discussed concurrently, since they are interrelated.

I. Whether or not the Court of Appeals erred in nullifying the Decision of the trial court confirming In the assailed Decision of the Court of Appeals, it ruled that petitioners failed to show possession
petitioners’ title over the subject property for not being allegedly supported by substantial evidence and occupation of the subject property under a bona fide claim of ownership since 12 June 1945 or
as required by law. earlier as provided for in Section 14(1) of the Property Registration Decree. It further said that the
testimonial evidence presented by petitioners was not sufficient to prove petitioners’ possession in
II. Whether or not the Court of Appeals gravely erred in declaring the subject property as pubic land the manner and within the period required by the aforesaid law because petitioners’ witnesses
and ignoring petitioners’ evidence of over 50 year possession in the concept of an owner and merely testified on their familiarity with the subject property.
completely unmolested by any adverse claim.

93
Section 14 of the Property Registration Decree speaks of who may apply for registration of land. The This is to certify that the parcel of land as shown and described on the reverse side of this plan- Lot
said provision of law refers to an original registration through ordinary registration proceedings.16 It 5001-B, Cad-299, Parañaque Cadastre situated at San Dionisio, Parañaque City, Metro Manila
specifically provides: containing an area of 3,520.92 square meters as prepared by Geodetic Engineer Mariano V. Flotildes
for Amado Buenaventura, et al., was verified to be within the Alienable and Disposable Land per L.C.
SEC. 14. Who may apply. – The following persons may file in the proper Court of First Instance [now Map 2623, Project No. 25 of Parañaque per Forestry Administrative Order No. 4-1141 dated January
Regional Trial Court] an application for registration of title to land, whether personally or through 3, 1968.21 (Emphasis supplied.)
their duly authorized representatives:
To our minds, the said certification is sufficient to establish the true nature or character of the subject
(1) Those who by themselves or through their predecessors-in-interest have been in open, property. The certification enjoys a presumption of regularity in the absence of contradictory
continuous, exclusive and notorious possession and occupation of alienable and disposable lands of evidence.22 As it is, the said certification remains uncontested and even the Republic itself did not
the public domain under a bona fide claim of ownership since June 12, 1945, or earlier. present any evidence to refute the contents of the said certification. Therefore, the alienable and
disposable character of the questioned parcel of land has been clearly established by the evidence of
(2) Those who have acquired ownership of private lands by prescription under the provisions of the petitioners, by 3 January 1968, at the latest.
existing laws.
Now, going to the requisites of open, continuous, exclusive and notorious possession and occupation
From the aforesaid provisions of the Property Registration Decree, we can deduce that there are three under a bona fide claim of ownership since 12 June 1945 or earlier, Republic alleges that no sufficient
requisites for the filing of an application for registration of title under the first category, to wit: (1) evidence was adduced by petitioners to show that they and their predecessors-in-interest have been
that the property in question is alienable and disposable land of the public domain; (2) that the in exclusive possession of the subject property since 12 June 1945 or earlier in the concept of an
applicants by themselves or through their predecessors-in-interest have been in open, continuous, owner, to which the Court of Appeals agreed. The Court of Appeals in its decision said that:
exclusive and notorious possession and occupation; and (3) that such possession is under a bona fide
claim of ownership since 12 June 1945 or earlier.17 The second classification relates to the acquisition Although they were able to show possession by their parents, their predecessors-in-interest, since
of private lands by prescription. 1948, they failed to prove the fact of possession since [12 June 1945] before the filing of the
application.23
In the case at bar, the Republic argues, through the Office of the Solicitor General, that petitioners’
own evidence tends to show that the subject property is not alienable and disposable because it was Emphasis should be given to the fact that the Court of Appeals, in its Decision, did not question
a salt bed and a fishpond and under Section 2, Article XII of the Constitution, except for agricultural petitioners’ possession of the subject property since 1948. Verily, it even stated in the said Decision
lands, all other natural resources shall not be alienated. Likewise, under the Regalian Doctrine, all that petitioners’ possession may be reckoned from 1948, the year of the execution of the Deed of
lands not otherwise appearing to be clearly within private ownership are presumed to belong to the Sale. The only reason posited by the appellate court in denying the Order of the trial court which
State. granted the application for registration of title of the petitioners was the fact that petitioners’
evidence was not sufficient to prove that their possession of the subject property was since 12 June
It is true that under the Regalian Doctrine all lands of the public domain belong to the State and all 1945 or earlier.
lands not otherwise appearing to be clearly within private ownership are presumed to belong to the
State.18 However, such presumption is not conclusive. It can be rebutted by the applicant’s We agree with the findings of the Court of Appeals that the evidence presented by petitioners was
presentation of incontrovertible evidence showing that the land subject of the application for not enough to prove that their possession of the subject property started since 12 June 1945 or earlier
registration is alienable and disposable.19 because the evidence established that the questioned parcel of land was acquired by petitioners’
parents only on 30 January 1948, the date of the execution of the Deed of Absolute Sale by its previous
After a thorough examination of the records of this case, this Court found out that petitioners offered owners. They can neither tack their possession to that of the previous owners because they failed to
in evidence a certification20 from the Department of Environment and Natural Resources, National present any evidence of possession by those prior owners. Moreover, petitioners’ possession of the
Capital Region dated 29 October 2001, to prove that the subject property was alienable and subject property could only ripen into ownership on 3 January 1968, when the same became alienable
disposable land of the public domain. The said certification contains the following statements: and disposable. "Any period of possession prior to the date when the [s]ubject [property was]
classified as alienable and disposable is inconsequential and should be excluded from the

94
computation of the period of possession; such possession can never ripen into ownership and unless
the land had been classified as alienable and disposable, the rules on confirmation of imperfect title By prescription, one acquires ownership and other real rights through the lapse of time in the manner
shall not apply thereto."24 and under the conditions laid down by law.

Be that as it may, this will not be an insurmountable bar to the petitioners to have the title to the Also in Article 1113 of the Civil Code, it is provided that:
subject property registered in their names.
All things which are within the commerce of men are susceptible of prescription, unless otherwise
In the case of Republic v. Court of Appeals,25 this Court closely examined the land registration laws provided. Property of the State or any of its subdivision not patrimonial in character shall not be the
governing land registration proceedings in the Philippines. In the aforesaid case, the Court made the object of prescription.
following pronouncements:
Likewise, Article 1137 of the Civil Code states that:
When the Public Land Act was first promulgated in 1936, the period of possession deemed necessary
to vest the right to register their title to agricultural lands of the public domain commenced from July Ownership and other real rights over immovables also prescribe through uninterrupted adverse
26, 1894. However, this period was amended by R.A. [Republic Act] No. 1942, which provided that possession thereof for thirty years, without need of title or of good faith. (Emphasis supplied.)
the bona fide claim of ownership must have been for at least thirty (30) years. Then in 1977, Section
48(b) of the Public Land Act was again amended, this time by P.D. No. 1073, which pegged the It is well-settled that properties classified as alienable and disposable land may be converted into
reckoning date at June 12, 1945. This new starting point is concordant with Section 14(1) of the private property by reason of open, continuous and exclusive possession of at least 30 years.28 Such
Property Registration Decree. property now falls within the contemplation of "private lands" under Section 14(2), over which title
by prescription can be acquired. Hence, because of Section 14(2) of Presidential Decree No. 1529,
Indeed, there are no material differences between Section 14(1) of the Property Registration Decree those who are in possession of alienable and disposable land, and whose possession has been
and Section 48(b) of the Public Land Act, as amended. True, the Public Land Act does refer to characterized as open, continuous and exclusive for 30 years or more, may have the right to register
"agricultural lands of the public domain," while the Property Registration Decree uses the term their title to such land despite the fact that their possession of the land commenced only after 12
"alienable and disposable lands of the public domain." It must be noted though that the Constitution June 1945.29
declares that "alienable lands of the public domain shall be limited to agricultural lands." Clearly the
subject lands under Section 48(b) of the Public Land Act and Section 14(1) of the Property Registration The aforesaid jurisprudential rule truly demonstrates that, in the present case, while petitioners’
Decree are of the same type. possession over the subject property can be reckoned only on 3 January 1968, the date when
according to evidence, the subject property became alienable and disposable, they can still have the
Did the enactment of the Property Registration Decree and the amendatory P.D. No. 1073 preclude subject property registered in their names by virtue of Section 14(2) of the Property Registration
the application for registration of alienable lands of the public domain, possession over which Decree.
commenced only after June 12, 1945? It did not, considering Section 14(2) of the Property
Registration Decree, which governs and authorizes the application of "those who have acquired The records, indeed, reveal that petitioners were in possession of the subject property for more than
ownership of private lands by prescription under the provisions of existing laws."26 (Emphasis 30 years, 32 years to be exact, reckoned from the year 1968, when the subject property was finally
supplied.) declared alienable and disposable by the DENR to the time they filed an application for registration
of title over the subject property on 5 June 2000. Petitioners’ possession of the subject property since
It becomes crystal clear from the aforesaid ruling of the Court that even if the possession of alienable 1968 has been characterized as open, continuous, exclusive and notorious possession and occupation
lands of the public domain commenced only after 12 June 1945, application for registration of the in the concept of an owner.
said property is still possible by virtue of Section 14(2) of the Property Registration Decree which
speaks of prescription. Petitioners presented as evidence their tax declarations covering the years from 1948 until the third
quarter of 2001. They also offered in evidence a certification30 from the Office of the Treasurer of
Under the Civil Code, prescription is one of the modes of acquiring ownership.27 Article 1106 of the the City of Parañaque to prove that realty taxes over the subject property had been duly paid by
Civil Code provides: petitioners. As a rule, tax declarations or realty tax payments of property are not conclusive evidence

95
of ownership, nevertheless, they are good indicia of possession in the concept of owner, for no one would otherwise result in an unjust and unwarranted situation. It would be the height of injustice if
in his right mind would be paying taxes for a property that is not in his actual or constructive petitioners’ registration of title over the said property will de denied solely on that ground.
possession. They constitute at least proof that the holder has a claim of title over the property. The
voluntary declaration of a piece of property for taxation purposes manifests not only one’s sincere WHEREFORE, premises considered, the instant Petition is hereby GRANTED. The Decision and
and honest desire to obtain title to the property and announces his adverse claim against the State Resolution of the Court of Appeals dated 23 August 2004 and 25 January 2005, respectively, are
and all other interested parties, but also the intention to contribute needed revenues to the hereby REVERSED and SET ASIDE. The Order of the trial court dated 29 October 2001 which granted
Government. Such an act strengthens one’s bona fide claim of acquisition of ownership.31 petitioners’ application for registration of the subject property and directing the issuance of a decree
of registration in petitioners’ favor once the judgment has become final and executory is hereby
In the same breath, it cannot be gainsaid that petitioners have been in actual possession of the subject REINSTATED. No costs.
property since 1968, at the latest. According to the testimony of their witnesses, parts of the subject
property are planted with bananas and some vegetables, and a bamboo grove. The other parts of the SO ORDERED.
subject property were used as a fishpond, as well as devoted to salt making until 1990.32 However,
when the property was no longer suitable for agricultural purposes, for fishpond, and for salt making G.R. No.. 195026
because of its conversion to non-agricultural purposes consistent with the zonal development of the
area, the petitioners backfilled the subject property with gravel and sand, for which they paid their CENTRAL MINDANAO UNIVERSITY, represented by its President, DR. MARIA LUISA R. SO LIVEN,
farm helpers just compensation. Thereafter, they enclosed the property with perimeter fence, Petitioner,
installed guards and a caretaker to prevent potential squatters from penetrating the area.33 When vs.
tax declarations and receipts are coupled with actual possession, they constitute evidence of great REPUBLIC OF THE PHILIPPINES, represented by the Department of Environment and Natural
weight and can be the basis of a claim of ownership through prescription.34 Resources, Respondent.

Conspicuously, the petitioners’ witnesses are one in pointing out that petitioners and their DECISION
predecessors-in-interest are the sole claimants of the subject property.
PERALTA, J.:
It bears stressing that the pieces of evidence submitted by petitioners are incontrovertible. No one,
not even the Republic, presented any evidence to contradict the claims of the petitioners that they For this Court's resolution is a petition for review on certiorari dated January 14, 2011 filed by
are in possession of the subject property and their possession of the same is open, continuous and petitioner Central Mindanao University (CMU), seeking to reverse and set aside the Decision1 dated
exclusive in the concept of an owner for over 30 years. Verily, even the appellate court mentioned in December 30, 2010 of the Court of Appeals (CA), which annulled the Decision2 dated December 22,
its Decision that petitioners were able to show possession of the subject property as early as 1948, 1971, the Amended Decision3 dated October 7, 1972 and the Second Amended Decision4 dated
the only basis for its Decision reversing the Order of the trial court being the insufficiency of the September 12, 1974 rendered by the then Court of First Instance (CFI), 15th Judicial District, Branch
evidence presented by petitioners to establish their possession of the subject property prior to 12 II of Bukidnon and annulled the Decrees No. N-154065, N-154066 and N-154067 issued in favor of
June 1945. petitioner and the Original Certificate of Title (OCT) No. 0-160, OCT No. 0-161 and OCT No. 0-162
registered in petitioner's name on January 29, 1975.
IN ALL, petitioners were able to prove sufficiently that they have been in possession of the subject
property for more than 30 years, which possession is characterized as open, continuous, exclusive, The facts follow:
and notorious, in the concept of an owner. By this, the subject alienable and disposable public land
had been effectively converted into private property over which petitioners have acquired ownership Petitioner Central Mindanao University (CMU) is an agricultural educational institution owned and
through prescription to which they are entitled to have title through registration proceedings. run by the State established by virtue of Republic Act No. 4498.5 It is represented by its President, Dr.
Petitioners’ right to have their title to the subject property registered cannot be defeated simply Maria Luisa R. So liven in accordance with CMU Board of Regents Resolution No. 02, s. 2011.6
because the possession of petitioners commenced on a date later than 12 June 1945, for the law and
supplementing jurisprudence amply, justly and rightfully provides the necessary remedy to what

96
The subjects of the controversy are two parcels of land situated at Musuan, Maramag, Bukidnon The cadastral court, in its Decision dated December 22, 1971 in Land Registration Case Cadastral Rec.
identified as "Sheet 1, Lot 1 of Ir-1031-D" consisting of 20,619, 175 square meters, and "Sheet 2, Lot No. 414, declared that the subject parcels of land as public land included in the reservation for CMU,
2 of Ir-1031-D" consisting of 13,391,795 square meters, more or less.7 and be registered in its name, except for specified portions adjudicated to other persons. 18 The court
also gave the other 18 claimants an opportunity to acquire full ownership in the subject parcels of
In 1946, CMU took possession of the subject parcels of land and started construction for the school land. 19 Hence, the court reduced the claim of CMU to 3,041 hectares of total land area. 20 The
site upon the confirmation of the Secretary of Public Instruction. 8 However, during the final survey dispositive portion of the decision reads:
in 1952, CMU discovered that there were several adverse claimants, holders, possessors and
occupants of the portions of lots identified as school sites. 9 In view of the foregoing considerations, judgment is hereby rendered declaring Lot No. 1 containing
an approximate area of 20,619,175 square meters and Lot No. 2 containing an area of 13,391,795
On January 16, 1958, upon the recommendation of the Secretary of Agriculture and Natural square meters, both situated in the barrio of Musuan, municipality of Maramag, Bukidnon, as
Resources and pursuant to the provisions of Section 8310 of Commonwealth Act (C.A.) No. 141, described in the survey plans and technical descriptions approved by the Director of Lands as IR-1031-
otherwise known as Public Land Act, President Carlos P. Garcia issued Proclamation No. 47611 which D, marked as Exhibits "D" and "D-1" of the Central Mindanao University, as public land included in the
reserved certain portions of the public domain in Musuan, Maramag, Bukidnon for petitioner CMU's reservation in favor of said University by virtue of Proclamation No. 476, series of 1958, of the
(formerly Mindanao Agricultural College) site purposes. 12 The said parcels of land were withdrawn President of the Philippines, which may be registered in its name, except such portions hereinbelow
from sale or settlement and reserved for CMU's school site purposes, "subject to private rights, if any specified which are adjudicated in favor of the following:
there be."
1. Venancio Olohoy, married, and Esmeralda Lauga, married to Julio Sagde, both of legal ages and
In a letter dated October 27, 1960, the Director of Lands Zoilo Castrillo formally requested the residents of Valencia, Bukidnon- 17.75 hectares of Lot No.1 as shown in the survey plan (Exh. "D");
Secretary of Agriculture and Natural Resources that he be authorized under Section 87 of C.A. No.
141, to file in the CFI of Bukidnon an application for the compulsory registration of the parcels of land 2. Martina Songkit, of legal age, married to Martin Binanos and resident of Maramag, Bukidnon - 3
reserved by President Garcia under Proclamation No. 476 as CMU's school site purposes. 13 hectares of Lot No. 2 as shown in the plan Exh. "D-1 ";

In the first indorsement dated November 9, 1960, the Office of the Secretary of Agriculture and 3. Pablo Saldivar, widower, of legal age and resident of Dologon, Maramag, Bukidnon- 12 hectares of
Natural Resources, through its Undersecretary Salvador F. Cunanan, forwarded to the Executive Lot No. 2 as indicated in the survey plan Exh. "D-1" abovementioned;
Secretary a recommendation that the Director of Lands be authorized to file the said application.14
4. Fernando Bungcas, married to Feliciana Gayonan and resident of Dologon, Maramag- 6 hectares of
Thereafter, the Office of the President, through the Assistant Executive Secretary Enrique C. Quema, Lot No. 2;
in the second indorsement dated December 12, 1960, authorized and directed the Director of Lands
to file the necessary petition in the CFI of Bukidnon for the compulsory registration of the parcels of 5. Cerilo Salicubay, married to Valentina Bento, and Virginia Salicubay, married to Ricardo Tunasan,
land reserved for CMU. 15 both of legal ages and residents of Panalsalan, Maramag, Bukidnon, share and share alike, -4 hectares
of Lot No. 2
Department Legal Counsel Alejandro V. Recto, in the indorsement dated December 28, 1960,
communicated the said directive and authority granted to the Director of Lands to file the application 6. Rosita Lupiahan, of legal age, married to Simplicio Alba and resident of Maramag, Bukidnon - 4
for compulsory registration.16 hectares of Lot No. 2.

On January 31, 1961, the Director of Lands filed a petition with the then Court of First Instance of The areas herein adjudicated to the above-named private individuals should be surveyed and each
Bukidnon for the settlement and adjudication of the title of the parcels of land reserved in favor of lot given a separate number with their corresponding technical descriptions. Considering, however,
CMU, and for the determination of the rights of adverse claimants in relation to the reservation of that the Court rejected most of the claim due to the dubious nature of the occupation of the claimants
the land. 17 prior to the take-over by the College, now University, in 1946 but most of them remained on the land
up to the present time, in order to avoid possible injustice and in line with the national objective of
providing land for the landless, it is hereby recommended that the claimants enumerated hereunder

97
who filed answers and presented evidence which, nevertheless, was found short of the requirements 18. Dativa P. Velez- 18 hectares of Lot No. 1.
for a decree of registration, be given the opportunity to acquire full ownership thereof through a
homestead, or free patent application if they are landless persons, otherwise by means of a sales Should the above recommendation be given due course, it is further suggested that those claimants
application if they are already owners of other pieces of real estate, after a corresponding included in the said recommendation who are now occupying portions of Lot No. 2 situated above
amendment of the Executive Proclamation through the avenues allowed by law. The following the university grounds on the hillside which they have already denuded, should be transferred to the
claimants may be considered for that purpose, namely: lower portions of the land near or along the Pulangi river in order to enable the University to reforest
the hillside to protect the watershed of its irrigation system and water supply.
1. Geronimo Aniceto and his sister Francisca Aniceto- 12 hectares of Lot No. 2;
After this decision become final and the portions adjudicated to private persons have been
2. Bonifacio Aniceto- 6 hectares of Lot No. 2; segregated and their corresponding technical descriptions provided, the order of the issuance of the
corresponding decree and the certificates of title shall be issued.
3. Julita Aniceto- 12 hectares of Lot No. 2;
SO ORDERED.21
4. Maximo Nulo- 5 hectares of Lot No. 2;
Upon the submission of the parties of the compromise agreement through a Joint Manifestation, the
5. Magno Sepada- 3 hectares of Lot No. 1; cadastral court rendered its Amended Decision dated October 7, 1972 adjudicating in full ownership
of some portions of the subject lots to the 29 groups of claimants.22 A portion of the fallo of the
6. Eulogio Guimba- 12 hectares of Lot No. 2; amended decision reads:

7. Mario Baguhin and his wife, Treponia Dagoplo 18 hectares of Lot No. 2; WHEREFORE, pursuant to the evidence presented and the compromise agreement submitted by the
parties, the decision rendered by this Court on December 22, 1971 is hereby AMENDED and another
8. Aniceto Nayawan- 12 hectares of Lot No. 2; one entered ADJUDICATING in full ownership to the claimants hereinbelow specified the following
portions of the lots in questions, to wit:
9. Eduardo Saloay-ay- 13 hectares of Lot No. 2;
xxxx
10. Arcadio Belmis and his wife Beatriz Lauga- 24 hectares of Lot No. 1;
The remaining portions of Lots 1 and 2 not otherwise adjudicated to any of the above-named private
11. Vitaliano Lauga- 24 hectares of Lot No. 1; claimants are hereby ADJUDICATED in full ownership to the Central Mindanao State University. It is
hereby directed that the different portions of Lots 1 and 2 hereinabove granted to private claimants
12. Procopio Abellar- 12 hectares of Lot No. 1; must [be segregated] by a competent surveyor and given their technical descriptions and
corresponding lot numbers for purposes of the issuance of certificates of title in their favor.
13. Rufino Dador- 12 hectares of Lot No. l;
It is, however, ordered that the area adjacent and around or near the watersheds or sources of Lot
14. Roque Larayan- 12 hectares of Lot No. 1; No. 2 adjudicated to any of the private claimants specified in the foregoing paragraph may be replaced
or substituted to the Central Mindanao State University with other areas of equal extent in either Lot
15. Benito Lutad- 12 hectares of Lot No. l; 1 or 2, should said University desire to do so in order to protect and conserve the watersheds.

16. Juliana Pasamonte- 11 hectares of Lot No. 1; The findings and resolutions made by the Court in its original decision not affected by the
amendments incorporated elsewhere herein shall stand.
17. Tirso Pimentel- 19 hectares of Lot No. 1; and

98
The petition from relief from judgment presented by Lucio Butad which the Court finds without merit 7. To Triponia Dagoplo, Lot 1-G with an area of 60.001 square meters;
is hereby denied.1âwphi1
8. To Mario Baguhin, Lot 1-H with an area of 60.001 square meters;
Once the decision becomes final and the subdivision directed in the preceding paragraph has been
accomplished, the order for the issuance of the corresponding decree of registration and the 9. To Celerina Guimba, Lot 1-1 with an area of 30.001 square meters;
certificates of title in favor of each and every adjudicatee shall likewise issue.
10. To Constantino Baston, Lot 1-J with an area of 30.001 square meters;
SO ORDERED.23
11. To Maximo Nulo, Lot 1-K with an area of 49.999 square meters;
Based on the Order made by the court that those portions of the private claimants in the area
adjacent and around, or near the watersheds of Lot No. 2 may be replaced or substituted by CMU 12. To Beatriz Lauga, Lot 1-L with an area of 100.00 square meters;
with areas of equal extent, the 16 grantees entered into an agreement with CMU for the replacement
of the areas adjudicated to them with those outside the watershed vicinity or beyond the area 13. To Evorcio Olohoy, Lot 1-M with an area of 177.500 square meters;
necessary for the proper development, administration, supervision and utilization of the portion
adjudicated to CMU.24 14. To Arcadia Belmis, Lot 1-N with an area of 140.000 square meters;

Thereafter, the cadastral court, in its second amendment of the Decision dated September 12, 1974, 15. To Luciano Namuag, Lot 1-0 with an area of 240.000 square meters;
ordered that the specific portions of the subject lots be adjudicated to the 33 claimants as indicated
in their agreement.25 It also awarded to CMU Lot 1-S (18,531,671 square meters), Lot 2-A (10,001 16. To Vitaliano Lauga, Lot 1-P with an area of 240.000 square meters;
square meters), and Lot 2-Q (12,266,524 square meters).26 On January 25, 1975, the court issued
Decrees No. N-154065, N-154066, and N-154067 in favor of CMU.27 Consequently, OCT Nos. 0-160, 17. To Rufino Dador, Lot 1-Q with an area of 120.00 square meters;
0-161 and 0-162 were registered in the name of CMU on January 29, 1975. 28 The decretal portion
of the decision reads: 18. To Procopio Abellar, Lot 1-B with an area of 120.001 square meters;

WHEREFORE, finding said manifestation and agreement of the parties in order, the dispositive 19. To Eduardo Saloay-ay, Lot 2-B with an area of 130.000 square meters;
portions of the amended decision rendered by this Court on October 7, 1972 aforementioned is
further amended such that the lots specified hereunder and more particularly indicated in the revised 20. To Francisco Anecito, Lot 2-C with an area of 120.000 square meters;
plans and technical descriptions above-mentioned are hereby adjudicated as follows:
21. To Julita Anecito, Lot 2-D with an area of 60.000 square meters;
1. To Roque Larayan, Lot 1-A with an area of 120.001 square meters;
22. To Vicente Buntan, Lot 2-E with an area of 30.000 square meters;
2. To Fernanda Bungcas, Lot 1-B with an area of 60.00 square meters;
23. To Victoria.no Lacorda, Lot 2-F with an area of 130.000 square meters;
3. To Tirso Pimentel, Lot 1-C with an area of 190.000 square meters;
24. To Cerilo Salicubay, Lot 2-G with an area of 40.000 square meters;
4. To Juliana Pasamonte, Lot 1-D with an area of 109.999 square meters;
25. To Julita Anecito, Lot 2-H with an area of 60.000 square meters;
5. To Dativa Velez, Lot 1-E with an area of 180.00 square meters;
26. To Benito Butad, Lot 2-I with an area of 120.000 square meters;
6. To Mario Bagubin, Lot 1-F with an area of 60.00 square meters;
27. To Pablo Zaldivar, Lot 2-J with an area of 120.000 square meters;

99
ACCORDINGLY, the instant petition is GRANTED. The 1) Decision dated December 22, 1971, 2)
28. To Magno Sepada, Lot 2-K with an area of 30.000 square meters; Amended Decision dated October 7, 1972 and 3) Second Amended Decision dated September 12, 197
4, all rendered by the Court of First Instance, 15th Judicial District, Branch II, Bukidnon Province, in
29. To Anecito Nayawan, Lot 2-L with an area of 120.000 square meters; "L.R.C. Cad. Rec. No. 414, Sec. 87 of Commonwealth Act 141, Ir-1031-D (Lots 1 & 2), Maramag,
Bukidnon, insofar as they adjudicated a portion of the land covered by Proclamation No. 476 to the
30. To Bonifacio Anecito, Lot 2-M with an area of 60.001 square meters; Central Mindanao University, are declared NULL and VOID.

31. To Eulogio Guimba, Lot 2-N with an area of 120.001 square meters; Consequently, 1) Decrees No. N-154065, N-154066 and N-154067 issued in favor of the University on
January 24, 1975; and 2) Original Certificates of Title (OCT) No. 0-160 (covering Lot 1-S), No. 0-161
32. To Martina Songkit, Lot 2-0 with an area of 30.000 square meters; (for Lot 2-A) and No. 0-162 (for Lot 2-Q) registered in the University's name on January 29, 1975, are
likewise declared NULL AND VOID.
33. To Rosita Lapianan, Lot 2-P with an area of 40.000 square meters;
SO ORDERED.34
34. To Central Mindanao State University; Lot 1-S with an area of 18,531.671 square meters;
The CA ruled that there was no sufficient proof of a positive act by the government, such as
35. To Central Mindanao State University; Lot 2-A with an area of 10.001 square meters; presidential proclamation, executive order, administrative action, investigation reports of Bureau of
Lands investigators, or a legislative act or statute, which declared the land of the public domain
36. To Central Mindanao State University, Lot 2-Q with an area of 12,266,524 square meters; alienable and disposable.35 The documents adduced by CMU did not expressly declare that the
covered land is already alienable and disposable and that one of such documents was merely signed
The findings and resolutions made by this Court in its original decision not affected by the by the Assistant Executive Secretary. 36
amendments incorporated herein shall remain in force.
According to the CA, CMU was unable to prove that the subject land ceased to have the status of a
Once this decision becomes final, the order for the issuance of the corresponding decrees of reservation.37 However, the CA clarified that despite nullification of the titles in its favor, CMU is still
registration and the certification of title in favor of each and every adjudicates shall likewise issue. the rightful possessor of the subject property by virtue of Proclamation No. 476.38

SO ORDERED.29 Hence, the petitioner CMU filed the present petition before this Court raising the sole issue:

On December 15, 2003, the Republic of the Philippines, represented by the Department of Whether or not the Court of Appeals:
Environment and Natural Resources through the Office of the Solicitor General (OSG), filed before the
CA a petition for annulment of the Decision dated September 12, 1974 by the cadastral court granting 1. committed a serious and grave error and gravely abused its discretion on a question of law, and
in favor of CMU the title to the subject parcels of land.
2. ruled and decided a question of substance in a way and manner not in accord with law and
The Republic argued that the cadastral court should have summarily dismissed the registration applicable decisions of this Honorable Court
proceedings since the Solicitor General did not sign or file the petition for compulsory registration of
the parcels of I as provided in Sections 5330 and 8731 of Commonwealth Act No. 141. 32 It also alleged in granting the petition for annulment of judgment filed by respondent on the ground that the
that the subject parcels of land are inalienable lands of public domain.33 It maintained that the cadastral court has no jurisdiction over the subject matter or the specific res of the subject matter of
cadastral court did not acquire jurisdiction over the res; hence, the entire proceedings of the case the petition below for the reason that the subject lands are inalienable and non-disposable lands of
should be null and void. the public domain.39

Accordingly, the CA ruled in favor of the respondent. The dispositive portion of the decision reads:

100
CMU maintains that the CA has completely misconstrued the facts of the cadastral proceedings since declared alienable under the provisions of this Act or by proclamation of the President. (Emphasis
the documents it presented showed that the subject property has already been declared, classified, supplied)
and certified by the Office of the President as alienable and disposable lands.40
In the case of Navy Officers' Village Association, Inc. v. Republic, 50 it was held that parcels of land
Particularly, CMU alleges that the specific and express authorization and the directive, as embodied classified as reservations for public or quasi-public uses: (1) are non-alienable and non-disposable in
in the Second Indorsement41 dated December 12, 1960, from the President, through the then view of Section 88 (in relation with Section 8) of C.A. No. 141, specifically declaring them as non-
Assistant Executive Secretary Enrique C. Quema, authorizing the Director of Lands to file the alienable and not subject to disposition; and (2) they remain public domain lands until they are
necessary petition in the CFI of Bukidnon for compulsory registration of the parcels of land reserved actually disposed of in favor of private persons. 51 In other words, lands of the public domain
for CMU's site purposes is equivalent to a declaration and certification by the Office of the President classified as reservations remain to be property of the public dominion until withdrawn from the
that the subject parcels of land are alienable and disposable.42 public or quasi-public use for which they have been reserved, by act of Congress or by proclamation
of the President, or otherwise positively declared to have been converted to patrimonial property. 52
CMU has cited the case of Republic v. Judge De la Rosa43 wherein the then President Quirino issued
on June 22, 1951 a directive authorizing the Director of Lands to file the necessary petition in the CFI In the case at bar, CMU relies on the Court's ruling in the De la Rosa53 case that the directive from
of Isabela for the settlement and adjudication of the titles to the tract of land involved in the Gamu the President authorizing the Director of Lands to file the necessary petition for the compulsory
Public Lands Subdivision, Pls-62, Case 5. This Court held that the said presidential directive was registration of the parcels of land so reserved is the equivalent of the declaration and certification
equivalent to a declaration and certification that the subject land area is alienable and disposable.44 that the subject land is alienable and disposable. As such, CMU avows that the subject lots, as declared
alienable and disposable, are properly registered in its name.
This Court finds the instant petition without merit.
This Court finds that the De la Rosa case does not apply in the instant petition because of the varying
Under the Regalian doctrine, all lands of the public domain belong to the State, and that the State is factual settings, to wit:
the source of any asserted right to ownership of land and charged with the conservation of such
patrimony.45 Also, the doctrine states that all lands not otherwise appearing to be clearly within a. In De la Rosa, the Mallig Plains Reservation was reserved by the President for settlement purposes
private ownership are presumed to belong to the State.46 Consequently, the person applying for under the administration of National Land Settlement Administration (NLSA), later replaced by Land
registration has the burden of proof to overcome the presumption of ownership of lands of the public Settlement and Development Corporation (LASEDECO), while the subject lots in the present case was
domain.47 reserved for educational purposes, e.g. as CMU's school site, under the administration of the Board
of Trustees of CMU.
To prove that a land is alienable, the existence of a positive act of the government, such as presidential
proclamation or an executive order; an administrative action; investigation reports of Bureau of Lands b. The National Resettlement and Rehabilitation Administration, when it replaced LASEDECO,
investigators; and a legislative act or a statute declaring the land as alienable and disposable must be excluded the Mallig Plains Reservation among the properties it needed in carrying out the purposes
established. 48 Hence, a public land remains part of the inalienable public domain unless it is shown and objectives of Republic Act No. 1160,54 thus, the Reservation eventually reverted to and became
to have been reclassified and alienated by the State to a private person. 49 public agricultural land. There was no evidence that CMU ceased to use and occupy the reserved lots
in Musuan, Maramag, Bukidnon as its school site or that its public purpose is abandoned, for the lots
As noted, Proclamation No. 476 issued by then President Garcia, decreeing certain portions of the to revert to and become public agricultural land.
public domain in Musuan, Maramag, Bukidnon for CMU's site purposes, was issued pursuant to
Section 83 of C.A. No. 141. Being reserved as CMU' s school site, the said parcels of land were c. At the time that President Quirino issued the directive, the Gamu Public Land Subdivision in the
withdrawn from sale and settlement, and reserved for CMU. Under Section 88 of the same Act, the Mallig Plains Reservation was not reserved for public or quasi-public purpose or has ceased to be so.
reserved parcels of land would ordinarily be inalienable and not subject to occupation, entry, sale, On the other hand, the subject lots in Bukidnon are reserved for public purpose when the President,
lease or other disposition, subject to an exception, viz.: through the Assistant Executive Secretary, issued the said directive.

Section 88. The tract or tracts of land reserved under the provisions of section eighty-three shall be
non-alienable and shall not be subject to occupation, entry, sale, lease, or other disposition until again

101
d. In the De la Rosa case, the private respondent was a qualified private claimant with the requisite was made. Hence, the lots did not revert to and become public agricultural land for them to be the
period of possession of the subject residential lot in his favor.1avvphi1 Meanwhile, CMU is not a subject of a declaration by the President that the same are alienable and disposable.
private claimant of the land so reserved.
We have ruled in the case of CMU v. DARAB56 that the CMU land reservation is not alienable and
It was explicated in De la Rosa55 that the authority of the President to issue such a directive, held as disposable land of public domain, viz.:
equivalent to a declaration and certification that the subject land area is alienable and disposable,
finds support in Section 7 of C.A. No. 141, to wit: It is our opinion that the 400 hectares ordered segregated by the DARAB and affirmed by the Court
of Appeals in its Decision dated August 20, 1990, is not covered by the [Comprehensive Agrarian
Sec. 7. For purposes of the administration and disposition of alienable or disposable public lands, the Reform Program] CARP because:
President, upon recommendation by the Secretary of Agriculture and Commerce, shall from time to
time declare what lands are open to disposition or concession under this Act. (Emphasis supplied). (1) It is not alienable and disposable land of the public domain;

However, the said directive by the President is limited to those enumerated in Section 8 of C.A. (2) The CMU land reservation is not in excess of specific limits as determined by Congress;
No.141, which provides that:
(3) It is private land registered and titled in the name of its lawful owner, the CMU;
Section 8. Only those lands shall be declared open to disposition or concession which have been
officially delimited and classified and, when practicable, surveyed, and which have not been reserved (4) It is exempt from coverage under Section 10 of R.A. 6657 because the lands are actually, directly
for public or quasi-public uses, nor appropriated by the Government, nor in any manner become and exclusively used and found to be necessary for school site and campus, including experimental
private property, nor those on which a private right authorized and recognized by this Act or any other farm stations for educational purposes, and for establishing seed and seedling research and pilot
valid law may be claimed, or which, having been reserved or appropriated, have ceased to be so. production centers.
However, the President may, for reasons of public interest, declare lands of the public domain open
to disposition before the same have had their boundaries established or been surveyed, or may, for The inalienable character of the lands as part of the long term functions of autonomous agricultural
the same reason, suspend their concession or disposition until they are again declared open to educational institution is reiterated in CMU v. Executive Secretary: 57
concession or disposition by proclamation duly published or by Act of the National Assembly.
(Emphases supplied) It did not matter that it was President Arroyo who, in this case, attempted by proclamation to
appropriate the lands for distribution to indigenous peoples and cultural communities. As already
As can be gleaned from the above provision, the lands which can be declared open to disposition or stated, the lands by their character have become inalienable from the moment President Garcia
concession are those which have been officially delimited and classified, or when practicable dedicated them for CMU's use in scientific and technological research in the field of agriculture. They
surveyed; those not reserved for public or quasi-public purpose; those not appropriated by the have ceased to be alienable public lands.58
Government; those which have not become private property in any manner; those which have no
private right authorized and recognized by C.A. No. 141 or any other valid law may be claimed; or This Court is not unmindful of its earlier pronouncement in CMU v. DARAB that the land reservation
those which have ceased to be reserved or appropriated. is a private land registered and titled in the name of its lawful owner, the CMU. This pronouncement,
which is now being argued by CMU as one of its bases in convincing this Court that the subject
For the said President's directive to file the necessary petition for compulsory registration of parcels property is owned by it and already alienable, is specious. The 1992 CMU case merely enumerated
of land be considered as an equivalent of a declaration that the land is alienable and disposable, the the reasons why the said portion of the property is beyond the coverage of CARP. Moreover, the fact
subject land, among others, should not have been reserved for public or quasi-public purposes. that the Court had already settled the inalienable character of the subject property as part of the long
term functions of the autonomous agricultural educational institution in the case of CMU v. DARAB
Therefore, the said directive on December 12, 1960 cannot be considered as a declaration that said and reiterated in CMU v. Executive Secretary, belies CMU's contention that this Court has recognized
land is alienable and disposable. Unlike in De la Rosa, the lands, having been reserved for public that the said land is a private property or that the land is alienable and disposable.
purpose by virtue of Proclamation No. 476, have not ceased to be so at the time the said directive

102
As to what constitutes alienable and disposable land of the public domain, this Court expounds in its 15th Judicial District, Branch II of Bukidnon is NULL and VOID. Accordingly, Original Certificate of Title
pronouncements in Secretary of the Department of Environment and Natural Resources v. Yap: 59 Nos. 0-160, OCT No. 0-161 and OCT No. 0-162 issued in the name of petitioner, are CANCELLED. Sheet
1, Lot 1 of Ir-1031-D and Sheet 2, Lot 2 of Ir-1031-D are ORDERED REVERTED to the public domain.
xxxx
SO ORDERED.
A positive act declaring land as alienable and disposable is required. In keeping with the presumption
of State ownership, the Court has time and again emphasized that there must be a positive act of the
government, such as an official proclamation, declassifying inalienable public land into disposable
land for agricultural or other purposes. In fact, Section 8 of CA No. 141 limits alienable or disposable
lands only to those lands which have been "officially delimited and classified."

The burden of proof in overcoming the presumption of State ownership of the lands of the public
domain is on the person applying for registration (or claiming ownership), who must prove that the
land subject of the application is alienable or disposable. To overcome this presumption,
incontrovertible evidence must be established that the land subject of the application (or claim) is
alienable or disposable. There must still be a positive act declaring land of the public domain as
alienable and disposable. To prove that the land subject of an application for registration is alienable,
the applicant must establish the existence of a positive act of the government such as a presidential
proclamation or an executive order; an administrative action; investigation reports of Bureau of Lands
investigators; and a legislative act or a statute. The applicant may also secure a certification from the
government that the land claimed to have been possessed for the required number of years is
alienable and disposable.60

In the case at bar, CMU failed to establish, through incontrovertible evidence, that the land
reservations registered in its name are alienable and disposable lands of public domain. Aside from
the series of indorsements regarding the filing of the application for the compulsory registration of
the parcels of land and the said directive from the President, CMU did not present any proof of a
positive act of the government declaring the said lands alienable and disposable.

For lack of proof that the said land reservations have been reclassified as alienable and disposable,
the said lands remain part of inalienable public domain, hence; they are not registrable under Torrens
system.

This Court will not discuss the other issue raised by CMU, e.g., the filing of the petition for cadastral
proceeding was pursuant to the written consent, authorization and directive of the OSG, as the same
was not discussed in the assailed Decision of the CA. This Court also dismisses the other issue raised
- that the titles in CMU's name were singled out by respondent - for lack of evidence.

WHEREFORE, the petition for review on certiorari dated January 14, 2011 filed by petitioner Central
Mindanao University is hereby DENIED. The Decision dated December 30, 2010 of the Court of
Appeals in CA-G.R. SP No. 81301 is hereby AFFIRMED. The proceedings in the Court of First Instance,

103
G.R. No. 199537 6. She, through her predecessors, had been in peaceful, open, continuous, exclusive, and notorious
possession of the subject lot in the concept of an owner for over thirty (30) years.
REPUBLIC OF THE PHILIPPINES, Petitioner,
vs. On 28 April 2004, the land registration court granted Tan’s application. The court confirmed her title
ANDREA TAN, Respondent. over the subject lot and ordered its registration.

DECISION The Republic appealed the case to the CA, arguing that Tan failed to prove that she is a Filipino citizen
who has been in open, continuous, exclusive, and notorious possession and occupation of the subject
BRION, J.: lot, in the concept of an owner, since June 12, 1945, or earlier, immediately preceding the filing of
her application. The appeal was docketed as CA-G.R. CEB-CV No. 00702.
This is a petition for review on certiorari filed by the Republic of the Philippines (Republic) from the
May 29, 2009 decision1 and October 18, 2011 resolution2 of the Court of Appeals (CA) in CA-G.R. CEB- On May 29, 2009, the CA denied the appeal. The CA observed that under the Public Land Act, there
CV No. 00702. The CA denied the Republic's appeal from LRC Case No. N-1443 wherein the Municipal are two kinds of applicants for original registration: (1) those who had possessed the land since June
Trial Court in Consolacion, Cebu, granted respondent Andrea Tan's application for land title 12, 1945; and (2) those who already acquired the property through prescription. The respondent’s
registration. application fell under the second category.

Antecedents The CA noted that before land of the public domain can be acquired by prescription, it must have
been declared alienable and disposable agricultural land. The CA pointed to the certification issued
On October 2, 2002, Tan applied for the original registration of title of Lot No. 4080, Cad. 545-D (new) by the Community Environment and Natural Resources Office (CENRO) as evidence that the subject
situated in Casili, Consolacion, Cebu (the subject lot). She alleged that she is the absolute owner in was classified as alienable and disposable on September 1, 1965, pursuant to Land Classification
fee simple of the said 7,807 square-meter parcel of residential land she purchased from a certain Project No. 28. The CA concluded that Tan had already acquired the subject lot by prescription.
Julian Gonzaga on September 17, 1992. Her application was docketed as LRC Case No. N-144.
On July 2, 2009, the Republic moved for reconsideration. Citing Republic v. Herbieto,4 it argued that
After complying with the jurisdictional requirements, the land registration court issued an order of an applicant for judicial confirmation of title must have been in possession and occupation of the
general default, excepting the State which was duly represented by the Solicitor General. subject land since June 12, 1945, or earlier, and that the subject land has been likewise already
declared alienable and disposable since June 12, 1945, or earlier.5
During the trial, Tan proved the following facts:
On October 18, 2011, the CA denied the motion for reconsideration citing the then recent case of
1. The subject lot is within Block 1, Project No. 28, per LC Map No. 2545 of Consolacion, Cebu; Heirs of Mario Malabanan v. Rep. of the Philippines6 which abandoned the ruling in Herbieto.
Malabanan declared that our law does not require that the property should have been declared
2. The subject lot was declared alienable and disposable on September 1, 1965, pursuant to Forestry alienable and disposable since June 12, 1945, as long as the declaration was made before the
Administrative Order No. 4-1063; application for registration is filed.7

3. Luciano Gonzaga who was issued Tax Declaration Nos. 01465 in 1965 and 02983 in 1972 initially On January 5, 2012, the Republic filed the present petition for review on certiorari.
possessed the subject lot.
The Petition
4. After Luciano’s death, Julian Gonzaga inherited the subject lot;
The Republic argues: (1) that the CA misapplied the doctrine in Malabanan; and (2) that the CENRO
5. Andrea Tan purchased the subject lot from Julian Gonzaga on September 17, 1992; certification and tax declarations presented were insufficient to prove that the subject lot was no
longer intended for public use.

104
Meanwhile, the respondent insists that she has already proven her title over the subject lot. She (1) Those who by themselves or through their predecessors-in-interest have been in open,
maintains that the classification of the subject lot as alienable and disposable public land by the DENR continuous, exclusive and notorious possession and occupation of alienable and disposable lands of
on September 1, 1965, per Land Classification Project No. 28, converted it into patrimonial property the public domain under a bona fide claim of ownership since June 12, 1945, or earlier;
of the State.
(2) Those who have acquired ownership of private lands by prescription under the provision of
From the submissions, the lone issue is whether a declaration that Government-owned land has existing laws;
become alienable and disposable sufficiently converts it into patrimonial property of the State,
making it susceptible to acquisitive prescription. (3) Those who have acquired ownership of private lands or abandoned river beds by right of accession
or accretion under the existing laws;
Our Ruling
(4) Those who have acquired ownership of land in any other manner provided for by law.13
We find the petition meritorious.
The PRD also recognizes prescription as a mode of acquiring ownership under the Civil Code.14
All lands of the public domain belong to the State. It is the fountain from which springs any asserted Nevertheless, prescription under Section 14(2) must not be confused with judicial confirmation of
right of ownership over land. Accordingly, the State owns all lands that are not clearly within private title under Section 14(1). Judicial confirmation of title requires:
ownership. This is the Regalian Doctrine which has been incorporated in all of our Constitutions and
repeatedly embraced in jurisprudence.8 Under the present Constitution, lands of the public domain 1. That the applicant is a Filipino citizen;15
are not alienable except for agricultural lands.9
2. That the applicant, by himself or through his predecessors-ininterest, has been in open, continuous,
The Public Land Act10 (PLA) governs the classification, grant, and disposition of alienable and exclusive and notorious possession and occupation of the property since June 12, 1945;16
disposable lands of the public domain. It is the primary substantive law on this matter. Section 11
thereof recognizes judicial confirmation of imperfect titles as a mode of disposition of alienable public 3. That the property had been declared alienable and disposable as of the filing of the application.17
lands.11 Relative thereto, Section 48(b) of the PLA identifies who are entitled to judicial confirmation
of their title: Only private property can be acquired by prescription. Property of public dominion is outside the
commerce of man.18 It cannot be the object of prescription 19 because prescription does not run
(b) Those who by themselves or through their predecessors-in-interest have been in open, against the State in its sovereign capacity.20 However, when property of public dominion is no longer
continuous, exclusive, and notorious possession and occupation of agricultural lands of the public intended for public use or for public service, it becomes part of the patrimonial property of the State.
domain, under a bona fide claim of acquisition or ownership, since June 12, 1945, immediately 21 When this happens, the property is withdrawn from public dominion and becomes property of
preceding the filing of the application for confirmation of title, except when prevented by war or force private ownership, albeit still owned by the State.22 The property is now brought within the
majeure. Those shall be conclusively presumed to have performed all the conditions essential to a commerce of man and becomes susceptible to the concepts of legal possession and
government grant and shall be entitled to a certificate of title under the provisions of this chapter. prescription.1avvphi1
(As amended by PD 1073.)
In the present case, respondent Tan’s application is not anchored on judicial confirmation of an
The Property Registration Decree12 (PRD) complements the PLA by prescribing how registrable lands, imperfect title because she does not claim to have possessed the subject lot since June 12, 1945. Her
including alienable public lands, are brought within the coverage of the Torrens system. Section 14 of application is based on acquisitive prescription on the claim that: (1) the property was declared
the PRD enumerates the qualified applicants for original registration of title: alienable and disposable on September 1, 1965; and (2) she had been in open continuous, public, and
notorious possession of the subject lot in the concept of an owner for over thirty (30) years.
Section 14. Who may apply. The following persons may file in the proper Court of First Instance an
application for registration of title to land, whether personally or through their duly authorized In our 2009 decision and 2013 resolution23 in Malabanan, we already held en banc that a declaration
representatives: that property of the public dominion is alienable and disposable does not ipso facto convert it into
patrimonial property. We said:

105
Accordingly, there must be an express declaration by the State that the public dominion property is
no longer intended for public service or the development of the national wealth or that the property
has been converted into patrimonial. Without such express declaration, the property, even if
classified as alienable or disposable, remains property of the public dominion, pursuant to Article
420(2), and thus incapable of acquisition by prescription. It is only when such alienable and disposable
lands are expressly declared by the State to be no longer intended for public service or for the
development of the national wealth that the period of acquisitive prescription can begin to run. Such
declaration shall be in the form of a law duly enacted by Congress or a Presidential Proclamation in
cases where the President is duly authorized by law.24

While a prior declaration that the property has become alienable and disposable is sufficient in an
application for judicial confirmation of title under Section 14(1) of the PRD, it does not suffice for the
purpose of prescription under the Civil Code.25 Before prescription can even begin to run against the
State, the following conditions must concur to convert the subject into patrimonial property:

1. The subject lot must have been classified as agricultural land in compliance with Sections 2 and 3
of Article XII of the Constitution;

2. The land must have been classified as alienable and disposable;26

3. There must be a declaration from a competent authority that the subject lot is no longer intended
for public use, thereby converting it to patrimonial property.

Only when these conditions are met can applicants begin their public and peaceful possession of the
subject lot in the concept of an owner.

In the present case, the third condition is absent. Even though it has been declared alienable and
disposable, the property has not been withdrawn from public use or public service. Without this,
prescription cannot begin to run because the property has not yet been converted into patrimonial
property of the State. It remains outside the commerce of man and the respondent’s physical
possession and occupation thereof do not produce any legal effect. In the eyes of the law, the
respondent has never acquired legal possession of the property and her physical possession thereof,
no matter how long, can never ripen into ownership.

WHEREFORE, we hereby GRANT the petition. The May 29, 2009 decision and October 18, 2011
resolution of the Court of Appeals in CA- G.R. CEB-CV No. 00702 are REVERSED and SET ASIDE. The
respondent's application for Land Registration is DENIED for lack of merit. No pronouncement as to
costs.

SO ORDERED.

106
G.R. No. 133465 September 25, 2000 default, denied the same in its order dated August 22, 1996. The trial court gave greater weight to
the report of the Land Registration Authority (LRA) that petitioner's certificate of title was issued
AMELITA DOLFO, petitioner, without any legal basis and the report of the National Bureau of Investigation (NBI) that the signature
vs. of Antonia Cabuco, the Register of Deeds of the Province of Cavite signatory on the certificate, was a
THE REGISTER OF DEEDS FOR THE PROVINCE OF CAVITE, TRECE MARTIRES CITY, THE REPUBLIC OF THE forgery. This is notwithstanding the documents proffered by petitioner allegedly showing the
PHILIPPINES, LAND REGISTRATION AUTHORITY, CESAR E. CASAL, RUSTICO A. CASAL, ERNESTO A. genuineness of the signature of Antonia Cabuco on the certificate of title. The trial court opined that
CASAL, RODOLFO A. CASAL, ALFREDO A. CASAL, JR., EMMANUEL A. B. CASAL, RAFAEL S. CASAL, JR., C. petitioner's title over the subject property was of doubtful nature and that allowing her to intervene
JOSEFINA S. CASAL, CELEDONIA S. CASAL, WILHELMINA S. CASAL, MELANIO MEDINA, ADELAIDA in the LRC cases would unduly delay the proceedings.4
MEDINA, AURORA MEDINA, C. P. G. AGRICOM CORPORATION and HEIRS OF DAMIAN ERMITANIO and
CELEDONIA MARTINEZ, respondents. Meanwhile, on August 1, 1996, Atty. Artemio Caña, in his capacity as Acting Register of Deeds of the
Province of Cavite, filed a complaint for the annulment of petitioner's certificate of title before the
DECISION Regional Trial Court, Branch 89, Bacoor, Cavite.5 The matter remains pending in that court.

MENDOZA, J.: On the other hand, the Regional Trial Court, Branch 19, Bacoor, Cavite rendered a joint decision
recognizing and confirming the rights of private respondents over the litigated property and ordered
This is a petition for review on certiorari of the decision1 of the Sixteenth Division of the Court of the issuance of a Decree of Registration in their favor.6
Appeals in CA-G.R. SP No. 41896 entitled, "Amelita Dolfo v. Hon. Novato T. Cajigal, et al." Said decision
upheld the orders dated May 7, 1996 and August 22, 1996 of the Regional Trial Court, Branch 19, Later, petitioner filed before the Court of Appeals a petition for certiorari and mandamus to annul
Bacoor, Cavite, in LRC Case Nos. B-89-14 and B-90-6 denying petitioner's motion for leave to intervene and set aside the above orders of the Regional Trial Court, Branch 19, Bacoor, Cavite. Petitioner also
and/or admit complaint in intervention as well as her motion for reconsideration. The petition prayed that the latter be compelled to give due course to her motion for leave to intervene and/or
likewise assails the appellate court's resolution denying petitioner's motion for reconsideration and admit complaint-in-intervention. The petition was later amended to include the LRA as party
the trial court's joint decision recognizing the rights of private respondents over a parcel of land respondent.7
located in Barangay Lantic, Carmona, Cavite which is the subject of the abovesaid LRC Cases.
On October 20, 1997, the Court of Appeals rendered its decision denying the petition due course, the
The pertinent facts are as follows: dispositive portion of which reads:

On March 5, 1996, petitioner and Yangtze Properties, Inc. (Yangtze) filed a motion for leave to file WHEREFORE, the petition is hereby DENIED DUE COURSE and is DISMISSED. Costs against petitioner.
and/or admit complaint-in-intervention in LRC Cases Nos. B-94-60, B-89-14 and B-90-6 pending
before the Regional Trial Court, Branch 19, Bacoor, Cavite. The first case is for reconstitution of SO ORDERED.8
Original Certificate of Title No. 362 purportedly covering the subject real property, while the last two
were cases for registration of title. Petitioner alleged that she is the registered owner of the real The Court of Appeals likewise denied petitioner's motion for reconsideration in its resolution dated
property subject of the said LRC Cases as shown by Transfer Certificate of Title No. T-320601 issued April 21, 1998.9
in her name by the Register of Deeds of Trece Martires City. Yangtze, petitioner's co-movant, had
earlier entered into a Contract to Sell with petitioner over the said property.2 In its order dated May Petitioner now contends that:
7, 1996, the trial court denied the aforementioned motion on the grounds that: 1) it is a procedural
error to file a complaint for intervention in cases involving original application for land registration, I. THE RESPONDENT COURT OF APPEALS GRAVELY ERRED IN HOLDING THAT THE PROPER REMEDY IN
the proceedings therein being in rem; and 2) there had already been an order of general default THE LAND REGISTRATION CASES IS AN OPPOSITION TO THE APPLICATION OF THE APPLICANTS, AND
entered by the court against those who failed to oppose the applications. The trial court noted NOT A MOTION TO INTERVENE IN THE PROCEEDINGS BEFORE THE TRIAL COURT.
petitioner's failure to exercise any act of dominion over the subject property consistent with her
allegation of ownership.3 On May 15, 1996, petitioner and Yangtze filed a motion for reconsideration
of the May 7, 1996 order. The trial court, treating the motion as a motion to lift the order of general

107
II. THE RESPONDENT COURT OF APPEALS SERIOUSLY ERRED IN NOT HOLDING THAT THE RESPONDENT
TRIAL COURT GRAVELY ERRED IN NOT ADMITTING PETITIONER'S MOTION FOR INTERVENTION ON The rule that a title issued under the Torrens System is presumed valid and, hence, is the best proof
THE BASIS OF PETITIONER'S POSSESSION OF HER INDEFEASIBLE TITLE OVER THE SUBJECT PROPERTIES. of ownership of a piece of land does not apply where the certificate itself is faulty as to its purported
origin.13
III. THE RESPONDENT COURT OF APPEALS SERIOUSLY ERRED IN NOT UPHOLDING THE TITLE OF THE
PETITIONER, TCT NO. T-320601, OVER THE PROPERTIES IN QUESTION DESPITE PETITIONER'S In this case, petitioner anchors her arguments on the premise that her title to the subject property is
OVERWHELMING EVIDENCE TO PROVE THE GENUINENESS AND DUE EXECUTION OF HER TITLE, AND indefeasible because of the presumption that her certificate of title is authentic. However, this
DESPITE ITS EXPRESS AND CATEGORICAL ACKNOWLEDGMENT OF THE FACT THAT PETITIONER INDEED presumption is overcome by the evidence presented, consisting of the LRA report dated May 24,
PRESENTED NUMEROUS DOCUMENTS TO PROVE THE AUTHENTICITY OF HER TITLE. 199614 that TCT No. T-320601 was issued without legal basis and the NBI report dated June 20,
199615 that the signature of Antonia Cabuco was a forgery. Although petitioner submitted
We find petitioner's contentions unmeritorious. documents purporting to show the genuineness of Antonia Cabuco's signature, she has not refuted
the findings contained in the LRA report that her certificate of title has no legal basis. Thus, in its
First. The provisions of §§ 14 and 25 of P.D. No. 1529 (Property Registration Decree) show that the report, the LRA stated:
applicant and the oppositor are the only parties in cases of original applications for land registration,
unlike in ordinary civil actions where parties may include the plaintiff, the defendant, third party Verification conducted in the Registry of Deeds of Cavite Province on 21 May 1996 disclosed that
complainants, cross-claimants, and intervenors. there is no document on file in the registry vault to support the issuance of TCT No.T-320601 in favor
of Amelita Dolfo. Even the Primary Entry Book for Act 496 under the date 18 November 1991 does
It is now settled that a motion to intervene in a land registration case cannot be allowed. A party not indicate that a document was presented for registration in favor of Amelita Dolfo affecting TCT
wishing to be heard should ask for the lifting of the order of general default, and then if lifted, file an No. 11520 which resulted in the issuance of TCT No. T-320601. Instead, page 232 of the Primary Entry
opposition to the application for registration. This is so because proceedings in land registration are Book, Volume 47 (Annex "B") shows that under the date - 18 November 1991 there appears no
in rem and not in personam, the sole object being the registration applied for, not the determination document entered therein at 11:05 a.m. in favor of Amelita Dolfo or in her behalf affecting the parcel
of any right connected with the registration.10 of land described in TCT No. T-320601.

Second. Both the trial court and the Court of Appeals made a factual finding that petitioner's title to This Investigator also failed to locate despite the thorough search of the vault clerks, TCT No. 11520
the land is of doubtful authenticity. the supposed title from whence the subject TCT No. T-320601 was derived. What are filed in the title
volume are certificates of titles, including TCT No. T-11519 and TCT No. T-11521 both issued by RD
Having jurisdiction only to resolve questions of law, this Court is bound by the factual findings of the Cuevas on 5 November 1964 at 9:00 a.m. (see Annexes "C" & "C-1"). In other words, TCT No. 11520
trial court and the Court of Appeals.11 Even if intervention is allowed in cases of original registration was supposed to have been issued by RD Cuevas in November 1964. In the absence of the title, it
of title, petitioner cannot rely on her certificate of title in view of the evidence respecting its cannot however be determined if TCT No. T-11520 covers the same parcel of land in the subject title
genuineness. As correctly held by the Court of Appeals: of Amelita Dolfo.

Moreover, even if intervention is proper, petitioner's reliance on her title is infirm. While she Records of this Authority show that Judicial Forms 109 and 109-D (CB printed) with Serial No. 2061717
presented numerous documents to prove its authenticity, however, they have been disputed by were requisitioned by and issued to Cavite Registry of Deeds on 21 October 1991. There appears no
Benjamin Flestado, Chief of the Inspection and Investigation Division of the Land Registration report of consumption pertaining to those title-forms was submitted by the Cavite Registry of Deeds
Authority (LRA), in his Report showing that her T.C.T. No. T-320601 was issued without legal basis and (see Annexes "D" to "D-5").
that no document was on file with the Primary Entry Book of the Registry of Deeds of Trece Martires
City to support the issuance thereof. This Report concludes that petitioner's T.C.T. No. T-320601 is On the other hand, records of the Cavite Registry of Deeds show that Judicial Forms 109-109-D with
spurious. Such finding is reinforced by the NBI Report dated June 20, 1996 showing that the signature Serial No. 2061717 were consumed and used for a certificate and of title, TCT No. 322182, in favor of
of Register of Deeds Antonia Cabuco appearing on petitioner's title is a forgery. Consequently, Atty. Manuel dela Cruz and not for issuance of TCT No. T-320601 in favor of Amelita Dolfo. This is confirmed
Artemio Cana, Acting Register of Deeds of Cavite, filed a complaint with the Regional Trial Court, by the Certification of Deputy Register of Deeds, dated 21 May 1996 (Annex "E"), which attests:
Branch 89 at Bacoor for annulment of petitioner's title.12

108
This is to certify that as per records on file in the issuance book dated October 25, 1991, Judicial Form Thus, petitioner cannot invoke the indefeasibility of her certificate of title.1âwphi1 It bears emphasis
109-109-D with Serial No. 2061717 was issued to Manuel dela Cruz with corresponding TCT No. that the Torrens system does not create or vest title but only confirms and records one already
322182 and not TCT No. T-320601 in favor of Amelita Dolfo. existing and vested.18 Thus, while it may be true, as petitioner argues, that a land registration court
has no jurisdiction over parcels of land already covered by a certificate of title, it is equally true that
It is, however, also unfortunate that TCT No. 322182 (with Serial No. 2061717) and the certificate of this rule applies only where there exists no serious controversy as to the authenticity of the certificate.
title with Serial No. 2061716 are not also filed in the corresponding title (book) volume. What are filed
therein are certificates of title, including TCT No. T-322180 (with Serial No. 2061715) and TCT No. T- Fourth. Indeed, to allow petitioner to intervene in the LRC cases would not avoid multiplicity of suits
322183 (with Serial No. 2061718) both issued by RD Cabuco on 8 August 1993 and 25 October 1991, in view of the case for annulment and cancellation of TCT No. T-320601 now pending before the
respectively (see Annexes "F" & "F-1"). Regional Trial Court, Branch 89, Bacoor, Cavite. It is premature for petitioner to intervene in the LRC
cases because her certificate of title, supposedly her best proof of ownership over the property
The Issuance Book of title-forms on file in the Cavite RD, particularly page 134 (Annex "G"), also described therein, is questionable. Besides, inasmuch as the authenticity of her certificate of title is
confirms that Judicial Form 2061717 was issued or released for TCT No. 322182 and not for TCT No. also being questioned in the LRC cases, the evidence that she will present to the prove the contrary
T-320601. would be the same evidence she will present in the case for annulment of title. At this point, where
there is already a decree of registration issued in favor of private respondents, it is moot and academic
Furthermore, registry records show, particularly the same Issuance Book of title-forms, page 88 to allow petitioner to participate in the LRC cases for the purpose of preventing possible double titling
(Annex "H"), that the Judicial Forms 109 and 109-D used for the genuine TCT No. 320601 were with of property. As the trial court correctly stated, petitioner is not left without remedy even if she was
Serial No. 2037534 (erroneously typed in the certification as 2037519); and that the said TCT No. not allowed to intervene. If it is shown that her certificate of title is genuine and that she is the true
320601 was issued in the name of Molino Homes and not in favor of Amelita Dolfo. This is confirmed owner of the litigated property, the proceedings in the land registration cases would then be null and
by the Certification of DRD Diosdado A. Concepcion, dated 21 May 1996, which states (Annex "H-1"): void because the trial court has no jurisdiction on the matter. Otherwise, she could sue for damages.

This is to certify that as per records on file in this registry, dated October 8, 1991 Judicial Form 109- WHEREFORE, the petition is DENIED and the decision and the resolution of the Court of Appeals are
109-D with Serial Number 2037519 TCT No. 320601 Book No. 1701 Page 101 issued in the name of AFFIRMED. Costs against petitioner.
Molino Homes and was received by Amania Jimenez.
SO ORDERED.
When interviewed, DRD Concepcion disclosed to this Investigator that the supposed original of TCT
No. T-320601 in the name of Amelita Dolfo is detached from the title volume. He could not tell where
it is now, not even the vault clerks of the registry. However, Atty. Concepcion surmised that the same
is in the possession/custody of Ms. Melany Victoria, OIC Deputy Registry of Deeds. Atty. Concepcion
furnished this Investigator a certified copy (of a xerox copy in his file) of the subject TCT No. T-320601
(see Annex "I"). He further intimated that based on his own personal verification he believes that the
supposed title in the name of Amelita Dolfo is of doubtful authenticity. In fact, Atty. Concepcion
further averred, that former Register of Deeds Antonia Cabuco disowned the signature above the
typewritten name "Antonia B. Cabuco, Register of Deeds" in the subject TCT No.T-320601.16

In an effort to remove any doubt on the veracity of her certificate of title, petitioner questioned the
credibility of Atty. Artemio Caña who filed an action for annulment of her title. However, her evidence
to prove the genuineness of her certificate of title was the letter of the LRA Administrator, Reynaldo
Maulit, who, in declaring the existence of TCT No. T-320601 in the vaults of the Register of Deeds of
the Province of Cavite, referred to the letter-report of the same Atty. Artemio Caña dated April 30,
1996.17

109
G.R. No. 171535 June 5, 2009 At this point, it becomes imperative to trace the chain of ownership over Lot No. 4763-D. It is
undisputed that the original owners of said property were the spouses Julian Cuison and Marcosa
MACTAN-CEBU INTERNATIONAL AIRPORT AUTHORITY, Petitioner, Cosef, who owned the entire Lot No. 4763, of which Lot No. 4763-D is a portion of (sic). Unfortunately
vs. for herein parties, this is where the similarity of facts end (sic), and the instant controversy begins.
SPOUSES EDITO and MERIAN TIROL and SPOUSES ALEJANDRO and MIRANDA NGO, Respondents.
According to plaintiffs-appellees: Originally, the entire Lot No. 4763 was decreed in the names of
DECISION spouses Julian Cuison and Marcosa Cosef under the provisions of the Land Registration Act on June
1, 1934. [In] January 1974, spouses Julian Cuison and Marcosa Cosef sold Lot No. 4763 to Spouses
PUNO, C.J.: Moises Cuizon and Beatriz Patalinghug. The latter spouses thereafter succeeded to secure the
reconstitution of Original Certificate of Title of Lot No. 4763, Opon Cadastre as evidenced by Court
Before the Court is a Petition for Review on Certiorari under Rule 45 of the 1997 Rules of Civil Order dated July 3, 1986. Said Court Order subsequently became final and executory, thus a
Procedure seeking to reverse, annul and set aside (i) the May 27, 2005 Decision1 of the Court of reconstituted title, OCT No. RO-2754, was issued in the name of the original owners-spouses Julian
Appeals in CA–G.R. CV No. 72867 entitled "Spouses Edito and Merian Tirol, et al. v. Mactan-Cebu Cuison and Marcosa Cosef. On September 12, 1986, the Deed of Absolute Sale between spouses Julian
International Airport Authority," and (ii) its February 17, 2006 Resolution2 denying petitioner’s Cuison/Marcosa Cosef and spouses Moises Cuizon/Beatriz Patalinghug was registered and annotated
motion for reconsideration. on OCT No. RO-2754, which was cancelled to give way to the issuance of TCT No. 16735 in the name
of spouses Moises Cuizon and Beatriz Patalinghug. Thereafter, the latter sold a portion, denominated
The instant case finds its genesis in a complaint for quieting of title filed on August 8, 1996 by as Lot No. 4763-D, to Mrs. Elma Jenkins on December 15, 1987, who[,] as earlier discussed, sold the
respondents, Spouses Edito and Merian Tirol and Spouses Alejandro and Miranda Ngo, against same lot to herein plaintiffs-appellees on September 15, 1993. Plaintiffs-appellees contend that all
petitioner Mactan-Cebu International Airport Authority (MCIAA). The facts were aptly summarized by throughout the chain of ownership, the titles – albeit from a reconstituted one – of the previous
the Court of Appeals as follows: owners were absolutely devoid of any annotations of liens, encumbrances, lis pendens, adverse claim,
or anything that may cause a reasonable man of ordinary prudence and diligence to suspect the
The instant appeal revolves around a certain parcel of land, Lot No. 4763-D, over which the parties to contrary. Furthermore, plaintiffs-appellees have been in actual, uninterrupted and peaceful
the above-entitled case assert ownership and possession. possession of the property since 1993, and if the possession of their predecessors-in-interest be
tacked, plaintiffs-appellees would be in constructive, uninterrupted and peaceful possession for sixty-
xxx xxx xxx two (62) long years as of the date of filing their Complaint for Quieting of Title in the court a quo.

Plaintiffs-appellees and business partners, Edito P. Tirol and Alejandro Y. Ngo, along with their According to the defendant-appellant: On March 23, 19863 , the original owners, spouses Julian
respective spouses, claim to have purchased a 2,000 square meter parcel of land, Lot No. 4763-D, Cuison and Marcosa Cosef sold Lot No. 4763 to the government, through the [then] Civil Aeronautics
from a certain Mrs. Elma S. Jenkins, a Filipino citizen married to a certain Mr. Scott Edward Jenkins, Administration (CAA, for brevity). In a Certificate dated March 19, 1959, vendor Julian Cuison
an American citizen, per Deed of Absolute Sale dated September 15, 1993. Plaintiffs-appellees bought confirmed that he was the possessor and actual owner of Lot No. 4763 which was located within the
the said property on the strength of the apparent clean title of vendor Jenkins as evidenced by the "Mactan Alternate International Airport" and that the duplicate copy of the certificate of title was lost
Tax Declaration and Transfer Certificate of Title No. 18216, all under Mrs. Elma Jenkins’ name, which or destroyed during the last war without him or his predecessor(s)-in-interest having received a copy
bear no annotation of liens, encumbrances, lis pendens or any adverse claim whatsoever. After the thereof. Since then, the government, through defendant-appellant MCIAA, has been in open,
sale wherein plaintiffs-appellees were purportedly purchasers for value and in good faith, they continuous, exclusive and adverse possession of the property in the concept of owner. Said lot
succeeded in titling the said lot under their names per Transfer Certificate of Title No. 27044 on allegedly became part of the Clear Zone of Runway 22 for purposes of required clearance for take-off
September 20, 1993, and further proceeded to pay realty taxes thereon. It was only in January 1996 and landing. Moreover, defendant-appellant asserts that plaintiffs-appellees are nothing more than
that plaintiffs-appellees discovered a cloud on their title when their request for a Height Clearance trustees of Lot No. 4763-D in favor of defendant-appellant MCIAA, being merely successors-in-
with the Department of Transportation and Communications was referred to the defendant-appellant interest of the original owners, spouses Julian Cuison and Marcosa Cosef, who undertook in
Mactan[-]Cebu International Airport Authority (MCIAA, for brevity), on account of the latter’s paragraph 4 of the Deed of Absolute Sale, to assist in the reconstitution of title so that the land may
ownership of the said lot by way of purchase thereof dating far back to 1958. be registered in the name of vendee government, through defendant-appellant MCIAA. In paragraph
5 of the same Deed of Absolute Sale, the parties also agreed that the property be registered under

110
Act 3344 pending the reconstitution and issuance of title. Purportedly, in gross and evident bad faith vacant for several decades should have alerted the respondents to the possibility that the lot could
and in open violation of their Deed of Absolute Sale, the spouses Julian Cuison and Marcosa Cosef be part of the airport complex and therefore owned by petitioner.
again sold the same property to spouses Moises Cuizon and Beatriz Patalinghug, who in turn sold the
lot to Mrs. Elma Jenkins, who eventually sold the same to herein plaintiffs-appellees. Defendant- Respondents filed their Motion for Reconsideration6 on January 23, 2001, and a Supplemental (sic)
appellant MCIAA further imputes bad faith to plaintiffs-appellees under the rationale that because to Motion for Reconsideration7 on May 17, 2001. Petitioner duly filed its Opposition8 to the said
their title came from a reconstituted one and that Lot No. 4763 was within the Clear Zone of Runway Motions on April 10, 2001 and June 13, 2001, respectively.
22 of the airport, plaintiffs-appellees should have exerted effort in researching the history of
ownership and cannot possibly claim to be innocent of MCIAA’s ownership and possession thereof.4 In an Order9 dated August 9, 2001, the trial court did a complete volte face and reversed its Decision.
Holding that Article 154410 of the New Civil Code – which set forth the rule on double sales – finds
In its December 4, 2000 Decision,5 the trial court ruled in favor of petitioner MCIAA in this wise: application to the instant case, the trial court ratiocinated:

WHEREFORE, premises considered, the Court rules in favor of defendant and thus DISMISSES the In the words of the Supreme Court in Cruz vs. Cabana, this Court finds that in the case of [a] double
complaint of plaintiffs for want of merit. sale of real property[,] Article 1544 of the New Civil Code applies. Defendant was certainly the first
buyer and the plaintiffs [were] the subsequent buyers, to be exact fourth (sic).
The Republic of the Philippines, represented by the defendant MCIAA, is adjudged as (sic) the lawful
owner of the entire Lot 4763, Opon Cadastre. But who among the parties herein has a better right to Lot No. 4763-D? To answer this question, it is
necessary to determine first the issue [of] whether or not the plaintiffs were buyers in good
The Deed of Absolute Sale involving Lot 4763-D in favor of plaintiffs is hereby declared null and void. faith.lawphil

Transfer Certificate of Title No. 27044 for Lot 4763-D under the names of plaintiffs is likewise deemed xxx xxx xxx
null and void.
The Court is not convinced that indeed the plaintiffs were buyers in bad faith. xxx The registration of
The Register of Deeds is directed to issue to the defendant MCIAA a transfer certificate of title the deed of absolute sale by the defendant at the Registry of Deeds under Act No. 3344 sometime in
covering the whole Lot 4763. 1959 is not the registration being contemplated under the law. "Registration under Act No. 3344
differs materially from registration under the Spanish Mortgage Law and under the Land Registration
The counterclaim of defendant, however, is denied for lack of merit. Act. In the Spanish Mortgage Law[,] there is [an] express provision (Article 17) to the effect that titles
recorded thereunder cannot be annulled or invalidated by prior unrecorded rights, while the Land
No pronouncement as to costs. Registration Act (No. 496) contains a special disposition that only transactions noted on the certificate
of title and entered in the registry books can bind the land. On the other hand, transactions registered
SO ORDERED. under Act No. 3344 cannot defeat a third person with a better right. Of course[,] the law does not
define exactly what may be considered a better right, leaving the matter of its construction to the
The trial court held that there was a valid transfer of title from Spouses Julian Cuison and Marcosa courts. The main reason for the difference in the operation of Act No. 3344 compared with the other
Cosef to the Civil Aeronautics Administration (CAA), and accordingly, the respondents did not buy Lot systems of registration lies obviously in the fact that recordings under said Act No. 3344 are not
No. 4763-D from a person who could validly dispose of it. It likewise ruled that the government preceded by any investigation, judicial or administrative, as to the validity or efficacy of the title
(through the CAA, and now respondent MCIAA) has been in possession of the disputed land since it sought to be recorded." It is undisputed that Lot No. 4763 was a registered land, only that at the time
bought the same in 1958, when a public deed of absolute sale was executed in its favor. Lastly, of registering defendant’s document of sale there was no copy of the certificate of title because the
respondents were considered as having bought Lot No. 4763-D in bad faith since they ignored same was not available due to the after effect of the last global war.
circumstances that should have made them curious enough to investigate beyond the four corners of
the Transfer Certificate of Title. In the trial court’s view, the facts that Lot No. 4763-D (i) is only about Hence, the Court agrees with the plaintiffs when they contended that "even at the time when OCT
320 meters from the center of the runway and therefore part of the clear zone and (ii) has been No. RO-2754 was issued[,] there was no document allegedly proving its (defendant) ownership being
annotated on the certificate of title." At the time when Transfer Certificates of Title Nos. 16735, 18216

111
and 27044 were issued to the plaintiffs and their predecessors-in-interest, there were no annotations
of the alleged claim of the defendant. Thus, the plaintiffs have all the good reasons to rely on the (b) The two (or more) sales transactions must pertain to exactly the same subject matter;
validity of the titles. xxx
(c) The two (or more) buyers at odds over the rightful ownership of the subject matter must each
xxx xxx xxx represent conflicting interests; and

xxx The fact that Lot No. 4763-D was within 320 meters from the center of the runway and within (d) The two (or more) buyers at odds over the rightful ownership of the subject matter must each
airport premises, was part of the clear zone, and had long been vacant are not enough warning to have bought from the very same seller.
third persons dealing [with] such land. It was undisputed that the lot in controversy is outside the
perimeter fence of the defendant. The fact that the said lot was part of the clear zone is not sufficient Obviously, said provision has no application in cases where the sales involved were initiated not by
justification to warn the plaintiffs in (sic) buying it. Such fact was merely for the purpose of just one vendor but by several successive vendors.16 In the instant case, respondents and petitioner
construction of buildings, not for realty ownership.11 (italics in the original) had acquired the subject property from different transferors. Petitioner, through its predecessor-in-
interest (CAA), acquired the entire Lot No. 4763 from its original owners, spouses Julian Cuison and
Aggrieved, petitioner then appealed to the Court of Appeals which rendered a Decision12 on May 27, Marcosa Cosef, on March 23, 1958. On the other hand, respondents acquired the subject parcel of
2005, the dispositive portion of which states: land, a portion of Lot No. 4763, from Mrs. Elma Jenkins, another transferee, some thirty-five years
later. The immediate transferors of Elma Jenkins were the spouses Moises Cuizon and Beatriz
WHEREFORE, premises considered, the appeal is hereby DENIED. Accordingly, the assailed Order Patalinghug who, in turn, obtained the subject property from spouses Julian Cuison and Marcosa
dated August 9, 2001 is AFFIRMED. Cosef. Therefore, the instant controversy cannot be governed by Article 1544 since petitioner and
respondents do not have the same immediate seller.
SO ORDERED.
This notwithstanding, we find that respondents have a better right to Lot No. 4763-D.
On June 21, 2005, petitioner seasonably moved for its reconsideration but the Court of Appeals
denied the same in its February 17, 2006 Resolution.13 Petitioner does not contest that Lot No. 4763, of which the property subject of this case is a part, was
registered under Act No. 496 (the Land Registration Act) even before the Second World War.
Hence this appeal under Rule 45 of the 1997 Rules of Civil Procedure, where petitioner argues that: Paragraph 4 of the Deed of Absolute Sale17 between petitioner and Spouses Julian Cuison and
Marcosa Cosef stipulates, in relevant part:
THE COURT OF APPEALS COMMITTED A SERIOUS ERROR OF LAW WHEN IT AFFIRMED THE AUGUST 9,
2001 ORDER OF THE TRIAL COURT EVEN IF THE SAME IS NOT SUPPORTED BY THE EVIDENCE ON That since the Original/Transfer Certificate of Title of the aforementioned property has been lost
RECORD.14lavvphi1 and/or destroyed, or since the said lot is covered by Cadastral Case No. 20 and a decree issued on July
29, 1930, xxx the VENDEE hereby binds itself to reconstitute said title at its own expense and that the
Simply stated, the issue may be synthesized as follows: Between respondents Spouses Tirol and VENDOR, his heirs, successors and assigns bind themselves to help in the reconstitution of title so
Spouses Ngo, on the one hand, and petitioner MCIAA, on the other, who has the superior right to the that the said lot may be registered in the name of the VENDEE in accordance with law. (italics
subject property? supplied)

We rule in favor of the respondents, but on grounds different than those relied upon by the Court of Additionally, in his Certification18 dated March 19, 1959, Julian Cuison stated that "the duplicate copy
Appeals and the trial court. of the certificate of title for [Lot No. 4763] was lost or destroyed during the last war without having
been received by [him] or [his] predecessor-in-interest."
Preliminarily, reliance on Article 1544 of the New Civil Code is misplaced. In Cheng v. Genato, et al.,15
we enumerated the requisites that must concur for Article 1544 to apply, viz.: In this regard, well-settled is the rule that registration of instruments must be done in the proper
registry in order to effect and bind the land.19 Prior to the Property Registration Decree of 1978, Act
(a) The two (or more) sales transactions must constitute valid sales; No. 496 (or the Land Registration Act) governed the recording of transactions involving registered

112
land, i.e., land with a Torrens title. On the other hand, Act No. 3344, as amended, provided for the
system of recording of transactions over unregistered real estate without prejudice to a third party In this case, since the Extra-Judicial Partition of Real Estate with Deed of Absolute Sale in favor of
with a better right.20 Accordingly, if a parcel of land covered by a Torrens title is sold, but the sale is AZNAR was registered under Act No. 3344 and not under Act No. 496, the said document is deemed
registered under Act No. 3344 and not under the Land Registration Act, the sale is not considered not registered. Rather, it was the sale in favor of Go Kim Chuan which was registered under Act No.
registered21 and the registration of the deed does not operate as constructive notice to the whole 496.
world.22
AZNAR insists that since there was no Torrens title on file in 1964, insofar as the vendors, AZNAR, and
Consequently, the fact that petitioner MCIAA was able to register its Deed of Absolute Sale under Act the Register of Deeds are concerned, the subject property was unregistered at the time. The
No. 3344 is of no moment, as the property subject of the sale is indisputably registered land. Section contention is untenable. The fact that the certificate of title over the registered land is lost does not
50 of Act No. 496 in fact categorically states that it is the act of registration that shall operate to convert it into unregistered land. After all, a certificate of title is merely an evidence of ownership or
convey and affect the land; absent any such registration, the instrument executed by the parties title over the particular property described therein. This Court agrees with the petitioners that AZNAR
remains only as a contract between them and as evidence of authority to the clerk or register of deeds should have availed itself of the legal remedy of reconstitution of the lost certificate of title, instead
to make registration, viz.: of registration under Act 3344. We note that in Aznar Brothers Realty Company v. Aying, AZNAR, beset
with the similar problem of a lost certificate of title over a registered land, sought the reconstitution
SECTION 50. An owner of registered land may convey, mortgage, lease, charge, or otherwise deal with thereof. It is unfortunate that, in the instant case, despite the sale of the subject property way back
the same as fully as if it had not been registered. He may use forms of deeds, mortgages, leases, or in 1964 and the existence of the remedy of reconstitution at that time, AZNAR opted to register the
other voluntary instruments like those now in use and sufficient in law for the purpose intended. But same under the improper registry (Act 3344) and allowed such status to lie undisturbed.25 (italics
no deed, mortgage, lease, or other voluntary instrument, except a will, purporting to convey or affect supplied)
registered land, shall take effect as a conveyance or bind the land, but shall operate only as a contract
between the parties and as evidence of authority to the clerk or register of deeds to make registration. In the instant case, petitioner MCIAA did not bother to have the lost title covering Lot No. 4763-D
The act of registration shall be the operative act to convey and affect the land, and in all cases under reconstituted at any time, notwithstanding the fact that the Deed of Absolute Sale was executed in
this Act the registration shall be made in the office of register of deeds for the province or provinces 1958, or more than fifty years ago. Vigilantibus, non dormientibus, jura subveniunt. Laws must come
or city where the land lies. (italics supplied) to the assistance of the vigilant, not of the sleepy.26 As a matter of fact, this entire controversy may
very well have been avoided had it not been for petitioner’s negligence.
Hence, respondents may not be characterized as buyers in bad faith for having bought the property
notwithstanding the registration of the first Deed of Absolute Sale under Act No. 3344. An improper Furthermore, under the established principles of land registration, a person dealing with registered
registration is no registration at all. Likewise, a sale that is not correctly registered is binding only land may generally rely on the correctness of a certificate of title and the law will in no way oblige
between the seller and the buyer, but it does not affect innocent third persons.23 him to go beyond it to determine the legal status of the property,27 except when the party concerned
has actual knowledge of facts and circumstances that would impel a reasonably cautious man to make
Petitioner, however, is of the impression that registration under Act No. 3344 is permissible because such inquiry.28 Applying this standard to the facts of this case, we rule that respondents exercised
the duplicate copy of the certificate of title covering Lot No. 4763-D had been lost or destroyed. This the required diligence in ascertaining the legal condition of the title to the subject property as to be
argument does not persuade. Our pronouncement in Amodia Vda. de Melencion, et al. v. Court of considered innocent purchasers for value and in good faith. We quote with favor the factual findings
Appeals, et al.24 is apropos: of the Court of Appeals in this respect:

In the case at bench, it is uncontroverted that the subject property was under the operation of the Defendant-appellant MCIAA also asseverates that the close proximity of the property to the runway
Torrens System even before the respective conveyances to AZNAR and Go Kim Chuan were made. of the airport (320 meters from the center line of the runway) and the fact that it has been vacant for
AZNAR knew of this, and admits this as fact. Yet, despite this knowledge, AZNAR registered the sale a considerable period should have caused [plaintiffs-appellees] to be dubious of the title of the
in its favor under Act 3344 on the contention that at the time of sale, there was no title on file. We previous owners thereof. This was, in Our opinion, satisfactorily explained by plaintiffs-appellees
are not persuaded by such a lame excuse. when witness Mr. Edito Tirol testified in open court that he never thought it strange that the land had
always been vacant, and that besides, there were private houses beside the vacant lot, suggesting
xxx xxx xxx that the property must be of private ownership and not that of the airport. Furthermore, he testified

113
that he undertook great care in verifying the clean title of the said land, [e.g.,] deputizing an employee
to do the necessary research, personally copying pertinent documents registered in the Registry of
Property and even consulting legal advice on the matter. These, for Us, are badges of good faith.
Besides, being allegedly part of the Clear Zone, ATO aviation rules proscribe merely the installation of
buildings and other physical structures, except landing facilities. Aviation rules (which, although
repeatedly invoked, interestingly were not presented before the court by defendant-appellant
MCIAA) do not prohibit realty ownership.29

IN VIEW WHEREOF, the Petition is hereby DENIED. The May 27, 2005 Decision and the February 17,
2006 Resolution of the Court of Appeals are AFFIRMED.

SO ORDERED.

114

You might also like